You are on page 1of 745

AN ESSENTIAL APPROACH TO

ORDINARY LEVEL CHEMISTRY


BY KAHWA IVAN (BSC.EDUC, MUST)
kahwaivanny@gmail.com/2015bs047@std.must.ac.ug.

1
Contents
1. Introduction to chemistry...........................................................................................
1.1. Definition of chemistry.......................................................................................
1.2. Applications of chemistry....................................................................................
1.3. A laboratory.....................................................................................................
1.4. Uses of a laboratory...........................................................................................
1.5. Precautions/ safety measures/rules and regulations taken in the laboratory........................
1.6. Apparatus used in the laboratory...........................................................................
1.7. Apparatus and their uses......................................................................................
1.7.1. The Bunsen burner.........................................................................................
1.7.2. Flames of the Bunsen burner.............................................................................
1.7.2.1. Luminous flame..........................................................................................
1.7.2.2. Characteristics of a luminous flame..................................................................
1.7.2.3. Non luminous flame....................................................................................
1.7.2.4. Characteristics of a non luminous flame............................................................
1.7.2.5. Parts of a non luminous flame........................................................................
1.7.2.6. Differences between luminous and non-luminous flame........................................
1.7.2.7. Steps followed when lighting a Bunsen burner....................................................
2. States of matter.......................................................................................................
2.1. Solids................................................................................................................
2.2. Liquids..............................................................................................................
2.3. Gases................................................................................................................
2.4. Change of state....................................................................................................
2.5. The Kinetic Particle Theory of Matter........................................................................
2.6. Diffusion............................................................................................................
2.6.1. Diffusion of gases..............................................................................................
2.6.2. Diffusion of liquids............................................................................................
2.6.3. Factors Affecting Rate of Diffusion........................................................................
2.6.3.1. Temperature..................................................................................................
2.6.3.2. Mass of particles............................................................................................
3. Chemical and physical changes...................................................................................
3.1. Differences between physical and chemical changes......................................................
4. Solutions, crystals, compounds and mixtures...................................................................
4.1. Solutions............................................................................................................
4.2. Solute................................................................................................................

2
4.3. Solvent..............................................................................................................
4.4. Types of solutions.................................................................................................
4.4.1. A saturated solution...........................................................................................
4.4.2. A super saturated solution....................................................................................
4.4.3. A suspension....................................................................................................
4.5. Differences between solutions and suspensions.............................................................
4.6. Crystals.............................................................................................................
4.7. How to grow a large crystal of copper (II) sulphate........................................................
4.8. Water of crystallization..........................................................................................
5. Compounds and mixtures..........................................................................................
5.1. Compound..........................................................................................................
5.2. Mixture..............................................................................................................
5.3. Differences between mixtures and compounds.............................................................
5.4. Methods of separation of mixtures............................................................................
5.5. The methods of separation of mixtures include the following...........................................
5.5.1. Filtration.........................................................................................................
5.5.2. Crystallisation & Evaporation to Dryness................................................................
5.5.3. Evaporation to dryness........................................................................................
5.5.4. Decanting........................................................................................................
5.5.5. Using separating funnel......................................................................................
5.5.6. Distillation......................................................................................................
5.5.6.1. Simple Distillation..........................................................................................
5.5.6.2. Separation of a mixture of water and ethanol by distillation.......................................
5.5.6.3. Fractional distillation......................................................................................
5.5.7. Sublimation.....................................................................................................
5.5.8. Magnetic Attraction...........................................................................................
5.5.9. Chromatography...............................................................................................
6. Elements, compounds, atoms and symbols.....................................................................
6.1. An element.........................................................................................................
6.2. An atom.............................................................................................................
6.3. A molecule.........................................................................................................
6.4. Radicals.............................................................................................................
6.5. Chemical symbols................................................................................................
6.6. Metals and non-metals...........................................................................................
6.7. Properties of metals and non-metals..........................................................................

3
7. Atomic structure and the periodic table..........................................................................
7.1. An atom.............................................................................................................
7.2. Composition of an atom.........................................................................................
7.3. Structure of an atom..............................................................................................
7.4. Atomic number....................................................................................................
7.5. Atomic mass.......................................................................................................
7.6. Electronic configuration.........................................................................................
7.7. Electronic configuration of ions................................................................................
7.8. Isotopes.............................................................................................................
8. The periodic table....................................................................................................
8.1. Chemical families.................................................................................................
8.2. Bonding.............................................................................................................
8.3. Ionic/Electrovalent Bonding....................................................................................
8.4. Covalent Bonding.................................................................................................
8.5. Dative bonding....................................................................................................
8.6. Metallic Bonding..................................................................................................
9. Valency.................................................................................................................
9.1. Elements and radicals with their valencies...................................................................
9.2. Radicals.............................................................................................................
10. Chemical formulae...............................................................................................
10.1. Writing chemical formulae...................................................................................
10.2. Calculating the number of atoms of elements in a compound........................................
11. Chemical equations...............................................................................................
11.1. Balanced chemical equations................................................................................
11.2. Balancing chemical equations...............................................................................
12. Types of chemical reactions.....................................................................................
12.1. Direct combination or direct synthesis....................................................................
12.2. Simple decomposition........................................................................................
12.3. Simple replacement...........................................................................................
12.4. Double replacement...........................................................................................
13. The atmosphere and combustion...............................................................................
13.1. Air is a mixture of gases......................................................................................
13.1.1. Oxygen...........................................................................................................
13.1.2. Uses of oxygen.................................................................................................
13.1.3. Nitrogen.........................................................................................................

4
13.1.4. Carbon dioxide.................................................................................................
13.1.5. Noble gases.....................................................................................................
13.1.6. Uses of noble gases............................................................................................
13.1.7. Water vapour....................................................................................................
13.2. Hygroscopic, deliquescent and efflorescent substances................................................
13.2.1. Hygroscopic substances......................................................................................
13.2.2. Deliquescent substances......................................................................................
13.2.3. Efflorescent substances.......................................................................................
13.2.4. Water of crystallisation.......................................................................................
13.3. Drying agents...................................................................................................
13.4. Burning substances in air.....................................................................................
13.6. Rusting...........................................................................................................
13.7. Combustion.....................................................................................................
14. Methods of gas collection.......................................................................................
15. Oxygen..............................................................................................................
15.1. Oxides............................................................................................................
16. Oxidation and reduction.........................................................................................
17. Water and hydrogen............................................................................................ 101
17.1. Sources of water............................................................................................. 101
17.2. Properties of pure water.................................................................................... 101
17.3. Test for water................................................................................................. 102
17.4. Purification of water........................................................................................ 102
17.5. Reactions of metals with water............................................................................103
17.6. Uses of water................................................................................................. 105
17.7. Reactivity Series............................................................................................. 106
18. Hydrogen......................................................................................................... 109
19. Acids, bases and salts.......................................................................................... 113
19.1. Acids........................................................................................................... 113
19.1.1. Common Acids............................................................................................... 113
19.1.2. Laboratory acids: 3 common laboratory acids/Mineral acids.......................................113
19.1.3. Basicity of an acid........................................................................................... 113
19.1.4. Some Acids with Their Basicity..........................................................................114
19.1.5. Strong and Weak Acids..................................................................................... 114
19.1.6. Comparing Strong and Weak Acids with Concentrated and Dilute Acids........................115
19.1.7. Properties of Dilute Acids.................................................................................. 115

5
19.1.8. Storage of Acids.............................................................................................. 116
19.1.9. Uses of Acids................................................................................................. 116
19.1.10. Acids and Hydrogen Ions...............................................................................116
19.2. Bases and Alkalis............................................................................................ 117
19.2.1. Properties of Alkalis......................................................................................... 117
19.2.2. Neutralisation reactions..................................................................................... 117
19.2.3. Preparation of bases......................................................................................... 118
19.2.4. Strong and Weak Bases..................................................................................... 118
19.2.5. Uses of Alkalis............................................................................................... 119
19.3. Indicators and pH............................................................................................ 119
19.3.1. pH............................................................................................................... 119
19.3.2. pH scale........................................................................................................ 119
19.4. Indicators...................................................................................................... 120
19.5. Measuring pH of a Solution...............................................................................120
19.6. Ionic Equations............................................................................................... 121
19.7. Salts............................................................................................................ 123
19.7.1. Types of salts................................................................................................. 124
19.7.2. Preparation of Salts.......................................................................................... 124
19.7.3. Soluble and Insoluble Salts................................................................................ 125
19.7.4. Preparation of Insoluble Salts............................................................................. 125
19.7.5. Preparation of Soluble Salts...............................................................................127
19.7.6. Action of heat on salts...................................................................................... 132
19.8. Solubility of salts............................................................................................ 134
19.8.1. Solubility curves............................................................................................. 135
19.8.2. Uses of solubility............................................................................................ 136
19.9. Determination of solubility of salts......................................................................137
20. Carbon and its compounds.................................................................................... 139
20.1. Oxides of carbon............................................................................................. 145
20.1.1. Carbon dioxide............................................................................................... 145
20.1.2. Carbon monoxide............................................................................................ 152
20.2. Carbonates and hydrogen carbonates....................................................................160
20.3. Calcium Oxide (quicklime)................................................................................ 164
20.4. Sodium carbonate (soda ash).............................................................................. 166
20.5. The Carbon cycle............................................................................................ 169
20.6. Hardness of water............................................................................................ 173

6
21. Electrolysis....................................................................................................... 176
21.1. Laws of electrolysis......................................................................................... 191
21.2. Application of electrolysis.................................................................................196
22. Formulae, stoichiometry and the mole concept...........................................................206
22.1. Relative Atomic Mass....................................................................................... 206
22.2. Percentage Composition....................................................................................208
22.3. Calculating the Mass of an Element in a Compound.................................................208
22.4. Calculating the Mass of Water in a Compound........................................................209
22.5. Mole............................................................................................................ 209
22.6. Molar Mass................................................................................................... 210
22.7. Different Kinds of Chemical Formulae..................................................................211
22.8. Calculating the Empirical Formula from Percentage Composition................................212
22.9. From Empirical formula to Molecular Formula.......................................................212
22.10. Molar Volume of Gases..................................................................................... 216
22.11. Calculations using chemical equations..................................................................217
22.11.1. Constructing Chemical Equations.....................................................................217
22.11.2. Calculations from Equations........................................................................... 218
22.11.2.1. Reacting Masses....................................................................................... 218
22.11.2.2. Reacting Masses and Volumes......................................................................220
22.11.2.3. Calculations involving energy changes...........................................................222
22.12. Concentration of Solutions................................................................................. 222
22.13. Quantitative analysis........................................................................................ 225
22.14. Uses of Titrations in Analysis.............................................................................228
22.14.1. Identification of Acids and Alkalis....................................................................228
22.14.3. Determination of basicity of an acid..................................................................231
22.14.4. Determination of water of crystallisation in oxalic acid (COOH) 2. xH2O.....................232
22.15. Calculations of volume of solutions......................................................................235
22.17. Gay Lussacs law............................................................................................ 236
22.18. Gas laws....................................................................................................... 237
23. Qualitative analysis............................................................................................. 239
24. Sulphur and its compounds................................................................................... 248
24.1. Sulphur......................................................................................................... 248
24.2. Extraction of sulphur by the Fraschs process..........................................................248
24.3. Extraction of sulphur from natural gas..................................................................249
24.4. Uses of sulphur............................................................................................... 249

7
24.5. Allotropes of sulphur........................................................................................ 249
24.5.1. Rhombic sulphur............................................................................................. 250
24.5.2. Monoclinic sulphur.......................................................................................... 250
24.5.3. Amorphous sulphur......................................................................................... 251
24.5.4. Plastic sulphur................................................................................................ 251
24.5.5. Colloidal sulphur............................................................................................. 251
24.6. Properties of sulphur........................................................................................ 252
24.6.1. Physical properties.......................................................................................... 252
24.6.2. Chemical properties of sulphur........................................................................... 252
a. Action of heat on sulphur (in absence of air).................................................................252
b. Combustion of sulphur (in a plentiful supply of air)........................................................252
c. Reaction with metals and non-metals..........................................................................252
d. Action of acids on sulphur....................................................................................... 253
24.7. Oxides of sulphur............................................................................................ 253
24.7.1. Sulphur dioxide.............................................................................................. 253
24.7.1.1. Laboratory preparation of sulphur dioxide..........................................................253
24.7.1.2. Properties of sulphur dioxide...........................................................................254
24.7.1.3. Uses of sulphur dioxide.................................................................................255
24.7.2. Sulphur trioxide.............................................................................................. 255
24.7.2.1. Preparation of sulphur trioxide.........................................................................255
24.8. Sulphuric acid................................................................................................ 256
24.8.1. Industrial manufacture of sulphuric acid by the contact process...................................256
24.8.2. Properties of sulphuric acid................................................................................ 257
24.8.2.1. Physical properties....................................................................................... 257
24.8.2.2. Chemical properties...................................................................................... 258
24.8.3. Uses of sulphuric acid...................................................................................... 259
24.9. Sulphates...................................................................................................... 260
24.10. Hydrogen sulphide.......................................................................................... 262
24.10.1. Laboratory preparation of hydrogen sulphide.......................................................262
24.10.2. Testing for hydrogen sulphide..........................................................................262
24.10.3. Properties of hydrogen sulphide.......................................................................263
24.10.3.1. Physical properties.................................................................................... 263
24.10.3.2. Chemical properties................................................................................... 263
25. Nitrogen and its compounds..................................................................................265
25.1. Nitrogen....................................................................................................... 265

8
25.2. Laboratory preparation of nitrogen from air............................................................265
25.3. Test for nitrogen.............................................................................................. 267
25.4. Properties of nitrogen....................................................................................... 267
25.5. Uses of nitrogen.............................................................................................. 268
25.6. Nitrogen monoxide (nitrogen(II) oxide).................................................................269
25.6.1. Laboratory preparation of nitrogen monoxide.........................................................269
25.6.2. Tests for nitrogen monoxide............................................................................... 269
25.6.3. Properties of nitrogen monoxide..........................................................................270
25.7. Nitrogen dioxide............................................................................................. 270
25.7.1. Laboratory preparation of nitrogen dioxide.............................................................270
25.7.2. Properties of nitrogen dioxide............................................................................. 271
25.8. Ammonia...................................................................................................... 272
25.8.1. Laboratory preparation of ammonia......................................................................272
25.8.2. Industrial preparation of ammonia (Haber process)...................................................273
25.8.3. Tests for ammonia........................................................................................... 273
25.8.4. Properties of ammonia...................................................................................... 274
25.8.5. Solubility of ammonia in water...........................................................................274
25.8.6. Experiment to demonstrate the high solubility of ammonia gas in water.........................274
25.8.7. Action of ammonia on copper (II) oxide................................................................275
25.8.8. Combustion of ammonia................................................................................... 276
25.8.9. Reaction with hydrogen chloride.........................................................................277
25.8.10. Reaction with chlorine................................................................................... 277
25.8.11. Uses of ammonia......................................................................................... 278
25.8.12. Ammonia solution........................................................................................ 279
25.8.12.1. Preparation of ammonia solution...................................................................279
25.8.13. Ammonium salts.......................................................................................... 279
25.8.13.1. Nitrogenous fertilizers................................................................................ 279
25.8.13.2. Effect of heat on ammonium salts..................................................................280
25.8.13.3. Test for ammonium salts.............................................................................281
25.8.14. Reactions of ammonia solution and sodium hydroxide solution................................282
25.8.15. Nitric acid.................................................................................................. 284
25.8.15.1. Laboratory preparation of nitric acid..............................................................284
25.8.15.2. Industrial preparation of nitric acid................................................................285
25.8.15.3. Uses of nitric acid..................................................................................... 285
25.8.15.4. Properties of nitric acid............................................................................... 286

9
25.8.15.5. Nitric acid acting as a strong acid..................................................................286
25.8.15.6. Nitric acid as an oxidizing agent...................................................................287
25.9. Nitrates......................................................................................................... 288
25.9.1. Action of heat on nitrates...................................................................................288
25.9.2. Test for nitrates............................................................................................... 290
26. Chlorine and its compounds..................................................................................298
26.1. Chlorine........................................................................................................ 298
26.1.1. Laboratory preparation of chlorine.......................................................................298
26.1.2. Industrial manufacture of chlorine.......................................................................299
26.1.3. Properties of chlorine....................................................................................... 300
26.1.4. Tests for chlorine............................................................................................. 306
26.1.5. Uses of chlorine.............................................................................................. 306
26.2. Hydrogen chloride........................................................................................... 306
26.2.1. Laboratory preparation of hydrogen chloride..........................................................306
26.2.2. Test for hydrogen chloride................................................................................. 307
26.2.3. Properties of hydrogen chloride...........................................................................307
26.3. Hydrochloric acid............................................................................................ 307
26.3.1. Preparation of hydrochloric acid..........................................................................307
26.3.2. Properties of hydrochloric acid........................................................................... 308
26.3.3. Uses of hydrochloric acid.................................................................................. 309
26.3.4. Properties of hydrogen chloride in methylbenzene....................................................309
26.4. Testing for soluble chloride................................................................................309
27. Extraction of metals............................................................................................ 313
27.1. Introduction................................................................................................... 313
27.2. Concentration of ores....................................................................................... 313
27.3. Sodium......................................................................................................... 314
27.3.1. Extraction of sodium........................................................................................ 314
27.3.2. Uses of sodium metal....................................................................................... 315
27.4. Copper......................................................................................................... 315
27.4.1. Extraction of copper......................................................................................... 316
27.4.2. Concentration of the ore.................................................................................... 316
27.4.3. Roasting and reduction..................................................................................... 316
27.4.4. Refining of the impure copper............................................................................ 317
27.4.5. Uses of copper................................................................................................ 318
27.5. Iron............................................................................................................. 318

10
27.5.1. Extraction of iron............................................................................................ 318
27.5.2. Casting iron (pig-iron)...................................................................................... 319
27.5.3. Wrought iron.................................................................................................. 319
27.5.4. Steel............................................................................................................ 320
27.5.5. Recycling of metals......................................................................................... 321
27.5.6. Alloy........................................................................................................... 321
28. Organic chemistry.............................................................................................. 324
28.1. Introduction................................................................................................... 324
28.2. Hydrocarbons................................................................................................. 324
28.3. Homologous series.......................................................................................... 324
28.4. Functional groups............................................................................................ 324
28.5. Alkanes........................................................................................................ 325
28.5.1. General properties of alkanes..............................................................................330
28.5.1.1. Physical properties....................................................................................... 330
28.5.1.2. Chemical properties...................................................................................... 330
28.6. Petroleum (crude oil)........................................................................................ 333
28.6.1. Fractional distillation of petroleum.......................................................................333
28.6.2. Cracking....................................................................................................... 335
28.6.3. Bio gas......................................................................................................... 335
28.6.4. Disadvantages of bio gas production.....................................................................336
28.6.5. Alkenes........................................................................................................ 336
28.7. Ethene.......................................................................................................... 337
28.7.1. Laboratory preparation of ethene.........................................................................337
28.7.1.1. Physical properties....................................................................................... 338
28.7.1.2. Chemical properties...................................................................................... 338
28.7.2. Uses of ethene................................................................................................ 340
28.7.3. Alkynes........................................................................................................ 341
28.7.4. Ethyne (acetylene)........................................................................................... 342
28.7.4.1. Physical properties of ethyne...........................................................................342
28.7.4.2. Chemical properties of ethyne.........................................................................342
28.7.4.3. Uses of ethyne............................................................................................. 343
28.7.5. Alkanols (alcohol)........................................................................................... 344
28.8. Ethanol (ethyl alcohol)...................................................................................... 344
28.8.1. Manufacture of ethanol..................................................................................... 344
28.8.2. Properties of ethanol........................................................................................ 346

11
28.8.2.1. Chemical properties...................................................................................... 346
28.8.2.2. Uses of ethanol............................................................................................ 347
28.9. Carboxylic acids and esters................................................................................ 347
28.9.1. Properties of carboxylic acids............................................................................. 347
28.10. Esters........................................................................................................... 348
28.10.1. Uses of esters.............................................................................................. 349
28.11. Soap............................................................................................................ 349
28.11.1. Manufacture of soap..................................................................................... 349
28.11.2. The cleansing action of soap........................................................................... 349
28.11.3. Soapless (synthetic detergents)........................................................................350
28.11.4. Advantages of synthetic detergents over soap......................................................350
28.11.5. Advantages of soap over synthetic detergents......................................................351
28.12. Polymerisation............................................................................................... 351
28.13. Addition polymerization.................................................................................... 351
28.14. Polyethene..................................................................................................... 351
28.15. Polypropene................................................................................................... 352
28.16. Polyvinyl chloride (P.V.C).................................................................................352
28.17. Synthetic rubber.............................................................................................. 352
28.18. Condensation polymerization.............................................................................353
28.19. Types of polymers........................................................................................... 353
28.20. Advantages of synthetic polymers over natural polymers...........................................355
28.21. Disadvantages of synthetic polymers....................................................................355
29. Energy changes.................................................................................................. 360
29.1. Introduction................................................................................................... 360
29.2. Enthalpy....................................................................................................... 360
29.3. Exothermic and endothermic reactions..................................................................360
29.4. Types of enthalpy changes................................................................................. 362
29.4.1. Determination of enthalpy (heat) of combustion of ethanol.........................................363
29.5. Determination of the enthalpy (heat) of solution o f sodium chloride.............................366
29.6. Enthalpy of neutralization.................................................................................. 367
29.7. Determination of heat of neutralization.................................................................367
29.8. Enthalpy of displacement..................................................................................369
29.8.1. Determination of the enthalpy (heat) of displacement of the reaction between copper(II)
sulphate solution and zinc.............................................................................................. 369
30. Rate of reaction and equilibrium............................................................................. 374

12
30.1. Rate of reaction.............................................................................................. 374
30.1.1. Determination of rate of reaction.........................................................................374
30.1.2. Determination of rates of reaction by measuring the volume of the gas evolved with time
374
30.1.3. Effects of temperature on the rate of reaction..........................................................378
30.1.3.1. Investigation of the effect of temperature on the rate of reaction...............................378
30.1.4. Effect of a catalyst on the rate of reaction...............................................................379
30.1.4.1. Investigation of the effect of catalyst on the rate of reaction.....................................379
30.1.5. Effect of surface area on the rate of reaction...........................................................381
30.1.5.1. Investigation of the effect of surface area on the rate of reaction...............................381
30.1.6. Effect of light on the rate of reaction.....................................................................382
30.1.6.1. Investigation of the effect of light on the rate of reaction.........................................382
30.1.7. Effect of pressure on the rate of reaction................................................................383
31. Equilibrium...................................................................................................... 383
31.1. Factors affecting equilibrium..............................................................................383
31.2 Trial questions for paper 1 and 2..384

13
1. Introduction to chemistry

1.1. Definition of chemistry


Chemistry is the scientific study of matter, its properties, composition and interactions with
other matter and with energy.

1.2. Applications of chemistry


Applied in:
Extraction of metals
Fermentation to form ethanol
Manufacture of ammonia, sulphuric acid and fertilizers
Water purification
Manufacture of soap
Manufacture of polymers for making plastics, clothes
Manufacture of medicine and other pharmaceuticals

1.3. A laboratory
This is a specialized and organized room where scientific experiments are conducted

1.4. Uses of a laboratory


It is where chemicals and apparatus are kept
It is a place where experiments are conducted
It is a place where all science subjects should be studied and taught from.

1.5. Precautions/ safety measures/rules and regulations taken in the laboratory


Students should never enter the laboratory before the teacher tells them to do so
Laboratory materials should be used on purposes directed by the teacher only
Students should never taste anything in the laboratory
Bottles should never be handled by their necks
Running is prohibited in the laboratory
Always wash or clean the apparatus and your hands after every chemical experiment
Anything taken in the mouth should be pushed out immediately and the mouth washed
several times with water
Do not carry out any experiments that are not authorized by the teacher
Place broken glasses and solids into bin always located in the laboratory

14
When your experiment is completed, turn off the water supply and gas supply and disconnect
any electrical connections and return all the materials and apparatus in their proper places

1.6. Apparatus used in the laboratory


Apparatus Name of apparatus Function
Droppers To allow liquids to be
dispensed drop by drop

Filter funnel Used for separating solids


from liquids via the laboratory
process of filtering

Filter paper Used to separate fine solids


from liquids

Flat bottomed flasks Used to keep a liquid or solid


on the flat surface for studying
purpose

Litmus paper Litmus paper contains a


chemical that turns blue in
basic solutions and reddish-
pink in acidic solutions
Pipette For transferring or measuring
out small quantities of liquid

15
Reagent bottles A glass container to hold
liquid chemicals

Retort stand Used to support apparatus (for


example a flask or
thermometer)

Round bottomed flasks A glass flask used in a


laboratory for holding
chemical liquids and solutions,
which has a spherical shape
for uniform heating, and one
or more long cylindrical
necks.
Separating funnel Laboratory equipment used to
separate liquids which do not
dissolve in one another, and
thus forms layers (immiscible
liquids)
Mortar and pestle Used for pounding and
grinding solid substances to
make them into smaller
particles for ease of handling
Beaker A beaker is a container used
to hold, mix, and pour liquid
or powdered chemicals.

Boiling tube A boiling tube is a small


cylindrical vessel used to
strongly heat substances in the
flame of a Bunsen burner

16
Bunsen burner The function of a Bunsen
burner is to heat substances.

Burette A buret (burette) is used to


dispense known amounts of a
liquid reagent in experiments
for which precision is
necessary, such as a titration
experiment.
Conical flasks Conical flask /
Erlenmeyer flasks are
shaped conically so that they
can be used to mix fluids with
a lesser chance of spilling
Tripod stand A tripod is a three legged
frame for supporting a few
apparatus during heating

Evaporating dish/crucible The Evaporating dish is used


to heat and evaporate liquids.

1.7. Apparatus and their uses

a. The Bunsen burner


The Bunsen burner is a device used as a source of heat in the laboratory. It uses gas fuel.
Bunsen burner

17
b. Flames of the Bunsen burner
The Bunsen burner produces two different flames depending on whether the air hole is open
or closed.
A flame is a combination of burning gases giving out heat and light.
The Bunsen burner produces two different flames namely
1. Luminous flame
2. Non-luminous flame

1.7.b.1. Luminous flame


A luminous flame is a flame formed when the air holes of the Bunsen burner are closed.

1.7.b.2. Characteristics of a luminous flame


This flame produces much light
This flame is yellow in colour, unsteady and quiet

18
The flame produces soot (smoke) when burning
It has four zones which have different colours

1.7.b.3. Non luminous flame


This is the flame which is formed when the air holes of the Bunsen burner are open

1.7.b.4. Characteristics of a non luminous flame


This flame produces little light
It is mainly blue in colour and produces no soot
It is very hot and noisy and therefore is commonly used for heating in the laboratories due to
its being hot and producing no soot
Non luminous flame has three zone s
Non-luminous and luminous flames

1.7.b.5. Parts of a non luminous flame


Inner zone: this is a zone of cool un-burnt gas
Middle zone: this is blue or green in colour, here some of the gas is burnt but not all because
there is no enough air
Pale blue/ purple zone: in this zone burning of the gas is complete

1.7.b.6. Differences between luminous and non-luminous flame


Luminous flame Non luminous flame
Is a yellow flame Is blue in colour
Is unstable Is stable

19
Has four zones Has three zone
Is quiet Is noisy
Is not hot enough Is very hot
Forms soot No soot is formed
Produced when air holes are closed Produced when air holes are open
Produces a lot of light Produces little light
1.7.b.7. Steps followed when lighting a Bunsen burner
Fix the Bunsen burner properly
Connect the Bunsen burner to the gas tap
Close the air holes by turning the metal rings (collar)
Put on the gas tap and switch on for a few seconds
Light the gas by applying a match stick on top of the chimney
Lastly open the air holes by turning on the metal rings

2. States of matter
Matter is anything that occupies space and has weight.
The states of matter are solids, liquids and gases

2.1. Solids
The characteristics of solids include:
- Fixed volume
- Fixed shape
- Incompressible
- Do not flow

Have a definite shape


The particles of solids are closely and regularly packed together
They are held together by strong forces which attract particles to each other
At a certain temperature the regular arrangement of particles in the solid state breaks down.
At this point the solid melts and turns into liquid state. The temperature at solids change to
liquid is called melting point.

2.2. Liquids
The characteristics of liquids include:
- Fixed volume

20
- No fixed shape
-Takes the shape of the container
- Incompressible
- Flow easily

Have definite volume but no definite shape and take up the shapes of their containers e.g.
water, milk
Liquids cannot be compressed by squeezing
When liquids are heated their particles move faster and finally turn into gas
The temperature at which a liquid changes into a gas is called the boiling point e.g. the
boiling point for pure water is 100oC.

2.3. Gases
The characteristics of gases include:
-No fixed volume
- No fixed shape
- Compressible
- Flow in all direction

Do not have a definite shape and volume


The particles cannot attract each other and can be compressed e.g. air
When gases are cooled, they turn into liquids and the process is called condensation.
The behaviour of particles in solids, liquids and gases is governed by the principle of
kinetic theory of gases.

2.4. Change of state


Changes in state are brought about by changes in temperature e.g if a liquid is cooled at very
low temperature; it will change to solid i.e. changing water to ice.

21
Solid

Freezing

Evaporation
Gases Condensatio
Liquids
n

Sublimation is the process of changing a substance from a solid to a gas state without passing
through the liquid state when heated. Substances which sublime include: iodine crystals,
ammonium chloride and iron (III) chloride
Note: water exists in all the three states. When in solid state, it is called ice, in liquid state, it
is called water, and in gaseous state, it is called water vapour/steam. But at room temperature
water is in liquid state.

Particles in solid: Particles in liquid: Particles in gas:


- Are packed close together in - Are packed closely but not - Are far apart and in random
orderly arrangement orderly arranged arrangement
- Have little empty space - Have little empty space - Are free to move anywhere
between them between them but more than in the container
- Can vibrate but cannot move in solids
freely about their fixed - Are not held fixed but free to
position move throughout liquid

Differences between properties of matter and particles in them

22
1. Matter can be coloured (e.g. sulphur is yellow) but particles are not.
2. Substances feel hot/cold but particles dont get hot/cold. The temperature is due to speed of
movement of particles. If hot, particles move fast.
3. Matter expands when heated but particles dont. They increase distance between particles
during expansion.

Changes of State
Melting
Melting is change from solid to liquid by absorbing heat to break force of attraction holding
particles together.
The temperature at which solid melts is melting point.

From the graph:


A-B: the temperature of solid increases to melting point.
B-C: the temperature remains constant as heat is absorbed to break forces of attraction instead
for raising temperature. Solid and liquid are present.
C-D: liquid heats as heat energy increases temperature.
D-E: liquid temperature rises to boiling point.
E: heat energy is absorbed by particles to break the attractive forces so that they move freely
and far apart as gas particles. Thats why the temperature remain constant

Freezing

23
Freezing is the change of liquid to solid by cooling down of liquid.
Freezing point is the temperature at which liquid freezes.

B C

A-B: liquid temperature decreases to freezing point.


B-C: heat energy is released as particles slow down to take up fixed and orderly position of a
solid. The temperature remain constant release of energy compensates for loss of heat to
surroundings.
C-D: solid cools to the temperature of surroundings.
Boiling
Boiling is the change of liquid to gas by absorbing heat to break the forces holding them
together.
Boiling point is the temperature at which liquid boils.
Evaporation
Evaporation is change of liquid to gas without boiling, occurs below boiling point on water
surface. It gives cooling effect heat energy absorbed from surroundings.
Condensation
Condensation is the change of gas to liquid. Heat energy is given out as gas particles slow
down and move closer to one another to form liquid.
Sublimation is the change of solid to gas without melting. Heat is absorbed.

2.5. The Kinetic Particle Theory of Matter


Kinetic theory assumes that
- Particles are too small to be seen directly

24
- There are spaces between particles of matter; the amount of space varies between each
states
- The particles are constantly move; each state moves in different speed

2.6. Diffusion
Diffusion is the spreading and mixing of particles in gases and liquids.

2.6.1. Diffusion of gases


Bromine drops are placed into a jar. Another jar full of air is placed on top of jar with
bromine, separated with cover. Cover is removed and bromine evaporates, filling both jars
with dense reddish-brown bromine vapour.
Explanation:
Bromine particles move from lower jar into spaces between air particles in upper jar. At the
same time, air particles move down from upper jar to mix with bromine particles in lower jar.
Eventually, bromine and air particles are mixed completely.

25
2.6.2. Diffusion of liquids
CuSO4 crystals placed in beaker of water, blue particles of the crystals is spread throughout the
water to form a uniformly blue solution.

26
2.7.

2.7.1. Factors Affecting Rate of Diffusion

2.7.1.1. Temperature
The higher the temperature, the more particles of matter absorb energy making them move faster,
the higher the rate of diffusion; the lower the temperature, the slower the rate of diffusion

2.7.1.2. Mass of particles

Greater mass, the slower it diffuses; smaller mass, the faster it diffuses

Cotton wool soaked in aqueous ammonia and another soaked in hydrochloric acid are placed on
opposite sides of the tube. Ammonium hydroxide (NH4OH) vapor and hydrogen chloride (HCl)
vapor diffuses in the tube and a compound is produced inside the tube closer to HCl soaked
cotton as the particles are heavier. The greater the mass, the slower particles diffuse and the
smaller the mass, the faster particles diffuse.
3. Chemical and physical changes

A chemical change is a process that occurs and a new substance is formed e.g.

rusting, explosion of hydrogen in air,

burning of magnesium to ash,

Burning of a paper to ash, etc

Physical change is a change in which no new substance is formed e.g. freezing (water to ice)

Evaporation of water (water to steam)

Magnetization of iron

Sublimation of solid iodine

Melting of ice

3.1. Differences between physical and chemical changes


Physical change Chemical change

No new Substance is formed A new Substance is formed

The change is reversible The change is irreversible

No energy is evolved or absorbed during the Energy is absorbed of evolved during the
reaction reaction

There is no change in mass There is change in Mass

The word MISE is used to recall these differences

Where M-Mass

I-Irreversible

S-Substance

E-energy

4. Solutions, crystals, compounds and mixtures

4.1. Solutions

A solution is a uniform mixture of two or more substances. When sugar is added to water and
stirred, the sugar dissolves in water.

In this process sugar is called a solute, water is called a solvent and a mixture of sugar and water
is called a solution.
4.2. Solute

A solute is a dissolved substance e.g. sugar, salt.

4.3. Solvent

A solvent: is a substance that dissolves a solute e.g. water, ethanol, petrol.

4.4. Types of solutions

4.4.1. A saturated solution

This is a solution which cannot dissolve any more solute at a given temperature in presence of
undissolved solute.

4.4.2. A super saturated solution

This is a solution which contains more solute than it can hold at a given temperature

4.4.3. A suspension

This is a liquid containing small particles of a solid which are spread throughout it and settle on
standing e.g. a solution of chalk in water.

4.5. Differences between solutions and suspensions

A suspension contains solid particles which can be seen but a solution contains no solid particles.
In suspension the mixture can be separated by filtration while in solution the mixture cannot be
separated by filtration.

In suspension the solid particles settle on standing but in solutions no solid particles can settle on
standing.

4.6. Crystals

A crystal is a solid that has solidified into regular fixed shape.

Crystals have regular shapes, flat surfaces and sharp edges

Crystals are formed when hot solutions cool. Is a hot saturated solution cools rapidly, crystals
formed are many and small but once cooled slowly crystals formed are few and big in size

Substances which form crystals are called crystalline substances i.e. sodium chloride and copper
(II) sulphate. Some solids do not form crystals e.g. charcoal and glass; and these are called non-
crystalline substances.

4.7. How to grow a large crystal of copper (II) sulphate

Fill the beaker with a saturated solution of copper (II) sulphate

Choose one good crystal of copper (II) sulphate and tie a thin thread around it. Hang it in the
solution and place the beaker in a warm place for several days. The crystals grow large and the
solvent slowly evaporates.
4.8. Water of crystallization

This is a definite amount of water some substances chemically combine with when they form
crystals from their solutions in water

A crystalline substance that contains water of crystallization is said to be hydrated

A hydrated substance is that which contains water of crystallization.

Examples of hydrated salts include

Copper (II) sulphate (CuSO4. 5H2O)

Iron (II) sulphate (FeSO4. 5H2O)

Sodium carbonate (Na2SO4. 10H2O) etc.

Some substances do not contain water of crystallization and are said to be anhydrous e.g.
sodium chloride (NaCl), potassium nitrate (KNO3) etc.

When a hydrated substance is heated, it loses its water of crystallization

CuSO4. 5H2O (s) CuSO4(s) + 5H2O (g)

Blue solid white solid


5. Compounds and mixtures

5.1. Compound

A compound is a substance which consists of two or more elements chemically combined


together.

Examples of compounds include:

Water (H2O): this is a compound made up of hydrogen (H) and oxygen (O) as elements

Common salt sodium chloride (NaCl): this is a compound made up of sodium (Na) and
chlorine (Cl)

Glucose (C6 H12O6): this is a compound made up of carbon (C), hydrogen (H) and oxygen (O)

Iron (II) sulphide (FeS): this is a compound made up of iron (Fe) and sulphur (S)

5.2. Mixture

A mixture is a substance which consists of two or more elements or compounds not chemically
combined together.

Examples include: Salt and water, Salt and sand, Water and alcohol, Chalk and water, Air

5.3. Differences between mixtures and compounds

Mixture Compound
Can be Separated by physical means e.g. Substances in it cant be separated by physical
filtration means

Physical Properties of mixtures e.g. colour and Physical properties of compounds are quite
density are the average of constituent different from those elements in them
substances

Energy is not usually given out or absorbed Energy is given out or absorbed when a
when a mixture is formed compound is made

Its Composition is variable, the substances can Its composition is not variable; the elements
be combined in any proportion by mass are combined in definite proportions by mass

The word SPEC is used to recall these differences

Where S- Separated

P- Properties

E- Energy

C- Composition

5.4. Methods of separation of mixtures


Liquid mixtures

There are two types of liquid mixtures i.e. immiscible and miscible liquid mixtures
Miscible liquids:

These are liquids which mix freely and form one layer
Immiscible liquids

These are liquids which do not mix easily and form more than one layer.

5.5. The methods of separation of mixtures include the following

5.5.1. Filtration
Filtration separates insoluble solid from a liquid.
- Mixture is poured through a filter paper with tiny holes
- Large solid particles cannot pass through the pores and trapped in it as residue while tiny liquid
particles pass through as filtrate.

5.5.2. Crystallisation & Evaporation to Dryness


Crystallisation separation of dissolved solid from a solution as well-formed crystals
Evaporation to Dryness separation of dissolved solid from a solution as crystals of salt by
evaporating all the liquid off.
Why crystallisation occurs?
- Solubility of most solutes decrease as temperature decrease, when solution cools, solution cant
hold more solute (saturated) so the extra solute separates as pure crystals.
5.5.3. Evaporation to dryness
- To obtain the solute from a solution
- Requires an evaporating dish
How to separate common salt (sodium chloride) from sand

Common salt and sand are placed in a beaker and water is then added. The mixture is warmed
gently while stirring until the salt completely dissolves. Salt dissolves but sand does not dissolve.

The solution is then filtered. After filtering, the salt solution is obtained separate from sand. The
salt solution is therefore called a filtrate and sand, the residue.

The salt solution is then poured into an evaporating basin. The water evaporates when the salt
solution is heated. It evaporates completely, leaving salt crystals behind on the evaporating dish.

This is called evapouration to dryness.

Using water bath carry out evapouration to dryness


To avoid spitting off (jumping off) of salt crystals formed, the evapourating basin is placed on a
water bath or sand bath. The steam produced from the water bath will heat up the salt solution
until crystals are formed.

5.5.4. Decanting

This method can also be used to separate a mixture of sand and salt in water

Put the mixture of sand and salt in the beaker and add water. Stir the mixture. Salt will dissolve
but sand will not. Leave the beaker to stand for a few minutes for the sand to settle.

The sand will settle on the bottom of the salt solution. Pour off the salt solution carefully without
disturbing the mixture.

The sand will be left on the bottom of the beaker. Then evaporate the salt solution to dryness.
NB:

This method is not as good as filtration and should always be discouraged in the laboratory

This method can also be used to separate chalk from water

Diagram showing decanting

5.5.5. Using separating funnel

- To separate two liquids that do not mix

- Requires the separating funnel

This method is used in separating immiscible liquids.

Immiscible liquids are those which do not mix at all. Such liquids separate into distinct layers
according to their densities. Examples include:
Water and paraffin

Water and mercury

Water and oil

To separate such liquids using a separating funnel, the following procedures are followed.

The mixture of paraffin/oil and water is poured into a separating funnel. Shake vigorously and
then allow to settle

Results

Water has a high density than paraffin and therefore water separates to the bottom and paraffin
goes on top of water making different layers

The tap is opened and the water layer runs out first

Close the tap and put another container where paraffin will be collected. Open the tap to collect
paraffin in a separate container

Separating funnel
5.5.6. Distillation

This is the process of heating a liquid to form vapour and then cooling it back to form a liquid

Distillation helps in separating of substances


(miscible liquid) and also in purification of liquids

5.5.6.1. Simple Distillation

- To obtain the solvent from the solution

- Requires a Liebig condenser and a distilling flask

Used to obtain the solvent from the solution

During distillation, the solution is heated so that its liquid component boils and escapes as a
vapour. The vapour is then cooled by running water and condensed into liquid called the
distillate.
To achieve even boiling and preventing too much bumping (frothing and bubbling) in the
flask, anti-bumping granules or boiling chips is added to the distilling flask containing the
mixture.

In some countries, distillation is used to obtain pure water.

The distillation apparatus can also be used to determine the boiling point of a liquid. This is done
by use of a thermometer. The thermometer is passed through one holed stopper (cork) of the
flask and the temperature of the vapour is noted. This will be the boiling point of the liquid.

NB: The impurities which contain dissolved salts remain in the flask and therefore pure water is
not good for drinking because it lacks mineral salts.

5.5.6.2. Separation of a mixture of water and ethanol by distillation

Water and ethanol are two miscible liquids and they have different boiling points, they are
therefore separated by fractional distillation

The boiling point of pure water is 100oC and that of ethanol is 78oC

The mixture of ethanol and water is poured into a boiling flask fitted with a fractionating
column. Glass beads are placed in the fractionating column. The use of the glass beads in the
fractionating column is to increase the surface area for effective separation of vapours

The mixture is then heated up to 78oC; temperature reading on the thermometer becomes steady
for some time. Ethanol vapourises and ethanol vapour is condensed by cold water flowing in the
condenser forming a liquid. The liquid which is received now is pure ethanol and is then
collected in the receiving flask.

Heating is continued up to 100o when water evapourates and is collected in a different container
The use of the thermometer is to record temperature of the two vapours

The use of porcelain pieces which are placed in the distilling flask is to enable the mixture boil
gently

5.5.6.3. Fractional distillation


- To separate two or more liquids with different boiling points into different fractions.

This is a process of separation of two or more liquids with different boiling points into different
fractions.

Fractional distillation is the process used to separate liquids which are miscible

Miscible liquids are those which mix freely in all proportions to form one uniform solution
The process can therefore be used to separate

Liquefied air into nitrogen and oxygen

Crude oil into petrol, diesel and kerosene

Crude oil is separated into its constituents such as petrol, diesel and kerosene
Fractional distillation of liquefied air
5.5.7. Sublimation

This is the process where when a solid is heated; it changes to a gas directly without passing
through the liquid state.
Separation of mixtures by sublimation

Sublimation can be used to separate a mixture of two substances where one sublimes and the
other does not. It can therefore be used to separate a mixture of iodine and common salt (sodium
chloride)

When a mixture of iodine and common salt (sodium chloride) is heated, iodine changes to a gas
and common salt remains in solid form. Therefore the two substances can be separated from one
another.

Other substances which sublime other than iodine include:

Ammonium chloride
Anhydrous aluminum chloride

Iron (III) chloride

Benzoic acid

5.5.8. Magnetic Attraction

- To separate magnetic substances from non-magnetic substances

In hospitals, magnets are often used to remove iron splinters from a patients eyes.

Electromagnets are also used for removing scrap steel and iron at the junkyard. These scrap
metal can then be sent for recycling.
Magnetic substances are substances which can be attracted by a magnet e.g. iron. Therefore a
magnet can be used to separate iron from sulphur because iron is attracted by a magnet and
sulphur cannot be attracted by a magnet and is therefore left behind.
5.5.9. Chromatography

- To separate liquid components in a mixture

This is a process of separating different coloured substances using a


porous paper. The coloured substances are moved over the paper at
different rates by a moving solvent.

The components of ink are separated by chromatography

Chromatography shows that ink consists of many coloured substances.

This can be done as follows

When a spot of ink is applied to the chromatography paper (usually filter paper), the dyes in the
ink are attracted to the surface of the paper. The chromatography paper is then immersed in
a solvent. The solvent level should not be above the ink spot.

As the solvent (usually water or ethanol) is soaked up by the paper, the solvent dissolves the
dyes.

A dye that is strongly attracted to the paper and not very soluble in the solvent will be left
behind. A dye that is weakly attracted to the paper and very soluble in the solvent will move up
with the solvent through the paper.
6. Elements, compounds, atoms and symbols

6.1. An element

An element is a substance which cannot be split up into two or more simpler particles by
chemical means. Examples of elements include

Copper

Sulphur

Carbon

Oxygen

Hydrogen

Iron

6.2. An atom

An atom is the smallest, indivisible particle of an element which can take part in a chemical
reaction.
6.3. A molecule

A molecule is the smallest indivisible particle of an element which can exist in free and separate
state. Molecules are formed when two or more atoms combine together e.g. water (H2O) is a
molecule made up of 2 atoms of hydrogen and 1 atom of oxygen

6.4. Radicals

A radical is a group of atoms which cannot exist on their own but exists in a compound.
Examples include:

Sulphate (SO4)

Carbonate (CO3)

Nitrate (NO3)

Sulphate radical cannot exist on its own but can exist in composition like in sulphuric acid
(H2SO4), calcium sulphate (CaSO4), sodium sulphate (Na2SO4)

6.5. Chemical symbols

A chemical symbol of an element is one or two letters which represent one atom of an element.
The letters used are the first letters of an element in English or Latin names of the element. The
first letter should be CAPITAL and the second letter should be small.

Common elements and their symbols include

Element Symbol
Hydrogen H

Helium He

Lithium Li

Beryllium Be

Boron B

Carbon C

Nitrogen N

Oxygen O

Fluorine F

Neon Ne

Sodium Na

Magnesium Mg

Aluminum Al

Silicon Si

Phosphorous P

Sulphur S

Chlorine Cl
Argon Ar

Potassium K

Calcium Ca

Scandium Sc

Titanium Ti

Vanadium V

Chromium Cr

Manganese Mn

Iron Fe

Cobalt Co

Nickel Ni

Copper Cu

Zinc Zn

Lead Pb

Mercury Hg

Silver Ag

Gold Au
Xenon Xe

Iodine I

Barium Ba

Elements whose symbols were derived from their Latin names are summarized below
Element Latin names Symbol
Potassium Kalium K
Sodium Natrium Na
Iron Ferrum Fe
Copper Cupium Cu
Lead Plumbium Pb
Mercury Hydrogyrum Hg
Silver Argentum Ag

6.6. Metals and non-metals

Elements are grouped into two;

Metals: Copper (Cu), Iron (Fe), Sodium (Na), Potassium (K), Mercury (Hg), Silver (Ag), Gold
(Au) etc

Non-metals: Chlorine (Cl), Carbon (C), Oxygen (O), Hydrogen (H), Helium (He), Nitrogen (N),
Fluorine (F), Bromine (Br) etc..

A metal: is an element which forms positive ions by losing electrons

An ion: is a charged particle e.g. Cu2+, Pb2+, Al3+, Zn2+

A non-metal: is an element which forms negative ions by gaining electrons e.g. Cl-, Br- etc
6.7. Properties of metals and non-metals
Metals Non-metals
Good conductors of heat and electricity Poor conductors of heat and electricity
Have high density Have low density
Solids with high melting points Most are gases. Solids have low melting points
Ductile i.e. can be drawn into wires Not ductile
Malleable i.e. can be made into sheets Not malleable
Strong and tough i.e. have high tensile strength Not strong i.e. have low tensile strength
Are rustrous i.e can be polished Not rustrous i.e. cannot be polished
7. Atomic structure and the periodic table

7.1. An atom

An atom is the smallest indivisible particle of an element which can take part in a chemical
reaction.

7.2. Composition of an atom

An atom is mainly composed of three particles namely:- electrons, protons and neutrons

Electrons

These are negatively charged particles of an atom.

They are found outside the nucleus of an atom.

They have a charge of negative one (-1).

NB: They are symbolized by e.

Protons

These are positively charged particles found in the nucleus of an atom.

Mass (1)

They have a charge of +1.


NB: They are represented by letter P.

Neutrons

These are neutral particles found in the nucleus of an atom.

Have no charge.

Mass (1)

NB: Neutrons are symbolized by N. Both neutrons and protons are found inside the nucleus and
are together called nucleons.

7.3. Structure of an atom

The central part of an atom is called the nucleus. Protons and neutrons are located in the nucleus.
These make up nucleon number.

Particle Symbol Relative mass Charge


Proton P +1 +1
Neutron N 0 0
Electron e- 1 -1
1836

Electrons move around nucleus in an orbit called energy level.

In a neutral atom the number of electrons is equal to the number of protons. The charge of an
electron is equal but opposite to that of a proton.

PROTON NUMBER is the number of protons in an atom.

NUCLEON NUMBER is the number of protons and neutrons in nucleus of an atom.


Therefore, to find the number of neutrons, we subtract proton number from nucleon number, i.e.:
Nucleon number Proton number = Neutrons

ELECTRONS have the same number as protons to balance the charges.

7.4. Atomic number

This is the number of protons in the nucleus of an atom. Since protons are equal to electrons in
an atom, atomic number can also be defined as the number of electrons found in an atom.

7.5. Atomic mass

This is the number of protons plus the number of neutrons in the nucleus of an atom. It is the
sum of protons and neutrons in an atom.

Atomic mass = protons + neutrons


Z
Consider the following element A X

Z is the atomic mass and A is the atomic number

The number of protons =number of electrons = b = atomic number

The number of neutrons = atomic mass atomic number i.e. Z-A

Example

16
1. An atom of an element is represented by the symbol 8 X . State

a. Its atomic number = 8

b. The mass number = 16

c. The number of neutrons = 8 (i.e. 16 -8 =8)

23
2. Work out the following, 11 Na . State

a. Its atomic number = 11

b. The mass number = 23

c. The number of neutrons = 12 (i.e. 23 - 11 = 12)

Examples of electronic structures of some elements:


16
Oxygen 8 O

40
Calcium 20 Ca

36
Argon 18 Ag

35
Chlorine 17 Cl

7.6. Electronic configuration

This is the arrangement of electrons in an atom.

In an atom, electrons occupy levels or shells and move in these shells around the nucleus. The
number of shells in an atom depends on the number of electrons present. The maximum number
of electrons occupying the innermost shell is 2. The rest of the shells contain a maximum of 8
electrons.

To write electronic configuration we write as n:n:n.... where first n denotes the first shell, second
the second shell and so and so for.
Examples

Draw the structure of the following atoms and state their electronic configuration.

1. Oxygen (O): atomic number = 8

2:6 (Oxygen)

2. Carbon (C): atomic number = 6


2:4 (Carbon)

3. Chlorine (Cl): atomic number =17

2:8:7 (Chlorine)

4. Sulphur (S): atomic number = 16

2:8:6 (Sulphur)
5.

40
a. Draw the electronic structure of element X represented by 20 X

b. State the number of neutrons and protons found inside the nucleus of X

6. Fill in the following table


Atom Atomic No. No. of electrons Electronic
configuration
Hydrogen (H) 1
Helium (He) 2
Lithium (Li) 3
Boron (B) 4
Nitrogen (N) 7
Neon (Ne) 10
Phosphorous (P) 15
Chlorine (Cl) 17
Potassium (K) 19

7.7. Electronic configuration of ions

An ion is an electrically charged particle. Ions are formed by gain or loss of electrons.

Metals form positively charged ions by losing electrons and non-metals form negatively charged
ions by gaining electrons.

The number of electrons gained or lost is equal to the valence of the atom e.g.

1. Sodium electronic configuration 2:8:1 loses 1 electron to form Na+. Na+ therefore has
electronic configuration of 2:8
2. Calcium electronic configuration 2:8:8:2 loses 2 electrons to form Ca2+. Ca2+ therefore
has electronic configuration of 2:8:8

3. Oxygen electronic configuration 2:6 gains 2 electrons to form O2-. O2- therefore has
electronic configuration of 2:8

4. Chlorine electronic configuration 2:8:7 gains 1 electron to form Cl-. Cl- therefore has
electronic configuration of 2:8:8

Exercise

The number of electrons, protons and neutrons in atoms W, X, Y and Z are shown in the table
below.
Atom Electrons Protons Neutrons
W 8 8 8
X 16 16 16
Y 13 13 14
Z A 3 4

a. Determine

i. The value of A

ii. The approximate atomic mass of Y

b. Write the electronic configuration of the following atoms and ions

i. W

ii. W2-
iii. Y

iv. Y3+

c. State the two atoms that are isotopes of the same element

d. Write the formula of the compound formed between X and Y

7.8. Isotopes

These are atoms of the same element with

Same atomic number

Different atomic masses due to difference in number of neutrons

Isotopes of an element have the same chemical properties.

E.g. Hydrogen has three isotopes


1 2 3
Isotopes H H H
Hydrogen 1 Deuterium 1 Tritium 1

Protons 1 1 1
Electrons 1 1 1
Neutrons 0 1 2
Mass No. 1 2 3

E.g. Chlorine has two isotopes


35 37
Isotopes Cl Cl
Chlorine - 35 17 Chlorine - 37 17

Protons 17 17
Electrons 17 17
Neutrons 18 20
Mass No. 35 37

Other examples include:

23 24
11 Na and 11 Na

24 25
12 Mg and 12 Mg

Calculating relative atomic masses

Examples

35 37
1. The relative abundances of 17 Cl and 17 Cl are 75% and 25% respectively by weight.

Calculate the relative atomic masses of chlorine

35 37
Relative atomic mass of chlorine = Mass due to 17 Cl + Mass due to 17 Cl

75 X 35 25 X 37
= 100 + 100

=35.5
Exercise
13 12 13
1. Carbon has two main isotopes C
6 and C .
6 C
6 has relative abundance of 1.11%

12
and C
6 has relative abundance 98.89%. Calculate the relative atomic mass of carbon.

23
2. Given the sodium atom, 11 Na

a. Give the examples of isotopes of sodium

b. Write down the electronic configuration of sodium

8. The periodic table

There over 103 elements so far discovered by scientists. The periodic table is a table in which all
the elements so far discovered are put. It is an arrangement of elements in order of their atomic
numbers.

There are 8 groups and 7 periods

Summary of the table

Horizontal rows are called periods

Vertical columns are called groups

Group
An element is put in a particular group depending on the number of electrons it has in its last or
outermost orbital e.g oxygen 2:6 has 6 electrons in its last orbital so its in group 6.

Sodium 2:8:1 has 1 electron in its outermost shell, so its put in group 1 etc.

Period

An element is put in a particular period depending on the number of orbitals it has e.g. oxygen 2:
6; Has 2 orbitals so it is period 2 and group 6.

Note

All elements in the same group have the same valency and similar chemical properties
Elements in group I III are metals

Elements in group V-VIII are non-metals

Elements in group IV are metalloids except Carbon which is a non-metal

Elements in group I are alkaline metals while elements in group II are called alkaline earth
metals.

Elements in group VII are halogens while elements in group VIII are Noble gases or inert
gases.

The number of electrons in the outermost shell corresponds to the group number.

For elements in groups I-IV the valency is given by the group number

For elements in groups V-VII the valency is given by 8-Group number.

Hydrogen is put in groups I and VII because it behaves as both a group I and group VII element

Helium is put in group VIII because it has a fully filled outermost orbital and behaves like a
group VIII element.
8.1. Chemical families

On the left of the periodic table there are metals, and to the right lie the nonmetals.

In the middle are metalloids and these exhibit both metallic and nonmetallic properties.

Metals are malleable, ductile, and have luster; most of the elements on the periodic table are
metals. They oxidize (rust and tarnish) readily and form positive ions (cations).

They are excellent conductors of both heat and electricity.

The metals can be broken down into several groups.

Transition metals (also called the transition elements) are known for their ability to refract light
as a result of their unpaired electrons.

They also have several possible oxidation states.


Ionic solutions of these metals are usually colored, so these metals are often used in pigments.

The actinides and lanthanides are collectively called the rare earth elements and are filling
the f orbitals.

They are rarely found in nature. Uranium is the last naturally occurring element; the rest are
man-made.

Non-metals do not conduct electricity well because they do not have free electrons.

All the elemental gases are included in the nonmetals. Notice that hydrogen is placed with the
metals because it has only one valence electron, but it is a nonmetal.

Specific families

Alkali metals (1A) Group one

They have one electron each in their outermost shell.

They are all metals which are highly electropositive i.e. they have a very high tendency to lose
electrons than other elements in the period. They form ions with a single charge by loss of one
electron e.g. Na+, Li+, K+.

They have a valency of one.


They are strong reducing agents.

They form compounds by either metallic or ionic bonding.

They are larger than any other elements in the period e.g. lithium (Li)

They have low densities, low melting points and low conductivities

They are soft and shinny and hence can be cut with a knife

The most reactive metal family, these must be stored under oil because they react violently with
water

They dissolve and create an alkaline, or basic, solution, hence their name

a) Lithium

Has atomic number 3 and electronic configuration 2:1

Has valency 1

Silvery white in colour

Extracted by electrolysis of lithium chloride

Its compounds are mainly deliquescent

Not very typical of group I elements, but resembles them


Harder than K and Na

Reactions

i. With water

Lithium reacts slowly with cold water to form its hydroxide and liberates hydrogen

Lithium + Water Lithium hydroxide + Hydrogen

2Li (s) + 2H2O(l) 2LiOH (aq) + H2 (aq)

ii. With air

Gives a red flame colour when burnt in air i.e. burns with a red flame

4Li (s) + O 2 (g) 2Li2O(s)

iii. With chlorine

Lithium reacts with chlorine to form lithium chloride

2Li (s) + Cl 2 (g) 2LiCl(s)

b) Sodium (Na)
Also a member of group 1

Has atomic number 11

Electronic configuration 2:8:1

Has atomic mass 23

Occurrence

Widely distributed in nature as sodium chloride NaCl, sodium nitrate NaNO3 or sodium
carbonate Na2 CO3 etc

Its a soft white metal

Reactions

i. With water

Sodium attacks cold water rapidly, evolving hydrogen and forming sodium hydroxide

Sodium + Water Lithium hydroxide + Hydrogen

2Na (s) + 2H2O(l) 2NaOH (aq) + H2 (aq)

ii. With air

Very little attack by dry air


Sodium + Oxygen Sodium oxide

4Na (s) + O 2 (g) 2Na2O(s)

The sodium oxide formed absorbs water i.e. deliquescent to form sodium hydroxide solution

Sodium oxide + Water Sodium hydroxide + Hydrogen

2Na2O (s) +H 2O (g) 2NaOH (aq) + H2 (g)

iii. With chlorine

Sodium combines directly with chlorine to form sodium chloride

2Na (s) + Cl 2 (g) 2NaCl(s)

Alkaline earth metals (Group 2A elements)

These are also reactive metals, but they dont explode in water; pastes of these are used in
batteries.

These are elements, which have two electrons each in their outermost orbital. Examples include
magnesium (Mg), calcium (Ca)
a. Magnesium (Mg)

Atomic number 12

Silvery white metal

Electronic configuration 2:8:2

Extracted electronically

Reactions

i. With air

Dry air does not attack magnesium

Damp air puts a layer of an oxide on magnesium later forming a hydroxide and carbonate

Burns in air with a bluish white flame leaving behind an oxide and nitride

2Mg(s) + O2 (g) 2MgO (s) (Magnesium oxide)

3Mg(s) + N2 (g) Mg3 N2 (s) (Magnesium Nitride)

ii. With water

Magnesium reacts with steam and not water


It burns brilliantly in steam producing an oxide and hydrogen

Mg(s) + H2O (g) MgO (s) + H2 (g)

iii. With dilute acids

All dilute mineral acids i.e. HCl, H2SO4, HNO3 react with magnesium liberating hydrogen

Mg(s) + H2SO4 (aq) MgSO4 (aq) + H2 (g)

Mg(s) + 2HNO3 (aq) Mg(NO3)2 (aq) + H2 (g)

Mg(s) + 2HCl (aq) MgCl2 (aq) + H2 (g)

NB: Hot concentrated sulphuric acid yields sulphur dioxide instead of hydrogen gas

iv. With halogens

When heated, magnesium reacts with halogens to produce salts called halides

Mg(s) + Cl2 (g) MgCl2 (s)

b. Calcium (Ca)

Widespread in the earths crust as CaCO3, CaSO4, CaF2, Ca3(PO4)2 etc

Extraction
Mainly by electrolysis of its fused calcium chloride

Properties of calcium

Silvery white in colour

Soft enough to cut with a knife

Reactions

i. With air

On exposure to air at room temperature, calcium tarnishes in color as it froms an oxide

Calcium + oxygen calcium oxide

2Ca(s) + O2 (g) 2CaO (s) (Calcium oxide)

Calcium oxide later combines with water to form calcium hydroxide

Calcium oxide + water calcium hydroxide

CaO (s) + H2O (aq) Ca(OH)2 (aq)

Calcium hydroxide formed combines with carbon dioxide to form calcium carbonate

Ca(OH)2 (aq) + CO2 (g) CaCO3 (s) + H2O (l)

Explanation
The calcium oxide (CaO) formed is deliquescent i.e. absorbs moisture from the atmosphere so it
forms calcium hydroxide Ca(OH)2 (aq). The calcium hydroxide Ca(OH)2 (aq) formed later
absorbs carbon dioxide (CO2), also from the atmosphere forming calcium carbonate (CaCO3)

ii. With water

Calcium reacts with water to form calcium hydroxide liberating hydrogen

Ca (s) + 2H2O (aq) Ca(OH)2 (aq) + H2 (g)

NB: The calcium hydroxide produced is sparingly soluble and tends to precipitate on the metal
and stops any further reaction from going on.

Halogens (7A)Known as the salt formers, they are used in modern lighting and always exist
as diatomic molecules in their elemental form.

They include fluorine (F2), chlorine (Cl2), bromine (Br2) and iodine (I2)

Have seven electrons in their outermost shell

Properties

Chlorine

Greenish yellow gas

Poisonous gas
Denser than air

Bleaches damp litmus paper

Atomic number 17

Electronic configuration 2:8:7

Atomic mass 35

Reactions

i. With water

Halogens react with water forming acids. Chlorine reacts with water forming two acids i.e.
hypochlorous acid (HOCl) and hydrochloric acid (HCl)

Cl2 (g) + H2O(l) HCl (aq) + HOCl (aq)

ii. With dilute sodium hydroxide

Halogens generally react with NaOH to produce a pale yellow solution of sodium chloride or
sodium bromide

NaOH(aq) + Cl2(g) NaCl(aq) + H2O(l)

iii. Displacement reactions


Written in order Cl, Br, I each halogen displaces those on its right from their solutions of simple
salts i.e. chlorine displaces bromine from its own solution and bromine forms a brown solution

2NaBr (aq) + Cl2(g) 2NaCl (aq) + Br2(aq)

iv. Oxidizing reactions

Halogens are oxidizing agents i.e. they accept electrons, therefore they are electronegative
elements

Noble gases (8A)Known for their extremely slow reactivity, these were once thought to never
react; neon, one of the noble gases, is used to make bright signs.
Sometimes referred to as Group O elements

Chemically inert i.e. they dont react

Their outermost orbitals are full

They include helium (He), neon (Ne), argon (Ar), Krypton (Kr), xenon (Xe)

i. Helium (He)

Has atomic number 2

Electronic configuration 2

Very stable because the only orbital is full

ii. Neon (Ne)

Has atomic number 10


Electronic configuration 2:8

Very stable because all the orbitals are full

iii. Argon (Ar)

Has atomic number 18

Electronic configuration 2:8:8

Very stable because all the three orbitals are full i.e. chemically satisfied

NB: their electron arrangement makes them very unreactive i.e. they do not lose or gain electrons
and this accounts for their low reactivity
8.2. Bonding

Elements always try to achieve the stable structure of the noble gases. In doing so, they combine
chemically forming bonds.

Types of bonds

They are prominently two types of bonds

The electrovalent bond and covalent bonds


Elements are made of atoms
Atom is smallest unit of an element, having properties of that element.
Molecule is group of two or more atoms chemically joined together, e.g. chlorine molecule has 2
chlorine atoms
Chemical formula shows the number and kinds of atoms in a molecule, e.g. chlorine molecule
has formula Cl2, where Cl is chlorine symbol and the subscript number (2) shows that there are 2
atoms in a chlorine gas molecule.
Compounds
Compound is substance containing two or more elements chemically combined together e.g.
Magnesium is an element; oxygen is an element they can only be burnt to form magnesium
oxide compound.
Composition of compounds
Ions or molecules make up compounds
Ions are atoms having electrical charge

E.g. NaCl made up of 2 ions; positively charged Na, negatively charged Cl.
8.3. Ionic/Electrovalent Bonding
Ionic bonding is the transfer of electrons from one atom to another to become/achieve an inert
gas configuration, forming ions.
Ionic bonds are formed between METALLIC and NON- METALLIC ATOMS ONLY.
- Metals lose electrons to form positive ions (cations)
- Non-metals gain electrons to form negative ions (anions)

The formation of ions is resulted from transfer of electrons from one atom to another atom(s).
The ions produced are of opposite charges and unlike charges attract, causing them to be held
together with a strong electrostatic force.
E.g. Formation of NaCl

Sodium atom loses an electron by transferring the electron to chlorine atom, making both stable.
The loss of electron forms a cation, Na+, and the gain of an electron forms anion, Cl-. The
opposite charges acquired by both ions attract each other, forming a strong ionic bond of NaCl.
E.g. Formation of MgF2
Sodium atom loses two electrons by transferring the electrons to fluorine atoms, one each,
making both stable. The loss of electron forms a cation, Mg2+, as it loses 2 electrons, and the gain
of electron forms anion, F-. The opposite charges acquired by both ions attract to each other,
forming a strong ionic bond of MgF2.
Deducing formula of ionic compounds
We can know the charge of elements by looking at groups of periodic table. Group I to group III
elements have a charge of +1, increasing to +3, going to the right. Group V to group VII
elements have a charge of -3, decreasing to -1, going to the right.
E.g. Aluminium sulfate
We have to balance the charges to make a stable bond
Ions present: Al3+ SO42-
SO42-
Al3+ SO42-
Total change: 6+ 6-

Therefore, the formula is Al2(SO4)3


1. The symbol of metal ion should always be first, e.g. NaCl
2. Polyatomic ion should be placed in brackets, e.g. Fe(NO3)2

Properties
1. Ionic compounds are hard crystalline solids with flat sides and regular shapes because the ions
are arranged in straight rows in strong ionic bonds.
2. Ionic compounds have very high melting points and boiling points.
3. The strong forces holding ionic compounds prevent them to evaporate easily. Hence, ionic
compounds have no smell.
4. Solid ionic compounds dont conduct electricity but they do when they are aqueous or molten.
This is because in liquid/aqueous state the ions which conduct electricity are free to move. In
solids, these ions are fixed in place.
5. Ionic compounds are soluble in water but insoluble in organic compounds. This is because the
ions attract water molecules which disrupts the crystal structure, causing them to separate and go
into solution. Vice versa is when in organic solvent.

8.4. Covalent Bonding


Covalent bonding is the sharing a pair of electrons to gain electronic configuration of an inert
gas, usually for molecules.
Covalent bonds occur between NON-METALLIC ATOMS ONLY.

In covalent bond, TRY TO SUBSTITUTE THE SHORT OF ELECTRONS OF


TWO/MORE ATOMS BETWEEN EACH OTHER TO FORM THE 2 OR 8 VALENCE
ELECTRONS. THE SHARED ELECTRONS APPEAR IN PAIRS!
E.g. H2 molecule

The hydrogen atom has one valency. To become stable, the hydrogen atom needs one more
electron, just like helium which has 2 valency electrons. When 2 hydrogen atoms join, they share
their electrons, on which, the share becomes 2 electrons, which is now a noble gas configuration,
being shared between these 2 atoms. Write the bond as H H single bond, which means they
share an electron pair (2 electrons).
E.g. Chlorine (Cl2) molecule
The chlorine (Cl) atom has 7 valency electrons and needs one electron, each, to form a noble gas
configuration between two Cl atoms. Hence they share an electron EACH to share 2 electrons
between the atoms. Hence, each Cl atom now has 8 valency electrons which is a noble gas
configuration.
E.g.O2 molecule

An O atom has 6 valency electrons and needs 2 electrons, each, to form a noble gas
configuration. Hence, EACH SHARE THE AMOUNT OF ELECTRONS EACH IS SHORT OF,
in this case 2 electrons, to form a stable molecule. The contribution hence now becomes 4
electrons and what left on each oxygen atom are 4 electrons. Combine each 4 electrons on
oxygen atom with the 4 electrons shared and hence get 8
E.g. H2O molecule

Apart from oxygen sharing between oxygen atoms, it can share electrons with other atoms.
Oxygen needs 2 electrons and when bonded with hydrogen, which need an atom each, they
combine to provide 2 electrons on both sides of oxygen bonded with hydrogen atoms. Each
hydrogen atom with oxygen atom form a single bond: O H.
E.g. CO2 molecule
Carbon needs 4, oxygen needs 2. Share two from oxygen part, WHICH HAS THE SMALLEST
NUMBER OF SHORT OF ELECTRONS, TO SHARE THE AMOUNT OF ELECTRONS
THAT EACH ATOM NEEDS, to form 4 shared atoms. Now oxygen is stable but carbon needs 2
more, which it can get from another oxygen atom. The atoms are now stable and since each bond
has 2 pairs of electrons, this is a double bond: C = O.

NB:
A pair of shared electrons between 2 atoms forms SINGLE BOND, X Y.
Two pairs of shared electrons between 2 atoms forms DOUBLE BOND, X = Y.

Three pairs of shared electrons between 2 atoms forms TRIPLE BOND, X Y.

8.5. Dative bonding

It is essentially another type of covalent bonding since it involves sharing of electrons. The
difference here is that the electrons to be shared are donated solely by one of the atoms in the
bond

This results from the existence of a lone pair of electrons, which are not directly concerned with
valency.

This kind of bonding exists in the ammonium molecule (NH4+). Coordinate bonding is
sometimes referred to as dative bonding.
8.6. Metallic Bonding
Metallic bonding is bonding within atoms of metals caused by attractive force between
positively charged metal ions and negatively charged free electrons. The atoms are packed
closely together in giant lattice structures.

Metallic bond formation


Each atom in metal gives up valence electrons to form positive ions. There are free electrons
moving between the spaces and positive metal ions are attracted to the sea of electrons which
hold the atoms together.

Structure and properties of metallic bonds


1. Metals can be bent (ductile) and can be stretched (malleable) because the layers of atoms in
metals slide over each other when force is applied but will not break due to attractive force
between electrons and metal ions.
2. Metals conduct electricity as it has free electrons which carry current.
3. Metals conduct heat as it has free electrons which gains energy when heated and moves faster
to collide with metal atoms, releasing heat in collisions.
4. Metals have high melting and boiling points because the bond between metals is very strong.
Hence very high heat energy needed to break the bonds.
9. Valency

A valency is the number of hydrogen atoms which combine with or displace one atom of an
element or one group of the radical e.g. one atom of oxygen combines with two atoms of
hydrogen to form water.

Therefore the valency of oxygen is 2. Hydrogen is regarded as the standard and its valency is 1.

A valency can also be defined as the number of electrons an element or radical must gain or lose
in order to attain a stable electronic configuration e.g. oxygen has electronic configuration 2:6,
therefore it needs two electrons to be a noble gas with electronic configuration of 2:8

9.1. Elements and radicals with their valencies

Element Symbol

Hydrogen 1

Helium 0

Lithium 1

Beryllium 2

Boron 3

Carbon 4

Nitrogen 3

Oxygen 2
Fluorine 1

Neon 0

Sodium 1

Magnesium 2

Aluminum 3

Silicon 4

Phosphorous 3 and 5

Sulphur 2

Chlorine 1

Argon 1

Potassium 0

Calcium 1

Chromium 3

Manganese 2

Iron 2 and 3

Cobalt 2

Nickel 2
Copper 2

Zinc 2

Lead 2

Mercury 2

Silver 1

Bromine 1

Iodine 1

9.2. Radicals

Radical Symbol Valency

Hydroxide OH 1

Bromide Br 1

Nitrate NO3 1

Chlorine Cl 1

Hydrogen carbonate HCO3 1

Hydrogen sulphate HSO4 1

Oxide O 2
Carbonate CO3 2

Sulphate SO4 2

Sulphite SO3 2

Phosphate PO4 3
10. Chemical formulae

This is a group of letters and numbers which represent the name of a compound. In writing
chemical formulae, we therefore use symbols of elements and their valences.

10.1. Writing chemical formulae

First write the symbols of the element or radical that makes up that compound e.g.

Sodium Chloride Calcium Sulphate Aluminium Oxide

Na Cl Ca SO4 Al O

Write the valences on top right side of the symbols of elements or radicals

Na1 Cl1 Ca2 SO42 Al3 O2

Rewrite the symbols again reversing the valences from top right side to the bottom right
side of symbols and radicals

Na1 Cl1 Ca2 SO42 Al3 O2

Giving
Na1 Cl1 Ca2 (SO4) 2 Al2 O3

Na1Cl1 Ca2 (SO4)2 Al2O3

Note: if the valency of any element is 1, it should not be written in the final formula. Also if
the two valencies are similar, they should not be written.

The valency of elements combined in a radical should be written outside the brackets and affects
all the elements enclosed when counting the number of atoms e.g. Al2(SO4)3

For valencies in a formula which are multiples should be cancelled to their lowest possible
values

Na1 Cl1 Ca2/2 (SO4) 2/2 Al2 O3

Formula: NaCl CaSO4 Al2O3

Examples

i. Sodium hydroxide

Sodium Hydroxide

Symbols Na OH

Valencies on top Na1 OH1

Reversing valencies Na1 OH1

Neglecting 1 in final formula: NaOH


ii. Magnesium chloride

Magnesium Chloride

Symbols Mg Cl

Valencies on top Mg2 Cl1

Reversing valencies Mg2 Cl1

Neglecting 1 in final formula: MgCl2

iii. Carbon dioxide

Carbon Oxide

Symbols C O

Valencies on top C4 O2

Reversing valencies C4 O2

Cancelling valencies to the lowest possible values from the final formula: CO2

iv. Ammonium sulphate

Ammonium Sulphate
Symbols NH4 SO4

Valencies on top NH41 SO42

Reversing valencies NH41 SO42

Neglecting 1 in the final formula: (NH4)2SO4

10.2. Calculating the number of atoms of elements in a compound

Examples:

Calculate the number of atoms of the elements contained in the following compounds

NaCl = one mole of sodium chloride contains

1 atom of sodium

1 atom of chlorine

CaSO4 = one mole of calcium sulphate contains

1 atom of calcium

1 atom of sulphur

4 atoms of oxygen

(NH4)2SO4 = one mole of ammonium sulphate contains


2 atom of nitrogen

8 atoms of hydrogen

I atom of sulphur

4 atoms of oxygen

5Al2O3 = 5 moles of aluminium oxide contains

10 atoms of aluminium

15 atoms of oxygen

3CuSO4.5H2O = 3 mole of copper sulphate. 5 water molecules contains

3 atoms of copper

3 atoms of sulphur

27 atoms of oxygen

30 atoms of hydrogen

NB: From the examples, the following should be noted

1. The numeral behind the formula represents the number of molecules and is multiplied
through each element in the whole formula to get total number of atoms of each element
2. The numeral in front of each element is multiplied only through that element

3. The numeral outside the brackets should be multiplied only through those elements inside
the brackets

Exercise

Write the chemical formulae of the following compounds and calculate the number of atoms of
each element

Chemical name Common name Formula

Calcium hydroxide (solid) Slaked lime Ca(OH)2

Calcium oxide Lime, quick lime CaO

Calcium hydroxide solution Lime water Ca(OH)2

Potassium hydroxide Caustic potash KOH

Sodium hydrogen carbonate Baking soda NaHCO3

Calcium carbonate Chalk, limestone or CaCO3


marble

Sodium hydroxide Caustic soda NaOH

Iron (III) oxide (hydrated) Iron rust Fe2O3.xH2O

Potassium nitrate Salt patre KNO3


Sodium carbonate hydrated Washing soda Na2CO3.10H2O
11. Chemical equations

Writing chemical equations

Steps

Write the formula for the reactants on the left hand side and that for the products on the right
hand side and check the valencies for the elements forming the formula to confirm if they are
right

i. Sulphuric acid reacts with sodium hydroxide to form sodium sulphate and water

H2SO4(aq) + 2NaOH(aq) Na2SO4(aq) + H2O(l)

ii. Hydrogen reacts with oxygen to form water

H2(g) + O2(aq) 2H2O(l)


NB: The following symbols represent

means to form + means reacts with (in reactants)

+ means and (in products)


States

The following are state symbols with their meanings

g= gas; l=liquid; aq=aqueous; s=solid

Write the states of matter for each element or compound in the equation:
H2SO4 (aq) + 2NaOH (aq) Na2SO4 (aq) + H2O (l)

11.1. Balanced chemical equations

A balanced equation is where the number of atoms on the left hand side is equal to the number of
atoms on the right hand side for each kind of element e.g. 2H2 (g) + O2 (g) H2O (l)

11.2. Balancing chemical equations

Write the formula for the reactants on the left hand side and that for products on the right
hand side. H2 (g) + O2 (g) H2O (l)

Check the valencies for the elements forming the compounds to confirm if formulae are
right

H2 (g) + O2 (g) H1 O2 (l)

=H2O

Count the number of atoms of each element on the right hand side and on the left hand
side to see if they balance

H2 (g) + O2 (g) H2O (l)

LHS RHS

H=2 H=2

O=1 O=1
If they dont balance, look for a number which can be multiplied in the formula above to
make all elements balance on both sides of the equation.

2H2 (g) + O2 (g) 2H2O (l)

LHS RHS

H=4 H=4

O=2 O=2

NB: Balanced

1. A numeral to balance the equation must be written behind the formula of the element and
affects the number of atoms of all elements in the formula e.g. 2H2O

2. Never fix the number in front of the element or formula to be balanced or in the middle of
the formula

3. Great care should be taken to ensure that all formulae are correctly written. Failure of the
equation to balance is an indicator that the equation may be wrong or some formulae are
wrongly written
4. Sometimes it is easier to balance an equation by using fractions and they multiplied by a
number to remove the fraction
Examples

Balance the following chemical equations

i. Na (s) + O2 (g) Na2O (s)

LHS RHS

Na=1 Na=2

O=2 O=1

Not balanced

4Na (s) + O2 (g) 2Na2O (s)

LHS RHS

Na=4 Na=4

O=2 O=1

Balanced

Or 2Na (g) + O2 (g) Na2O (l) multiplying by two through out to remove
the fraction
4Na (s) + O2 (g) 2Na2O (s)

ii. Fe (s) + O2 (g) Fe2O3 (s)

LHS RHS

Fe=1 Fe=2

O=2 O=3

Not balanced

iii. 4Fe (s) + 3O2 (g) 2Fe2O3 (s)

LHS LHS

Fe=4 Fe=4

O=6 O=6

Balanced

3
Or 2Fe (s) + 2 O2 (g) Fe2O3 (s)) multiplying by two through out to

remove the fraction

4Fe (s) + 3O2 (g) 2Fe2O3 (s)


iv. Zn (s) + HCl (aq) ZnCl2 (aq) + H2 (g)

LHS RHS

Zn =1 Zn =1

H=1 H=2

Cl =1 Cl =2

Not balanced

Zn (s) + 2HCl (aq) ZnCl2 (aq) + H2 (g)

LHS RHS

Zn =1 Zn =1

H=2 H=2

Cl =2 Cl =2

Balanced

Note:
All the above steps are not required in examinations, but they are important to make you
understand. After you have got the basics then it is advised to balance the equation using
your head and directly write the balanced equation which is always required

Some equations are already balanced and therefore do not need to be balanced by
fractions

More examples:

i. KClO3 (s) KCl (s) + O2(g)

2KClO3 (s) 2KCl (s) + 3O2(g)

ii. Mg (s) + HCl(aq) MgCl2(aq) + H2(g)

Mg (s) + 2HCl(aq) MgCl2(aq) + H2(g)

iii. CaCO3 (s) CaO (s) + CO2(g)

CaCO3 (s) CaO (s) + CO2(g) Directly balanced

iv. NaOH (aq) + HCl(aq) NaCl (s) + H2O(l)

NaOH (aq) + HCl(aq) NaCl (s) + H2O(l) Directly balanced

Exercise

Balance the following equations


a) C(s) + O2(g) CO2 (g)

b) N2(g) + H2(g) NH3 (g)

c) 4Na (s) + O2 (g) 2Na2O (s)

d) NaNO3 (s) NaNO2 (s) + O2 (g)

e) Pb(NO3)2(s) PbO (s) + NO2 (g) +O2 (g)


12. Types of chemical reactions

There are four types of chemical reactions

12.1. Direct combination or direct synthesis

When two or more elements or compounds combine directly to form a more complex compound,
the process is called direct combination.

This reaction has a general formula of A +B AB. Examples include

a. C(s) + O2(g) CO2 (g)

b. N2(g) + 3H2(g) 2 NH3 (g)

c. 4Na (s) + O2 (g) 2Na2O (s)

d. H2(g) + Cl2(g) 2 HCl (g)

e. Ca(s) + Cl2(g) CaCl2 (s)

f. 2K(s) + Cl2(g) 2KCl (s)


Simple decomposition

When a compound is broken down into two or more simpler substances, the type of reaction is
called decomposition e.g. during heating. It has a general formula of AB A + B.

Examples

CaCO3 (s) CaO (s) + CO2(g)

2KClO3 (s) 2KCl (s) + 3O2 (g)

2NaNO3 (s) 2NaNO2 (s) + O2 (g)

2H2O2 (l) 2H2O (l) + O2 (g)

12.2. Simple replacement

When an element in a free state replaces another element in a compound, the reaction is called
simple replacement.

It has a general formula of AB + CD AC + B

E.g. Fe (s) + CuCl2 (aq) FeCl2 (aq) + Cu (s)


12.3. Double replacement

When an element in a compound replaces another element in another compound, this type of
reaction is called double displacement.

It has a general formula of AB + CD AD + BC

Examples

ZnSO4(s) + Na2CO3 (s) ZnCO3(s) + Na2SO4 (s)

2KOH (aq) + H2SO4 (aq) K2SO4 (aq) + 2H2O (l)

NaOH (aq) + HCl (aq) NaCl (aq) + H2O (l)

NH4OH (aq) + HCl (aq) NH4Cl (aq) + H2O (l)

12
13. The atmosphere and combustion

The composition of the atmosphere

13.1. Air is a mixture of gases

The average composition of dry air in the atmosphere by volume is:-

Oxygen -21%

Nitrogen- 78%

Carbon dioxide - 0.03%

Noble/inert/rare gases -1%

13.1.1. Oxygen

It occupies 21% of the atmospheric air

It is added to the atmosphere during the process of

photosynthesis

thermal decomposition of nitrates

it is removed from the atmosphere during respiration, rusting and combustion


13.1.2. Uses of oxygen

It is used for respiration in living organisms

It supports burning (combustion) and rusting of metals

It is used as fuel; liquid oxygen is used to burn fuel in some air rockets

It is used in steel production

Oxyacetylene flame which is produced when acetylene burns in oxygen is used for cutting
metals

13.1.3. Nitrogen

This gas occupies the largest volume of 78%

It is used to dilute air in the atmosphere so that burning and rusting do not take place so fast.
Without nitrogen in the air, burning and rusting would be very fast.

13.1.4. Carbon dioxide

This occupies a volume of 0.03% in the atmosphere. Carbon dioxide comes as a result
respiration and burning fossil fuels e.g. petrol, diesel etc...

It is removed from the atmosphere by green plants during the process of photosynthesis.

CO2(g) + H2O (l) C6 H12O6(aq) + O2 (g)


13.1.5. Noble gases

These include Helium (He), Neon (Ne), Argon (Ar), Krypton (Kr), Xenon (Xe), Radon (Rn)

They do not react with any substance under ordinary conditions because they are inert and have
full outermost shells.

13.1.6. Uses of noble gases

Argon is used in some electric bulbs to stop the hot filament from darkening the glass of the bulb

Neon is used in coloured lights used for advertising

Helium is a very light gas used for filling balloons

13.1.7. Water vapour

In addition to the gases, the atmosphere also contains water vapour. This water vapour comes as
a result of evapouration from the oceans, seas, rivers etc The percentage composition of
vapour varies from 1% to 4% by volume.

13.2. Hygroscopic, deliquescent and efflorescent substances

13.2.1. Hygroscopic substances

Are substances which absorb water from the atmosphere e.g. sodium chloride, calcium chloride,
calcium oxide, concentrated sulphuric acid and anhydrous copper (II) sulphate
Once these substances are left exposed to the atmosphere, they absorb water from the
atmosphere.

13.2.2. Deliquescent substances

These are substances which absorb water from the atmosphere forming solution. Examples
include calcium chloride, sodium hydroxide, phosphorous oxide, iron (III) chloride and sodium
nitrate.

Note:

All deliquescent substances are hygroscopic but not all hygroscopic substances are deliquescent

Deliquescent substances are hygroscopic substances which absorb water from the atmosphere
forming a solution

13.2.3. Efflorescent substances

These are substances which lose water of crystallisation to the atmosphere

Efflorescence: is the loss or giving up of water of crystallisation to the atmosphere

Examples include:

Sodium carbonate -10-water( Na2CO3. 10H2O)

Sodium sulphate -10-water (Na2SO4. 10H2O)


13.2.4. Water of crystallisation

This is a definite amount of water which some substances chemically combine with, when they
form crystals from their solutions in water.

13.3. Drying agents

They are used to dry gases because they have a high affinity for water and therefore can absorb
water from moist gases
Drying agent Gases dried
Calcium oxide Ammonia
Concentrated sulphuric acid All gases except ammonia
Anhydrous calcium chloride All gases except ammonia
Phosphorous (V) oxide All gases except ammonia

13.4. Burning substances in air

The part of air used for burning is oxygen

Metals burn in air to form basic oxides which when dissolved in water form alkaline solutions

Sodium

This burns with a bright yellow flame forming a yellow solid of sodium peroxide and a little
sodium oxide

2Na(s) + O2 (g) Na2O2 (s) (sodium peroxide)

4Na(s) + O2 (g) 2Na2O (s) (sodium oxide)


When sodium oxide is dissolved in water, sodium hydroxide is formed

Na2O (s) + H2O (l) 2NaOH (aq) (sodium hydroxide)

When sodium peroxide is dissolved in water effervescence/bubbles of a colourless gas (oxygen)


are observed and a colourless solution (sodium hydroxide solution) are observed

2Na2O2(s) + 2H2O (l) 4NaOH (aq) + O2 (g)

Magnesium

This burns with a brilliant flame forming a white smoke and ash i.e. an oxide and a nitride

2Mg(s) + O2 (g) 2MgO (s) (Magnesium oxide)

3Mg(s) + N2 (g) Mg3 N2 (s) (Magnesium Nitride)

Aluminium

This burns in air when heated strongly and becomes very hot

It forms aluminium oxide and a little nitride

4Al(s) +3 O2 (g) 2 Al2 O3 (s) (Aluminium oxide)

2Al(s) + N2 (g) 2AlN (s) (Aluminium nitride)

Zinc
Zinc burns in air with a green flame to form zinc oxide. It does not form a nitride.

2Zn(s) + O2 (g) 2ZnO (s) (Zinc oxide)

Lead

This melts on heating to shiny beads and then forms lead oxide which is brown when hot and
turns yellow on cooling

2Pb(s) + O2 (g) 2PbO (s) (Lead II oxide)

Brown when hot

Yellow on cooling

Copper

This burns with a green flame to form a black oxide

2Cu(s) + O2 (g) 2CuO (s) (copper II oxide)

Non metals

Carbon

This burns in a plentiful supply of air to form Carbon dioxide


C(s) + O2 (g) CO2 (g)

If oxygen is in short supply, carbon burns with little oxygen to form carbon monoxide

2C(s) + O2 (g) 2CO (g)

Phosphorous

This burns with a bright yellow flame and produces dense white fumes of phosphorous pentoxide

P4 (s) + 5O2 (g) P2 O5 (s)

Or 4P (s) + 5O2 (g) P2 O5 (s)

Sulphur

This burns in air with a bright blue flame forming cloudy fumes of sulphur dioxide

S(s) + O2 (g) SO2 (g)

An experiment to determine the percentage of oxygen in the air

Put wet iron wool in a marked test tube and invert the test tube in a beaker of water. Note and
mark the length of the air column X
Leave the experiment to stand for a week. After a week, water will rise to a certain height in the
test tube. Note the new height of the air column Y.
The length of the air column used in the rusting of iron wool is X-Y.

This is equal to the volume of oxygen used up because oxygen is used up in rusting.

volume of oxygenused up
X 100
The percentage of oxygen used up = volume of total air

XY
= X X100

From the experiment, if the value of X is 10 and that of Y is 7.9, the percentage of oxygen used
up

107.9
= 10 X100

2.1
= 10 X100

=21%

Therefore the volume of oxygen used in rusting is 21% which is the composition of oxygen in
the atmosphere.

1.8. Burning candle

Burning is a chemical reaction in which a substance chemically combines with oxygen and
usually heat is produced.
When a candle burns, Carbon dioxide and water are produced.

An experiment to show that Carbon dioxide and water are produced when a candle burns

Arrange the apparatus as shown below

Pass the gases from the burning candle through a U-tube placed in cold water

Also pass the gases through a test tube containing lime water

Wait for a few seconds

After a short time, drops of a colourless liquid collect at the bottom of the U-tube which turn
anhydrous copper (II) sulphate from white to blue
The lime water in the test tube soon turns milky

The colourless liquid is water since it turns anhydrous copper (II) sulphate from white to blue

Lime water in the test tube turns milky due to the presence of Carbon dioxide produced.

Ca(OH)2 (aq) + CO2 (g) CaCO3 (s) + H2O(l)

Lime water

Therefore when a candle burns, Carbon dioxide and water are produced

13.5. Rusting

Rusting is a chemical change where by iron combines with oxygen under moist conditions to
form a brown substance called iron rust.

Rust is a brown coat formed when iron is left in damp air. Rust is chemically called hydrated iron
(III) oxide (Fe2O3.xH2O)

4Fe(s) +3O2 (g) 2Fe2O3 (s)

Fe2O3 (s) + xH2O (l) Fe2O3.xH2O (s)


Conditions necessary for rusting

There are two conditions necessary for rusting to take place. These are:

Oxygen
Water

For rusting to take place both of these must be present.


An experiment to show that iron rusts when left exposed to damp air

Place iron nails in the test tube and close the test tube with wet cotton wool. Leave the
experiment to stand for a week.

After a week, the iron nails were found to have turned brown i.e. had rusted.

An iron nail rusts in presence of water and air/damp air.

An experiment to show that water is necessary for rusting


Clean iron nails or iron fillings are placed at the bottom of a test tube. Cotton wool is pushed half
way the tube and anhydrous calcium chloride placed on top of the cotton wool, to absorb all the
moisture from air, leaving it dry.

Close the mouth of the test tube with another cotton wool. This reduces on the amount of damp
air reaching the calcium chloride such that it does not become damp so quickly.

Leave the experiment to stand for several weeks.


The iron nails do not rust because water is absent or because the air is dry. This proves that for
rusting to take place, the air must be damp.

An experiment to show that air is necessary for rusting

Water is first boiled for a few minutes to drive off any air in it. The iron nails are then placed at
the bottom of the test tube. Oil or grease is added on top of water to form a layer on its surface.
The oil layer prevents any entry of air.

At the end of the week the iron nails did not rust due to absence of air. Therefore iron nails
cannot rust in water if it does not contain air.

An experiment to investigate the conditions needed for rusting

Set up the experiment as shown below

Leave the experiment to stand for some time


Observe what happens to the nails in each test tube and record the results

Results

1. Rusting took place

2. Rusting did not take place


3. Rusting did not take place

4. Rusting did not take place

Explanation

A- For A all conditions for rusting are present

B- For B oxygen gas is absent

C- For C both water and oxygen gas are absent

D- For D water is is absent

Therefore rusting takes place in presence of water and oxygen gas.

Prevention of rusting

Iron can be prevented from rusting when either air or water is kept away from it. The methods of
preventing rusting include:
- Surface protection
- Sacrificial protection
- Use of stainless steel

Surface Protection covers metal with a layer of substance


1) Paint
2) Grease or oil (also help to lubricate)
3) Plastic
4) Metal Plating covering metal with thin layer of another metal (e.g. tin, chromium, silver)

Advantage These methods are cheap (except metal plating)

Disadvantage If the layer is broken, air and water can reach metal to rust
Sacrificial Protection
Is to sacrifice more reactive metal to corrode with water and air by layering it over less reactive
metal e.g. iron covered by magnesium.
If the layer is broken, water & air reach underneath layer, overlying metal still protect it.
Applications:
1) Galvanised Iron is steel coated with zinc, usually used on roofs.
2) Protecting ships blocks of zinc are attached to hulls to corrode instead of steel which is the
ship metal.
3) Underground steel pipes these are attached to magnesium block using insulated copper
cables. Magnesium corrodes first than steel.

13.6. Combustion
Combustion is the burning of substances in air. It is divided into
Complete and incomplete combustion
Complete combustion occurs when a substance is completely burnt in excess oxygen
C(s) + O2 (g) CO2(g)

Incomplete combustion occurs when a substance burns partially in a limited amount of oxygen
e.g.
2C(s) + O2 (g) 2CO(g)
14. Methods of gas collection

The method of gas collection depends on the properties of the gas.

It depends on whether the gas is;

Soluble or insoluble in water

Lighter or denser than air

There are three methods of gas collection:


a. Over water method/downward displacement of water
The method is used to collect gases which are insoluble or slightly soluble in water e.g. carbon
monoxide , oxygen, nitrogen and hydrogen.
b. Downward delivery or upward displacement of air:

This method is used to collect gases which are insoluble in water and denser than air e.g. sulphur
dioxide and hydrogen chloride

c. Upward delivery or downward displacement of air:

This method is used to collect gases which are soluble in water and less dense than air or lighter
than air e.g. ammonia
d. Gas syringe

This is used to collect all gases


15. Oxygen
Laboratory preparation of oxygen

a. Preparation of oxygen by heating a mixture of potassium chlorate and manganese


(IV) oxide

Potassium chlorate is heated in presence of manganese (IV) oxide and oxygen gas is produced. It
is collected in the gas jar over water.

2KClO3 (s) 2KCl (s) + 3O2 (g)

Manganese (IV) oxide acts as a catalyst.

A catalyst: is a substance which alters the rate of a chemical reaction and remains chemically
unchanged at the end of the reaction.
b. Preparation of oxygen by decomposition of hydrogen peroxide in the presence of
manganese (IV) oxide

Hydrogen peroxide is added to manganese (IV) oxide in a round bottomed flask. Effervescence
of a colourless gas occurs and oxygen gas is produced according to the equation

2H2O2 (l) 2H2O (l) + O2 (g)

The gas is then collected over water.

No heat is applied.

Manganese (IV) oxide acts as a catalyst.

Test for oxygen

It is a colorless gas which relights a glowing splint

Other substances from which oxygen can be prepared may include


Thermal decomposition of nitrates

Balance of oxygen in air

The balance is maintained by plants and animals

Plants add more oxygen to the atmosphere through photosynthesis

Animals use up oxygen from the air through respiration

Industrial scale preparation of oxygen

By fractional distillation of liquefied air since nitrogen has a lower boiling point of 90K remains
as a liquid

Properties of oxygen

Categorized into physical and chemical properties


Physical

It is colourless with no smell i.e. odorless

It is gaseous at room temperature

Slightly soluble in water, dissolved oxygen in water supports aquatic life

It is slightly denser than air

It is neutral to litmus paper


Has a boiling point of -183oC

It relight a glowing wooden splint


Chemical properties

It reacts with metals to form basic oxides

2Mg(s) + O2 (g) 2MgO (s) (Magnesium oxide)

4Na(s) + O2 (g) 2Na2O (s) (sodium oxide)

It reacts with non metals to acidic oxides

S(s) + O2 (g) SO2 (g)

P4 (s) + 5O2 (g) P2O5 (s)

C(s) + O2 (g) CO2 (g)


Uses of oxygen

- As rocket fuel
- In steel making, to burn off impurities
- In oxy-acetylene cutting and welding
- In oxygen tanks for deep sea divers and mountain climbers to provide oxygen
- For respiration for most animals
- Used as oxygen tents in hospital to aid patients with respiratory problems
15.1. Oxides

An oxide is a compound formed when an element combines with oxygen e.g. magnesium oxide
(MgO), Calcium oxide (CaO) etc
Types of oxides

a. Basic oxides

These are oxides of metals that react with water to form alkalis. Examples of basic oxides
include calcium oxide (CaO), zinc oxide (ZnO), potassium oxide (K2O), sodium oxide (Na2O)
etc

These basic oxides react with water to form corresponding alkalis e.g.

Na2O (s) + H2O (l) 2NaOH (aq) (Sodium hydroxide)

K2O (s) + H2O (l) 2KOH (aq) (Potassium hydroxide)


b. Acidic oxides/Acid anhydrides

There are oxides of non-metals which react with water to produce acids.

Examples of acidic oxides include Carbon dioxide (CO2), sulphur dioxide (SO2), sulphur
trioxide (SO3), nitrogen dioxide (NO2) etc

Examples of acids that can be produced from the reactions of these oxides with water are H2CO3,
H2SO3, H2SO4, HNO3 etc

CO2 (g) + H2O (l) H2CO3 (aq)

SO2 (g) + H2O (l) H2SO3 (aq)


SO3 (g) + H2O (l) H2SO4 (aq)

2NO2 (g) + H2O (l) 2HNO3 (aq)

Note:

Acidic oxides are also called acid anhydrides.

An acid anhydride is an oxide of a non-metal which reacts with water to form an acid.
c. Amphoteric oxides

These are oxides of metals which show both acidic and basic properties e.g. ZnO, Al2O3 and PbO
(Lead oxide)
d. Neutral oxides

These are oxides which show neither basic nor acidic characters. They are usually the lower
oxides of non-metals e.g. water (H2O), dinitrogen oxide (N2O), carbon monoxide (CO), nitrogen
monoxide (NO) etc

Mixed oxides

These are oxides which react like a mixture of two or more simpler oxides e.g trilead tetraoxide
(Pb3O4), triiron tetraoxide (Fe3O4) and dinitrogen tetraoxide (N2O4)

Peroxides

These are oxides which produce twice as much oxygen as would be expected from the usual
valency of the element in the oxide e.g. sodium peroxide (Na2O2), hydrogen peroxide (H2O2)
etc
16. Oxidation and reduction

Oxidation

Oxidation is the addition of oxygen to a substance

Oxidation is the removal of hydrogen from a substance

Oxidation is the loss of electrons by a substance

Examples of oxidation reactions include;

C(s) + O2 (g) CO2 (g)

4Na(s) + O2 (g) 2Na2O (s) (sodium oxide)

2Mg(s) + O2 (g) 2MgO (s)

H2S (s) + Cl2 (g) HCl(g) + S(s)

Na(s) Na+ (g) + e

Reduction

Reduction is the removal of oxygen from a substance

Reduction is the addition of hydrogen to a substance


Reduction is the gain of electrons by a substance

Examples of reduction reactions include:

H2 (g) + Cl2 (g) 2HCl(g)

CuO (s) + H2 (g) Cu(s) +H2O(l)

PbO (s) + H2 (g) Pb(s) +H2O(l)

Oxidizing agents

An oxidizing agent is a substance which adds oxygen to another substance

Or

It is a substance which removes hydrogen from another substance

Examples of oxidizing agents include: oxygen (O2), concentrated sulphuric acid (H2SO4),
potassium permanganate (KMnO4) manganese (IV) oxide (MnO2), nitric acid (HNO3), potassium
dichromate VII (K2CrO4), chlorine (Cl2) etc
Reducing agents

A reducing agent is a substance which adds hydrogen to another substance.

Or it is a substance which removes oxygen from another substance.

Examples of reducing agents include Hydrogen (H2), carbon (C), carbon monoxide (CO),
hydrogen sulphide (H2S), sulphur dioxide (SO2) and ammonia (NH3).
Redox reactions

A redox reaction is a reaction in which reduction and oxidation occur at the same time.
Therefore in such a reaction, one substance is reduced and another one oxidized.
17. Water and hydrogen
Water

Water is a compound made up of hydrogen and oxygen in the ratio of 2:1

17.1. Sources of water

Rain water: this is fairly purely than other kinds of water. It usually contains dissolved
gases like Carbon dioxide, oxygen and dust particles.

Well or spring water: this is good for drinking because it passes through soil and rock and
therefore some filtration has occurred. It also contains some dissolved gases.

River water: This contains a little dissolved matter e.g. salt and plenty of dirt and mud

Sea/lake water: This contains a lot of salt and therefore has a salty taste. Due to the
presence of salt in seas and lakes, such water is not good for drinking.

Underground water e.g. wells, bore holes etc

Surface water e.g. ponds, swamps etc


Synthesis of water

Experiment to show synthesis of water


17.2. Properties of pure water

It is a colourless liquid

It has a flat taste

It is neutral to litmus

It is freezes at 0oC and boils at 100oC and pressure of 1 atmosphere

It has a maximum density of 1g/cm3 at 4oC

17.3. Test for water

It turns anhydrous copper (II) sulphate from white powder to blue crystals

It turns anhydrous cobalt (II) chloride from blue to pink


17.4. Purification of water

Impure water is purified by using the process of distillation. Impure water is placed in a
distillation flask and heated to 100oC. The steam comes out while solid impurities remain in the
flask. The steam is cooled and condensed by cold water in the condenser.

Examples of pollutants of water

Sewage Fertilizers
Acidic gases
Oil
Hot water
Detergents
Insecticides
17.5. Reactions of metals with water
Potassium
Sodium React violently with cold water
Calcium

Magnesium
Aluminum React with steam
Zinc
Iron
Lead
Copper Do not react with cold water or steam
Mercury
Silver
Gold

Potassium

This is a bright silvery metal. It reacts explosively with cold water to produce a colourless
solution of potassium hydroxide and a colourless gas of hydrogen
2K(s) +2 H2O(l) 2KOH(aq) +H2(g)
Sodium

This reacts vigorously with cold water to produce a colourless


solution of sodium hydroxide and a colourless gas of hydrogen
2Na(s) +2 H2O(l) 2NaOH(aq) +H2(g)
During the reaction of sodium with water, a hissing sound is
produced. However, sodium is less reactive than potassium

Calcium

This reacts quietly with cold water producing calcium hydroxide and hydrogen
Ca(s) +2 H2O(l) Ca(OH)2 (aq) +H2(g)

Magnesium

This metal reacts with cold water very slowly but when magnesium is hot, it catches fire with
steam and burns with a bright light flame to produce a white ash of magnesium oxide and
hydrogen
Mg(s) +H2O(g) MgO (s) +H2(g)
In cold water, it slowly forms magnesium hydroxide and hydrogen
Mg(s) +2 H2O(l) Mg(OH)2 (aq) +H2(g)
Reaction of magnesium with steam

Clean about 6cm of magnesium ribbon with sand paper. Place wet sand in a small test tube. Heat
is applied to both ends of the wet sand and magnesium ribbon.

The wet sand will produce steam on heating which will react with magnesium ribbon to form a
white ash of magnesium oxide and hydrogen gas is produced
Mg(s) +H2O(g) MgO (s) +H2(g)
Iron

This reacts slowly with steam and produces a black triiron tetraoxide (Fe3O4) and hydrogen gas
3Fe(s) +4H2O(g) Fe3O4 (s) +4H2(g)

17.6. Uses of water


Acts as a solvent
Raw material in the manufacture of ammonia and ethanol
Used in the production of steam to drive turbines
For heating and cooling in industries

17.7. Reactivity Series


This is a list of metals in order of their reactivity with air, water or dilute acids
The most reactive being at the top and the least reactive at the bottom
Potassium
Sodium
Calcium
Magnesium
Aluminum
Zinc
Iron
Lead
Copper
Mercury
Silver
Gold

1) Reaction of Metals with Water


Potassium, Sodium, and Calcium reacts with cold water to form:
2M(s) + 2H2O(l) 2MOH(aq) + H2(g)
Metal + Water Metal Hydroxide + Hydrogen
Magnesium, Zinc, Iron reacts with steam to form:
M(s) + H2O(g) MO(s) + H2(g)
Metal + Water Metal Oxide + Hydrogen
Iron does not react with water
Copper and gold have no reaction with water and steam

2) Reaction of Metals with Dilute Hydrochloric Acid


Potassium, sodium, calcium, magnesium, zinc and iron reacts with dilute hydrochloric acid to
form:
M(s) + 2HCl(aq) MCl2(aq) + H2(g)
Metal + Acid Metal Chloride + Hydrogen
Lead reacts with warm hydrochloric acid slowly
Copper and gold have no reaction with dilute hydrochloric acid

3) Displacement Reactions
Displacement reaction is the displacement of ions of metal from compounds of metals lower in
reactivity series by metals higher in reactivity series.

E.g. Magnesium displaces copper(II) chloride


Mg(s) + CuCl2(aq) MgCl2(aq) + Cu(s)
For observation, silver magnesium metal coated with brown copper metal
Displacement is due to magnesium atoms transfer electrons to copper (II) ions forming copper
atoms.
Mg(s) Mg2+(aq) + 2e-
Cu2+(aq) + 2e- Cu(s)
Loss of electrons is due to its being less reactive, as the less reactive metal has higher chances of
losing electrons.
Thats why when magnesium is placed in potassium chloride solution, no reaction occurs

i. E.g. Displacement from metal oxides


Metals higher in the reactivity series displace oxides of metals lower in reactivity series.
When calcium burns with silver oxide, Calcium displaces silver to produce calcium oxide and
silver metal.
Ca(s) + Ag2O(s) CaO(s) + 2Ag(s)
This is called thermit reaction large amount of heat is produced.

4) Reaction of Metal Oxides with Carbon


The lower the position of a metal in reactivity series, the easier for carbon to remove oxygen
from the metal oxide by heating. At higher position, stronger heat is needed.
E.g. Copper (II) oxide reacts with carbon and can be reduced by bunsen burner flame
temperature
CuO(s) + C(s) Cu(s) + CO(g)
For iron (III) oxide to be reduced, it needs very high temperature.
5) Reaction of Metal Oxides with Hydrogen
The lower the position of metal in reactivity series, the easier hydrogen removes oxygen from the
metal oxide by heating. At higher position, stronger heat is needed.
E.g. Lead (II) oxide is reduced by hydrogen at Bunsen burner flame temperature
PbO(s) + H2(g) Pb(s) + H2O(l)

6) Decomposition of Metal Carbonates


The lower the position of a metal in the reactivity series, the easier it decomposes by heating. At
a higher position, stronger heat is needed.
E.g. Copper (II) carbonate is decomposed by heat at bunsen burner flame temperature
CuCO3(s) CuO(s) + CO2(g)
18. Hydrogen
Laboratory preparation of dry hydrogen gas

Hydrogen is prepared by the action of dilute acids on metals e.g. action of dilute sulphuric acid
or hydrochloric acid on zinc, magnesium or iron in the presence of a little copper (II) sulphate to
speed up the reaction.
Set up:

Procedure

Dilute hydrochloric acid or dilute sulphuric acid is added through a thistle funnel to zinc granules
in a conical flask
The hydrogen produced is passed through anhydrous calcium chloride in a U-tube or
concentrated sulphuric acid in a wash bottle for drying.
The dry hydrogen gas is then collected by upward delivery because it is lighter than air.
If the reaction is too slow, copper (II) sulphate solution is added to the flask to speed up the
reaction hence acting as a catalyst.
Equation

Zn(s) + 2HCl(aq) ZnCl2 (aq) + H2(g)


If sulphuric acid is instead used, the equation of reaction becomes
Zn(s) + H2SO4 (aq) Zn SO4 (aq) + H2(g)
When anhydrous calcium chloride is used as a drying agent, the diagram becomes;
NB: Dilute nitric acid is not used in preparation of hydrogen because it is an oxidizing agent and
would therefore oxidize hydrogen produced to water.

Preparation of hydrogen by action of water on metals

Metals like sodium, potassium and calcium react with water to produce hydrogen.
Mg(s) +2H2O(l) MgO (aq) +H2(g)
Ca(s) +2 H2O(l) Ca(OH)2 (aq) +H2(g)
2K(s) +2 H2O(l) 2KOH(aq) +H2(g)
2Na(s) +2 H2O(l) 2NaOH(aq) +H2(g)
Test for hydrogen gas

It is tested by using a burning splint. When a burning splint is placed in a gas jar full of hydrogen
gas, it burns with a pop sound.

Properties of hydrogen

Physical properties

It is a colourless, ordourless gas


It is neutral to litmus paper i.e. has no effect on litmus paper
It is the lightest gas (much less dense than air) and diffuses fast
It is not very soluble in water
It is the first element in the periodic table
Has an atomic number of one
Exists as a diatomic element
Chemical properties

It does not support burning but burns readily in air with a faint blue flame forming water
2H2(g) + O2(g) 2H2O(l)
It reacts with non-metals e.g.
Hydrogen reacts with chlorine in presence of sun light to produce hydrogen chloride
H2(g) + Cl2(g) 2HCl(g)
It also combines with nitrogen to produce ammonia gas
3H2(g) + N2(g) 2NH3(g)
Hydrogen is a reducing agent i.e. it reduces oxides of copper, lead and iron to their metals
CuO(s) + H2(g) Cu(s) + H2O(l)
PbO(s) + H2(g) Pb(s) + H2O(l)
Fe2O3 (s) + 3H2(g) 2Fe(s) +3H2O(l)
Action of hydrogen on copper (II) oxide

When hydrogen is passed over heated copper (II) oxide, copper is formed. The black copper (II)
oxide turns brown. A colourless liquid (water) forms on the cooler parts of the test tube.
CuO(s) + H2(g) Cu(s) + H2O(l)

Hydrogen also reacts with alkenes to give alkanes (hydrogenation)


CH2=CH2(g) + H2(g) CH3CH3(g)
It combines with highly electropositive metals to form hydrides
2Na(s) + H2(g) 2NaH (s)
NB: Hydrogen chloride (HCl), Hydrogen sulphide (H2S ), ammonia (NH3) and water (H2O ) are
called hydrides.
A hydride is a compound of an element with hydrogen only.
Uses of hydrogen
Used in the manufacture of methanol
It is used a fuel in rockets
Used in the manufacture of hydrogen chloride gas
It is used in hardening of oils
Used in the manufacture of ammonia (Haber process)
Used for filling hydrogen balloons
19. Acids, bases and salts

19.1. Acids
An acid is a substance which when dissolved in water produces hydrogen ions as the only
positively charged ions.

19.1.1. Common Acids


Acids in daily life:
Ethanoic acid found in vinegar and tomato juice
Citric acid found in citrus foods like lemons, oranges and grapefruit
Lactic acid found in sour milk and yoghurt, and in muscle respiration
Tartaric acid found in grapes
Tannic acid found in tea and ants body
Formic acid found in bee stings
Hydrochloric acid found in stomach juices

19.1.2. Laboratory acids: 3 common laboratory acids/Mineral acids


Hydrochloric acid (HCl)
HCl (aq) H+ (aq) + Cl-(aq)
Sulphuric acid (H2SO4)
H2SO4 (aq) 2H+ (aq) + SO42-(aq)
Nitric acid (HNO3)
H NO3 (aq) H+ (aq) + NO3-(aq)
Dilute acids solution containing small amount of acid dissolved in water
Concentrated acids solution containing large amount of acid dissolved in water

19.1.3. Basicity of an acid


Basicity of an acid is maximum number of hydrogen ions (H+)ions produced by a molecule of
an acid
19.1.4. Some Acids with Their Basicity
Acids Reaction with water Basicity
Hydrochloric acid HCl(aq) H+(aq) + Cl-(aq) Monobasic
Nitric acid HNO3(aq) H+(aq) + NO3-(aq) Monobasic
Ethanoic acid CH3COOH(aq) H+(aq) + CH3COO-(aq) Monobasic
Sulphuric acid H2SO4(aq) 2H+(aq) + SO42-(aq) Dibasic
Phosphoric acid H3PO4(aq) 3H+(aq) + PO43-(aq) Tribasic
The fizz of drinks
Soft drink tablets contains solid acid (e.g. citric acid, C6H8O7) & sodium bicarbonate
When tablet is added to water, citric acid ionises and the hydrogen ions produced reacts with
sodium bicarbonate to produce carbon dioxide gas, making them fizz

19.1.5. Strong and Weak Acids


Acids are grouped into weak and strong acids.
An acid is termed as strong or weak depending on its ease to release its hydrogen ions.
Strong acids They are those which are completely ionised in aqueous solution and they are
strong electrolytes. All mineral acids are strong acids. Their reactions are irreversible. E.g.
H2SO4, HNO3, HCl
H2SO4 (aq) 2H+(aq) + SO42-(aq)
In the above equation H2SO4 has completely been ionised in water, forming 3 kinds of particles:
- H+ ions
- SO42- ions
- H2O molecules
HCl(aq) H+(aq) + Cl-(aq)
Strong acids react more vigorously with metals than weak acids hydrogen gas bubbles are
produced rapidly
Weak Acids - acids that partially ionise in water. The remaining molecules remain unchanged as
acids. They are weak electrolytes. Their reactions are reversible. E.g. Ethanoic acid
(CH3COOH), carbonic acid (H2CO3), phosphoric acid (H3PO4)
H3PO4(aq) 3H+(aq) + PO43-(aq)
Weak acids react slowly with metals than strong acids hydrogen gas bubbles are produced
slowly.
19.1.6. Comparing Strong and Weak Acids with Concentrated and Dilute Acids
Concentration Strength
Is the amount of solute (acids or alkalis) dissolved in 1 dm3 of a Is how much ions can be
solution disassociated into from
acid or alkali
It can be diluted by adding more water to solution or concentrated The strength cannot be
by adding more solute to solution changed
Comparing 10 mol/dm and 0.1 mol/dm of hydrochloric acids and 10 mol/dm3 and 0.1 mol/dm3
3 3

of ethanoic acids
- 10 mol/dm3 of ethanoic acid solution is a concentrated solution of weak acid
- 0.1 mol/dm3 of ethanoic acid solution is a dilute solution of weak acid
- 10 mol/dm3 of hydrochloric acid solution is a concentrated solution of strong acid
- 0.1 mol/dm3 of hydrochloric acid solution is a dilute solution of strong acid

19.1.7. Properties of Dilute Acids


1) Acids have a sour, sharp taste e.g. lemons are sour due to citric acid
2) - Acids change the colour of indicators
Acids turn common indicator litmus blue litmus to red
3) - Acids react with metals
Acids react with metals to produce hydrogen gas. The gas is tested with a burning splint which
shows hydrogen burns with a pop sound.
2Na(s) + 2HCl (aq) 2NaCl (aq) + H2 (g)
4) - Acids react with carbonates and hydrogen carbonates (bicarbonates)
A salt, carbon dioxide gas and water are formed. To test for Carbon dioxide gas, the gas
produced is bubbled into limewater which forms a white precipitate.
Carbonates:
MgCO3(s) + 2HCl (aq) MgCl2 (aq) + CO2 (g) + H2O (l)
Hydrogen carbonates/Bicarbonates:
NaHCO3(s or aq) + HCl(aq) NaCl(aq) + CO2(g) + H2O (l)
5) - Acids react with metal oxides and hydroxides
Metal oxides & hydroxides react slowly with warm dilute acid to form salt and water
Cu(OH)2(s) + H2SO4(aq) CuSO4(aq) + 2H2O (l)
19.1.8. Storage of Acids
Acids are stored in clay pots, glass or plastic containers as sand, glass and plastic do not react
with acids. If its stored in metal container, metal would react with acids

19.1.9. Uses of Acids


1) Sulphuric acid is used in car batteries
2) Manufacture of ammonium sulphate for fertilisers
3) Manufacture of detergents, paints, dyes, artificial fibres & plastics
4) Hydrochloric acid can remove rust (iron(III) oxide) which dissolves in acids
5) Acids are used in preservation of foods (e.g. ethanoic acid)

19.1.10. Acids and Hydrogen Ions


The Need for Water in Acids

Acids are covalent compounds and do not behave as acids in the absence of water as water reacts
with acids to produce hydrogen ions (H+) ions, responsible for its acidic properties.
E.g. Citric acid crystals dont react with metals and dont change colours of indicators; citric acid
in water reacts with metals and turns litmus red.

Hydrogen Ions

Hydrogen gas is formed by acids as hydrogen (H+(aq))ions are present in acid solutions
This means when a solid/gas acid dissolved in water, they produce hydrogen (H+(aq))ions in it

Chemical equation: HCl(s) water HCl(aq)


Ionic Equation: HCl(aq) H+(aq) + Cl-(aq)
*Note that for ionic equation only aqueous solutions are ionised*
- However when dissolved in organic solutions, they dont show acidic properties

When metals react with acids, only the hydrogen ions react with metals, e.g.:
Chemical equation: 2Na(s) + 2HCl (aq) 2NaCl (aq) + H2 (g)
Ionic equation: 2Na(s) + 2H+ (aq) 2Na+ (aq) + H2 (g)
19.2. Bases and Alkalis
Bases are oxides or hydroxides of metals and react with an acid to from a salt and water only
Alkalis are bases(basic hydroxides) which are soluble in water
Laboratory Alkalis

- Sodium Hydroxide, NaOH


- Aqueous Ammonia, NH4OH
- Calcium Hydroxide, Ca(OH)2

All alkalis produce hydroxide ions (OH-) when dissolved in water. Hydroxide ions give the
properties of alkalis. They dont behave as acids in absence of water.
Alkalis are therefore substances that produce hydroxide ions, OH-(aq), in water.

19.2.1. Properties of Alkalis


1) Alkalis have a slippery feel
2) Alkalis are hazardous
Dilute alkalis are irritants
Concentrated alkalis are corrosive and burn skin (caustic (i.e. burning) alkalis)
3) Alkalis change the colour of indicators
Alkalis turn common indicator litmus red litmus to blue
4) Alkalis react with acids
The reaction is called a neutralisation reaction.

19.2.2. Neutralisation reactions


A reaction in which an acid reacts with a base to form a salt and water only
Energy is given out during the reaction. Common neutralisation reactions include:
NaOH(aq) + HCl(aq)NaCl(aq) + H2O(l)
2NaOH(aq) + H2SO4 (aq)Na2SO4 (aq) +2H2O(l)
KOH(aq) + HCl(aq)KCl(aq) + H2O(l)
Na2CO3 (aq) + H2SO4 (aq) Na2SO4 (aq) +H2O(l) + CO2(g)
5) Alkalis react with ammonium compounds
They react with heated solid ammonium compounds to produce ammonia gas
(NH4)2SO4(s) + Ca(OH)2(aq) CaSO4(aq) + 2NH3(g) + 2H2O(l)
6) Alkalis react with solutions of metal ions
Barium sulphate, BaSO4(aq), contains Ba2+(aq) ions
Ca(OH)2(aq) + BaSO4(aq) Ba(OH)2(s) + CaSO4(aq)
The solid formed is a precipitate the reaction is called a precipitation reaction

19.2.3. Preparation of bases


1) By burning metals in oxygen
2Mg(s) + O2 (g) 2MgO (s)
2) By addition of water to a metal
2Na(s) +H2 O (l) 2NaOH (aq) + H2(g)
3) By heating a metal hydroxide, metal carbonate or metal nitrate, except those of
potassium and sodium. This is because sodium and potassium salts are not decomposed
by heat
CaCO3(s) CaO(s) + CO2(g)
4) By reacting sodium hydroxide, potassium hydroxide or aqueous ammonia with a
solution of a salt
FeSO4(aq) + 2NaOH(aq) Fe(OH)2(s) + Na2SO4 (aq)

19.2.4. Strong and Weak Bases


Strong base - Base that completely ionises in water to form hydroxide, OH-(aq) ions
Their reactions are irreversible. E.g. NaOH, KOH, Ca(OH)2
Ca(OH)2(s) Ca2+(aq) + 2OH-(aq)
Weak base - Base that partially ionises in water. The remaining molecules remain unchanged as
a base. Their reactions are reversible. E.g. NH3
NH3(g) + H2O(l) NH4+(aq) + OH-(aq)

19.2.5. Uses of Alkalis


Alkalis neutralise acids in teeth (toothpaste) and stomach (indigestion)
Soap and detergents contain weak alkalis to dissolve grease
Floor and oven cleaners contain sodium hydroxide (strong alkalis)
Ammonia (mild alkalis) is used in liquids to remove dirt and grease from glass

19.3. Indicators and pH

19.3.1. pH
pH is the acidity or alkalinity of a substance. The pH scale ranges from 0 to 14. Solution with pH
7 are neutral, those with a pH less than 7 are acidic and those greater than 7 are alkaline. i.e

19.3.2. pH scale
Is used in measuring acidity and alkalinity in aqueous solutions
The PH scale is normally made up of pH values or numbers e.g. pH 7 for neutrality
Acidity ranges from 1 to 6 and alkalinity ranges from 8 to 14
Strength of an acid increases as the value of the numbers (pH) decreases i.e. (6<5<4<
1) represents increasing acidity
Strength of an alkali increases as the value of the numbers increases i.e. (8>9>10>14)
represents increasing alkalinity

19.4. Indicators
Indicators are substances that have different colours in acidic and alkaline solutions
Common indicators:
Litmus
Methyl orange
Phenolphthalein
The table shows the change of colours made by some indicators
Indicator Colour in acids colour changes at pH Colour in alkalis
Phenolphthalein Colourless 9 Pink
Methyl orange Red 4 Yellow
Litmus Red 7 Blue
Screened methyl orange Red 4 Green
Bromothymol blue Yellow 7 Blue

19.5. Measuring pH of a Solution


1. Universal indicators
It can be in paper or solution form. Universal paper can be dipped into a solution then pH found
is matched with the colour chart. It gives approximate pH value.
2. pH meter

A hand-held pH probe is dipped into solution and a meter will show the pH digitally or by a
scale. Measures pH of water in lakes and streams accurately
3. pH sensor and computer
A probe is dipped into solution and will be sent to computer through interface used to measure
pH of solution. The pH reading is displayed on computer screen.

pH Around Us
- Substances in body involved in good digestion have different pH values
- Blood to heart and lungs contains carbon dioxide making blood slightly acidic
- Acids are used in food preservations (ethanoic acid to preserve vegetables; benzoic acid used in
fruit juices, jams and oyster sauce)
- pH affects plant growth some plants grow in acidic soil; some need alkaline soil
- When hair is cleaned with shampoo which is alkali to dissolve grease, hair can be damaged
unless its rinsed or acid conditioner is used to neutalise excess alkali

19.6. Ionic Equations


Ionic equation is equation involving ions in aqueous solution, showing formation and changes
of ions during the reaction
Rule to make ionic equations:
- Only formulae of ions that change is included; ions dont change = omitted
- Only aqueous solutions are written as ions; liquids, solids and gases written in full

Reaction between Metals and Acids


For example, reaction of sodium with hydrochloric acid
2Na(s) + 2HCl(aq) 2NaCl(aq) + H2(g)
Its ionic equation is written as:
2Na(s) + 2H+(aq) + 2Cl-(aq) 2Na+(aq) + 2Cl-(aq) + H2(g)
Since 2 CI-(aq) ions dont change, theyre not involved in reaction. As the ionic equation is used
to show changes in reactions, we omit CI-(aq) ions. So, what is left is:
2Na(s) + 2H+(aq) 2Na+(aq) + H2(g)

Reaction between Soluble Ionic Compounds and Acids


E.g. Reaction of sodium hydrogencarbonate with hydrochloric acid
NaHCO3(aq) + HCl(aq) NaCl(aq) + CO2(g) + H2O (l)
Its ionic equation is:
Na+(aq) + H+(aq) + CO32-(aq) + H+(aq) + Cl-(aq) Na+(aq) + Cl-(aq) + CO2(g) + H2O(l)
Since Na+(aq) and Cl-(aq) ions dont change, we omit them, leaving:
H+(aq) + CO32-(aq) + H+(aq) CO2(g) + H2O (l)
CO32-(aq)) + 2H+(aq) CO2(g) + H2O(l)

Reaction between Insoluble Ionic Compounds and Acids


E.g. Reaction between iron (II) oxide and sulphuric acid
FeO(s) + H2SO4(aq) FeSO4(aq) + H2O(g)
Its ionic equation is:
FeO(s) + 2H+(aq) + SO42-(aq) Fe2+(aq) + SO42-(aq) + H2O(g)
Note: FeO is written in full as its solid (although its an ionic compound)
Since SO42-(aq) ions dont change, omit SO42- ions, leaving:
FeO(s) + 2H+(aq) Fe2+(aq) + H2O(g)
E.g. Reaction between calcium carbonate and hydrochloric acid
CaCO3(s) + 2HCl(aq) CaCl2(aq) + CO2(g) + H2O(l)
Its ionic equation is:
CaCO3(s) + 2H+(aq) + 2Cl-(aq) Ca2+(aq) + 2Cl-(aq) + CO2(g) + H2O(l)
Since 2 Cl-(aq) ions dont change, omit Cl- ions, leaving:
CaCO3(s) + 2H+(aq) Ca2+(aq) + CO2(g) + H2O(l)
Reactions Producing Precipitate
E.g. Reaction between calcium hydroxide and barium sulphate
Ca(OH)2(aq) + BaSO4(aq) Ba(OH)2(s) + CaSO4(aq)
Its ionic equation is written as:
Ca2+(aq) + 2OH-(aq) + Ba2+(aq) + SO42-(aq) Ba(OH)2(s) + Ca2+(aq) + SO42-(aq)
Since Ca2+(aq) and SO42-(aq) ions dont change, omit them, leaving:
Ba2+(aq) + 2OH-(aq) Ba(OH)2(s)

Displacement Reactions
E.g. Reactions between magnesium with zinc sulphate
Mg(s) + ZnSO4(aq) MgSO4(aq) + Zn(s)
Its ionic equation is written as:
Mg(s) + Zn2+(aq) + SO42-(aq) Mg2+(aq) + SO42-(aq) + Zn(s)
Since SO42-(aq) ions dont change, omit them, leaving:
Mg(s) + Zn2+(aq) Mg2+(aq) + Zn(s)

Neutralization
Neutralization is the reaction between acid and base to form salt and water only.
From ionic equation, it is known that the reaction only involves H+ ions from acids with OH- ions
from alkali to form water.
E.g. NaOH + H2SO4 forms Na2SO4 + H2O
H2SO4 (aq) + 2NaOH(aq) Na2SO4 (aq) + H2O(l)
Ionic equation is:
H+(aq) + OH-(aq) H2O(l)
Plants dont grow well in acidic soil. Quicklime (calcium hydroxide) is added to neutralise the
acidity of soil according to equation:
Acid (aq) + Ca(OH)2(aq) Ca(acid anion)(aq) + H2O(l)

Reaction between Base and Ammonium Salts


E.g. Reaction between NaOH and NH4OH
NaOH(aq) + NH4Cl(aq) NaCl(aq) + NH3(g) + H2O(l)
Ionic equation:
NH4+(aq) + OH-(aq) NH3(g) + H2O(l)
Oxides
Acidic Oxide Basic Oxide Amphoteric Neutral Oxide
Oxide
Oxides of non- Oxides of metals, Oxides of Oxides that dont
metals, usually usually solid transition metals, react with either
gases which which reacts with usually solid, acids/alkalis,
reacts with water water to produce which reacts with hence do not form
to produce acids, alkalis, e.g. CaO, acids/alkalis to salts, e.g. H2O,
e.g. CO2, NO2, K2O, BaO form salt and CO, NO
P4O10, SO2 water, e.g. Al2O3,
FeO, PbO

19.7. Salts
A salt is a substance consisting of positive metallic ions and negative ions derived from an acid
OR:
A salt is a compound formed when the replaceable ionisable hydrogen of an acid is replaced by a
metal or an ammonium ion either wholly or partially

Examples of salts and their corresponding salts


Acid Salt Name of the salt
HCl NaCl Sodium chloride
HCl KCl Potassium chloride
HCl NH4Cl Ammonium chloride
HNO3 AgNO3 Silver nitrate
H2SO4 CuSO4 Copper sulphate
H2SO4 MgSO4 Magnesium sulphate
H2SO4 Na2SO4 Sodium sulphate
H2SO4 NaHSO4 Sodium hydrogen sulphate

19.7.1. Types of salts


Normal salts
Acid salts

A normal salt: is formed when all the ionisable hydrogen in an acid has been replaced by a
metal or metallic radical e.g sodium chloride (NaCl), Sodium carbonate (Na2CO3)
An acid salt: is formed when only part of the ionisable hydrogen in an acid has been replaced by
a metal or metallic radical e.g sodium hydrogen sulphate (NaHSO4), sodium hydrogen carbonate
(NaHCO3) etc

19.7.2. Preparation of Salts


The method of preparation of a salt depends on whether the salt is soluble or insoluble in water.
Soluble salts are prepared by crystallisation method while insoluble salts are prepared by
precipitation method.
Soluble salts can also be prepared by reacting a metal and an acid
Zn(s) + 2HCl(aq)ZnCl2(aq) + H2(g)

Some soluble salts can also be prepared by direct synthesis


E.g. chlorides of aluminum and iron can be prepared from their elements directly
Al(s) + 3Cl2(g) 2AlCl3(s)
Fe(s) + 3Cl2(g) 2FeCl3(s)
NB: The gas is passed over the heated metal in each case

19.7.3. Soluble and Insoluble Salts

Soluble Insoluble
All Nitrates -
All Sulphates Barium sulphate (BaSO4), Lead sulphate (PbSO4),
Calcium sulphate (CaSO4) is slightly soluble
All Chlorides Lead (II) chloride (PbCl2), silver chloride (AgCl)
Potassium, Sodium, Ammonium salts -
Potassium carbonate (K2CO3), sodium All Carbonates
carbonate (Na2CO3), ammonium
carbonate (NH4CO3)
Potassium oxide (K2O), sodium oxide All Oxides
(Na2O)
Oxides and hydroxides of potassium, All other oxides and hydroxides (those of calcium
sodium and ammonium and magnesium are slightly soluble)

19.7.4. Preparation of Insoluble Salts


Insoluble salts, e.g. BaSO4, CaSO4, PbSO4, PbCl2 , AgCl and most carbonates, can be prepared
by reacting a compound containing the wanted cation with another compound containing the
wanted anion. This is a precipitation reaction/double decomposition.

E.g. Preparation of barium sulphate (BaSO4) from Barium chloride and dilute sulphuric
acid
Barium chloride (BaCl2) which contains the wanted barium ion, is reacted with dilute sulphuric
acid (H2SO4) which contains the wanted sulphate ion, to produce solid barium suphate (BaSO4)
and aqueous potassium chloride (KCl). BaSO4 is then separated from KCl by filtration, leaving a
colourless filtrate (KCl(aq)) & BaSO4 left on filter paper. The salt is then washed with water to
completely remove KCl & filter paper is squeezed with another filter paper to dry BaSO4.
Preparation of lead (II) sulphate
An insoluble salt of lead (II) sulphate is prepared by precipitation. This is done by mixing
solutions containing a sulphate and lead (II) ions
A soluble salt of lead is mixed with a soluble sulphate
When lead ions combine with the sulphate ions, lead (II) sulphate is formed
Pb2+(aq) + SO42-(aq) PbSO4(s)
Lead (II) nitrate solution is put in a beaker and the solution is heated. Dilute sulphuric acid is
then added. A white precipitate of lead (II) sulphate is formed. The precipitate is filtered off as a
residue and washed with distilled water. The crystals are then dried.
Pb(NO3)2( aq) + H2SO4 (aq) 2HNO3(aq) + PbSO4(s)
Ionically
Pb2+(aq)+ 2NO3-(aq) + 2H+(aq) + SO42-(aq) 2NO3-(aq) + 2H+(aq) + PbSO4(s)
Pb2+(aq) + SO42-(aq) PbSO4(s)

Preparation of lead (II) chloride


Dilute hydrochloric acid is put in a beaker and lead (II) nitrate solution is added. A white
precipitate is formed which is filtered off. The precipitate is washed and dried.
Pb(NO3)2( aq) + 2HCl (aq) 2HNO3(aq) + PbCl2(s)
Ionically
Pb2+(aq)+ 2NO3-(aq) + 2H+(aq) + 2Cl-(aq) 2NO3-(aq) + 2H+(aq) + PbCl2(s)
Pb2+(aq) + 2Cl-(aq) PbCl2(s)

19.7.5. Preparation of Soluble Salts


Soluble salts are prepared by the action of an acid on a metal, metal carbonate, metal oxide,
metal hydroxide or alkali
Soluble salts of lead, copper, iron and zinc can be prepared as follows;
1. Add metal, metal oxide or metal carbonate to dilute acid until the metal is in excess and
heat
2. Filter off the excess solid and concentrate the filtrate by heating
3. Cool the solution to crystallize and filter off the crystals
4. Dry the crystals to obtain a dry salt

Preparation of copper (II) sulphate from sulphuric acid and copper (II) oxide
Dilute sulphuric acid is put in a beaker and copper (II) oxide is added little by little until in
excess. The excess oxide is the filtered off.
The filtrate which remains in the container is concentrated by heating. The solution is left to cool
for some time.
Crystals are formed and then filtered off and put on a filter paper for drying
CuO(s) + H2SO4(aq) CuSO4(aq) + H2O(l)
CuSO4(aq) + 5H2O(l) CuSO4.5H2O(s)
NB: Salts of lead (II) nitrate, zinc sulphate and magnesium sulphate can be prepared in the same
way.

Preparation of zinc sulphate from zinc


A beaker is half filled with dilute sulphuric acid and zinc is added
Effervescence of hydrogen gas occurs
If the reaction is slow, add copper (II) sulphate to speed up the reaction
Add more zinc metal to make sure that all the acid has been used up
Warm gently
Filter to remove excess zinc and other insoluble impurities. The filtrate is a colourless solution
containing zinc sulphate
Allow the filtrate to crystallize by heating it so that water can freely evaporate
Colourless needle like crystals will be obtained after evaporation
Zn(s) + H2SO4(aq) ZnSO4(aq) + H2(g)
ZnSO4(aq) +7 H2O(g) ZnSO4.7 H2O(s)
NB: Other salts such as magnesium sulphate, copper (II) sulphate and iron (II) sulphate can be
prepared in the same way. But with copper (II) sulphate, concentrated sulphuric acid is used
instead of dilute sulphuric acid with copper metal.

Preparation of salts by action of an acid on a carbonate


E.g. Preparation of lead (II) nitrate from lead (II) carbonates
Dilute nitric acid is put in a beaker and lead (II) carbonate added little at a time until no more
reacts
Filter off excess lead (II) carbonate and collect a colourless filtrate
Evapourate the filtrate. After evapouration crystals of lead (II) nitrate are formed.
PbCO3(s) + 2HNO3(aq) Pb(NO3)2(aq)+ CO 2 (g) + H2O(l)
Salts of copper (II) sulphate, copper (II) nitrate, zinc sulphate and calcium chloride can be
prepared in the same way.

Preparation of salts from an acid and an alkali (Neutralisation)


E.g. Preparation of sodium chloride
Pipette 25cm3 of sodium hydroxide into a conical flask
Add 2 drops of phenolphthalein indicator
Run dilute hydrochloric acid from the burette dropwise until the colour of the solution turns from
pink to colourless. The solution at this time is neutral.
Note the volume of the acid that was used. The whole process is repeated using exactly the same
volume of the acid and base but without the addition of the indicator.
The solution obtained contains sodium chloride
Transfer this solution to an evaporating basin and evapourate it to dryness
After evapourating crystals of sodium chloride are formed
NaOH(aq) + HCl(aq) NaCl(aq) + H2O(g)
The process can also be used to prepare salts of potassium and ammonium

Experiment illustrating neutralization


25.0cm3 of acid, as a standard solution, is placed in a conical flask using pipette. A few drops of
the indicator are added & titrated with an alkali from the burette until the indicator changes
colour, showing that all the acid has just reacted. Volume of alkali added is measured. Prepare
another 25.0cm3 of acid again, using the same pipette add the same volume of alkali as before to
prevent excess alkali/acid because both the reactants & products are aqueous. The product is then
evaporated to dryness to obtain the salt.
Reacting Metal with Acid
Only metals like zinc and magnesium, which moderately react with dilute acids, are used.
E.g. Reacting zinc with dilute sulphuric acid to prepare zinc sulphate
Zn(s) + H2SO4(aq) ZnSO4(aq) + H2O(l)
Zinc is added to dilute sulphuric acid until in excess to ensure no more sulphuric acid is present.
Then the mixture is filtered off to separate zinc from zinc sulphate. The filtrate which contains
zinc sulphate is then placed in evaporating dish to evaporate most of water then its cooled after
zinc sulphate crystals are formed. The crystals are then filtered and squeezed between filter
papers to dry.
By Reacting an Insoluble Base with Acid
E.g. Reacting magnesium oxide with Acids
MgO(s) + H2SO4(aq) MgSO4(aq) + H2(g)
The same method as reaction of acid with metal is used, so refer to diagram and above
explanation, substituting reactants and products.

By Reacting a Carbonate with Acid


E.g. Reacting calcium carbonate with Acids
K2CO3(s) + H2SO4 (aq) K2SO4 (aq) + CO2 (g) + H2O (l)
The same process is used as reaction of acid with metal, just that carbon dioxide is produced.
Carbon dioxide can be tested by bubbling it into limewater which will turn limewater colourless
to milky.
19.7.6. Action of heat on salts
Carbonates
Potassium and sodium carbonates are stable and do not decompose on heating
Ammonium carbonate decomposes when heated to give ammonia gas, Carbon dioxide
and water
(NH4)2CO3(s) NH3(g) + CO2(g) + H2O(l)
The rest of the carbonates decompose on heating to form metal oxides and Carbon
dioxide gas
MgCO3(s) MgO(s) + CO2(g)
White powder White residue Colourless gas
ZnCO3(s) ZnO(s) + CO2(g)
White powder Yellow residue - hot colourless gas
White residue cold
PbCO3(s) PbO(s) + CO2(g)
White powder White residue hot colourless gas
Yellow residue - cold
CuCO3(s) CuO(s) + CO2(g)
Green powder Black residue Colourless gas
NB: The difference between lead oxide and zinc oxide is that lead oxide is brown when hot and
yellow when cold while zinc oxide is yellow when hot and white when cold.

Nitrates
Ammonium nitrate decomposes on heating to give dinitrogen oxide and water
NH4NO3(s) N2O(g) + H2O(l)
Metal nitrates decompose following the reactivity series

Potassium Decompose to pale yellow metal nitrites and oxygen


Sodium
2MNO3(s) 2MNO2 (s) + O2 (g)

Calcium
Magnesium
Decompose on heating to form metal oxides, nitrogen dioxide and oxygen

2M (NO3)2(s) 2MO(s) + 4NO2 (g) + O2 (g)


Aluminum
Zinc
Iron
Lead
Copper

Mercury Decompose on heating to form their corresponding metals, nitrogen


Silver dioxide and oxygen M(NO3)2(s) M(s) + 2NO2(g) + O2 (g)

Potassium and sodium nitrate melt into colourless liquids and then decompose to pale yellow
nitrites and oxygen
2KNO3(s) 2KNO2 (s) + O2 (g)
2NaNO3(s) 2NaNO2 (s) + O2 (g)
Mercury and silver nitrates decompose on heating to form their corresponding metals, nitrogen
dioxide and oxygen
Hg(NO3)2(s) Hg(s) + 2NO2 (g) + O2 (g)
Ag(NO3)2(s) Ag(s) + 2NO2 (g) + O2 (g)
The rest of the nitrates i.e. from calcium to copper nitrates decompose on heating to form metal
oxides, nitrogen dioxide and oxygen
2Ca (NO3)2(s) 2CaO(s) + 4NO2 (g) + O2 (g)
2Cu (NO3)2(s) 2CuO(s) + 4NO2 (g) + O2 (g)
2Pb (NO3)2(s) 2PbO(s) + 4NO2 (g) + O2 (g)
Sulphates and hydrates
Most sulphates are hydrated and when heated, they lose their water of crystallisation to form
anhydrous salts which are resistant to further heating and therefore do not decompose. Therefore
hydrated sulphates do not decompose on heating e.g.
MgSO4.7 H2O(s) MgSO4(s) + 7 H2O(l)
CuSO4.5 H2O(s) CuSO4(s) + 5H2O(l)
Blue White
Na2SO4.10H2O(s) Na2SO4(s) + 10H2O(l)
FeSO4.7 H2O(s) FeSO4(s) + 7 H2O(l)
On further heating, the anhydrous iron (II) sulphate formed decomposes to give sulphur dioxide,
sulphur trioxide and leaves a brown solid of iron (III) oxide
FeSO4(s) Fe2O3(s) + SO2(g)+ SO3(g)
When ammonium sulphate is heated it decomposes to give ammonia, sulphur trioxide and water
(NH4)2SO4(s) NH3 (g) + SO3(g) +H2O(l)

19.8. Solubility of salts


Solubility: is defined as the mass of solute dissolved in a fixed mass of a solvent at a particular
temperature to give a saturated solution.
Or it is the number of grams of solute dissolved in 100g of water to give a saturated solution at a
particular temperature
Or solubility of a solute in a solvent at a given temperature is the number of moles of a solute
necessary to saturate one kilogram of a solvent at that temperature
NB: Solubility increases with temperature for most of the salts
Saturated solution: Is one which cannot dissolve any more solute at a given temperature and
pressure
Or
A saturated solution is one which contains much more solute than it can dissolve at that
temperature
Solubility can be expressed as
1. Mass of solute (g) per 100g of a solvent i.e. grams per 100g of solvent (g/100g of
solvent)
2. Mass of solute (g) per 1000g of a solvent i.e grams per 1000g of solvent (g/1000g of
solvent)
Some salts are highly soluble in water e.g. potassium nitrate, sodium nitrate, sodium chloride,
potassium chloride, sodium carbonate, potassium carbonate.
Others are sparingly soluble in water at room temperature e.g. lead chloride, silver chloride,
barium sulphate, lead sulphate, calcium carbonate etc
The solubility of these sparingly soluble salts increases as temperature increases.
19.8.1. Solubility curves
A solubility curve is a plot of mass of solute dissolved in 100g of water against temperature
Or: solubility curve is a graph of solubility versus temperature for a solute
From solubility curves above:
The salt whose solubility increases most with increase in temperature is potassium nitrate
The salt whose solubility decreases most with increase in temperature is calcium sulphate
The temperature at which two curves intercept is the temperature at which solubility of the two
salts is the same
The salt whose solubility shows very little dependence on temperature is sodium chloride

19.8.2. Uses of solubility


If a solubility curve of a salt is available, it can be used to obtain the solubilities of theh salt at
various temperatures
The solubility curve can be used to determine the temperature at which a certain quantity of salt,
when dissolved in water can produce a saturated solution
The solubility curve can be used to calculate the mass of salt obtained by cooling the solution
from a higher to a lower temperature

Example

a. Calculate the mass of potassium chlorate that can be crystallized by cooling the solution
from 70oC to 30oC
b. Use the solubility curve to determine the solubility of potassium chlorate at 50oC
Solution:
The solubility of potassium chlorate at 70oC is 160g/100g water and solubility at 30oC is
60g/100g water. Therefore the mass of potassium chlorate that can be crystallized by cooling the
solution from 70oC to 30oC = 160-60 =100g
The solubility of potassium chlorate at 50oC is 100g/100g water. This is obtained by
extrapolation of the line indicated by the dotted line on the graph

19.9. Determination of solubility of salts


E.g. Determination of solubility of potassium nitrate
Some potassium nitrate crystals are crushed in a mortar and placed in a boiling tube. Little water
is added to about halfful. The boiling tube is put in a beaker of water and the beaker is warmed.
More potassium nitrate crystals are added while stirring until some remain undissolved at the
bottom of the tube.
Read the temperature of the solution. When the temperature reads 55oC, put off the flame and
allow the apparatus to cool. When the temperature drops to 50oC, decant the saturated solution
into a weighed dish leaving the solid potassium nitrate behind
Calculate the mass of potassium nitrate dissolved in 100g of water at 50oC, using the results
obtained. This will be the solubility of potassium nitrate at 50oC
Sample results
Mass of dish =14.32g
Mass of dish + potassium nitrate solution =35.70g
Mass of dish + potassium nitrate solid =18.60g
Temperature of the saturated solution = 50oC
Therefore
Mass of water =35.70 18.60 =17.10g
17.10g of water dissolved 4.28g of potassium nitrate =18.60-14.32 = 4.28g
17.10g of water dissolved 4.28g of potassium nitrate
4.28
18.60g of water will dissolve = 17.10

4.28
( X 100)
100g of water dissolve = 17.10 g
=25g
Therefore, the solubility of potassium nitrate is 25g/100g of water at 50oC
Exercise
1. 10g of a saturated sodium chloride solution was evaporated and 6g of solid sodium
chloride was left. Calculate
a. Solubility of sodium chloride (Ans = 150g)
b. The percentage of sodium chloride in a saturated solution (Ans = 60%)
2. 75g of a saturated solution contains 30g of a salt. Calculate its solubility(Ans =
66.67g/100g of water)
3. The solubility of X is 40g/100g of water. Calculate the mass of X that can be dissolved in
60g of water to give a saturated solution (Ans = 24g)
4. The table below shows the solubilities of salt P in water at different temperatures
Temperature /oC 10 20 30 40 50 60
Solubilities (g/100g of solvent) 18 20 24 30 38 50
a. Plot a graph of solubility of P
b. Use your graph to determine solubility of P at 25oC (Ans =22g/100g of water)
c. Calculate the mass of P that would dissolve in 45g of water at 25oC (Ans =9.9g)
20. Carbon and its compounds
Carbon
This is the element in group IV of the periodic table. It has atomic number 6 and atomic mass 12

Occurrence of carbon
Pure carbon is found in the form of diamond and impure carbon as graphite. Carbon is a
constituent of numerous naturally occurring substances such as coal, mineral oils, carbonates,
organic matter and in air as carbon dioxide.

Allotropes of carbon
Allotropy: is the existence of an element in more than one form, without change in physical
state. These different forms are called allotropes.

An allotrope is one of two or more distinct forms of an element. Carbon has three allotropes:
diamond, graphite and amorphous carbon. Others elements that show allotropy are:

1. Oxygen with two allotropes, that is, ordinary oxygen (O2) and ozone (O3).
2. Tin with two allotropes, that is, grey tin and white tin.
3. Sulphur has five allotropes, that is, amorphous sulphur, monoclinic sulphur, rhombic
sulphur, plastic sulphur and colloidal sulphur.
4. Phosphorus with two allotropes, that is white/yellow phosphorus and red phosphorus.

Graphite
Graphite is a black, soft, slippery, hexagonal crystalline substance. Its atoms are joined by strong
covalent bonds.

Structure of graphite
Graphite is a two dimensional layered structure. The carbon atoms within the layers are arranged
in hexagonal rings and each carbon atom is covalently bonded to three other carbon atoms.

For each carbon atom, three out of the four valence electrons localized during the formation of
the covalent bonds. The remaining electron is delocalized (mobile) over the whole layer. The
mobile electrons are free to move through the structure and therefore enable graphite to conduct
heat and electricity.

The hexagonal layers lie on top of one another and are joined by weak van der waals forces
which enable layers to slide over each other easily. That is why graphite is soft and can be used
as a lubricant.

Properties of graphite
1. It is a black material which feels greasy on touching
2. It is opaque and shiny
3. It has a density of 2.3g/cm3
4. It has hexagonal structures
5. It conducts electricity. This because it contains delocalised electrons. When an electric
field is applied, these electrons move freely conducting electric current.
6. Writes well on papers
7. Graphite is soft and slippery because the layers within the structure are held together by
weak van de waals forces which makes it possible for the layers to slide over each other
easily.
Uses of graphite
1. Used as protective coating for iron substances to prevent rusting.
2. Used as an electrode in electrolytic cells. Since graphite conducts electricity
3. As a lubricant for dynamos and electric motors.
4. For making pencil leads

Diamond
It is a colourless, transparent and sparkling crystalline substance. It is the hardest substance
known.

Structure of diamond
Diamond has a tetrahedral structure in which each carbon atom is joined by covalent bonds to
four other carbon atoms.

The valence electrons are all used in forming covalent bonds and therefore they are localized.

This structure is extended throughout the substance in three dimensions.

Diamond is a poor conductor of heat and electricity because it does not have free and mobile
electrons.
Uses of diamond
1. Used for manufacturing drilling and cutting hard substances such as glass and rock.
2. It is used to make jewelry e.g. necklaces and ear rings because of its high refractive index
giving it a shiny appearance.

Amorphous carbon
Amorphous carbon is black and has the lowest density among all the allotropes of carbon. It is a
fair conductor of electricity. It is a non-crystalline substance. Amorphous carbon exists in several
forms including wood charcoal, animal charcoal and lampblack. Coke and soot are other forms
of impure amorphous carbon. Animal charcoal is made by heating animal bones and remains in
a limited supply of air. Coke is made by heating coal in absence of air. Wood charcoal is formed
by burning in a limited supply of air. It can be used to remove poisonous gases such as ammonia,
sulphur dioxide and chlorine. It is also useful as fuel. Lamp black is made by burning oil in a
limited supply of air e.g. kerosene, petroleum and turpentine. It is used in making printers ink,
shoe polish, carbon paper and car tyres.

Uses of amorphous carbon


1. It is found in wood charcoal formed by burning wood with a limited supply of air. Wood
charcoal is used as a source of fuel for cooking. Also coke is used as a source of energy.
2. It is found in animal charcoal, which is used to remove the brown colour from sugar
during its manufacture.
3. Lampblack obtained by burning oils with a limited supply of air, is used in sugar during
its manufacture.
4. Coke is used as a reducing agent in extraction of metals from their ores e.g. iron.

Proof for allotropy of carbon


When equal masses of graphite, diamond and amorphous carbon are separately burnt in excess
pure oxygen, each produces the same quantity of carbon dioxide and no other product. The
carbon dioxide is tested using limewater which turns milky. This proves that graphite, diamond
and amorphous carbon are made up of the same substance, carbon.

C(s) + O2(g) CO2(g)

Differences between diamond, graphite and amorphous carbon


Property Diamond Graphite Amorphous carbon
Appearance Colourless and Grey-black, opaque Black, opaque and
transparent and shiny dull
Hardness Hardest Soft, greasy and Soft
soapy
o
Burning in air Burns at 900 C Burns at 700 oC Burns at 500 oC
Density 3.5g/cm3 2.3g/cm3 1.5g/cm3
Conduction of Poor Good Fair
electricity
Structure Giant molecular Hexagonal Indefinite
Bonding electrons All 4 electrons Uses only three
electrons

Chemical properties of carbon


All allotropes of carbon have similar chemical properties but different physical properties.

a) Reaction with oxygen


When carbon is burnt in excess pure oxygen, carbon dioxide is formed.

C(s) + O2(g) CO2(g)

When carbon is burnt in limited supply of oxygen, carbon monoxide is formed.


2C(s) + O2(g) 2CO(g)

Exhaust fumes from cars contain carbon monoxide because of incomplete combustion of petrol
or diesel.

Combustion of charcoal in oxygen


Some powdered wood charcoal is placed in a deflagrating spoon and heated over a Bunsen
burner flame until it glows red-hot. The spoon is immediately transferred into a jar of oxygen

Charcoal continues to burn slowly with a yellow flame without any further heating. The amount
of charcoal gradually decreases and finally only a small amount of ash is left. The presence of
ash implies that wood charcoal is not pure carbon.

C(s) + O2(g) CO2(g)


b) Reaction of carbon with metallic oxides
Carbon reduces oxides of metals that are lower than it in the activity series. Such oxides
include copper (II) oxide, zinc oxide, lead(II) oxide and iron(III) oxide. All these are reduced
to free metal while carbon is oxidised to carbon dioxide.
2CuO(s) + C(s) 2Cu(s) + CO2 (g)

2ZnO(s) + C(s) 2Zn(s) + CO2 (g)

2PbO(s) + C(s) 2Pb(s) + CO2 (g)

2Fe2O3(s) + 3C(s) 4Fe(s) +3CO2 (g)

This reaction is used in extraction of the metals. Those metals higher in reactivity series than
carbon have a higher affinity for oxygen and will not give it up to carbon.

c) It reduces concentrated nitric acid to nitrogen dioxide


4HNO3 (aq) + C(s) 2H2O (l) + 4NO2 (g) + CO2 (g)

d) It reduces concentrated sulphuric acid to sulphur dioxide

2H2SO4 (aq) + C(s) 2H2O (l) + 2SO2 (g) + CO2 (g)

20.1. Oxides of carbon

20.1.1. Carbon dioxide


The amount of carbon dioxide in the atmosphere is about 0.03 percent by volume. It is produced
as a waste product during respiration of plants and animals. It is also formed during combustion
of fuels such as wood, petrol etc.

Experiment:

Laboratory preparation of carbon dioxide


Dilute hydrochloric acid from a tap funnel is added to calcium carbonate in a flat-bottomed flask
Effervescence occurs and a colourless gas, which is carbon dioxide, is formed according to the
equation.

CaCO3(s) + 2HCl (aq) CaCl2 (aq) + H2O + CO2 (g)

The gas is then passed through a bottle containing water or potassium hydrogen carbonate
solution to absorb any hydrochloric acid fumed.

It is then passed through concentrated sulphuric acid to dry it.

The gas is collected by downward delivery in a gas-jar since the gas is denser than air.

Ionic equation

Ca2+(aq) + CO32- (aq) + 2H+(aq ) + 2Cl- (aq) Ca2+(aq) + 2Cl- (aq) + H2O(l) + CO2 (g)

2H+ (aq) + CO32- (aq) H2O(l) + CO2 (g)

NB: If the gas is not required dry it can be collected over water. This is possible because carbon
dioxide is only slightly soluble in water.

Dilute sulphuric acid is not used with calcium carbonate because the reaction produces calcium
sulphate which is sparingly soluble and thus forms a coating on the calcium carbonate which
stops further reaction.

CaCO3(s) + H2SO4 (aq) CaSO4(s) + H2O (l) + CO2 (g)


Lead (II) carbonate is also not used because when it reacts with dilute hydrochloric acid or
sulphuric acid, the reaction soon slows down and then stops. This is due to the formation of lead
(II) chloride or lead (II) sulphate, both of which are insoluble salts.

The insoluble salt coats the carbonate preventing it from reacting with the acid.

PbCO3(s) + 2HCl(aq) PbCl2(s) + CO2(g) + H2O(1)

PbCO3(s) + H2SO4(aq) PbSO4(s) + CO2(g) + H2O(1)

Kipps apparatus

A continuous supply of carbon dioxide can be obtained from a Kipps apparatus using calcium
carbonate and dilute hydrochloric acid.

Industrial preparation of carbon dioxide

In industries, carbon dioxide is obtained as a by-product of fermentation of sugars to alcohol.

C6H12O6(aq) 2C2H5OH(aq) + 2CO2(g)

It is also obtained from the manufacture of cement. Cement is made by heating limestone
(calcium carbonate) with some sand and silicates to form impure calcium oxide.

CaCO3(s) CaO(s) + CO2(g)

Properties of carbon dioxide

Physical properties

1. It is a colourless gas
2. It has a faint sharp test
3. It has a very faint smell
4. It does not support burning
5. It is slightly soluble in water forming carbonic acid.
H2O(l) + CO2(g) H2CO3(aq)

6. It turns litmus pink since it is a weak acidic gas.

7. It is denser than air. When carbon dioxide in a jar is poured into another jar containing a
lighted candle, the candle is extinguished. This shows that carbon dioxide is denser than
air. It displaces air from the jar containing a lighted candle hence starves the candle of
oxygen.

Chemical properties

(a) Effect of carbon dioxide on burning magnesium


When a piece of burning magnesium ribbon is lowered into a gas-jar containing carbon dioxide,
it continues to burn for a short time with a spluttering flame. Black particles of carbon are
formed on the sides of the gas-jar and white ash of magnesium oxide is also formed.

2Mg(s) + CO2(g) 2MgO(s) + C(s)

Magnesium continues to burn in carbon dioxide because of its higher affinity for oxygen than
carbon. The heat from the burning magnesium decomposes carbon dioxide into carbon and
oxygen. The decomposition of carbon dioxide provides more oxygen which supports continued
burning of magnesium oxide.

(b) Effect of carbon dioxide on calcium hydroxide solution (lime-water)


When carbon dioxide is bubbled through lime-water, the lime-water turns milky. The white
precipitate is due to the formation of an insoluble substance, calcium carbonate, in water.

Ca(OH)2(aq) + CO2(g) CaCO3(s) + H2O(l)

The above test is used to distinguish carbon dioxide from any other gas. However, if excess
carbon dioxide is bubbled through the milky solution, the white precipitate dissolves to form a
colourless solution due to the formation of calcium hydrogen carbonate, which is soluble in
water.

CaCO3(s) + H2O (l) + CO2 (g) Ca(HCO3)2(aq)


(c) Reaction with alkalis
Sodium hydroxide solution readily absorbs carbon dioxide to produce sodium carbonate.

2NaOH(aq) + CO2(g) Na2CO3(aq) + H2O(l)

With excess carbon dioxide, a white precipitate of sodium hydrogen carbonate is formed. The
precipitate is sparingly soluble in cold water.

Na2CO3(aq) + H2O(l) + CO2(g) 2NaHCO3(s)

When a jar of carbon dioxide is placed in a trough containing sodium hydroxide solution, the
solution quickly rises into the jar. This is because the gas is rapidly absorbed into the solution.
Carbon dioxide reacts with sodium hydroxide solution.

2NaOH(aq) + CO2(g) Na2CO3(aq) + H2O(l)

When solid sodium hydroxide is exposed to air, a colourless solution is formed and later a white
crystalline solid is formed. Sodium hydroxide is deliquescent and therefore absorbs water from
air to form a solution. The solution absorbs carbon dioxide from air forming a white crystalline
solid of sodium carbonate decahydrate.

2NaOH(s) + CO2(s) + 9H2O(l) Na2CO3.10H2O(s)


Uses of carbon dioxide

1. Carbon dioxide is used in the manufacture of carbonated drinks because of its pleasant
taste in water.
2. Carbon dioxide is used as a refrigerating agent for perishable goods
3. Pieces of solid carbon dioxide are sometimes dropped into clouds to cool them to form
rain.
4. Carbon dioxide is used in fire extinguishers. Carbon dioxide being denser than air forms
a layer around the burning material. It covers the fire and starves it of oxygen hence the
fire is put out.
5. It is used during photosynthesis by green plants
6. It is used in the manufacture of sodium carbonate and sodium hydrogen carbonate
Exercise

1.
a.
i. Draw a labeled diagram of the set-up of the apparatus that can be used to prepare a dry sample of
carbon dioxide in the laboratory
ii. Write an equation that leads to the formation of carbon dioxide
iii. Write an ionic equation for the reaction leading to the formation of carbon dioxide
b. Carbon dioxide was passed through calcium hydroxide solution. Describe and explain the
reaction that took place.
c.
i. State what would be observed if burning magnesium ribbon was lowered into a jar of carbon
dioxide
ii. Write equation for the reaction that takes place
2.
a. Describe the structure of graphite
b. State two properties in which graphite differs from diamond
c. Graphite was heated in excess air and the gas given off passed through aqueous calcium
hydroxide for a long time
i. State what was observed
ii. Write equations for the reaction (s)

20.1.2. Carbon monoxide


Carbon monoxide is a poisonous, colourless gas with practically no smell. It is present in coal
gas and other gaseous fuels. It is formed by the partial combustion of carbon.
2C(s) + O2(g) 2CO(g)

Laboratory preparation of carbon monoxide

Charcoal is put in a combustion tube and heat is applied until red hot. Carbon dioxide is passed
over heated charcoal in a combustion tube. Carbon monoxide is produced.

C(s) + CO2(g) 2CO(g)

The mixture of excess carbon dioxide and carbon monoxide is passed over concentrated
potassium hydroxide solution which absorbs carbon dioxide. Carbon monoxide is then collected
over water because it is insoluble in water.

2KOH(aq) + CO2(g) K2CO3(aq) + H2O(l)


Preparation of carbon monoxide from sodium methanoate (HCOONa)

Sodium methanoate is put in a flask and concentrated sulphuric acid is added drop wise through
a tap funnel.

Effervescence occurs and carbon monoxide is collected over water. In the flask, sodium
methanoate is first converted to methanoic acid which is later dehydrated with concentrated
sulphuric acid

2HCOONa(aq) + H2SO4(aq) HCOOH(aq) + Na2SO4(aq)

HCOOH(aq) CO(g) + H2O(l)


Preparation of carbon monoxide from ethanedioic acid

Oxalic acid crystals are placed in a flask and concentrated sulphuric acid added through a thistle
funnel. The mixture is then warmed, effervescence occurs and a mixture of carbon dioxide and
carbon monoxide is produced. The mixture is then passed over concentrated potassium
hydroxide which absorbs carbon dioxide. Carbon monoxide is then collected over water.

H2C2O4 (aq) CO2 (g) + CO(g) + H2O(l)

2KOH(aq) + CO2(g) K2CO3(aq) + H2O(l)

Test for carbon monoxide

It burns with a blue flame forming carbon dioxide

2CO(g) + O2(g) 2CO2(g)


Properties of carbon monoxide

1. It is a colourless gas.
2. It has no effect on litmus paper, that is, it is a neutral gas.
3. It burns in air with a blue flame forming carbon dioxide.
2CO(g) + O2(g) 2CO2(g)

This reaction also takes place in a charcoal burner when there is a sufficient supply of air.

At A, there is plentiful supply of oxygen and charcoal burns to form carbon dioxide.

C(s) + O2(g) CO2(g)

At B, the rising carbon dioxide is reduced by red-hot charcoal to form carbon monoxide.

CO2(g) + C(s) 2CO(g)

At the surface of the burner, the hot carbon monoxide burns in the air with a blue flame to form
carbon dioxide.

2CO(g) + O2(g) 2CO2(g)

If the charcoal burner is in a poorly ventilated room with insufficient air, the reaction at the
surface fails to takes place. The poisonous carbon monoxide is released into the room. If
someone stays in such a room, he or she may die within a short while due to carbon monoxide
poisoning.

4. It is insoluble in water.
5. It is a reducing agent. It reduces some metallic oxides of copper, lead, zinc and iron, that
is, oxides of metals below carbon in activity series. The porcelain boat is heated strongly
and the excess carbon monoxide is lighted at the jet.
CuO(s) + CO(g) Cu(s) + CO2(g)

(black) (brown)

Fe2O3(s) + 3CO(g) 2Fe(s) + 3CO2(g)

(red-brown) (grey)

ZnO(s) + CO(g) Zn(s) + CO2(g)

(white) (grey)

PbO(s) + CO(g) Pb(s) + CO2(g)

Lead(II) oxide (yellow) is reduced to a grey solid. Carbon monoxide does not, however, reduce
the oxides of metals higher than carbon in the reactivity series. Such metals have a higher affinity
for oxygen than carbon monoxide.
6. It is a poisonous gas because it forms a fairly stable compound with haemoglobin which
reduces the oxygen-carrying capacity of blood.

Uses of carbon monoxide

1. In the manufacture of synthetic petrol.


2. In the reduction of ores and refining of nickel.
3. In the manufacture of methanol used in anti-freezer mixtures in cold countries to prevent
ice from forming in car radiators.
4. Used as fuel in form of producer gas. Producer gas is produced when air is passed over
red-hot coke.
Differences between carbon monoxide and carbon dioxide

Carbon dioxide Carbon monoxide


Has a faint smell Has no smell
Turns lime water milky Does not
It is not a reducing agent It is a strong reducing agent
Slightly soluble in water and alkalis Insoluble in water and alkalis
Reacts with alkalis forming carbonates Does not
Turns blue litmus paper red Neutral to litmus
Not poisonous Poisonous
Denser than air Less dense than air
Is an acidic gas Is not acidic
Does not burn Burns with a blue flame
No action on oxides Reduces metallic oxides
Reacts with heated charcoal to form carbon No action on charcoal
monoxide

Exercise

1)
a) Name the element present in pure charcoal
b) Explain why it is dangerous to use charcoal stove in a poorly ventilated room.
c) Write an equation for the reaction between charcoal and heated iron (III) oxide.
2) The figure below shows an experimental setup to investigate the effect of carbon monoxide
on oxides of metals.
a)
i) State the conditions for the reaction taking place in the combustion tube.
ii) Write the equation for the reaction taking place in the combustion tube.
b)
i) Name the gas X being burnt at the jet.
ii) Why is it necessary to burn gas X?
iii) Write equation for the combustion of gas X.
c) Name any other oxide that can be used instead of lead(II) oxide.
d) What would you expect to happen if lead (II) oxide was replaced with magnesium oxide?
Give a reason for your answer.
20.2. Carbonates and hydrogen carbonates
Carbonates

Carbonates are salts derived from carbonic acid (H2CO3). Aluminium carbonate does not exist.

Summary of properties of carbonates

Potassium Soluble in water


Sodium
Do not decompose on heating

Calcium
Magnesium
Zinc Insoluble in water
Iron Decompose on heating to form metal oxides and carbon dioxide
Lead
PbCO3(s) PbO(s) + CO2 (g)
Copper

Effects of heat on carbonates

Carbonates of potassium and sodium are not decomposed by heat. It is only lithium carbonate in
group I that decomposes on heating.

Li2CO3(s) Li2O(s) + CO2(g)

Carbonates of calcium, magnesium, zinc, iron, lead and copper are decomposed by heat to an
oxide and carbon dioxide.

When a white solid (powder) of lead (II) carbonate is heated strongly in a test-tube, a
colourless gas which turns lime-water milky is given off and a brown residue of lead (II) oxide
when hot and yellow when cold is formed.

PbCO3(s) PbO(s) + CO2(g)


Brown residue when hot
Yellow residue on cooling

When a green solid (powder) of copper (II) carbonate is heated, a black residue of copper(II)
oxide is formed.

CuCO3(s) CuO(s) + CO2(g)

Black residue
When a white solid (powder) of zinc carbonate is strongly heated, a yellow residue when hot
and white when cold is formed

ZnCO3(s) ZnO(s) + CO2(g)

Yellow residue when hot


White residue on cooling
When a green solid (powder) of iron (II) carbonate is heated, a brown residue of copper(II)
oxide is formed.

FeCO3(s) FeO(s) + CO2(g)

Brown residue
White solids of magnesium carbonate and calcium carbonate decompose to white solids of
magnesium oxide and calcium oxide respectively.

MgCO3(s) MgO(s) + CO2(g)

CaCO3(s) CaO(s) + CO2(g)

Ammonium carbonate sublimes when heated. The cause of this sublimation is that ammonium
carbonate dissociates on heating to ammonia, water and carbon dioxide, which recombine on
cooling.

(NH4)2CO3(s) 2NH3(g) + CO2(g) + H2O(l)

Solubility of carbonates in water


Carbonates of lithium, potassium, sodium and ammonium are soluble in water. The other
carbonates are insoluble in water.

Testing for carbonate (CO32-)

Add a dilute hydrochloric or sulphuric or nitric acid to the solution or solid to be tested.
Effervescence with liberation of a colourless gas that turns lime-water milky indicates the
presence of a carbonate (CO32-) or a hydrogen carbonate.
2H+ (aq) + CO32- (aq) H2O(l) + CO2 (g)

Hydrogen carbonates

These are salts derived from carbonic acid and are formed by partial replacement of hydrogen in
the acid by a metal. Therefore hydrogen carbonates are acidic salts. Common hydrogen
carbonates include sodium hydrogen carbonate (NaHCO 3) and calcium hydrogen carbonate
(Ca(HCO3)2)

All hydrogen carbonates are soluble in water

Hydrogen carbonates are decomposed by heat to produce carbonates, carbon dioxide and water

Ca(HCO3)2(s) CaCO3(s) + CO2(g) + H2O (l)

2NaHCO3(s) Na2CO3(s) + CO2 (g) + H2O (l)

2KHCO3(s) K2CO3(s) + CO2 (g) + H2O (l)

What happens when sodium hydroxide is exposed to air?

When sodium hydroxide is exposed to air, it absorbs water forming a solution. The solution then
absorbs carbondioxide from the air and forms a crystalline solid of washing soda (sodium
carbonate decahydrate Na2CO3.10 H2O)

2NaOH(s) + CO2 (g) + 9H2O (l) Na2CO3.10 H2O(s)

On further exposure, the hydrated sodium carbonate decahydrate loses its water of crystallisation
forming a white powder of sodium carbonate monohydrate
Na2CO3.10 H2O(s) Na2CO3.H2O(s) + 9H2O (l)

This white powder later absorbs carbon dioxide to form sodium hydrogen carbonate

Na2CO3.H2O(s) + CO2 (g) 2NaHCO3(s)

Distinguishing between a carbonate and a hydrogen carbonate

Add magnesium sulphate or magnesium chloride solution to the test solution. A white precipitate
indicates the presence of a carbonate.

Mg2+(aq) + CO32-(aq) MgCO3(s)

Mg2+(aq) + 2HCO3-(aq) Mg(HCO3)2(aq)

Hydrogen carbonates gives no precipitate but on heating, the magnesium hydrogen carbonate
decomposes to the insoluble magnesium carbonate (white precipitate).

Mg(HCO3)2(aq) MgCO3(s) + H2O(l) + CO2(g)

Exercise

1. Element X belongs to group II in the periodic table.


(a) Write the formula of the carbonate of X.
(b) The carbonate of X was heated strongly until no further change.
(i) Write the equation for the reaction.
(ii) Name one reagent which can be used to identify the gaseous product.
(c) The residue in (b) was added to excess dilute nitric acid and warmed.
(i) State what would be observed.
(ii) Write the equation for the reaction.

2. The figure below shows an experimental setup to investigate the effect of heat on lead (II)
carbonate.
(a) Write the equation for the reaction taking place in test-tube W.
(b) State what is observed in test-tube Q.
(c) What is observed in test-tube Q if lead (II) carbonate is replaced with sodium carbonate?
Give a reason for your answer.

3. (a) Write the equation for the reaction that would take place if
(i) Dilute hydrochloric acid is added to sodium hydrogen carbonate.
(ii) Sodium hydrogen carbonate is strongly heated.
(b) State what would be observed and write equation for the reaction that would take
place if magnesium sulphate solution is added to a solution containing

(i) Carbonate ions.

(ii) Hydrogen carbonate ions.

20.3. Calcium Oxide (quicklime)


Manufacture of calcium oxide

Calcium oxide is manufactured mainly from limestone, which is heated to very high
temperatures in a kiln. The limestone is mixed with coke or coal and it is fed into the kiln at the
top. Coke or coal burns and the heat decompose the limestone into the oxide and carbon dioxide.

CaCO3(s) CaO(s) + CO2(g)

The lime sinks to the bottom of the kiln and is removed; carbon dioxide is allowed to escape.
It can also be obtained by strongly heating sea shells. Sea shells contain calcium carbonate which
decomposes into the oxide and carbon dioxide.

Uses of calcium oxide

1. Used in extraction of iron, to remove silica impurities.


2. Used in production of lime-water.
20.4. Sodium carbonate (soda ash)
Laboratory preparation of sodium carbonate

Dilute hydrochloric acid is poured on to calcium carbonate.

Effervescence occurs and the gas (carbon dioxide) produced is passed through water to remove
traces of acid.

CaCO3(s) + 2HCl(aq) CaCl2(aq) + H2O(l) + CO2(g)

Then carbon dioxide is passed into a moderately concentrated solution of sodium hydroxide for
some time until finally a white precipitate of sodium hydrogen carbonate appears.

2NaOH(aq) + CO2(g) Na2CO3(aq) + H2O(l) (1st stage)

Na2CO3(aq) + H2O(l) + CO2(g) 2NaHCO3(s) (2nd stage)

The white precipitate is filtered off and washed two or three times with cold water. The solid is
transferred into a dish and heated to a constant mass. Sodium carbonate is obtained as a fine
white powder.

2NaHCO3(s) Na2CO3(s) + H2O(l) + CO2(g)

Manufacture of soda ash (sodium carbonate) by solvary process


Soda ash is obtained at Lake Magadi in Kenya by the solvary process.

The raw materials in this process are calcium carbonate (limestone) and sodium chloride in form
of brine. The ammonia dissolves in sodium chloride.

The mixture is reacted with carbondioxide down a large tower called the carbonator in which
there is an upwards flow of carbon dioxide (from decomposition of calcium carbonate) under
pressure. Sodium hydrogen carbonate is produced

NH3(g) + CO2(g) + H2O(l) NH4HCO3(aq)

NH4HCO3(aq) + NaCl(aq) NaHCO3(s) + NH4Cl(aq)

Sodium hydrogen carbonate precipitates in the lower part of the tower in form of a wet sludge,
which is tapped off from the bottom of the tower.

After filtration and washing to remove ammonium compounds, sodium hydrogen carbonate is
heated to produce sodium carbonate.
2NaHCO3(s) Na2CO3(s) + CO2(g) + H2O(g)

Carbon dioxide is recycled for use. Ammonia is recovered from the ammonium chloride by
reacting ammonium chloride with calcium hydroxide, obtained by adding water to calcium oxide
(from decomposition of calcium carbonate). Ammonia is recycled for use.

CaO(s) + H2O(l) Ca(OH)2(s)

2NH4Cl(aq) + Ca(OH)2(s) 2NH3(g) + CaCl2(aq) + 2H2O(l)

Therefore, the end products of solvary process are calcium chloride and sodium carbonate.
Calcium chloride is used in extraction of sodium.

Uses of sodium carbonate

1. It is used for softening of water for domestic purpose. Calcium ions which are the
principal cause of hardness in water; are precipitated from water as calcium carbonate by
the addition of sodium carbonate.
Ca2+ (aq) + CO32- (aq) Ca CO3 (s)

2. It is used in manufacture of glass. Ordinary glass is made by mixing calcium carbonate,


silicon dioxide (sand) and sodium carbonate together and melting them. The resulting
mixture of sodium and calcium silicates produces glass on cooling.
3. It is used to make dry soap powders.

Washing soda

Washing soda is sodium carbonate decahydrate (Na 2CO3.10H2O).When exposed to air, the
crystals lose mass and become coated with a fine white powder. Each molecule of washing soda
gives up, to the atmosphere, nine molecules of water of crystallization forming sodium carbonate
monohydrate (Na2CO3.H2O).

Na2CO3.10H2O(s) Na2CO3.H2O(s) +9H2O(l)


Such an action, that is, the giving up of water of crystallization to the atmosphere is termed as
efflorescence.

Washing soda is used for softening water by precipitating the calcium ions from solution as
calcium carbonate.

Ca2+(aq) + CO32-(aq) CaCO3(s)

Exercise

1. (a) State what would be observed if sodium carbonate solution was added to
(i) Aqueous calcium hydroxide.
(ii) Dilute sulphuric acid.
(b) Write ionic equations for the reactions in (a) (i) and (ii).

2. A mixture containing copper (II) sulphate and copper (II) carbonate was shaken with
water and filtered.
(a) Identify the residue.
(b) To the residue was added dilute sulphuric acid.
(i) State what was observed.
(ii) Write the equation for the reaction.

20.5. The Carbon cycle


Carbon cycle is the balancing of carbon dioxide in air i.e. describes the processes that increase or
decrease the carbon dioxide concentration in the environment (atmosphere). The atmosphere
contains about 0.03% of carbondioxide by volume and this volume is kept almost constant.

Processes that add carbondioxide to the atmosphere

1) Combustion: Carbon and its compounds burn in air to produce carbon dioxide e.g. burning
of coke, coal, wood, petrol, oils etc.
C(s) + O2 (g) CO2 (g)
2) Respiration: When sugars are oxidized in the body, carbondioxide is produced

C6H12O6 (l) + 6O2 (g) 6CO2 (g) + 6H2O (l)


3) Thermal decomposition of calcium carbonates: Carbon dioxide passes into air when
limestone or chalk is heated.
CaCO3 (s) CaO (g) + CO2 (l)
4) Fermentation: in the manufacture of ethanol during fermentation carbon dioxide is
produced as a bi-product.

C6H12O6 (l) 2C2H5OH (l) + CO2 (g) + Energy

Processes that remove carbondioxide from the atmosphere

1) Photosynthesis: Green plants absorb carbon dioxide from the atmosphere to make their own
food

6CO2 (g) + 6H2O (l) C6H12O6 (l) + 6O2 (g)

2) Hardening of mortar: Mortar and white ash remain slaked lime which slowly absorbs
carbon dioxide is produced

Ca(OH)2 (aq) + CO2 (g) CaCO3 (s) + H2O (l)

3) Solution in water: Rain dissolves carbon dioxide to form a weak acid (carbonic acid) which
runs into rivers, lakes, seas and oceans
Exercise

1. (a) Zinc carbonate was strongly heated in a test-tube until no further change.
(i) State what was observed.
(ii) Write the equation for the reaction which took place.
(b) The residue formed in (a) above was added to dilute sulphuric acid and heated.

(i) Write the equation for the reaction.

(ii) State what was observed.

2. (a) Define allotropy.


(b) Give the three allotropes of carbon.

(c) Give two examples of other elements which show allotropy and name their allotropes.

3. (a) Name two common reagents used in the laboratory preparation of carbon dioxide.
(b) State what is observed when carbon dioxide is bubbled in fairly concentrated sodium
hydroxide solution for some time.

(c) Write the equation(s) of the reaction(s) that take(s) place.

4. (a) Describe the structure of graphite.


(b) Explain why graphite conducts electricity whereas diamond does not.

(c) State any two uses of diamond.

(d) Describe how you would show by a chemical test that graphite is made up carbon
atoms.

5. Carbon monoxide was passed over strongly heated copper (II) oxide.
(i) State what was observed.
(ii) Write the equation for the reaction.
(iii) Name any other oxide that shows similar reaction with carbon monoxide.
6. (a) Draw a well labeled diagram for preparation of sodium carbonate in the laboratory.
(b) (i) What is observed when washing soda (Na2CO3.10H2O) is exposed to atmosphere
for some time.

7. (a) Copper (II) carbonate was heated strongly until there was no further change.
(i) State what was observed.
(ii) Write an equation for the reaction.
(iii) Name one reagent which can be used to identify the gaseous product.
(b) Excess dilute sulphuric acid was added to the residue in (a) and the mixture warmed.

(i) State what was observed.

(ii) Write an equation for the reaction.

8. (a) (i) How can calcium oxide (quicklime) be obtained on large scale?
Diagram not required.

(ii) Write equation for the reaction that occurs.

(b) (i) What would be observed when fresh calcium oxide is added to water in a beaker?
(ii) Write equation for the reaction that would occur.

(c) Dilute hydrochloric acid was added to calcium oxide.

(i) State what is observed.

(ii) Write the equation for the reaction that occurs.

(a) To the product in (c) was added a solution of carbonate ions.


(i) State what would be observed.
(ii) Write the equation for the reaction that occurs.
(b) State any two uses of calcium oxide.

20.6. Hardness of water


Hard water is that which does not form lather (foam) readily with soap.

Hardness of water is due to presence of calcium ions (Ca 2+) or magnesium ions (Mg2+) present in
water

Soft water is that which readily forms lather with soap

There are two types of hard water

Temporary hard water


Permanent hard water
Temporary hard water
This is the water which becomes soft on boiling. Temporary hard water is due to the presence of
calcium hydrogen carbonate (Ca(HCO3)2) or magnesium hydrogen carbonate (Mg(HCO3)2)
dissolved in water

Treatment of temporary hard water

Temporary hard water can be softened by:

1) Boiling
When temporary hard water is boiled, it becomes soft. This is because boiling decomposes
calcium hydrogen carbonate or magnesium hydrogen carbonate to calcium carbonate or
magnesium carbonate respectively thus removing the calcium (Ca2+) ions or magnesium (Mg2+)
ions from water making it soft

Mg(HCO3)2(aq) MgCO3(s) + CO2(g) + H2O(l)

Ca(HCO3)2(aq) CaCO3(s) + CO2(g) + H2O(l)

2) Addition of washing soda (Na2CO3.10H2O)

This removes temporary hardness from water as it reacts with hydrogen carbonates ions
dissolved in water

Ca(HCO3)2(aq) + Na2CO3(aq) CaCO3(s) + 2NaHCO3(aq)

3) Addition of slaked lime to water

This precipitates calcium carbonate hence removing calcium ions from water making it soft

Ca(HCO3)2(aq) + Ca(OH)2(aq) 2CaCO3(s) + 2H2O(l)

4) Addition of aqueous ammonia

This also precipitates calcium ions out of water making it soft

Ca(HCO3)2(aq) + NH4OH (aq) CaCO3(s) + NH4 HCO3(aq) + H2O(l)

5) Addition of naturally occurring zeolite or the manufactured permutit (sodium


aluminium silicate (Na2Al2Si2O8.xH2O)

This removes calcium (Ca2+) ions by precipitation of calcium aluminium silicate


(CaAl2Si2O8.xH2O)

CaSO4(aq) + Na2Al2Si2O8.xH2O(aq) CaAl2Si2O8.xH2O(s) + Na2SO4(aq)

Advantages of hard water

1) Strengthens teeth and bones since it contains calcium


2) Allows for use of cheap lead pipes for water supply. Does not cause lead poisoning
3) Necessary for formation of animal shells. Some aquatic animals need calcium to form their
shells e.g. water snails.

Disadvantages of hard water

a. Leads to wastage of soap as it needs much soap before it forms lather. Initially soap is
used in removing calcium sulphate from water before a lather is formed
b. Produces scum which leaves dirty marks on clothes
Scum can also damage silk and nylon clothes.
Scum: is a solid precipitate formed when hard water reacts with soap. Scum is chemically
called calcium stearate (C17H35COOCa)
C17H35COONa(aq) + CaSO4(aq) (C17H35COO)2Ca(s) + Na2SO4(aq)
c. Leaves fur in kettles and pans in which it is boiled. This fur is a poor conductor of heat
and therefore wastes fuel or energy.
Fur: fur is solid calcium carbonate or magnesium carbonate formed inside kettles or pans
d. It forms boiler scales inside boilers; this is also a coat of calcium carbonate or magnesium
carbonate formed inside boilers. Both fur and boiler scales waste fuel as heat cant pass
through them easily.

21.
22. Electrolysis
Electrolysis is the decomposition of an electrolyte in aqueous solution or molten state by passing
an electric current through it.

Definitions

a. Conductors and non conductors

Conductors: These are substances that allow electricity to pass through them

In electrolytes, the conducting particles are called ions while in metals the conducting particles
are electrons.

Some substances do not conduct electricity in solid state e.g. solid sodium chloride or gaseous
state e.g. hydrogen chloride gas but conduct well in aqueous (solution) or molten form. This is
because in solid, the compounds consist of ions held together by strong forces of attraction but
ions separate in molten or solution form and can move freely.

Non-conductors/Insulators: These are substances that do not allow the passage of electricity
through them

b. Electrolytes and non-electrolytes

An electrolyte is an ionic compound which conducts an electric current in aqueous solution or in


molten state and is decomposed by it.

Electrolytes are composed of ions. In the solid state, the ions are rigidly held in regular positions
and are not able to move freely. Melting the solid breaks the forces between the ions and
therefore the ions are free to move in a molten electrolyte. Dissolving a solid in water or any
other polar solvent, causes the breakdown of the lattice setting the ions free in aqueous state.

Types of electrolytes

Strong electrolytes

Weak electrolytes
Non electrolytes

Strong electrolyte: This is a compound which is completely ionised in dilute solution and in
molten state e.g. salts such as sodium chloride and mineral acids e.g. hydrochloric acid

HCl(aq) H+(aq) + Cl-(aq)

NaCl(aq) Na+(aq) + Cl-(aq)

Weak electrolyte: This is a compound which is only slightly ionized in dilute solution and in the
molten state. They have very few mobile ions and therefore slightly conduct an electric current
e.g. water, carbonic acid, ethanoic acid and ammonia solution.

H2CO3(aq) 2H+(aq) + CO32-(aq)

CH3COOH(aq) H+(aq) + CH3COO-(aq)

H2O(l) H+(aq) + OH-(aq)

NH4OH(l) NH4+(aq) + OH-(aq)

Non electrolyte: This is a solution or molten compound which cannot be decomposed by an


electric current e.g. sugar, alcohols, benzene and most organic compounds

A non-electrolyte is a solution or a molten compound which does not conduct electricity and
therefore cannot be decomposed by an electric current e.g. paraffin, sugar solution, ethanol etc.
Non-electrolytes exist only in the form of molecules and are incapable of ionisation. The
molecules have no charge and are therefore not able to carry an electric current.

Electrodes

These are poles of carbon (graphite) or metal where current enters and leaves the electrolyte. The
types of electrodes include;

Cathode: This is a negative electrode at which electrons enter the electrolyte or leave the
external circuit
Anode: This is the positive electrode at which the electrons leave/the electrolyte or enter the
external circuit.

NB: An electrode must be a good conductor of electricity and should not react with the
electrolyte.

A simple arrangement during electrolysis

Ions

An ion is a charged particle. Types of ions include:

Cation: This is a positively charged ion that will move to the cathode during electrolysis e.g. all
metallic ions e.g. Na+, NH4+, H+, Cu2+, Pb2+ etc.

Anion: This is a negatively charged ion that moves to the anode during electrolysis e.g. all non-
metal ions and radicals e.g. Cl-, SO42-, OH-, NO3-, Br- etc.

Faradays

This is the quantity of electricity carried by one mole of electrons and is equal to 96500
coulombs (C)

(1Faraday = 1mole of electrons =96500C)

Exercise

The shows the electrical conductivity of substances W, X and Y

Substance Solid state Molten state Aqueous solution

W Conducts Conducts Not soluble


X Doesnt conduct Conducts Conductors

Y Doesnt conduct Doesnt conduct Not soluble

(a) Which of the substances is likely to be copper(II) sulphate? Explain.


(b) Give the type of bonding that is present in substance
(i) W.

(ii) X.

(iii) Y.

(c) Which one of the substances is likely to be plastic? Explain.


(d) What particles are responsible for conducting electricity in substance
(i) W?

(ii) X?

Theory of electrolysis (Ionic theory)

This states that electrolytes consist of ions which are positively and negatively charged particles
that move to different electrodes during electrolysis.

In ionic compounds, these charges are held together by electrostatic forces but in solution or
molten state, these ions are free to move. The positive ions move to the cathode and the negative
to the anode.

What happens to anions at the anode?

When an electric current is applied to the electrolyte, the negatively charged ions called anions
move to the positively charged electrode called the anode. Once there, they lose electrons to
become atoms and are said to be discharged i.e. 2Xn-(aq or l) X2 (g) + ne

What happens to cations at the cathode?


The positively charged ions called cations move to the negatively charged electrode called
cathode where they gain electrons and become atoms which are then said to be discharged i.e.
Mn+(aq or l) + ne M (s)

Selective discharge of ions

When two or more ions of similar charges are present under similar conditions in a solution e.g.
K+ and H+ or SO42- and OH-, one is preferentially selected for discharge. The selective discharge
depends on the following factors.

(a) Position of the metal or group in the electrochemical (activity) series


If there are two ions with the same charge in the solution, the least reactive ion is discharged
first. For example, in the electrolysis of sodium chloride solution, both Na+ and H+ (from water)
are present and migrate to the cathode. The H+ being less reactive than the Na+ is discharged first.

If Cu2+ and H+ ions are both present in solution, both migrate to the cathode but the H + being less
reactive than the Na+ is discharged first.

The electrochemical series for ions is given below.

Cations Anions

K+ SO42-

Na+ NO3-

Ca2+ Cl- Reactivity

Reactivity Mg2+ Br- Decreasing

Decreasing Al3+ I-

Zn2+ OH-

Fe2+

Pb2+
H+

Cu2+

Hg2+

Ag+

(b) The nature of electrodes


This factor sometimes influences the choice of ion discharge.

E.g. when a solution of copper (II) sulphate is electrolysed using copper electrodes, copper ions
are discharged at the cathode but neither SO 42- nor OH- are discharged at the anode. Instead the
anode dissolves.

Reaction at the cathode

Cu2+(aq) + 2e Cu (s)

Reaction at the anode

Cu (s) Cu2+(aq) + 2e

When copper (II) sulphate solution is electrolysed using the copper cathode and carbon anode,
the copper is discharged at the cathode and OH- is discharged at the anode.

Reaction at the cathode

Cu2+(aq) + 2e Cu (s)

Reaction at the anode

4OH- (aq) 2H2O(l) + O2(g)

E.g. Electrolysis of a solution of sodium chloride with mercury as a cathode and with platinum as
cathode. With platinum, the hydrogen ion is discharged in accordance with the order of the
activity series, sodium ion being higher in the series. The cathode product is hydrogen gas. If the
mercury cathode is used, there is a possibility of discharging sodium ion to form sodium
amalgam with mercury. This requires less energy than the discharge of hydrogen ions to form
hydrogen gas and so occurs in preference.

(c) Concentration of the ions


Increase of concentration of an ion tends to promote its discharge. For example in concentrated
sodium chloride solution, there are hydroxide ions and chloride ions but the concentration of the
chloride ions exceeds that of the hydroxide ions and therefore chloride ions are discharged. The
more concentrated ions are discharged in preference to ones which are less concentrated.

Electrolysis of dilute sulphuric acid

This is commonly called electrolysis of water.

Ions present:

From sulphuric acid are H+ and SO42-

From water are H+ and OH-.

Reaction at cathode:

The hydrogen ions migrate to the cathode, gain electrons and become hydrogen gas.

2H+(aq) + 2e- H2(g)

Reaction at the anode:

The hydroxide ions and sulphate ions migrate to the anode. The hydroxide ions being less
reactive than sulphate ions are discharged and oxygen gas is formed.

4OH-(aq) 2H2O(l) + O2(g) + 4e-

Overall equation
4H+(aq) + 4OH-(aq) 2H2(g) + O2(g)+ 2H2O(l)

Electrolysis of dilute sulphuric acid in an apparatus called a voltammeter

Note:

1. pH changes at the anode and cathode:

The acidity at the cathode decreases (pH increases) because the hydrogen ions are discharged as
hydrogen gas and therefore the concentration of hydrogen ions in solution decreases. At the
anode, the acidity increases (pH decreases). The discharge of hydroxide ions disturbs the ionic
equilibrium of water and therefore more water ionizes to restore it.

H2O(l) H+(aq) + OH-(aq)

Therefore the excess hydrogen ions produced, with incoming sulphate ions, is equivalent to
increased concentration of sulphuric acid. This means that the total acidity at anode and cathode
together remains constant. This implies that the final change is that water is decomposed to
produce hydrogen and oxygen. That is why it is called electrolysis of water.

2. Two volumes of hydrogen are produced at the cathode and one volume of oxygen
produced at the anode i.e. Hydrogen : Oxygen =2:1
Overall equation

4H+(aq) + 4OH-(aq) 2H2(g) + O2(g)+ 2H2O(l)

Electrolysis of molten lead (II) bromide

The bulb does not give out light while the lead (II) bromide is solid showing that no electric
current passes through the solid lead (II) bromide. As the lead (II) bromide melts, the bulb
gives out light. After a while, a brown colouration is observed at the anode and a shiny grey
solid (lead) is deposited at the cathode.

Reaction at the anode:

2Br-(1) Br2 (l) + 2e-

Reaction at the cathode:

Pb2+(l) + 2e- Pb(s)

1. Write equations which represent the discharge of the following ions at the
(a) Cathode
(i) Ag+(aq)

(ii) Cu2+(aq)

(iii) Al3+(aq)

(iv) Na+(l)

(b) Anode
(i) Cl-(aq)

(ii) OH-(aq)

(iii) I-(aq)

2. Write down the ions present in the following substances.


(a) Dilute sulphuric acid.
(b) Copper(II) sulphate solution.
(c) Molten sodium chloride.
(d) Concentrated sodium hydroxide solution.
(e) Glucose solution.
3. Write down the ions that migrate to the anode and cathode when an electric current is
passed through the following substances.
(a) Dilute hydrochloric acid
(b) Silver nitrate solution
(c) Molten magnesium chloride
(d) Sodium hydroxide solution

4. The table below shows the observations made when an electric current was passed
through two substances Q and Z.
Substance Observation

Molten Q Conducts an electric current and a brown


substance is deposited at the cathode.

Molten Z Conducts an electric current and it is not


decomposed.

(a) Which of the two substances would not conduct electricity in solid state? Explain.
(b) In what other state would you expect substance Q to conduct electricity?
(c) Name the particles that are responsible for conducting electricity in substance Q and
Z.
(d) Give the type of bonding that is present in substances Q and Z.

5.
a. Which ions would be discharged at the electrodes during electrolysis of a dilute
solution containing
(i) K+ and Mg2+?

(ii) Zn2+ and Ag+?

(iii) SO42- and OH-?

(iv) NO3- and OH-?

b. Write the equation for the discharge of the ions in (a) (i) and (iv).

Electrolysis of copper(II) sulphate solution (using copper electrodes active electrodes)


Ions present:

From copper (II) sulphate: Cu2+ and SO42-

From water: H+ and OH-

Reaction at cathode:

Copper (II) ions and hydrogen ions migrate to cathode. Copper (II) ions are discharged
because they are less reactive than hydrogen ions. Copper (II) ions gain electrons from the
cathode and copper is deposited. A brown layer of copper is deposited at the cathode and thus
the mass of the cathode increases.

Cu2+(aq) + 2e- Cu(s)

Reaction at anode:

Both the sulphate and hydroxide ions migrate to the anode but none loses its electrons.
Instead the copper anode itself loses electrons and as it does so, it becomes copper (II) ions
which dissolves in solution. The anode electrode dissolves and its mass decreases.

Cu(s) Cu2+(aq) + 2e-

Electrolysis of copper (II) sulphate solution (using copper cathode and platinum anode)
Ions present: From copper (II) sulphate: Cu2+ and SO42-

From water: H+ and OH-.

Copper (II) ions and hydrogen ions move to the cathode. Copper (II) ions being less reactive
than hydrogen ions are discharged. Copper (II) ions gain electrons and copper is deposited.
The blue colour of the electrolyte (copper (II) sulphate solution) fades as copper is deposited
because copper (II) ions are removed from the solution.

Cu2+(aq) + 2e- Cu(s)

Sulphate ions and hydroxide ions move to the anode. Hydroxide ions being less reactive than
sulphate ions are discharged by giving up their electrons. Bubbles of a colourless gas
(oxygen) are formed at the anode.

4OH- (aq) 2H2O(l) + O2(g) + 4e-

The overall equation is obtained by adding the two equations after multiplying the first
equation by 2, to obtain the same number of electrons in both equations.

2Cu2+(aq) + 4e- 2Cu(s)

4OH-(aq) 2H2O(l) + O2(g) + 4e-

Overall equation: 2Cu2+(aq) + 4OH- (aq) 2Cu(s) + 2H2O(l) + O2 (g)

Electrolysis of concentrated copper (II) chloride solution

(using carbon anode and copper cathode)

Copper (II) ions from copper (II) chloride and hydrogen ions from water migrate to the
cathode. Copper (II) ions are discharged because they are more concentrated than hydroxide
ions, thus chlorine gas (greenish yellow gas) is liberated.

Cu2+(aq) + 2e- Cu(s)


Chloride ions from copper (II) chloride and hydroxide ions from water move to the anode but
chloride ions are discharged because they are more concentrated than hydroxide ions, thus
chlorine gas (greenish yellow gas) is liberated.

2Cl- (aq) Cl2 (g) + 2e-

The overall equation is obtained by adding the two equations.

Cu2+ (aq) Cl2(g) Cu(s) + 2e-

However, if the copper (II) chloride solution is very dilute, some discharge of hydroxide ions
will also occur. As the copper (II) chloride solution is diluted, there will not be a point at
which chlorine ceases to be produced and oxygen replaces it. Instead, a mixture of the two
gases will come off, with the proportion of oxygen gradually increasing. The same case
arises in the electrolysis of sodium chloride solution and hydrochloric acid, because the same
anions are involved.

Summary of the effects of electrolysis

Solution Ions in solution Cathode of Anode of Product at Product at Observation


electrolyse cathode anode
d

Conc. HCl H+, OH-, Cl- Platinum or carbon Carbon Hydrogen Chlorine Solution becomes alkaline

Bubbles of colourless gas at the


cathode , greenish yellow gas at the
anode

NaOH Na+, H+, OH- Platinum Platinum Hydrogen Oxygen Bubbles of colourless gas at the
cathode and anode

Solution becomes neutral

Conc. NaCl Na+, H+, OH-, Cl- Platinum or carbon carbon Hydrogen Chlorine Bubbles of colourless gas at the
cathode, greenish yellow gas at the
anode

Solution becomes alkaline

NaOH is formed

Conc. NaCl Na+, H+, OH-, Cl- Mercury Carbon Sodium Chlorine Solution is diluted, grey metal at the
cathode, greenish yellow gas at the
anode

CuSO4 Cu2+, H +, OH-, Copper Carbon/Pt Copper Oxygen Blue colour fades
solution SO42-
Brown solid at the cathode

Bubbles of the colourless gas at the


anode

Solution becomes more acidic

H2SO4 is produced

CuSO4 Cu2+, H +, OH-, Copper Copper copper Copper No change in pH


2-
solution SO4 anode
dissolves Brown solid at the cathode and
Cathode mass increases

Anode mass decreases

Molten Pb2+, Br- Carbon Carbon Lead Bromine No change


PbBr2
Grey solid at the cathode

Brown liquid at the anode

Dilute H+, OH-, SO42- Pt Pt Hydrogen Oxygen Total acidity at cathode and anode
H2SO4 reduces, Bubbles of colourless gas at
the cathode and anode

Conc. CuCl2 Cu2+, H+, OH-, Cl- Carbon Carbon Copper Chlorine Solution is diluted

Brown solid at the cathode

Greenish yellow gas at the anode

Conc. CuCl2 Cu2+, H+, OH-, Cl- Copper Copper Copper Copper No change in pH
anode
dissolves Brown solid at the cathode and
Cathode mass increases

Anode mass decreases

Exercise

1. Acidified water was electrolysed using platinum electrodes.


(a) Write an equation for the reaction that took place at the
(i) Anode
(ii) Cathode

(b) Name one other substance that can be used as electrode in the electrolysis of acidified
water.
2. An aqueous solution of silver nitrate solution was electrolysed using platinum electrodes.
(a) Write an equation for the reaction at
(i) Anode

(ii) Cathode

(b) Write the overall equation for the reactions taking place at anode and cathode.
(c) Write the equations for the reactions at anode and cathode is silver nitrate solution
was electrolysed using silver electrodes.
3. Dilute hydrochloric acid was electrolysed using carbon electrodes.
(a) Name the product(s) formed at the anode.
(b) Write the equation for the reaction at cathode.
4. An aqueous solution of zinc nitrate was electrolysed using platinum electrodes. Write an
equation for the reaction at
(i) Anode

(ii) Cathode.

5. A concentrated solution of copper(II) chloride was electrolysed using copper anode and
copper cathode. Write an equation for the reaction at
(i) Anode

(ii) Cathode

22.1. Laws of electrolysis


The laws of electrolysis were stated by Faraday. According to his laws, the amount of substance
produced during electrolysis depends on the

(i) Time of passing a steady current (t).


(ii) Magnitude of the steady current (I).

(iii) Charge on the ion of the element (n).

Faradays first law

It states that the mass of a substance deposited at the electrodes is directly proportional to the
quantity of electricity passed. This can be illustrated by the graph below.

Mass of

substance

deposited

(g)

Quantity of electricity (c)

Quantity of electricity = current x time

i.e. Q = I x t.

Therefore, the quantity of electricity can be found by measuring the current (I) in amperes and
the time (t) in seconds for which it flows. The unit for quantity of electricity is a coulomb (C).
One coulomb is equivalent to one ampere of current flowing for one second.

Exercise

Calculate the quantity of electricity used when a current of

(i) 2 A is passed through an electrolyte for 20 minutes.


(ii) 10 A is passed through an electrolyte for 2 hours.

(iii) 12 A is passed for 2 minutes and 20 seconds.

Faradays second law

It states that the mass of a substance deposited at the electrodes is inversely proportional to the
charge on its ion. For example, if the same quantity of electricity is separately passed through a
solution of silver ions and copper (II) ions, it is found that the number of moles of silver
deposited are twice the number of moles of copper deposited.

Faraday

This is the quantity of electricity required to deposit one mole of a substance from an ion with a
single charge. One mole of silver ions requires one faraday (1 mole of electrons) to discharge at
the cathode.

Ag+(aq) + e- Ag(s)

One mole of copper(II) ions requires 2 faradays (2 moles of electrons) to discharge at the
cathode.

Cu2+(aq) + 2e- Cu(s)

One mole of aluminium ions requires 3 faradays to discharge at the cathode.

A13+(aq) + 3e- A1(s)

Two moles of chloride ions require 2 faradays to discharge as one mole of chlorine molecules at
the anode.

2Cl-(aq) Cl2(g) + 2e-

From the above examples, one mole of electrons is equivalent to 1 faraday. 1 faraday is
equivalent to 96500 coulombs.
Example I: Quantity of electricity

A current of 4 amps was passed through a solution of magnesium sulphate solution for 5
minutes. Calculate the quantity of electricity

(i) Required to deposit one mole of magnesium.

(ii) Used.

Solution:

(i) Mg2+(aq) + 2e- Mg(s)

2 faradays are required to liberate 1 mole of magnesium.

Quantity of electricity required to liberate 1 mol of magnesium

= 2 x 96500 = 193000 C

(ii) Q = It

Q = 4 x 5 x 60 = 1200 C

Example II: Atomic mass of a substance deposited

When a current of 0.45 amps was passed through a solution of copper (II) sulphate for 1500
seconds, 0.222 g of copper were deposited. Calculate the relative atomic mass of copper.

Solution:

Q = It

Q = 0.45 x 1500 = 675 C

Cu2+(aq) + 2e- Cu(s)

2 faradays are required to liberate 1 mol of copper.

Quantity of electricity required to liberate 1 mol of copper.


= 2 x 96500 = 193000 C.

675 C deposit 0.222 g of copper.

1 C deposits 0.222 g of copper.

675

193000 C deposit 0.222 x 193000 g of copper.

675

Thus the relative atomic mass of copper is 63.5.

Example III: Moles of a substance deposited

A current of 10 amps is passed through molten magnesium chloride for 4 hours. How many
moles of magnesium metal are produced by this electrolysis?

Solution:

Q = It

But t = 4 x 60 x 60

Q = 10 x 4 x 60 x 60

Q = 144000 C

Mg2+(aq) + 2e- Mg(s)

2 faradays liberate 1 mol of magnesium.

So 2 x 96500 C produces 1 mol of magnesium.

1 C produces 1 mol of magnesium.

2 x 96500

144000 C would produce 1 x 144000 mol of magnesium.


2 x 96500

0.746 mol of magnesium

Exercise

1. How many coulombs are required to discharge the following ions.


(i) Zn2+

(ii) Na+

(iii) Fe3+

(iv) Pb2+

(v) OH-

(vi) H+

(vii) Br-

2. Molten aluminium chloride was electrolysed for 200 seconds using a current of 40A.
(a) Write the equation for the reaction at
(i) Cathode.

(ii) Anode.

(b) Calculate the quantity of electricity used.


(c) Calculate the quantity of electricity in coulombs required to deposit one mole of
(i) Aluminium

(ii) Chlorine molecules.

3. Calculate the mass of aluminium deposited when a current of 960 A passes through a
solution of aluminium (III) oxide in fused cryolite for 800 seconds.
4. What volume of oxygen measured at s.t.p would be liberated in electrolysis of dilute
sulphuric acid by 96000 coulombs. (1 mole of gas occupies 22.4 litres at s.t.p.)
22.2. Application of electrolysis
(i) Electroplating

This is the process of coating a metal with another metal by the process of electrolysis.
Electroplating is done to protect metals from corrosion and to improve their appearance. The
metal to be plated is made the cathode in a suitable electrolyte containing ions of the plating
material. For example, during silver plating, the metal to be plated is made the cathode in a silver
salt solution as an electrolyte and pure silver is made the anode.

The silver salt solution contains positively charged silver ions which are attracted to the cathode
(metal to be plated). Once there, they gain electrons to form silver atoms.

Ag+(aq) + e+ Ag(s)

The anode (plate of pure silver) loses its electrons and forms silver ions which dissolve in the
solution to replace the ones moving to the cathode. The process continues until an adequate layer
of silver has been deposited on the metal being plated.

Other metals which can be used to coat other metals include chromium, nickel, copper and gold.
When iron is to be chromium plated, it is first electroplated with nickel to prevent corrosion and
then with chromium.
For successful electroplating, the material to be plated should be clean and the electric current,
temperature and concentration of the electrolyte should be exactly right. When a very low current
is used, electrolysis proceeds very slowly and a very smooth deposit can be obtained.

Exercise

1)
(a) Name the substance that can be used as the anode and cathode during
(i) Chromium plating of iron metal.

(ii) Nickel plating of steel.

(b) Name the positive ions that must be present in the electrolyte used in a(i) and (ii).
(c) Write the equation for the reaction taking place at cathode in a(ii)
[valency of nickel (Ni) = 2]

(ii) Anodizing

Anodizing is the electrolytic process of coating objects made of aluminium with a very thin
oxide film to protect the metal from corrosion.
To ensure a very thin film of oxide, the oxidation is carried out by electrolysis, using the
aluminium object as the anode. The electrolyte is usually dilute sulphuric acid which gives
oxygen at the anode on electrolysis (refer to electrolysis of dilute sulphuric acid).

4OH-(aq) 2H2O(l) + O2(g) + 4e-

Under the correct conditions, oxygen reacts with the surface of the aluminium and coats it with a
thin invisible but protective coating of aluminium oxide.

4Al(s) + 3O2(g) 2Al2O3(s)

(iii) Purification of metals


Metals such as copper and zinc may be refined, that is purified by electrolysis. The impure metal
is made the anode and the pure metal the cathode. The electrolyte is a solution containing the
metal.

(iv) Extraction of metals


Reactive metals such as aluminium and sodium are extracted by electrolysis of the fused
electrolytes

(v) Manufacture of chemicals


The most important example is the manufacture of sodium hydroxide, chlorine and hydrogen
using the flowing mercury cathode cell. In the manufacture of sodium hydroxide, concentrated
chloride solution (brine) is electrolysed using a graphite (carbon) anode and a flowing mercury
cathode. The mercury cathode enables sodium ions to discharge in preference to hydrogen ions.
Sodium is deposited at the cathode and chlorine is liberated at the anode.

Na+(aq) + e- Na(s)

2Cl+(aq) Cl2(g) + 2e+

The sodium formed dissolves in the mercury cathode to form a solution called sodium
amalgam. The amalgam is mixed with water producing sodium hydroxide solution, hydrogen
and pure mercury.
2NaHg(l) + 2H2O(l) 2NaOH(aq) + H2(g) + 2Hg(l)

Sodium amalgam

Mercury is then recycled by use of a pump. Hydrogen is collected as a by-product. Sodium


hydroxide solution is evaporated to dryness.

Sodium hydroxide is used in the manufacture of soap, in the textile industry (bleaching and
dyeing processes, in rayon manufacture and mercerising cotton to give it a silky sheen), in the
purification of bauxite for aluminium extraction and in the preparation of phenols and cresols
from coal-tar.

Exercise
1. Copper (II) sulphate solution was electrolysed using the set up shown in the figure below.

(i) Name electrodes A and B

(ii) Write down all the ions present in aqueous copper (II) sulphate.

(iii) Describe what happens to each electrode during electrolysis.

(i) Write an equation for the reaction that took place at A and B.
(ii) Comment on the colour of the solution after electrolysis.

2. The figure below shows electrolysis of dilute sulphuric acid using carbon electrodes. A
current of 11.0 amps was passed for 5 minutes and 20 seconds through the circuit.
(a) Name gas
(i) X.

(ii) Y.

(b) Write the equation for the reaction at


(i) Anode.

(ii) Cathode.
(c) Calculate the volume of gas Y produced at room temperature.
(IF = 96500 C, 1 mole of gas at room temperature occupies 24 dm3)

3. (a) Molten lead(II) bromide was electrolysed between carbon electrodes.


(i) State what was observed at the cathode and anode.

(ii) Write an equation for the reaction that took place at each electrode.

(b) Calculate the mass of the product formed at the cathode when a current of 2 amps is
passed for 1 hour and 30 minutes. (Pb = 207, Br = 80)

4. (a) What is meant by electroplating?


(b) Draw a labeled diagram of an apparatus that can be used in the laboratory to copper
plate an object.

5. A current of 0.25 amps was passed through copper(II) sulphate solution for 40 minutes.
Calculate the
(a) Quantity of electricity used.
(b) Quantity of electricity which deposits one mole of copper.
(c) Mass of copper deposited during the electrolysis.
(d) Moles of copper deposited. (Cu = 64, 1 F = 96500 C)
6. The circuit shown in the figure below was used in an experiment to study the effect of
electricity on lead (II) bromide.
(a) State what was observed.
(i) Before lead(II) bromide had melted.

(ii) After lead(II) bromide had completely melted.

(b) Explain your answer in (a)


(c) Write equation for the reaction that took place at
(i) Y.

(ii) X.

7. Copper(II) sulphate solution was electrolysed using carbon electrodes.


(a) State what was observed at the
(i) Anode.

(ii) Cathode.

(b) Explain your answer in (a) (ii).


(c) Write equation(s) which took place at the anode.
(d) Write overall equation for the reactions.
8. The figure below shows an arrangement of the apparatus used for the purification of
copper.
(a) Name the substance used as
(i) Anode.

(ii) Cathode.

(b) Name the electrolyte.


(c) Write equation for the reaction that took place at
(i) X.

(ii) Y.

9. An aqueous solution of potassium iodide was electrolysed in a U-tube using carbon


electrodes. Iodide was formed at the anode and it dissolved to form a yellowish brown
solution around the electrode. At the cathode, bubbles of a colourless gas were seen to
evolve. The solution near the cathode had a pH of about 11.
(a) Explain with an equation how the change from iodide ions to iodine took place at the
anode.
(b) What was the gas evolved at the cathode?
(c) A solution of potassium iodide in water is neutral (pH = 7). Explain why the pH
increased to 11 near the cathode during electrolysis.
23. Formulae, stoichiometry and the mole concept

23.1. Relative Atomic Mass

Comparing Atomic Masses with the Carbon Atom


To compare to a carbon atom, a carbon-12 atom is used. The mass of the isotope is 12 times

1
greater than hydrogen atom so 12 of carbon-12 atoms is equivalent to the mass of one

hydrogen atom.

Relative Atomic Mass - the average mass of one atom of the element (averaging isotopes) when

1
compared with 12 mass of a carbon-12 atom.

Average mass of one atom of the element


Ar = 1
X The mass of one atom of carbon12
12

Note: The Relative Atomic Masses are already stated on the periodic table above each chemical
formula.

Relative Molecular Mass and Relative Formula Mass


Using the relative atomic masses (Ar), its possible to calculate Relative Masses of molecules
and ionic compounds

Relative Molecular Mass


Molecules contain atoms joined together, e.g. Cl2
Average mass (molecular mass) of Cl2= add relative atomic masses of both atoms.
Examples:
The molar mass of oxygen molecules (O2) =2 X O
=2X16
=32g
The molar mass of water molecules (H2O) =2 X H + 1 X O
=2X1+ 1 X 16
=18g

Relative Molecular Mass the average mass of one molecule of substance (averaging isotopes)

1
when compared with 12 mass of a carbon-12 atom.

Total mass of all atoms of a molecule


In short: Mr = = 1
X The mass of one atom of carbon12
12

Relative Formula Mass same as relative molecular mass but for ions or ionic compounds only
Relative Formula Mass total relative atomic masses of all atoms in a formula of ionic
compound
E.g. Relative formula mass of MgSO4
Mr = 24x1 + 32x1 + 4x16 = 120

Note: Relative molecular mass and relative formula mass have no units
Exercise
Calculate the relative formula masses or relative molecular masses of the following
compounds (H=1, Cl=35.5, Cu=64, S=32, O=16, Na=23, C=12, Fe=56, Zn=65, N=14,
Pb=207, Ag=108 )
a. Hydrogen chloride
b. Copper (II) sulphate
c. Sodium hydroxide
d. Sodium carbonate
e. Iron (II) sulphate
f. Copper (II) chloride
g. Zinc nitrate
h. Lead (II) carbonate
i. Silver chloride
j. Copper (II) sulphate pentahydrate
k. Sodium carbonate decahydrate

23.2. Percentage Composition


atomicmass of an element
Percentage by mass = ( RMMorRFM of a compound x 100)%

E.g.

a. Determine the percentage composition of each element in sodium chloride

RFM of NaCl = 23 + 35.5 =58.5

23
% of Na = 58.5 x 100 = 39.32%

b. Determine which oxides of iron of Fe2O3 or Fe3O4 has more iron.

Mr (Fe2O3) = 2(56) + 3(16) = 160


Percentage of Fe in Fe2O3
mass of Fe 2
=( Relative formula mass of Fe 2 O 3 x 100) %

2( 56)
=( 160 x 100) %
112
=( 160 x 100) %

= 70%
Mr (Fe3O4) = 3(56) + 4(16) = 232
3 (56)
Percentage of Fe in Fe2O3 = ( 232 x 100) %

= 72%
Fe3O4 has more iron composition than that of Fe2O3.

23.3. Calculating the Mass of an Element in a Compound


Use the example of Fe2O3 in the example above. The percentage mass of iron in iron (III) oxide
is 70%. Therefore to calculate mass of iron in a 200g compound of iron (III) oxide is (0.7 x
200)g = 140g
e.g. Determine the mass of iron in 200g of Fe2O3.
Mr(Fe2O3)= 2(56) + 3(16) = 160
2( 56)
Mass of Fe in Fe2O3 = ( 160 x 200g

2( 56)
=( 160 x 200g

= 140g

23.4. Calculating the Mass of Water in a Compound


Compound with water mass is hydrated and has H2O in their formula.
e.g. Calculate water mass in 12.5g hydrated copper sulphate, CuSO4.5H2O
Mass of water
Mass of 5H2O in CuSO4.5H2O = ( Relative formula mass of hydrated salt x mass of sample

RFM of CuSO4.5H2O = 64 + 32 + 4(16)+5(2+16) =250


5 (18)
=( 250 x 12.5g

= 4.5g

23.5. Mole
Counting Particles
Unit for particles = mole
Symbol = mol
1 mol = 6 x 1023 atoms

Moles of Particles
Calculating the Number of Moles
Number of particles
n= 6 x 10
23

E.g 1: How many moles are in 3 x 1024 molecules of water, H2O?


3 x 1024
n = 6 x 1023

= 5 mol

e.g 2: Calculate the number of molecules in 0.25 mole of CO2. Hence, how many atoms are
present?
Number of particles
0.25mol = 6 x 10
23

Number of particles = 0.25 mol x 6 x 1023


= 1.5 x 1023 molecules
Number of atoms = total number of atoms in CO2 x number of particles
= 3 x 1.5 x 1023
= 4.5 x 1023 atoms
23.6. Molar Mass
Molar mass the mass of one mole of any substance

For substances consisting of atoms


It is the mass of the element in grams. Eg. Ar(C) = 12, molar mass = 12g

For substances consisting of molecules


It is the mass of the substance in grams. Eg. Ar(H2O) = 18, molar mass = 18g

For substances consisting of ions


It is the mass of substance in grams. Eg. Ar(NaCl)= 58.5, molar mass= 58.5g

Calculations Using Molar Mass

Mass(m)
n = Relative formulaatomicmass ( Mr)

E.g. Find the mass of 0.4 mol of iron atom.


Mass
0.4 = 56

m = n x Mr
m = 0.4 x 56 = 22.4 g

Or
1 mole of iron weighs 56g
0.4mol of iron weigh 0.4 x 56 = 22.4 g
23.7. Different Kinds of Chemical Formulae
Ethene formula is C2H6
Molecular Formula shows the actual formula and kinds of atoms present, e.g. C2H6. It
expresses the composition of a compound showing the actual number of atoms in the compound.
Empirical Formula shows the simplest whole number ratio of the atoms present in a
compound, e.g. C2H6, ratio 1:3, therefore C1H3, simply CH3
Structural Formula shows how atoms are joined in the molecule. It can be represented by ball-
and-stick model or diagrammatically.

Ball-and-Stick Diagrammatic
a. Calculating the Empirical Formula of a Compound
Find the empirical formula of an oxide of magnesium consisting of 0.32g of oxygen and 0.48g of
magnesium. (Mg = 24, O = 32)
Solution
Elements present Mg O
Composition by mass 0.48 0.32
0.48 0.32
Number of moles 24 16

= 0.02 mol = 0.02 mol


0.02 0.02
Divide by the smallest number 0.02 0.02

1 1
Mg1O1
Therefore, the empirical formula is MgO
23.8. Calculating the Empirical Formula from Percentage Composition
An oxide of sulphur consists of 40% sulphur and 60% oxygen (S = 32, O = 16)

Solution
Elements present S O
Percentage composition 40 60
40 60
Number of moles 32 16

= 1.25 mol = 3.75 mol


1.25 3.75
Divide by the smallest number 1.25 1.25

1) 3
S1O3
Therefore, the empirical formula is SO3

23.9. From Empirical formula to Molecular Formula


Find the molecular formula of propene, CH2, having molecular mass of 42.
Solution Molecular formula will be CnH2n
Relative molecular mass = 12 n (from carbon Ar) + 2n (2 x hydrogen Ar) = 14n
14 n = 42
n=3
Therefore, C3H6
Solution
The complete combustion of 7.5g of an organic compound, Q containing carbon, hydrogen and
oxygen gave 17.8g of carbon dioxide and 9.27g of water. Calculate the simplest formula of Q.

Solution:

Molar mass of CO2 = 12 + 16 x 2 = 44g

44g of carbon dioxide contain 12g of carbon.


12
1g of carbon dioxide contains 44 g of carbon.

12 x 17.8
17.8g of carbon dioxide contains 44 g of carbon.

= 4.86g of carbon

Molecular mass of H2O = 1 x 2 + 16 = 18g

18g of water contain 2g of hydrogen.

2
1g of water contains 18 g of hydrogen.

2 x 9.27
9.27g of water contain 18 g of hydrogen

= 1.03g of hydrogen

Mass of oxygen = 7.5 (4.86 + 1.03) = 1.61g of oxygen

Elements present: C H O

Composition by mass: 4.86 1.03 1.61

Number of moles: 4.86 1.03 1.61

12 1 16

0.41 1.03 0.10

Mole ratio: 0.41 1.03 0.10

0.10 0.10 0.10

Simplest ratio: 4 : 10 : 1
Thus the simplest formula of Q is C4H10O.

Example

A compound Y contains 15.8% aluminium, 56.2% oxygen and 28% sulphur. (S=32, Al = 27 O
=16). (i) Calculate the empirical formula of Y.

(ii) The molecular mass of Y is 342g. Determine the molecular formula of Y.

Solution:

Elements present: A1 S O

Percentage composition: 15.8 28 56.2

Number of moles: 15.8 28 56.2

27 32 16

0.59 0.88 3.5

Mole ratio: 0.59 0.88 3.5

0.59 0.59 0.59

1 1.5 6

Multiply by 2 throughout 1x2 1.5 x 2 6x 2

to obtain whole numbers:

Simplest ratio: 2 : 3 : 12

The empirical formula of compound Y is given by Al2S3O12 or Al2(SO4)3.

ii) (Al2S3O12) n = 342

[(27 x 2) + (32 x 3) + (16 x 12)]n = 342


342n = 342

342 342

n = 1

The molecular formula of Y is Al2S3O12 or Al2(SO4) 3.

Exercise
1) A compound contains 43.4% by mass of sodium, 11.3% carbon and 45.3% oxygen.
Calculate the simplest formula of the compound (Na = 23, C = 12, O = 16)

2) A compound contains 40% carbon, 6.67% hydrogen, the rest being oxygen. The relative
molecular mass of the compound is 180 (C=12, H =1, O = 16). Determine the empirical
formula of the compound and the molecular formula of the compound

3) An oxide of an element X was made of 50% X. Calculate the simplest formula of the
oxide (X = 32, O = 16)

4) A compound of molar mass 400 with 28% iron, 48% oxygen and the rest being sulphur
was dissolved in water. Calculate the empirical formula and molecular formula of the
compound .(Ans =Fe2(SO4)3)

5) When hydrated sodium carbonate crystals (Na2CO3.xH2O) were exposed to air for a long
time, there was loss of mass of 62.9%. What is the amount of the water of
crystallisation? (Na=23, C=12, O=16, H=1)
Na2CO3.xH2O(s) Na2CO3(s) + xH2O(l) .(Ans x=10)
6) A white crystalline salt (Z.xH2O) contains 51.2% of water of crystallisation. If the
formula mass of the crystals is 120, calculate the amount of water of crystallisation.(Ans
x=7)

23.10. Molar Volume of Gases

The Avogadros Law


Equal volume of gases at the same temperature and volume contain equal number of particles or
molecules.

Molar Volume of Gas volume occupied by one mole of gas


One mole of all gases at standard temperature and pressure (s.t.p.) = 22.4dm3

One mole of all gases at room temperature and pressure (r.t.p.) = 24dm3
1dm3 = 1000cm3
Formulae:
Volume of a gas
Number of moles of a gas (n) = Molar volume

Volume of a gas = Number of moles (n) x Molar volume (Mr)

Example
What is the number of moles of 240cm3 of Cl2 at r.t.p.?
Solution
Since 24000cm3 of chlorine gas contain 1 mole
1
Then 1cm3 of chlorine gas contains 24000 moles

1
x 240
240cm3 of chlorine gas contain 24000 moles = 0.01 mol
Volume of a gas 1
x 240
Or Number of moles of a gas (n) = Molar volume = 24000 moles =

0.01 mol

Molar Volume and Molar Mass


Gases have same volume but not necessarily same mass
Example: 1 mole of hydrogen gas (H2) has a mass of 2g,
1 mole of Carbon Dioxide gas (CO2) has a mass of 44g
Example
Find the volume of 7g of N2 at r.t.p.
Solution
Step 1: Find the number of moles from the mass of nitrogen
Molar mass of N2 =2 x 14 =28g
Mass of a gas 7
Number of moles of a gas (n) = Molar mass of gas = 28 moles = 0.25 mol

Step 2: Find the volume of nitrogen, now with formula of gas


Volume of a gas = Number of moles (n) x Molar volume (Mr)

Volume of gas = 0.25 mol x 24


= 6 dm3 (or 6000cm3)

23.11. Calculations using chemical equations

23.11.1. Constructing Chemical Equations


E.g. Reaction Between Hydrogen and Oxygen
Word Equation: Oxygen + Hydrogen Water
To write the chemical equation, we use symbols of atoms/molecules:
O2 + H2 H2O
BUT THIS IS IMBALANCED! A BALANCED EQUATION MUST HAVE THE SAME
NUMBER OF ATOMS OF EACH ELEMENT ON BOTH SIDES! THEREFORE...
O2 + H2 H2O
O H H
O H H
O
From above, we know that H2O is short 1 oxygen atom. Therefore we multiply the product by 2
first. Note: all atoms in molecules are automatically multiplied by 2.
O2 + H2 2H2O
O H H
O H H
H
H
O
O
Now we can cancel off oxygen atoms. However, hydrogen atoms on the reactant side are short of
2 atoms. Therefore, we multiply the hydrogen molecule by 2 so that the short is balanced. The
equation is fully balanced when we are able to cancel off all atoms of that element on both sides.
O2 + 2H2 2H2O
O H H
O H H
H H
H H
O
O

23.11.2. Calculations from Equations

23.11.2.1. Reacting Masses


In every equation, each atom is rational to each other. Suppose we want to find moles of X atoms
that reacted to form 0.25 mole of Y atoms. We always put the atom we want to find as numerator
and the denominator being the atom we know.
E.g. X + 2Z 2Y
Find the ratio first:
Number of moles of X 1
=
Number of moles of Y 2

Then multiply the ratio by no. of moles of Y to find the reacting mole of X.

1
Number of moles of X = 2 x 0.25 = 0.125 mole

Therefore 0.125 mole of X reacted with 0.25 mole of Y.

To find the reacting mass of X, e.g. Y is given as 35g, we simply multiply the mole by the mass
of Y as they are always in ratio:
0.125 x 35 = 4.375 g

Example
Lead (II) nitrate reacts with potassium iodide according to the equation
Pb(NO3)2(aq) + 2KI (aq) PbI2(s) + 2KNO3 (aq)
Calculate the mass of lead (II) iodide that will be formed when 33.2g of potassium iodide reacts
with excess lead (II) nitrate (K=39, N=14, O=16, Pb=207, I=127)
Solution
Find the ratio first:
Number of moles of Lead ( II ) iodide 1
=
Number of moles of potassium iodide 2

RFM of potassium iodide (KI) =39 + 127 = 166


33.2
Moles of potassium iodide = 166 = 0.2moles

1
Number of moles of PbI2 = 2 x 0.2 = 0.1 mole
Therefore 0.1 mole of PbI2 reacted with 0.2 mole of KI.

The reacting mass of PbI2 = moles of PbI2 X RFM of PbI2


RFM of PbI2 = 207 + 2(127) = 461
Mass of PbI2 = 0.1 x 461
=46.1g
Exercise
1. Ammonium chloride reacts with calcium hydroxide according to the equation
Ca(OH)2(s) + 2NH4Cl (aq) CaCl2(s) + 2NH3(g) +2H2O (l)
If 14.8g of calcium hydroxide was reacted completely with ammonium chloride, what mass of
ammonia gas will be evolved? (H=1, N=14, O=16, Ca=40) (Ans = 6.8g of ammonia)

2. Calculate the loss in mass when 10g of calcium carbonate is heated to a constant mass
(Ca=40, C=12, O=16) (Ans =4.4g)
3. Calculate the mass of ammonium chloride that will just react completely with 14.8g of
calcium hydroxide (N=14, H=1, Cl=35.5) (Ans = 21.4g)

23.11.2.2. Reacting Masses and Volumes


First, find the ratio of moles and multiply the mole of the gas volume you want to find with the
volume of gas at room temperature (24dm3)
Example
MgCl2 is formed by reacting Mg and HCl according to equation:
Mg(s) + 2HCl (aq) MgCl2(s) + H2 (g)
Find the amount of hydrogen gas, in cm3, formed when 14.6g of HCl is reacted at room
temperature. (H=1, Cl=35.5)
Solution

Ratio: Find the ratio first:


Number of moles of H 2 1
=
Number of moles of HCl 2

Then multiply the ratio by no. of moles of HCl to find the reacting mole of H2.
RFM of HCl = 1 + 35.5 = 35.5
14.6
Moles of HCl = 36.5 = 0.4moles

1 1
Number of moles of H2 = 2 x moles of HCl= 2 x 0.4moles = 0.2 moles

To find the volume of H2, simply multiply the mole by the molar volume at room temperature:
Volume of H2 = moles of H2 x Molar gas volume at room temperature
= 0.2 x 24 dm3= 4.8 dm3
But 1dm3 = 1000cm3
4.8dm3 x 1000 = 4800 cm3
4 800cm3 of gas is formed

Exercise

1. Ammonium chloride reacts with calcium hydroxide according to the equation


Ca(OH)2(s) + 2NH4Cl (aq) CaCl2(s) + 2NH3(g) +2H2O (l)
If 14.8g of calcium hydroxide was reacted completely with ammonium chloride, what volume of
ammonia gas will be evolved? (H=1, N=14, O=16, Ca=40, I mole of gas occupies 24dm3 ) (Ans
= 9.6dm3 of ammonia)

2. Carbon burns in oxygen according to the following equation


C (s) + O2 (g) CO2(g)
Calculate the volume of carbondioxide collected at s.t.p when 10g of carbon is burnt (c =12, one
mole of a gas occupies 22.4l at s.t.p) (Ans = 18.6 litres)
3. When heated strongly, potassium nitrate decomposes according to the equation
2KNO3(s) 2KNO2(s) + O2(g)
Calculate the volume of oxygen evolved at s.t.p by heating 5g of potassium nitrate (K=39,
N=14, O=16, 1mole of a gas occupies 22.4litres at s.t.p)
4. 3.4g of a mixture containing calcium carbonate and calcium sulphate was reacted with
excess hydrochloric acid and 448cm3 of carbon dioxide measured at s.t.p was evolved
a. Write the equation for the reaction
b. Calculate
i. Mass of calcium carbonate in the mixture
ii. Percentage of calcium carbonate in the mixture
(Ca=40, C=12, O=16, 1mole of a gas occupies 22400cm3 at s.t.p)

23.11.2.3. Calculations involving energy changes


Carbon reacts with sulphur according to the equation
C(s) + 2S(s) CS2(s) H = +116kJmol-1
The amount of heat absorbed when 16g of sulphur reacts with excess carbon is (C=12, S=32)
Solution
2moles of sulphur absorb 116kJ

2 x 32g of sulphur absorb 116kJ

1 16
1g of sulphur absorb 2 X 32 kJ

1 16
x 16
16g of sulphur absorbs 2 X 32 kJ = 29kJ

Exercise

1. The formation of methanol from hydrogen and carbon dioxide is represented by the
following equation
2H2(g) + 2CO(g) CH3OH (l) H = -92kJmol-1
What would be the energy released when 3.2g of methanol is formed? (C=12, H=1,
O=16)
2. Methane burns in oxygen according to the equation
CH4(g) + 2O2(g) CO2(g) + 2H2O (l) H = -890kJmol-1
Calculate the volume of methane at s.t.p that will turn in excess oxygen to produce 2670kJ (1
mole of a gas occupies 22.4dm3 at s.t.p)

3. Calculate the heat produced when 48g of graphite is burnt in excess oxygen (C=12,
O=16, H=1, H = -390kJmol-1)

23.12. Concentration of Solutions


Concentration of solution tells the number of solute in a volume of solution
It is the amount of a substance dissolved in a certain amount of solution. It is expressed either in
grams per litre (gl-1 or gdm-3) or moles per litre (moll-1 or moldm-3)

dm
Volume of solution( 3)
Concentration (C) = Number of moles ( mol )grams of a solute( g)

Molarity
Molarity: Is the number of moles of a substance contained in one litre of a solution
But 1 litre = 1000cm3
Therefore, molarity can also be defined as the number of moles of a substance contained in
1000cm3 of a solution

Units of molarity = moles per litre (moll-1) or moles per dm3 (moldm-3)

Molar solution
This is the solution containing one mole of a substance per litre of solution i.e. solutions which
are 1M.

Standard solution
This is a solution whose concentration is exactly known or is the solution which contains a
known mass and a known volume.
Calculating the Amount of Solute
Examples
1) Calculate the number of moles that are 20g of sodium hydroxide (Na=23, O=16, H=1)
Solution
RFM of NaOH =23+16+1 = 40
Mass(m) 20
Number of moles of NaOH, n = Relative formulaatomicmass (Mr ) = n = 40 =

0.5mol
2) 4g of sodium hydroxide was dissolved in water to make 200cm3 of a solution. Calculate
the molarity of the standard solution formed
Solution
RFM of NaOH =23+16+1 = 40
Mass(m) 4
Number of moles of NaOH, n = Relative formulaatomicmass (Mr ) = n = 40 =

0.1mol
200cm3 of solution contain 0.1mol of sodium hydroxide
0.1
1cm3 contains 200 mol of sodium hydroxide

0.1
3 x 1000
1000cm contains 200 mol of sodium hydroxide

=0.5moles per dm3 of sodium hydroxide


=0.5M

3) What is the mass of solute in 600cm3 of 1.5M NaOH solution?


Solution
1000cm3 contains 1.5 mol of sodium hydroxide
1.5
1cm3 contains 1000 mol of sodium hydroxide
1.5
600cm3 contains 1000 x 600 mol of sodium hydroxide

=0.9mol

From
Mass(m)
Moles (n) = Relative formulaatomicmass (Mr )

Mass (m) = moles (n) x Relative formula mass (Mr )

RFM of NaOH =23+16+1 = 40

m = 0.9 x 40
= 36g

4) How many grams of sodium sulphate crystals Na2SO4.10H2O would be required to make
500cm3 of 0.01M solution (Na=23, S=32, O=16, H=1)
Solution
1000cm3 contains 0.01 mol of Na2SO4.10H2O
0.01
3
1cm contains 1000 mol of Na2SO4.10H2O

0.01
500cm3 contains ( 1000 x 500) mol of Na2SO4.10H2O

=0.005mol

From
Mass(m)
Moles (n) = Relative formulaatomicmass (Mr )
Mass (m) = moles (n) x Relative formula mass (Mr)

RFM of Na2SO4.10H2O = 2x23 +32 +4x16 +10(2x1 + 16) = 323

m = 0.005 x 323
= 1.615g

Exercise

1) How many moles of sulphuric acid are contained in 250cm3 of 0.1M sulphuric acid? (Ans
= 0.0025M)
2) Calculate the mass of nitric acid (HNO3) required to make 200cm3 of 2M solution
(Ans=25.2g)

23.13. Quantitative analysis


Is the measurement of the concentration of the elements/compounds in a known substance

Volumetric Analysis
Is a measure of the concentrations of an acids/alkalis in solution

Acid-alkali Titrations in Volumetric Analysis


It needs: - a standard solution: a solution of known concentration, and
- a solution of unknown concentration

Detecting the End Point


End point is the point at which neutralisation of acid and alkali is complete /is the point at which
the concentration of the titrant and titrand are the same in the titration.
- Sharp indicators (phenolphthalein and methyl orange) are used to detect end points effectively
- Litmus and universal indicators are not used as the changes at the end point are not sharp

A Typical Acid-alkali Titration


The diagram shows how titration is used to find concentration of sulphuric acid (H2SO4) using
Sodium hydroxide (NaOH) solution

Calculating molarity of a solution using a standard solution


When concentration of one of the solutions is given in a neutralisation reaction, the concentration
of the other solution can be calculated using the following steps
1. Write a balanced equation for the reaction
2. Calculate the number of moles of the standard solution
3. Using the mole ratio, calculate the number of moles of the second solution
4. State the volume which contains the moles
5. Calculate the number of moles in the specified volume e.g. 1000cm3
6. State the molarity with units i.e. mol/litre or M

Note: Calculate concentration in grams per litre using the formula


Concentration = Molarity x RFM

Examples

30.0 cm3 of 0.100 M NaOH reacted completely with 25.0 cm3 of H2SO4 in a titration. Calculate
the concentration of H2SO4 in mo mol/dm3 given that:
2NaOH(aq) + H2SO4(aq) Na2SO4(aq) + 2H2O(l)
Solution
Step 1: Find the reacting mole of NaOH
1000 cm3 of solution contain 0.1moles of sodium hydroxide

0.1
1cm3 contains 1000 moles

0.1
X 30
30cm3 contain 1000 moles = 0.0030 moles

Step 2: Write the chemical equation for the reaction


2NaOH(aq) + H2SO4(aq) Na2SO4(aq) + 2H2O(l)

Step 3: Find the ratio of number of moles of H2SO4 to number of moles of NaOH
2NaOH(aq) : H2SO4(aq) =2:1

Step 4: Use ratio to find number of moles of H2SO4 that reacted


2 moles of the NaOH reacts with 1 moles of H2SO4
1
1 moles of the NaOH reacts with 2 moles of H2SO4

1
X 0.030
0.030 moles of the NaOH reacts with 2 moles of H2SO4

=0.0015moles

Step 5: Find the concentration of H2SO4 in mol/dm3

25cm3 of solution contains 0.0015 moles of H2SO4


0.0015
1cm3 contains 25 moles of H2SO4

0.0015
3 X 1000
1000cm contain 25 moles of H2SO4

= 0.06M
Molarity of H2SO4 = 0.06M
23.14. Uses of Titrations in Analysis

23.14.1. Identification of Acids and Alkalis


Example:
An acid has formula of H2XO4. One mole of H2XO4 reacts with 2 moles of NaOH.
A solution of the acid contain 5.0g/dm3 of H2XO4. In titration, 25.0cm3 of acid reacted with
25.5cm3 of 0.1 M NaOH. Calculate the concentration of acid in mol/dm3 and find X of the acid
and its identity
Solution
Step 1: Find the reacting mole of NaOH
1000 cm3 of solution contain 0.1moles of sodium hydroxide

0.1
3
1cm contains 1000 moles

0.1
X 25.5
25.5cm3 contain 1000 moles = 0.00255 moles

Step 2: Write the chemical equation for the reaction


2NaOH(aq) + H2XO4(aq) Na2XO4(aq) + 2H2O(l)

Step 3: Find the ratio of number of moles of H2XO4 to number of moles of NaOH
2NaOH(aq) : H2XO4(aq) =2:1

Step 4: Use ratio to find number of moles of H2XO4 that reacted


2 moles of the NaOH reacts with 1 moles of H2XO4
1
1 moles of the NaOH reacts with 2 moles of H2XO4

1
X 0.0255
0.0255 moles of the NaOH reacts with 2 moles of H2XO4

=0.01275moles
Step 5: Find the concentration of H2XO4 in mol/dm3

25cm3 of solution contains 0.01275 moles of H2XO4


0.01275
1cm3 contains 25 moles of H2XO4

0.01275
X 1000
1000cm3 contain 25 moles of H2XO4

= 0.51M
Molarity of H2XO4 = 0.51M

Since 1000cm3 of H2XO4 contains 0.051 mol and 5 g of H2XO4.


0.051 mol of H2XO4 has a mass of 5g of H2XO4
5
1 mole of H2XO4 has a mass of 0.051 = 98 g

Hence, Mr of X = 98 2(1) 4(16) = 32.


X is sulphur and H2XO4 is sulphuric acid

b. Percentage Purity of Compounds


Mass of actual substance/dm 3
Percentage purity = ( Mass of impure substance X 100)%

Example:
7.2g of an impure sample of hydrated sodium carbonate (Na 2CO3.10H2O) was dissolved in 250
cm3 of solution. 20 cm3 of this solution was required to completely react with 25 cm 3 of 0.1M
hydrochloric acid. Calculate:
a) The molarity of pure hydrated sodium carbonate
b) The mass of pure hydrated sodium carbonate per litre of solution
c) The percentage;
i. Purity
ii. Impurity of hydrated sodium carbonate

Solution

i. Moles of hydrochloric acid that reacted

1000 cm3 of hydrochloric acid contain 0.1moles


0.1
1cm3 contains 1000 moles

0.1
X 25
25 cm3 contain 1000 moles = 0.0025 moles

From the equation

Na 2 CO 3 ( aq ) +2 HCl 2 NaCl(aq) + H 2 O ( l ) +CO2 ( g)

2 moles of the acid reacts with 1 moles of sodium Carbonate


1
1 moles of the acid reacts with 2 moles of the Carbonate

1
X 0.025
0.025 moles of the acid reacts with 2 moles of the Carbonate

=0.00125moles

20 cm3 of sodium carbonate solution contains 0.00125 moles of the carbonate


0.001255
3
1cm contains 20 moles of the carbonate

0.00125
X 1000
1000cm3 contain 1cm3 contain 20 moles of sodium hydroxide

= 0.0625M
Molarity of sodium hydroxide = 0.0625M
ii. Molarity
concentrationgrams per litre
Molarity=
Molar mass (RMM /RFM )

Molar mass of Na2CO3.10H2O (Na = 23, C = 12, O = 16, H = 1)


=23X2 + 12 + 16X3 + 10X (2+16) = 286
concentrationgrams per litre
0.625=
286

Concentration in grams per litre = 17.875g


iii. Percentage
i. Purity Note: 7.2g of impure hydrated sodium carbonate was dissolved in 250 cm3 of
solution. To calculate the percentage purity, the mass of the pure sample in 250 cm3 of
solution should be calculated
1000 cm3 contains 17.785g of pure hydrated sodium carbonate
17.785
3
1 cm contains 1000 g of pure hydrated sodium carbonate

17.785
250 cm3contains 1000 X 250 g of pure hydrated sodium carbonate

= 4.4688g of pure hydrated sodium carbonate


4.4688
X 100
Percentage purity = 7.2

=62.07%
7.24.4688
X 100
ii. Percentage impurity = 7.2 = 37.93%

Or Percentage impurity = 100 -62.07 = 37.97%

23.14.2. Determination of basicity of an acid


The basicity of an acid is the number of hydrogen ions, H + (aq) which can be produced by one
mole of the acid in solution. The general formula of an acid can be written as HnX or HnA; where
n= basicity of the acid
A or X = the acid radical e.g. chloride (Cl), sulphate (SO4), phosphate (PO4) etc.
Hence HX is monobasic
H2X is dibasic
H3X is tribasic
In an experiment 20.0 cm3 of a 0.1M acid HnA required 10.0 cm3 of a 0.1M required 10.0 cm3 of
0.2M sodium hydroxide solution for complete neutralisation.
a. Write the equation for the reaction
b. Calculate:
i. The number of moles of the acid that reacted
ii. The number of moles of sodium hydroxide solution that
reacted
c. Determine the basicity of the acid

Solution

a. H n A ( aq ) +nNaOH Nan A (aq)+nH 2 O ( l )

b.
i. Moles of the acid
1000 cm3 of acid contain 0.1moles

0.1
1cm3 contains 1000 moles

0.1
X 20
20 cm3 contain 1000 moles = 0.00250 moles

ii. Moles of sodium hydroxide soltuion


1000cm3 contain 1cm3 contain 0.2 moles of sodium hydroxide
0.2
3
1cm contain 1000 s moles of sodium hydroxide

0.2
X 10
10 cm3 of solution contains 1000 moles of sodium hydroxide

= 0.002 moles of sodium hydroxide


c. Basicity of the acid
To determine the basicity of the acid, the value of n should be determined
From the equation:
Mole ratio HnA : NaOH
1 : n
Moles: 0.02 : 0.002
n = 1
Hence HnA is a monobasic acid (HA)

c. Determination of water of crystallisation in oxalic acid (COOH)2. xH2O


20 cm3 of 0.1 M sodium hydroxide solution was required to completely neutralise 20 cm 3 of solution
containing 6.4g of the oxalic acid (COOH) 2. xH2O solution. 2moles of sodium hydroxide react with 1
mole of oxalic acid(C=12, O=16, H=1)
Calculate:-
a. The moles of sodium hydroxide that reacted
b. The molecular mass of (COOH)2.xH2O
c. The value of X (number) of moles of water of crystallisation in (COOH) 2.xH2O
Solution
a. Moles of sodium hydroxide

1000 cm3 contain 0.1moles of sodium hydroxide

0.1
1cm3 contains 1000 moles of sodium hydroxide
0.1
20 cm3 contain
X 20 moles of sodium hydroxide
1000

= 0.002 moles of sodium hydroxide

b. Molecular mass of (COOH)2.xH2O


Hint: - To calculate molecular mass in this case, the molarity and concentration in grams per litre should
be known;
Molarity of (COOH)2. xH2O
Given that:-
2 moles of sodium hydroxide react with 1 mole of oxalic acid
1 mole of sodium hydroxide reacted with moles of oxalic acid
1
0.02 moles of sodium hydroxide react with
X 0.02 moles of oxalic acid
2
=0.01 moles of oxalic acid
20 cm3 of oxalic acid solution contains 0.001 moles
0.01
1 cm3 of oxalic acid solution contains 20 moles
0.01
1000 cm3 of oxalic acid solution contains
X 1000 moles =0.005M
20

concentrationgrams per litre


Molarity=
Molar mass (RMM /RFM )

6.4
0.05=
Molar mass(RMM /RFM )

6.4
Molar mass ( RMM /RFM )= = 128
0.05

Note: Relative molecular mass and relative formula mass have no units

c. Value of x
(COOH)2. xH2O = 128
12 + 32 + X (2+16) = 128
90 + 18X = 128
12890
X = 18
X = 2.1
Number of moles of water of crystallisation =2. Hence (COOH) 2. 2H2O
Exercise
1. 25cm3 of sodium hydroxide reacted completely with 20cm3 of 0.1M hydrochloric acid.
Calculate the concentration of sodium hydroxide in
a. moles per litre (Ans = 0.08mol/litre)
b. Gram per litre (Ans = 3.2g/litre)
2. 25cm3 of a solution of sulphuric acid required 32cm3 of 0.1M sodium hydroxide for
neutralization. Calculate the molarity of the acid(Ans = 0.1M)
3. 20cm3 of sodium carbonate reacted completely with 25cm3 of 0.8M hydrochloric acid.
Calculate the concentration of sodium carbonate in
a. moles per litre (Ans = 0.5mol/litre)
b. Gram per litre (Ans = 53g/litre)
4. 20cm3 of an acid R.xH2O were dissolved in 1 litre of aqueous solution. 25cm3 of this
solution required 16cm3 of 0.5M sodium hydroxide solution. Calculate the relative
formula mass of the acid and determine x (Acid : alkali=1:2, R=89, H=1, O=16)Ans: x=2
5. 25cm3 of an acid HX was neutralized by 24 cm3 of a solution containing 5g of sodium
hydroxide per litre.
a. Calculate the molarity of the acid
b. If the acid solution contained 24g/l; calculate
i. The RFM of the acid
ii. The RAM of X (Ans X= 19)
6. 0.008g of a metallic oxide MO was dissolved in 80cm3 of 0.05M sulphuric acid. The
resultant solution which contained excess acid required 10 cm3 of a solution containing
16g of sodium hydroxide per litre for complete neutralisation (H = 1, O =16, S =32, Na =
23).
a. Write an equation for the reaction between
i. MO and sulphuric acid
ii. Sodium hydroxide and sulphuric acid
b. Calculate the number of moles of
i. Sodium hydroxide used
ii. Excess sulphuric acid
iii. Sulphuric acid which reacted with MO
c. Calculate the molar mass of MO and the atomic mass of M (Ans M = 24)
7.
a. If the acid solution contained 24g/l; calculate
i. The RFM of the acid
ii. The RAM of X (Ans X= 19)

23.15. Calculations of volume of solutions


Example
What volume of 0.2M sodium hydroxide is required to neutralise 25cm3 of 0.1M hydrochloric
acid?
Solution
NaOH(aq) + HCl(aq) NaCl(aq) + H2O(l)
0.1 x 25
Moles of HCl = 1000 moles

=0.0025Moles
But 1 mole of acid reacts with 1 mole of base
Moles of NaOH = moles of HCl
0.2moles of NaOH are dissolved in 1000cm3

1000
1 moles of NaOH are dissolved in 0.2 cm3

1000 x 0.0025
0.0025Moles of NaOH are dissolved in 0.2 cm3

=12.5 cm3
Exercise
1. 25cm3of a 0.02M sodium hydroxide solution reacted with Vcm3 of an aqueous solution
containing 0.0025moles/cm3 of Z. calculate the volume V (2 moles of NaOH react with 1
mole of Z)
2. What volume of 0.1M hydrochloric acid would react with 25.0cm3 of sodium carbonate
(Na2CO3) solution containing 5.20g of anhydrous salt in 1dm3 of solution (Na=23, C=12,
O=16)
3. What volume of 0.05M sulphuric acid is required to neutralise completely 2.80g of
potassium hydroxide? (K=39, O=16, H=1)
4. 7.5g of compound U occupy, 5.6dm3 at s.t.p. Determine the molar mass of U.
1.9. Volumes of gases
According to Avogadros law, equal volumes of all gases at the same temperature and pressure
contain equal molecules
E.g. 2H2(g) + O2(g) 2H2O(g)
2moles 1mole 2mole
2Volumes 1Volume 2Volumes
Example
20cm3 of carbon monoxide is mixed with 30cm3 of oxygen and exploded. What is the
composition by volume of the resulting gas after cooling the mixture to the original temperature?
Solution
2CO(g) + O2(g) 2CO2 (g)
2moles 1mole 2mole
2Volumes 1Volume 2Volumes
20cm3 10cm3 20cm3

Therefore, 20cm3 of carbondioxide react with 10cm3 of oxygen and (30-10)cm3 of oxygen
remained un reacted. The carbon dioxide produced is equal to 20cm3. Thus the gaseous product
contained 20cm3 of carbon dioxide and 20cm3 of excess oxygen.

23.16. Gay Lussacs law


The law states that when gases react, they do so in volumes which bear a simple ratio to one
another and so the volume of gaseous products produced, all the volumes are measured at the
same temperature.
The law does not apply to solids and liquids
Example
10cm3 of carbon monoxide is mixed with 20cm3 of oxygen in limited air and allowed to cool at
original temperature and pressure. What is the volume of the gas after the reaction?

2CO(g) + O2(g) 2CO2 (g)


2moles 1mole 2mole
2Volumes 1Volume 2Volumes
10cm3 5cm3 10cm3
So the volume of excess oxygen =20-5 = 15cm3
Volumes of carbon dioxide =10cm3
Total volume = product + excess oxygen
Total volume = (15 x 10) = 25cm3

23.17. Gas laws


Boyles law
This states that, the volume of a fixed mass of a gas is inversely proportional to its pressure at a
constant temperature
1
V P and PV = constant

Where P = Pressure and V = Volume


Therefore
P1 V1 = P2 V2

Charles law
This states that, the volume of a fixed mass of a gas is directly proportional to its absolute
temperature at a constant temperature

V
V T and T = a constant

Where T = absolute temperature (Temperature in Kelvins (K)) and V = Volume


Therefore
V1 V2
T1 = T2

NB: To convert oC to K i.e. K= oC + 273

The Ideal gas equation


This is the general equation obtained by combining both Charles and Boyles laws
From Boyles law P1 V1 = P2 V2
V1 V2
From Charles law T1 = T2

P1 V 1 P2 V 2
T1 = T2

Exercise
1. The volume of a fixed mass of a gas is 200cm3 at 0oC and 760mmHg pressure. Calculate
the volume of the gas at 100 oC and 380mmHg pressure.
2. Calculate the volume of hydrogen measured at 25 oC when 88g of potassium react with
water at a pressure of 760mmHg (K=39, volume of the gas at s.t.p is 22400cm3)
24. Qualitative analysis
25. Sulphur and its compounds

25.1. Sulphur
Non metal

Yellow solid

Atomic number 16 and atomic mass 32

Has 3 allotropes

Insoluble in water but soluble in organic solvents

Sulphur occurs as hydrogen sulphide in petroleum gases

Sulphur occurs in; Louisiana and Texas, USA, as free sulphur.


25.2. Extraction of sulphur by the Fraschs process
A hole about 30 cm in diameter is dug into the soil and the hole is lined with an iron pipe. A
sulphur pump is sunk into the iron pipe. It consists of three concentric tubes.

In the outermost tube, superheated water at 170oC and at a pressure of 10 atmospheres, to keep it
in a liquid form, is sent down to the beds or deposits of sulphur. The sulphur melts and flows into
the reservoir at the base of the pump.

Hot compressed air under a pressure of about 15 atmospheres is sent down through the innermost
tube. It pushes the molten sulphur and water up through the middle tube and its collected in
containers. Water is evaporated off and almost 99% pure sulphur is obtained.

Extraction of sulphur

25.3. Extraction of
sulphur from natural gas
Natural gases obtained during the
refining of petroleum contain
hydrogen sulphide which is absorbed by special solvents. The gas is removed from the solvent
and a small portion of the gas is burnt in air to form sulphur dioxide.

2H2S (g) + 3O2 (g) 2H2O (l) + 2SO2 (g)

The remaining portion of the gas is left to react with the sulphur dioxide to form sulphur and
water. The water is evaporated off.

2H2S (g) + SO2 (g) 3S (s) + 2H2 O (l)

25.4. Uses of sulphur


1. Used in the manufacture of sulphuric acid
2. Used to dust vines to prevent the growth of fungi
3. Used in making calcium hydrogen sulphite (Ca(HSO3)2), which is used as a bleacher of
wood pulp in manufacture of paper
4. For the vulcanisation of rubber, a process which converts the soft pliable rubber into the
hard, tough substance of which motor tyres and similar products are made
5. Used in the manufacture of dyes, fireworks, sulphur compounds such as carbon
disulphide (CS2)
6. Used in making ointments for treatment of skin diseases such as ringworm

25.5. Allotropes of sulphur


Sulphur has five allotropes: monoclinic sulphur, rhombic sulphur, amorphous sulphur, plastic
sulphur, and colloidal sulphur

They are basically two allotropes

25.5.1. Rhombic sulphur


Rhombic sulphur is also referred to as octahedral sulphur or -sulphur.

It has an octahedral shape hence its name.

It has a melting point of 113 oC and has a density of 2.06g/cm3

It is stable and exists below 96 oC

Preparation of rhombic or octahedral sulphur (alpha sulphur)

Dissolve some powdered sulphur in carbon disulphide in a boiling tube. Place it in a beaker after
extinguishing all flames in the area around. Filter off the solution into another dry beaker and
place a clean filter paper on top of the beaker. Pierce some small holes in the filter paper and
place the set up near a window for a day to allow the carbon disulphide to evaporate. Large
rhombic crystals of sulphur will form.
25.5.2. Monoclinic sulphur
Monoclinic sulphur is also referred to as prismatic sulphur or -sulphur.

Its crystals are needle shaped.

It has a melting point of 119 oC and has a density of 1.98 g/cm3.

It exists only above 96 oC

Preparation of monoclinic sulphur (Beta sulphur)

Place some powdered sulphur in an evaporating dish. Carefully heat it until it melts. Stir and
gradually add more sulphur until the crucible is full of molten sulphur. Stop heating and allow it
to cool. A crust will form on the surface of sulphur. Carefully pierce through the crust and
immediately pour off the liquid sulphur inside. Cut away the crust by cutting around the edge of
the crucible with a knife. Small needle shaped crystals will be seen inside the evaporating dish.

Transition temperature

This is the temperature at which a change from one form of sulphur to another form takes place.
It is 96oC. Rhombic sulphur is stable below 96oC. Above this temperature, it slowly changes to
the monoclinic form. Monoclinic stable is stable above 96oC and therefore below this
temperature it slowly changes to the rhombic form.

Differences between rhombic sulphur and monoclinic sulphur

Rhombic sulphur consists of relatively large yellow, translucent, octahedral crystals with a
melting point of 114oC while monoclinic sulphur consists of needle shaped, pale yellow
transparent crystals with a melting point of 119oC

Rhombic sulphur has a density of 2.06g/cm3 while monoclinic sulphur has a density of 1.98
g/cm3.

Crystals of rhombic sulphur are stable below 96oC while monoclinic sulphur is stable above 96
o
C.

25.5.3. Amorphous sulphur


Amorphous sulphur is formed by oxidation of hydrogen sulphide by air. When a saturated
solution of hydrogen sulphide is left exposed to air for two days, an almost white powder of
amorphous sulphur is formed.
2H2S(aq) + O2(g) 2S(s) + 2H2O(l)

25.5.4. Plastic sulphur


When boiling sulphur is quickly cooled by pouring it into cold water, yellow/brown elastic solid
called plastic sulphur is formed.

25.5.5. Colloidal sulphur


When dilute hydrochloric acid is added to dilute sodium thiosulphate, a yellow suspension of
colloidal sulphur is formed

Na2S2O3(aq) +2HCl(aq) 2NaCl(aq) +H2O(l) + S(s)+ SO2(g)

25.6. Properties of sulphur

25.6.1. Physical properties


It is yellow non-metal. It is insoluble in water. It is a poor conductor of heat and electricity.

25.6.2. Chemical properties of sulphur

25.6.2.1. Action of heat on sulphur (in absence of air)


When a yellow solid of sulphur is heated, it melts at about 113oC to a clear amber liquid which
flows easily like water. Sulphur contains small rings of atoms, S8; the liquid flows with ease
because the rings have been separated. On further heating at about 160oC, the sulphur becomes
brown and viscous. It flows slowly because the small rings of 8 atoms break and longer chains
are formed. These chains twist together and do not flow readily over one another.

On further heating, the liquid becomes very dark, reddish brown in colour and less viscous again.
The chains break and become shorter which can flow more readily. Sulphur boils at 444oC and
forms a brown vapour. On cold surfaces, the vapour condenses directly into a yellow sublimate.

25.6.2.2. Combustion of sulphur (in a plentiful supply of air)


When burning sulphur is plunged into a gas-jar of air, the sulphur burns with a blue flame
leaving a misty gas

The mist is due to traces of sulphur trioxide formed simultaneously with sulphur dioxide.
S(s) + O2 (g) SO2 (g)

25.6.2.3. Reaction with metals and non-metals


The mixture of iron filings and sulphur when heated glows giving a black solid of iron (II)
sulphide

Fe(s) + S(s) FeS(s)

A hot copper foil glows in sulphur vapour forming a black solid, copper (I) sulphide.

2Cu(s) + S(s) Cu2S(s)

Carbon combines directly with sulphur to form a liquid, carbon disulphide. Very high
temperatures are required for this reaction to occur.

C(s) + 2S(s) CS2(l)

25.6.2.4. Action of acids on sulphur


Dilute acids do not act upon sulphur. It is oxidized by hot concentrated sulphuric acid with
formation of sulphur dioxide

S(s) + 2H2SO4(s) 3SO2(g) + 2H2O(l)

Sulphur is oxidized by hot concentrated nitric acid to sulphuric acid. Bromine is added to speed
up the rate of reaction.

S(s) + 6HNO3(s) H2SO4(aq) + 6NO2(g) + 2H2O(l)

25.7. Oxides of sulphur

25.7.1. Sulphur dioxide

25.7.1.1. Laboratory preparation of sulphur dioxide


Preparation of sulphur dioxide

Hot concentrated sulphuric acid reacts with copper turnings giving off sulphur dioxide. The gas
is dried by passing it through a bottle containing concentrated sulphuric acid and collected in a
gas-jar by downward delivery since it is denser than air

Cu(s) + 2H2SO4(l) CuSO4 (aq) + 2H2O(l) + SO2(g)

Instead of using copper turnings, it is also possible to prepare the gas by using sodium sulphite
and dilute sulphuric acid

Na2SO3 (aq) + H2SO4 (aq) Na2SO4 (aq) + H2O (l) + SO2 (g)

Sulphur dioxide is very soluble in water and therefore heating reduces its solubility in water
formed in the flask. Since sulphur dioxide is very soluble in water, it cannot be collected over
water.

25.7.1.2. Properties of sulphur dioxide


1. It is a colourless gas with an irritating smell
2. It is more dense than air
3. It turns damp blue litmus paper red because it is an acidic gas
H2O (l) + SO2 (g) H2SO3 (aq)

4. It is very soluble in water and its solubility in water can be shown by the fountain
experiment
5. It is a powerful reducing agent
a. It reduces iron (III) ions in the brown iron (III) sulphate solution to iron (II) ions
in the green iron (II) sulphate solution. Sulphur dioxide is oxidised to sulphuric
acid
2Fe3+(aq) + SO2(g) + H2O(l) 2Fe2+(aq) + 4H+(aq) + SO42-(aq)

b. Sulphur dioxide reduces concentrated nitric acid to nitrogen dioxide, the sulphur
dioxide being oxidized to sulphuric acid. Brown fumes are observed

SO2 (g)+ 2HNO3 (l) H2SO4 (l) + 2NO2 (g)

c. Sulphur dioxide reduces yellow acidified potassium dichromate solution to green


chromium (III) sulphate solution and itself oxidized to sulphuric acid. This is a
characteristic test for sulphur dioxide

3SO2 (g) + Cr2O72- (aq) + 2H+ (aq) 3SO42- (aq) + 2Cr3+ (aq) + H2O (l)

d. It reduces acidified potassium permanganate (VII) solution to a colourless


solution

6. Sulphur dioxide acts as an oxidizing agent when it reacts with a more powerful reducing
agent than itself.
a. When sulphur dioxide is dissolved in water, it forms sulphurous acid which is a
bleaching agent. Sulphurous acid takes up oxygen from the dye to form sulphuric
acid. The removal of oxygen from a dye converts the dye to a colourless
compound. This is essentially a different reaction from that of other bleaching
agents, which oxidize the dye to a colourless compound.

3SO2 (g) + 2H2O (l) + Dye H2SO4 (aq) + (Dye + 2H (colourless)

b. Sulphur dioxide reacts with hydrogen sulphide to form a yellow deposit of


sulphur. Sulphur dioxide oxidizes hydrogen sulphide to water by supplying
oxygen and itself reduced to sulphur

2H2S (g) + SO2 (g) 2H2O (l) + 3S (s)

c. Burning magnesium continues to burn in sulphur dioxide for sometime forming a


white powder of magnesium oxide and sulphur
2Mg (s) + SO2 (g) 2MgO (s) + S (s)

25.7.1.3. Uses of sulphur dioxide


1. It is used for the manufacture of sulphuric acid in the contact process
2. It is used as a bleaching agent in paper industry and also to bleach wool, silk and sponges
3. It is used as a preservative of foods e.g. orange juice and fruits where it reacts with
oxygen and prevents oxidation
4. It is used for fumigation of houses since it is poisonous and kills microorganisms

25.7.2. Sulphur trioxide

25.7.2.1. Preparation of sulphur trioxide

It is prepared by passing a mixture of dry sulphur dioxide and dry air over heated vanadium (V)
oxide (or platinised asbestos) at a temperature of 450 500 o C under a pressure of 200
atmospheres. Sulphur trioxide is seen as dense white fumes which are solidified in a freezing
mixture of ice and sodium chloride.

2 SO2 (g) + O2 (g) 2 SO3 (g)


25.8. Sulphuric acid
Sulphuric acid is a dibasic mineral acid.

Industrial manufacture of sulphuric acid by the contact process

The raw materials are sulphur dioxide, oxygen and water.

Stage 1: preparation of sulphur dioxide

Sulphur is burnt in air to produce sulphur dioxide

S(s) + O2 (g) SO2 (g)

Sulphur dioxide may contain some impurities such as arsenic compounds which may poison the
catalyst, that is, make the catalyst ineffective. Therefore sulphur dioxide is cleaned to remove
the impurities then is dried.

Stage II: Conversion of sulphur dioxide to sulphur trioxide

Then sulphur dioxide is mixed with air and passed along heated pipes containing pellets of
vanadium pentoxide (V2O5) (catalyst) at a temperature of 450 500 oC under a pressure of 200
atmospheres. Sulphur trioxide is formed.

2 SO2 (g) + O2 (g) 2 SO3 (g)

Sulphur trioxide formed is dissolved in concentrated sulphuric acid to produce a fuming liquid
called oleum.

SO3 (g) + H2SO4 (l) H2S2O7 (l)

Stage III: Conversion of sulphur trioxide to acid

The oleum is diluted with a known amount of water to give concentrated sulphuric acid.

H2S2O7 (l) + H2O (l) 2H2SO4 (aq)

Note: Sulphur trioxide is not dissolved in water directly because the reaction is too exothermic
and the heat produced from the reaction vapourises the acid forming only tiny droplets of the
acid leading to a spray of sulphuric acid which would affect the workers in the factory.

The whole process is summarized in the figure below


25.8.1. Properties of sulphuric acid

25.8.1.1. Physical properties


It is a colourless and ordourless oily liquid

Has a high affinity for water (hygroscopic) and that is why it is used as a drying agent

It has a density of 1.86g/cc and boils at 338oC

It does not show any acidic properties unless water is present.

Note: Sulphuric acid has a high affinity for water. Never add water to the concentrated acid
because it can explode. It is therefore advisable to add the acid to water rather than water to
acid.

25.8.1.2. Chemical properties


1) As an acid
Sulphuric acid behaves as a strong acid in dilute concentration
a) Metals above hydrogen in the reactivity series displace hydrogen from sulphuric acid

Zn (s) + H2SO4 (aq) ZnSO4 (aq) + H2 (g)

b) Reacts with bases to form salt and water only

ZnO (s) + H2SO4 (aq) ZnSO4 (aq) + H2O (l)

c) It neutralises alkalis like sodium hydroxide solution to form a salt and water only

2NaOH(aq) + H2SO4 (aq) Na2SO4 (aq) + 2H2O (l)

d) It reacts with carbonates and hydrogen carbonates liberating carbon dioxide

Na2CO3 (s) + H2SO4 (aq) Na2SO4 (aq) + H2O (l)+ CO2(g)

2NaHCO3 (s) + H2SO4 (aq) Na2SO4 (aq) + H2O (l) + 2CO2(g)


2) As an oxidizing agent
When concentrated and hot, it acts as an oxidizing agent to both metals and non metals and it is
reduced to sulphur dioxide. It accepts electrons or supplies oxygen in its reaction.
a) It oxidizes copper to copper (II) sulphate. Reaction with zinc and iron produces similar
results.

Cu(s) + 2H2SO4(l) CuSO4 (aq) + 2H2O(l) + SO2(g)

b) It oxidizes hydrogen sulphide to sulphur forming a yellow deposit.

H2S (g)+ H2SO4 (l) S (s) + 2H2O(l) + SO2(g)

c) It oxidizes sulphur and carbon to their oxides.

3S(s) + 2H2SO4(l) 2H2O(l) + 3SO2(g)

C(s) + 2H2SO4(l) CO2 (g) + 2SO2(g) + 2H2O(l)

3) As a dehydrating agent.
Sulphuric acid has a very high affinity for water and can remove it from substances including air,
that is, it is hygroscopic. It can be used as a drying agent for most gases.
a) When concentrated sulphuric acid is poured onto sugar (sucrose) in a beaker the sugar
turns yellow then brown and finally a black spongy mass of charcoal rises filling the
beaker. Steam is given off and the whole mass becomes very hot. The acid takes out the
elements of water from sugar leaving a black mass of carbon.

C12H22O11(s) 12C (s) + 11H2O(l)

Similar reactions take place when other carbohydrates are used

C6H12O6(s) 6C (s) + 6H2O(l)

b) When concentrated sulphuric acid is added to blue crystals of copper (II) sulphate
(hydrated) and warmed, they change to a white solid of anhydrous copper (II) sulphate as
water of crystallisation is removed by concentrated sulphuric acid.

CuSO4 .5H2O(s) CuSO4 (s) + 5H2O(l)

c) Other substances which are dehydrated by concentrated sulphuric acid include ethanol,
methanoic acid and oxalic aid
C2H5OH(l) C2H4 (g) + H2O(l)
Ethanol
HCOOH(l) CO(g) + H2O(l)
Methanoic acid
H2C2O4 (s) CO (g) + CO 2 (g) + H2O(l)
Oxalic acid
4) Action of concentrated sulphuric on nitrates and chlorides
Concentrated sulphuric acid displaces hydrochloric acid from metallic chlorides and nitric
acid from nitrates e.g. it reacts with sodium chloride when heated forming white fumes of
hydrogen chloride gas which dissolves in water forming hydrochloric acid.

2NaCl (s) + H2SO4 (aq) Na2SO4 (aq) + 2HCl (g) (With heating)

NaCl (s) + H2SO4 (aq) NaHSO4 (aq) + HCl (g) (Without heating)

It also reacts with potassium nitrate to form nitric acid

KNO3 (s) + H2SO4 (aq) KHSO4 (aq) + HNO3(aq)

25.8.2. Uses of sulphuric acid


1. It is used in the manufacture of detergents
2. It is used in car batteries and accumulators as an electrolyte
3. It is used in manufacture of fertilizers such as ammonium sulphate
NH3(g) + H2SO4 (aq) (NH4)2SO4 (s)
4. Used in extraction of metals and also cleaning them prior to plating
5. Used in manufacture of paints, dyes and explosives

25.9. Sulphates
Action of heat on sulphates (SO42-)

Most sulphates are hydrated and when heated, they lose their water of crystallisation to form
anhydrous salts which are resistant to further heating and therefore do not decompose. Therefore
hydrated sulphates do not decompose on heating e.g.
MgSO4.7 H2O(s) MgSO4(s) + 7 H2O(l)
Na2SO4.10H2O(s) Na2SO4(s) + 10H2O(l)

When a blue solid of hydrated copper (II) sulphate is heated, water vapour is given off as water of
crystallisation is lost, giving a white solid (residue). On further heating, it decomposes to form white
fumes of sulphur trioxide and a black residue of copper (II) oxide.
CuSO4.5 H2O(s) CuSO4(s) + 5H2O(l)
Blue White
CuSO4(s) CuO(s) + SO3 (g)

Overall equation:

CuSO4.5H2O CuO(s) + SO3 (g) + 5H2O(l)

On heating hydrated iron (II) sulphate (green), it loses its water of crystallisation
FeSO4.7 H2O(s) FeSO4(s) + 7 H2O(l)
On further heating, the anhydrous iron (II) sulphate formed decomposes to give white fumes of
sulphur trioxide together with sulphur dioxide and leaves a brown residue of iron (III) oxide
FeSO4(s) Fe2O3(s) + SO2(g)+ SO3(g)
When ammonium sulphate is heated it decomposes to give ammonia, sulphur trioxide and water
(NH4)2SO4(s) NH3 (g) + SO3(g) +H2O(l)

Testing for soluble sulphate (SO42-)

a. Using dilute hydrochloric acid and barium chloride solution.

To the solution add dilute hydrochloric acid followed by barium chloride solution.

A white precipitate of barium sulphate is formed

BaCl2(aq) + Na2SO4(aq) BaSO4(s) + 2NaCl(aq)

b. Using dilute nitric acid and barium nitrate solution.

To the solution add dilute nitric acid and barium nitrate solution. A white precipitate is formed.

Ba(NO3)2(aq) + Na2SO4(aq) BaSO4(s) + 2NaNO3 (aq)

Ionic equation

Ba2+(aq) + SO42-(aq) BaSO4(s)

Note: Carbonate ions (CO32-) and sulphite ions (SO32-) are precipitated as barium carbonate and
barium sulphite respectively, if carbonate and sulphite ions are present in solution.

Ba2+(aq) + CO32-(aq) BaCO3(s)

Ba2+(aq) + SO32-(aq) BaSO3(s)


The purpose of adding dilute nitric acid or hydrochloric acid is to remove the carbonate ion and
sulphite ions if they are present in solution.

2H+(aq) + CO32-(aq) H2O(l) + CO2(g)

2H+(aq) + SO32-(aq) H2O(l) + SO2(g)

The sulphite ions (SO42-) remain in solution because they do not react with dilute hydrochloric
acid acid or nitric acid.

c. Using lead (II) nitrate solution

Also lead (II) nitrate solution forms a white precipitate of lead (II) sulphate with a sulphate.

Pb2+(aq) + SO42-(aq) PbSO4(s)

25.10. Hydrogen sulphide

25.10.1. Laboratory preparation of hydrogen sulphide

Add concentrated hydrochloric acid to iron (II) sulphide. Effervescence occurs and the hydrogen
sulphide formed is collected over warm water because it is quite soluble in cold water

FeS(s) + 2HCl(aq) FeCl2(aq) + H2S(g)

Dilute sulphuric acid may be used

FeS(s) + H2SO4(aq) FeSO4(aq) + H2S(g)

To prepare hydrogen sulphide from sulphur, iron (II) sulphide is first prepared by heating the
mixture of iron and sulphur.
Fe(s) + S(s) FeS(s)

Hydrogen sulphide can be dried by using anhydrous calcium chloride. Concentrated sulphuric
acid is not used to dry the gas because it reacts with hydrogen sulphide forming a yellow
precipitate of sulphur.

H2SO4(aq) + 3H2S(g) 4H2O(l) + 4S(s)

25.10.2. Testing for hydrogen sulphide


A strip of filter paper is soaked in lead (II) ethanoate (lead (II) acetate) solution and dropped into
a gas jar of hydrogen sulphide. The paper turns black. This colour change is caused by
precipitation of black lead (II) sulphide.

(CH3COO)2Pb(aq) + H2S(g) PbS(s) + 2CH3COOH(aq)

Black

25.10.3. Properties of hydrogen sulphide

25.10.3.1. Physical properties


1. The gas is colourless
2. It has a repulsive smell similar like rotten eggs.
3. It is also fairly soluble in water forming a weak acidic solution
4. It is denser than air
5. The gas is extremely poisonous

25.10.3.2. Chemical properties


1. Hydrogen sulphide is a powerful reducing agent

It reduces reducing agents and itself oxidized to sulphur which appears as a pale yellow
precipitate

a. Hydrogen sulphide reduces iron (III) chloride to iron (II) chloride. When
hydrogen sulphide is passed through iron (III) chloride solution (yellow solution)
a yellow precipitate of sulphur appears. On filtering, a green solution of iron (II)
chloride appears as the filtrate

H2S(g) + 2FeCl3(aq) 2FeCl2(aq) + 2HCl(aq) + S(s)


b. Hydrogen sulphide reacts with concentrated nitric acid to form brown fumes of
nitrogen dioxide and a yellow deposit of sulphur. Also the mixture becomes hot.
The hydrogen sulphide reduces the nitric acid to nitrogen dioxide.

2HNO3(l) + H2S(g) 2H2O(l) + 2NO2(g) + S(s)

2. Combustion of hydrogen sulphide


a. With plentiful supply of air

Hydrogen sulphide burns with a blue flame forming water and sulphur dioxide.

2H2S(g) + 3O2(g) 2SO2(g) + 2H2O(l)

b. With a limited supply of air

When hydrogen sulphide is burnt in a limited supply of air, a yellow deposit of sulphur is
formed. The oxygen supply cannot oxidize the gas completely and therefore free sulphur is
deposited.

2H2S(g) + O2(g) 2H2O(l) + 2S(s)

3. Reacting with salts of metals


Hydrogen sulphide can precipitate insoluble sulphides of copper and lead only by
reacting with solutions of their salts. A dark brown precipitate of copper (II) sulphide and
a black precipitate of lead (II) sulphide are formed

CuSO4(aq) + H2S(g) CuS(s) + H2SO4(aq)

Pb(NO3)2(aq) + H2S(g) PbS(s) + 2HNO3(aq)

Pb(NO3)2(aq) + H2S(g) PbS(s) + 2HNO3(aq)


26. Nitrogen and its compounds

26.1. Nitrogen
Occurrence of nitrogen

Atomic number 7, atomic mass 14. It is one of the main elements needed for plant growth.

Nitrogen is the most abundant gas in the atmosphere, occupying about 78 per cent by
volume. It occurs in nature in a combined state as in minerals such as sodium nitrate. It is
found in living things in form of proteins.

26.2. Laboratory preparation of nitrogen from air


The methods include:

a. Thermal decomposition of ammonium nitrite

Ammonium nitrate when heated decomposes to give nitrogen.

NH4NO2(s) N2 (g) + 2H2O(l)


Ammonium nitrite is prepared from the reaction between sodium nitrite and ammonium chloride

NH4Cl(s) + NaNO2(s) NH4NO2(s) +NaCl(s)

b. Reduction of copper (II) oxide using ammonia

3CuO(s) + 2NH3(g) 3Cu(s) + N2(g) + 3H2O(l)


c. Direct combination of chlorine and ammonia gives nitrogen and ammonium
chloride

Cl2(g) + 8NH3(g) N2(g) + 6NH4Cl(s)

26.3. Industrial preparation of nitrogen from air


The raw material is air. It is done by systematically removing carbondioxide and oxygen leaving
nitrogen.

It is done in stages. These include:

a. The air is pumped into the set up


b. It is then passed through sodium hydroxide solution which removes carbondioxide

Sodium hydroxide absorbs and removes carbondioxide from the air mixture and forms sodium
carbonate

2NaOH (aq) + CO2 (g) Na2CO3 (aq) + H2O (l)


c. It is then passed over heated copper in the furnace to remove oxygen

It removes the oxygen by reacting it with the hot copper leading to formation of copper oxide

2Cu (s) + O2 (g) 2CuO (s)


d. The remaining air is mainly composed of nitrogen which is collected over water
since it is slightly less dense than air.
Note:

It can be dried by passing the gas through a U-tube containing glass beads wetted
with concentrated sulphuric acid to dry it and then collected in a syringe.
Nitrogen formed by this method is not pure. It contains several impurities, mainly the
noble gases as well as unreacted oxygen.
Commercially nitrogen is manufactured through fractional distillation of liquid air.

26.4. Test for nitrogen


Nitrogen is almost inactive (inert) at ordinary temperatures. There is almost no positive test
for it. This is because nitrogen is composed of diatomic molecules, the atoms of which are
held together very strongly by three (triple) covalent bonds. Only when sufficient energy is
supplied to break these bonds does nitrogen react. Because of being inert, it has no simple
positive test. It can only be identified by its negative response to the following tests for other
common gases.

1. Nitrogen extinguishes a burning splint and the gas does not burn. This distinguishes it
from other gases that support burning like oxygen and dinitrogen oxide or any
combustible gas such as hydrogen, carbon monoxide, hydrogen sulphide.
2. Nitrogen has no smell. This distinguishes it from gases such as sulphur dioxide ammonia,
hydrogen chloride.
3. Nitrogen has no action on lime-water. This distinguishes it from carbon dioxide.
26.5. Properties of nitrogen

26.5.1.1. Physical properties


1. Nitrogen is a colourless and tasteless gas.
2. It is slightly soluble in water under ordinary conditions.
3. It is slightly less dense than air.
4. Nitrogen and hydrogen combine at high temperatures and pressure in the presence of a
catalyst to form ammonia.
N2(g) + 3H2(g) 2NH3(g)

5. Nitrogen reacts only with the reactive metals (magnesium and calcium). When these
metals are heated strongly, they burn in nitrogen forming the corresponding nitride,
which is white in colour.
3Mg(s) + N2(g) Mg3N2(s)

3Ca(s) + N2(g) Ca3N2(s)

The heat produced by the burning magnesium ribbon or calcium is strong enough to
break the triple bond in the nitrogen molecule forming free nitrogen atoms. The free
atoms are very reactive and combine with these metals to form a nitride. If a burning
wooden splint is placed in a jar of nitrogen, it gets extinguished. This is because the heat
it produces is not sufficient to break the tripple bond between the nitrogen atoms

The nitrides dissolve in water to form the corresponding hydroxide and ammonia.

Mg3N2(s) + 6H2O(1) 3Mg(OH)2(aq) + 2NH3(g)

Ca3N2(s) + 6H2O(1) 3Ca(OH)2(aq) + 2NH3(g)

6. In thunderstorms, a small amount of nitrogen reacts with the oxygen in the air to form
nitrogen monoxide and nitrogen dioxide.
N2(g) + O2(g) 2NO(g)

2NO(g) + O2(g) 2NO2(g)

The electrical discharge in a thunderstorm provides sufficient energy for this reaction to
occur.

26.6. Uses of nitrogen


1. Used in the Haber process for the manufacture of ammonia.

N2(g) + 3H2(g) 2NH3(g)


2. It is used in food packaging, for example in crisp packets, to keep the food fresh and in
this case to prevent the crisps being compressed.

3. Liquid nitrogen is used as a refrigerant.

4. Because of its unreactive nature, nitrogen is used as an inert atmosphere for some
processes and chemical reactions. For example, empty oil tankers are filled with nitrogen
to prevent fires.

26.7. Nitrogen monoxide (nitrogen(II) oxide)

26.7.1. Laboratory preparation of nitrogen monoxide


The experiment is set up as shown in figure 2.2. Copper is covered with water and
concentrated nitric acid added, about equal in volume to the water. Vigorous effervescence
occurs and brown fumes dissolve in water over which nitrogen monoxide (main product) is
collected as a colourless gas. A green solution of copper(II) nitrate is left in the flask.

3Cu(s) + 8HNO3(aq) 3Cu(NO3)(aq) + 4H2O(l) + 2NO(g)


Nitrogen dioxide (the brown fumes) is produced partly by the action of the acid upon the
copper and partly by the oxidation of the main product (nitrogen monoxide) by the oxygen of
the air in the flask.

Cu(s) + 4HNO3(1) Cu(NO3)2(aq) + 2NO2(g) + 2H2O(l)

2NO(g) + O2(g) 2NO2(g)

26.7.2. Tests for nitrogen monoxide


1. Exposure to air

Remove the cover from a gas-jar of nitrogen monoxide. Brown fumes are at once
produced due to oxidation of the gas by oxygen of the air to nitrogen dioxide.

2NO(g) + O2(g) 2NO2(g)

2. Action on iron(II) sulphate solution

When a cold solution of iron(II) sulphate is poured in a gas-jar containing nitrogen


monoxide, a dark brown or black colouration is observed due to formation of nitros-
iron(II) sulphate.

FeSO4(aq) + NO(g) FeSO4.NO(aq)

26.7.3. Properties of nitrogen monoxide


1. It is colourless and almost insoluble in water
2. It is slightly denser than air.
3. It is neutral to litmus.
4. It supports the combustion of those burning materials whose flames are hot enough to
decompose it and so liberate free oxygen with which the material may combine. The gas
extinguishes a splint, candle, sulphur and glowing charcoal, but the gas supports the
combustion of strongly burning phosphorus or magnesium.
P4(s) + 10NO(g) 2P2O5(s) + 5N2(g)

2Mg(s) + 2NO(g) 2MgO(s) + N2(g)


26.8. Nitrogen dioxide

26.8.1. Laboratory preparation of nitrogen dioxide

The experiment is set up as shown in figure 2.3. When lead(II) nitrate is heated, it makes a
cracking sound giving off a brown gas (nitrogen dioxide) and oxygen. Nitrogen dioxide is
liquefied in the freezing mixture and collects in the tube as green liquid. The oxygen passes
on as gas and escapes or it is collected over water.

2Pb(NO3)2(s) 2PbO(s) + 4NO2(g) + O2(g)

Lead(II) nitrate is the most suitable to use because it crystallizes without water of
crystallization, which is found in crystals of most nitrates and which would interfere with the
preparation.

Nitrogen dioxide may also be prepared by the action of concentrated nitric acid on copper
turnings. The gas is collected by downward delivery in a gas-jar.

Cu(s) + 4HNO3(1) Cu(NO3)2(aq) + 2NO2(g) + 2H2O(1)

26.8.2. Properties of nitrogen dioxide


1. It is a brown gas.
2. It has a pungent, irritating smell and it is a poisonous gas.
3. It is soluble in water forming a pale blue solution containing nitric acid and nitrous acid.
Nitrogen dioxide is a mixed acid anhydride.
2NO2(g) + H2O(1) HNO3(aq) + HNO2(aq)

4. Nitrogen dioxide neutralizes alkalis forming a mixture of their corresponding nitrates and
nitrites. In this case nitrogen dioxide acts as an acid.
2NaOH(aq) + 2NO2(g) NaNO3(aq) + NaNO2(aq) + H2O(1)

2KOH(aq) + 2NO2(g) KNO3(aq) + KNO2(aq) + H2O(1)


5. Nitrogen dioxide does not burn, but supports combustion of substances whose flames are
hot enough to decompose it and so liberate free oxygen with which the substance may
combine. It supports the combustion of carbon, sulphur, phosphorus and magnesium.
2C(s) + 2NO2(g) 2CO2(g) + N2(g)

2S(s) + 2NO2(g) 2SO2(g) + N2(g)

2P4(s) 10NO2(g) 2P4O10(s) + 5N2(g)

4Mg(s) + 2NO2(g) 4MgO(s) + N2(g)

6. Nitrogen dioxide oxidizes red hot metals and itself reduced to nitrogen.
4Cu(s) + 2NO2(g) 4CuO(s) + N2(g)

4Zn(s) + 2NO2(g) 4ZnO(s) + N2(g)

7. When nitrogen dioxide is heated at temperatures above 150oC, it dissociates to give


nitrogen monoxide and oxygen. The brown colour of nitrogen dioxide fades to give a
colourless gas which is the mixture of nitrogen monoxide and oxygen.
2NO2(g) 2NO(g) + O2(g)

26.9. Ammonia

26.9.1. Laboratory preparation of ammonia


The apparatus is set up as shown in figure 2.4. The flask has to be slanted to prevent water
produced by the reaction from running into the hot flask when it causes the glass to crack. A
mixture of calcium hydroxide and ammonium chloride is first ground thoroughly. Then it is
placed in the flask and heated, producing ammonia which is collected by downward
displacement of air since it is less dense than air. The gas is dried by calcium oxide
(quicklime).

Ca(OH)2(s) + 2NH2C1(s) CaC12(s) + 2H2O(1) + 2NH3(g)


Instead of calcium hydroxide, sodium hydroxide or potassium hydroxide solution may be
used, in which case the flask would be placed in the vertical position and heated.

NaOH(aq) + NH4Cl(s) NaCl(aq) + H2O(l) + NH3(g)

KOH(aq) + NH4Cl(s) KCl(aq) + H2O(l) + NH3(g)

Ammonium sulphate may be used instead of ammonium chloride.

Ca(OH)2(s) + (NH4)2SO4(s) CaSO4(s) + 2H2O(l) + 2NH3(g)

The usual drying agents such as concentrated sulphuric acid and anhydrous calcium chloride
are not used because ammonia reacts with them to form ammonium sulphate and tetraamine
calcium chloride respectively.

2NH3(g) + H2SO4(l) (NH4)2SO4(s)

CaCl2(s) + 4NH3(g) Cac12.4NH3(s)

26.9.2. Industrial preparation of ammonia (Haber process)


Dry nitrogen and hydrogen in the ratio of one to three respectively, are passed over a catalyst
of iron with some traces of aluminium oxide present. The reaction occurs at the surface of the
catalyst, therefore, the catalyst should be finely divided to increase the surface area over
which the reaction occurs. Aluminium oxide improves the performance of the catalyst by
making it more porous thus providing a higher surface area for the reaction. The temperature
is between 450oC 500oC. The gases are under a pressure of 250 atmospheres to 500
atmospheres. Ammonia is produced.

N2(g) + 3H2(g) 2NH3(g)

Ammonia is removed from the mixture of gases by cooling the mixture with a freezing
mixture. It is only ammonia that liquefies and can be removed from the mixture. The
unreacted nitrogen and hydrogen are recycled (figure 2.5).

Nitrogen used in this process is obtained by fractional distillation of liquid air and hydrogen
is obtained from natural gas or electrolysis of brine.

26.9.3. Tests for ammonia


The gas has a characteristic chocking smell. It turns damp red litmus paper blue and forms
white fumes with concentrated hydrochloric acid or hydrogen chloride.

26.9.4. Properties of ammonia


It is a colourless gas with a choking smell. It is less dense than air and thus collected by
upward delivery. It is an alkaline gas and therefore turns red litmus blue. It is the only
common alkaline gas.

26.9.5. Solubility of ammonia in water


It is very soluble in water to give an alkaline solution. The great solubility of ammonia is due to
the reaction of the gas with water. Ammonia is a base and removes protons from water to
produce ammonium ions and hydroxide ions.

NH3(g) + H2O(l) NH4+(aq) + OH-(aq)


The solution is only weakly alkaline because of the reversible nature of this reaction, which
results in a relatively low concentration of hydroxide ions. Ammonia gas dissolved in water is
usually known as aqueous ammonia.

26.9.6. Experiment to demonstrate the high solubility of ammonia gas in water

This is an experiment to demonstrate the high solubility of ammonia gas in water. A large round
thick walled flask is filled with ammonia gas. It is then fitted with two glass tubes C and D with
clips at one end (figure 2.6). The flask is inverted over a trough of water and the clip on tube D
opened to allow in a few drops of water and then closed. These are shaken with ammonia to
dissolve it. If the red litmus solution is added to water in the trough, water in the flask will turn to
blue indicating that it is an alkaline gas which dissolved in water. The clip on the tube C is
opened. Water runs up the tube and spreads at the end of the tube forming a fountain.

The few drops of water, which entered through tube D, dissolved all the ammonia gas in the flask
so that a partial vacuum was created in the flask. When the clip on tube C was opened,
atmospheric pressure pushed the water up the tube forming a fountain.

26.9.7. Action of ammonia on copper (II) oxide


When a stream of dry ammonia is passed over very strongly heated copper(II) oxide as shown in
figure 2.7, a colourless liquid (water) forms in the U-tube. The black oxide turns brown and a
colourless gas collects in the jar over water. Ammonia reduces the copper(II) oxide to copper and
itself oxidized to nitrogen.
3CuO(s) + 2NH3(g) 3H2O(l) + N2(g)

Here ammonia behaves as a reducing agent. A similar reaction takes place with the oxides of lead
and iron.

3PbO(s) + 2NH3(g) 2Pb(s) + 3H2O(l) + N2(g)

Fe2O3(s) + 2NH3(g) 2Fe(s) + 3H2O(l) + N2(g)

This experiment can also be used to demonstrate that ammonia contains nitrogen.

26.9.8. Combustion of ammonia


Ammonia is a good reducing agent, which means that it can be easily oxidised. Ammonia burns
with a green/yellow flame, in an atmosphere of air slightly enriched by oxygen forming nitrogen
and water.

4NH3(g) + 3O2(g) 2N2(g) + 6H2O(l)

The figure 2.8 shows how ammonia is burnt. The role of the glass wool is to distribute oxygen
evenly throughout the gas vessel.
In presence of catalyst, ammonia is oxidised to nitrogen monoxide.

The figure 2.9 shows that set of the experiment. A hot platinum or copper wire which acts
as a catalyst is suspended in a beaker of concentrated ammonia and oxygen is bubbled
through the solution. The metal catalyst remains red-hot because the reaction is
exothermic. Brown fumes of nitrogen dioxide, which are formed due to oxidation of
nitrogen monoxide, are observed.

4NH3(g) + 5O2(g) 4NO(g) + 6H2O(l)

2NO(g) + O2(g) 2NO2(g)


The fumes later turn white due to formation of ammonium nitrate.

4NO2(g) + O2(g) +2H2O(g) 4HNO3(g)

NH3(g) + HNO3(g) NH4NO3(g)

26.9.9. Reaction with hydrogen chloride


Ammonia reacts with hydrogen chloride to form white fumes, which turn to a white solid
of ammonium chloride on standing.

NH3(g) + HCl(g) NH4Cl(s)

26.9.10. Reaction with chlorine


Ammonia burns spontaneously in chlorine forming a mist of hydrogen chloride.

2NH3(g) + 3Cl(g) N2(g) + 6H4Cl(g)

In excess ammonia, dense white fumes of ammonium chloride are formed. Hydrogen
chloride formed reacts with excess ammonia to form the white fumes, which later settle
to a white solid.

2NH3(g) + 3Cl2(g) N2(g) + 6HCl(g)

HCl(g) + NH3(g) NH4Cl(s)

This can be represented by one equation.

3Cl2(g) + 8NH3(g) N2(g) + 6NH4Cl(s)

26.9.11. Uses of ammonia


1. Ammonia solution is used in laundry work to remove temporary hardness.
2. Ammonia is used to manufacture ammonium sulphate and ammonium nitrate used as
fertilizers.
3. It is used in manufacture of nitric acid.
4. It is used in production of nylon.
5. It can be used as a refrigerant because it evapourates readily, removing heat from the
surrounding as it does so. It can be easily liquefied by compression.
Exercise

Study the following reaction scheme.

Air Gas W Sulphuric acid Solid Y

Ammonia

Natural gas Gas X Nitric acid Solid Z

(a) Identify the substances W, X, Y and Z.


(b) State the method used to obtain gas W from the air.
(c) Write the reaction leading to the formation of
(i) ammonia
(ii) Y
(iii) Z
(d) State any use of solid Z.

26.9.12. Ammonia solution

26.9.12.1. Preparation of ammonia solution


It can be prepared by dissolving ammonia in water using the setup shown in figure 2.10. The
filter is used in passing ammonia into water in order to prevent the sucking back of water
from the beaker into the reaction flask. Ammonia is so soluble in water that, in the process of
dissolving, too much of it could dissolve at one time creating a low gaseous pressure in the
reaction flask as well as in the delivery tube by the atmospheric pressure outside. The funnel
is arranged with its rim only just immersed in order to ensure that when water is sucked into
the funnel, contact with the water is broken and the water falls back into the beaker rather
than being sucked back along the delivery tube.
26.9.13. Ammonium salts

26.9.13.1. Nitrogenous fertilizers


Fertilizers supply crops with inorganic elements required for their growth. Such elements
include nitrogen, phosphorus, potassium and calcium. Nitrogen is one of the most important
elements required for the growth of plants. However, the plants are not able to take in
elemental nitrogen. They obtain it in the form of nitrates. Ammonium salts also supply plants
with nitrogen since they are converted by soil bacteria to nitrates. Nitrogenous fertilizers
supply nitrogen together with various other elements to plants. Majority of them are
ammonium salts. They include ammonium sulphate (AS), ammonium nitrate (AN), di-
ammonium phosphate (DAP), calcium ammonium nitrate (CAN) and ammonium sulphate
nitrate (ASN).

In the laboratory, ammonium salts are made by reacting the appropriate acid with ammonia.
For example, ammonium sulphate is made by neutralizing sulphuric acid with ammonia.

2NH3(g) + H2SO4(aq) (NH4)2SO4(aq)

In industry, ammonium sulphate cannot be made using sulphuric acid, as the later is very
expensive. Instead, it is made by reacting ammonium carbonate with calcium sulphate.
Ammonium carbonate is first prepared by saturating ammonia solution with carbon dioxide.

2NH3(g) + CO2(g) + H2O(l) (NH4)2CO3(aq)

Solid calcium sulphate is added and the mixture is stirred forming ammonium sulphate
solution and calcium carbonate.
(NH4)2CO3(aq) + CaSO4(s) (NH4)2SO4(aq) + CaCO3(s)

Calcium carbonate is removed by filtration and solid ammonium sulphate is obtained by


crystallization.

26.9.13.2. Effect of heat on ammonium salts


1. Ammonium chloride sublimes when heated. The cause of this sublimation is that
ammonium chloride dissociates on heating to ammonia and hydrogen chloride, which
recombine on cooling.
NH4Cl(s) NH3(g) + HCl(g)

Also ammonium carbonate sublimes.

(NH4)2CO3(s) 2NH3(g) + CO2(g) + H2O(g)

2. Ammonium sulphate decomposes on heating into ammonia and sulphuric acid. Although
the reaction is similar to that of ammonium chloride no sublimation occurs because
sulphuric acid is less volatile than ammonia. The ammonia gas escapes before sulphuric
acid volatiles such that the two cannot recombine.
(NH4)2SO4(s) 2NH3(g) + H2SO4(g)

3. Ammonium nitrate is decomposed to nitrogen(I) oxide (dinitrogen oxide) and water.


NH4NO3(s) N2O(g) + 2H2O(g)

Dinitrogen oxide is a colourless gas. It is fairly soluble in water and neutral to litmus. It is
denser than air and a glowing splint is relit when lowered into a gas-jar containing
dinitrogen oxide. The heat decomposes dinitrogen oxide into oxygen and nitrogen. It is
oxygen that relights the glowing splint.

2N2O(g) 2N2(g) + O2(g)


Caution: Ammonium nitrate should not be heated in the laboratory because it explodes
on storng heating.

4. Ammonium nitrate decomposes to nitrogen and water.


NH4NO2(s) N2(g) + 2H2O(g)

26.9.13.3. Test for ammonium salts


When ammonium salts are heated with an alkali, a colourless gas (ammonia) which has a
pungent choking smell and turns wet red litmus paper blue is given off.

NH4+(aq) + OH-(aq) NH3(g) + H2O(l)

Exercise

State what would be observed and write an ionic equation for the reaction that would take
place when aqueous ammonium chloride was

(a) Heated with sodium hydroxide solution.


(b) Added to silver nitrate solution.

26.9.14. Reactions of ammonia solution and sodium hydroxide solution

Ammonia solution neutralizes acids forming a salt and water only.

2NH4OH(aq) + H2SO4(aq) (NH4)2SO4(aq) + 2H2O(l)

Ammonia solution precipitates metal hydroxides from solutions containing the metal ions.
When a few drops of ammonia solution are added to a solution of copper(II) ions, a blue
precipitate is formed.

Cu2+(aq) + 2OH-(aq) Cu(OH)2(s)


When excess aqueous ammonia is added to the blue precipitate, the precipitate dissolves to
form a deep blue solution containing complex tetraamine copper(II) ions.

Cu(OH)2(s) + 4NH3(aq) [Cu(NH3)4]2+(aq) + 2OH-(aq)

A solution of zinc ions forms a white precipitate with a few drops of aqueous ammonia. The
precipitate dissolves in excess ammonia solution to form a colourless solution containing
complex tetraamine zinc ions.

Zn2+(aq) + 2OH-(aq) Zn(OH)2(s)

Zn(OH)2(s) + 4NH3(aq) [Zn(NH3)4]2+(aq) + 2OH-(aq)

Iron(II), iron(III), lead(II) and aluminum ions form precipitates of the hydroxides with
aqueous ammonia which are insoluble in excess ammonia solution.

Fe2+(aq) + 2OH-(aq) Fe(OH)2(s)

(green)

Fe3+(aq) + 3OH-(aq) Fe(OH)3(s)

(brown)

Pb2+(aq) + 2OH-(aq) Pb(OH)2(s)

(white)

A13+(aq) + 3OH-(aq) A1(OH)3(s)

(white)

A solution of aluminium, Zinc and lead(II) ions reacts with sodium hydroxide solution to
form a white precipitate that is soluble in excess sodium hydroxide solution to form a
colourless solution.
A13+(aq) + 3OH-(aq) A1(OH)3(s)

A1(OH)3(s) + OH-(aq) A1(OH)4-(aq)

(aluminate ion)

Zn2+(aq) + 2OH-(aq) Zn(OH)2(s)

Zn(OH)2(s) + 2OH-(aq) Zn(OH)42-(aq)

(zincate ion)

Pb2+(aq) + 2OH-(aq) Pb(OH)2(s)

Pb(OH)2(s) + 2OH-(aq) Pb(OH)42-(aq)

(plumbate ion)

Iron(II) and iron(III) ions in solution, react with sodium hydroxide solution to give a green
and brown precipitate respectively, insoluble in excess sodium hydroxide solution.

Fe2+(aq) + 2OH-(aq) Fe(OH)2(s)

Fe3+(aq) + 3OH-(aq) Fe(OH)3(s)

Magnesium and calcium ions in solution react with sodium hydroxide solution to give a
white precipitate insoluble in excess sodium hydroxide solution.

Ca2+(aq) + 2OH-(aq) Ca(OH)2(s)

Mg2+(aq) + 2OH-(aq) Mg(OH)2(s)

26.9.15. Nitric acid

26.9.15.1. Laboratory preparation of nitric acid


When a mixture of potassium nitrate and concentrated sulphuric acid is heated gently,
potassium nitrate gradually dissolves and effervescence occurs givning off nitric acid which
is condensed in another flask placed in a sink and cooled by tap water as shown in figure
2.11.

KNO3(s) + H2SO2(l) KHSO4(s) + HNO3(g)

Brown fumes of nitrogen dioxide are produced during heating because of thermal
decomposition of nitric acid.

4HNO3(g) 2H2O(l) + 4NO2(g) + O2(g)

The experiment must be carried out in all-glass apparatus because nitric vapour attacks
rubber and cork.

26.9.15.2. Industrial preparation of nitric acid


Nitric acid is manufactured by the catalytic oxidation of ammonia. Ammonia and excess air
are passed over a heated platinum catalyst at about 800oC, forming nitrogen monoxide. The
reaction is exothermic.

4NH3(g) + 5O2(g) 4NO(g) + 6H2O(l)

Nitrogen monoxide is cooled and reacts with oxygen from excess air to produce brown fumes
of nitrogen dioxide.

2NO(g) + O2(g) 2NO2(g)

Nitrogen dioxide together with excess air is dissolved in hot water to form nitric acid.

2H2O(l) + 4NO2(g) + O2(g) 4HNO3(aq)

26.9.15.3. Uses of nitric acid

1. In the manufacture of fertilizers such as ammonium nitrate. It is manufactured by reacting


ammonia gas and nitric acid.
NH3(g) + HNO3(aq) NH4NO3(aq)

2. Used for the manufacture of dyes and explosives.


3. Used in manufacture of drugs.

26.9.15.4. Properties of nitric acid

It behaves chemically in two ways.

1. It is a strong acid.
2. It is a powerful oxidizing agent.
26.9.15.5. Nitric acid acting as a strong acid

Nitric acid is a very strong acid, being almost completely ionized in dilute solution with the
production of the hydrogen ion and the nitrate ion.

HNO3(aq) H+(aq) + NO3-(aq)

This ionization confers on it the usual acidic properties, modified to some extent by powerful
oxidizing action of the acid.

(a) It liberates carbon dioxide from carbonate and hydrogencarbonate.


CuCO3(s) + 2HNO3(aq) Cu(NO3)2(aq) + H2O(l) + CO2(g)

NaHNO3(s) + HNO3(aq) NaNO3(aq) + H2O(l) + CO2(g)

(b) It reacts with oxides and alkalis to form salt and water only.
CuO(s) + 2HNO3(aq) Cu(NO3)2(aq) + H2O(l)

NaOH(aq) + HNO3(aq) NaNO3(aq) + H2O(l)

(c) Hydrogen is liberated when very dilute acid ia added to magnesium.


Mg(s) + 2HNO3(aq) Mg(NO3)2(aq) + H2(g)

Magnesium is the only metal that liberates hydrogen with nitric acid and only when the
acid is very dilute. Other metals are oxidised by the acid to the corresponding nitrates.

26.9.15.6. Nitric acid as an oxidizing agent


(a) When concentrated nitric acid is added to a green solution of iron(II) sulphate and
warmed, it oxidizes it to a yellow or brown solution of iron(III) sulphate.
Fe2+(aq) Fe3+(aq) +e-
(b) Concentrated nitric acid reacts with copper giving off nitrogen dioxide.
4HNO3(l) + Cu(s) Cu(NO3)2(aq) + 2H2O(l) + 2NO2(g)

If the acid is 50% concentrated (equal volume of water as the volume of acid), nitrogen
monoxide is formed.

3Cu(s) + 8HNO3(aq) 3Cu(NO3)2(aq) + 4H2O(l) + 2NO(g)

Lead reacts with nitric acid in a similar way. Aluminium and iron are made assive
because of the formation of the oxide layer, which forms a protective layer over the metal
and stops further reaction.

(c) Reaction with non-metals


(i) Concentrated nitric acid reacts with sulphur to give brown fumes of nitrogen
dioxide.
S(s) + 6HNO3(l) H2SO4(aq) + 2H2O(l) + 6NO2(g)

(ii) When a piece of red-hot charcoal is put into concentrated nitric acid, it continues to
burn and brown fumes are formed.
C(s) + 4HNO3(l) CO2(g) + 4NO2(g) + 2H2O(l)

(iii) When red phosphorus is gently heated with moderately dilute nitric acid, brown
fumes are formed.
P(s) + 5HNO3(aq) H3PO4(aq) + H2O(l) + 5NO2(g)

(d) Other oxidation reactions


When hydrogen sulphide is passed through moderately dilute nitric acid, a pale yellow
precipitate of sulphur and nitric acid is reduced to nitrogen monoxide. With concentrated
nitric, nitrogen dioxide is formed.

3H2S(g) + 2HNO3(aq) 3S(s) + 2NO(g) + 4H2O(l)

H2S(g) + 2HNO3(l) S(s) + 2NO2(g) + 2H2O(l)


26.10. Nitrates

26.10.1. Action of heat on nitrates


Nitrates of potassium and sodium when heated melt to a colourless liquid and then slowly
decompose to give a pale yellow nitrate and a colourless gas which rekindles (re-lights) a
glowing splint.

2NaNO3(s) 2NaNO2(l) + O2(g)

2KNO3(s) 2KNO2(l) + O2(g)

Lead(II) nitrate makes a cracking sound when heated. The sound is due to the fact that the air
inside the crystals splits them when it expands due to heating. A brown mixture of nitrogen
dioxide and oxygen is given off. Lead(II) oxide (residue) is brown when hot and yellow
when cold.

2Pb(NO3)2(s) 2PbO(s) + 4NO2(g) + O2(g)

Most metallic nitrates decompose to a metal oxide, nitrogen dioxide (brown fumes) and
oxygen gas which relights a glowing splint.

2Ca(NO3)2(s) 2CaO(s) + 4NO2(g) + O2(g)

(white) (white)

2Mg(NO3)2(s) 2MgO(s) + 4NO2(g) + O2(g)

(white) (white)

2Zn(NO3)2(s) 2ZnO(s) + 4NO2(g) + O2(g)

(white)

2Cu(NO3)2(s) 2CuO(s) + 4NO2(g) + O2(g)

(green) (black)
Zinc oxide is yellow when hot and white when cold. Zinc nitrate and copper(II) nitrate are
hydrated and when heated do not produce a cracking sound. They melt first and dissolve in
their water of crystallization forming a solution. The solution then evaporates and when most
of the water has evapourated, decomposition starts. Mercury(II) nitrate and silver nitrate
decompose to the metal, nitrogen dioxide and oxygen.

Hg(NO3)2(s) Hg(1) + 2NO2(g) + O2(g)

2AgNO3(s) 2Ag(s) + 2NO2(g) + O2(g)

Exercise

When a green compound W was heated strongly, a brown gas was given off and a black
residue remained.

(a) Name the


(i) brown gas
(ii) black residue
(b) Write the equation for the reaction.
(c) Dilute nitric acid was added to the black residue and warmed.
(i) State what was observed.
(ii) Write the equation for the reaction.
(d) To the products in (c) was added aqueous ammonia drop wise until in excess.
(i) State what was observed.
(ii) Write the equation(s) for the reaction(s) that took place.

26.10.2. Test for nitrates

1. Brown ring test:


To a solution of a nitrate in a test-tube, an equal volume of freshly prepared iron(II)
sulphate solution is added. The test-tube is held in a slanting position and very carefully
concentrated sulphuric acid is poured down the sides of the test-tube. Concentrated
sulphuric acid is denser than the solution and therefore sinks to the bottom. A brown ring
forms where the two layers meet as shown in figure 2.12.
The formula of the brown ring is FeSO4.NO. Concentrated sulphuric acid reacts with
nitrate ions to give nitric acid.

H+(aq) + NO3-(aq) HNO3(aq)

Nitric acid formed then oxidizes iron(II) to iron(III) and itself reduced to nitrogen
monoxide.

Fe2+(aq) Fe3+(aq) + e-

4HNO3(aq) 2H2O(l) + 4NO(g) + 3O2(g)

Nitrogen monoxide combines with the remaining iron(II) sulphate to form the dark brown
compound, nitroso-iron(II) sulphate.

FeSO4(aq) + NO(g) FeSO4.NO(aq)

The ring disappears if the solution is shaken. This is because when concentrated sulphuric
acid and water mix, a lot of heat is evolved which decomposes the compound.

FeSO4.NO(aq) FeSO4(aq) + NO(g)

2. Using hot concentrated sulphuric acid


A solid nitrate is gently heated with concentrated sulphuric acid in a test-tube forming
nitric acid. The top part of the tube is heated to decompose nitric acid forming brown
fumes of nitrogen dioxide.
4HNO3(g) 2H2O(l) + 4NO2(g) + O2(g)

3. Copper and concentrated sulphuric acid


Mix a solid nitrate with copper and heat gently with concentrated sulphuric acid. Nitric
acid formed reacts with copper forming brown fumes of nitrogen dioxide.

Cu(s) + 4HNO3(l) Cu(NO3)(aq) + 2H2O(l) + 2NO2(g)

Exercise

1. Nitric acid is manufactured by catalytic oxidation of ammonia.


(a) Name
(i) Two raw materials, other than ammonia that are used in the manufacture of nitric
acid.
(ii) The catalyst used.

(b) Write equation for the reaction between nitric acid and ammonia.
(c) State one use of the product in (b).

2. (a) Describe the industrial preparation of nitric acid from ammonia. Your
description should include equations for the reactions that occur.

(b) Explain what happens when concentrated nitric acid is added to copper.

(c) Describe one chemical test that can be used to confirm the presence of a

nitrate.

(d) State what would be observed if concentrated nitric acid was heated with

iron(II) sulphate solution.

3. (a) Draw a labeled diagram of the apparatus that can be used to prepare ammonia
in the laboratory.

(b) Describe an experiment that can be carried out to show that ammonia is a

soluble alkaline gas.

(c) A copper coil was heated strongly and held over a concentrated solution of

ammonia in a beaker. Oxygen was then bubbled into ammonia solution.

(i) State what was observed.


(ii) Explain the observation in (i).

4. (a) A piece of burning magnesium was introduced into a jar of nitrogen.


(i) State what was observed.
(ii) Write an equation for the reaction that took place.
(b) Water was added to the product of the reaction in (a) and the resultant mixture

tested with litmus.

(i) State what was observed.

(ii) Write an equation for the reaction.

(c) Name one other metal that reacts with nitrogen in a similar way to magnesium.

5. When compound x is heated with concentrated sulphuric acid, a gas which forms dense
white fumes with ammonia is liberated.
(a) Identify the anion in x.
(b) Write an ionic equation for the reaction between a solution of x and silver nitrate.
(c) State what would be observed if lead(II) nitrate solution was added to solution of x
and the mixture heated.

6. In the industrial preparation of ammonia, nitrogen is reacted with hydrogen.


(i) Write the equation for the reaction.
(ii) Give the conditions under which the reaction takes place.
(iii) State what is observed when aqueous ammonia is added drop wise to copper(II)
sulphate solution until in excess.
(iv) Write an equation for the reaction between ammonia and hydrogen chloride gas.
(v) State what was observed in (iv) above.

7. Dilute nitric acid reacts with copper to form a colourless gas, which on exposure to air
gives brown fumes soluble in water.
(a) Write the equation for the reaction between copper and nitric acid.
(b) Name the colourless gas.
(c) Explain how the brown fumes are formed.
(d) Write the equation to show the reaction between water and the brown fumes.

8. Excess lead(II) oxide was added to warm dilute nitric acid and the mixture was stirred.
After cooling, the mixture was filtered and a solution of ammonium hydroxide was added
to the filtrate.
(a) Write an equation for the reaction between lead(II) oxide and nitric acid.
(b) State what was observed when ammonium hydroxide solution was added to the
filtrate drop wise until in excess.
(c) Write an equation for the reaction in (b) above.

9. What would be observed if dilute sodium hydroxide solution was added drop wise until
in excess to a solution of
(i) Fe2+ salt.
(ii) Fe3+ salt.

10. (a) Describe how zinc sulphate crystals can be prepared from zinc in the
laboratory.

(b) A small amount of zinc sulphate was dissolved in dilute nitric acid and the

resultant solution divided into two portions.

(i) State what would be observed when sodium hydroxide solution is added to

the first portion drop wise until in excess.

(ii) Write the equation(s) of the reaction(s) that took place.

(c) (i) State what would be observed when aqueous ammonia is added to the

second portion drop wise until in excess.

(ii) Write the equation(s) for the reaction(s)

11. Study the figure 2.13 and answer questions that follow.
(a) Name
(i) gas X
(ii) liquid W.
(iii) one reagent that can be used to identify W.

(b) (i) State what is observed in the combustion tube.


(ii) Write the equation for the reaction.

(iii) Name another oxide that shows similar reaction with gas X.

(c) Why is it possible to collect nitrogen gas as shown?


(d) Nitrogen gas can be obtained from air on a large scale. State the method used.

12. Lead(II) nitrate was heated strongly in the apparatus shown in figure 2.14.
(a) Identify
(iii) Liquid Q.
(iv) Gas X.
(b) (i) State what was observed in the test-tube during the heating.
(ii) Write the equation for the reaction that took place.

(c) To the residue was added dilute nitric acid and the mixture warmed.
(i) Write the equation for the reaction.
(ii) State what was observed.
(d) To the resultant product in (c) was added sodium hydroxide solution drop wise until
in excess.
(i) State what was observed.
(ii) Write the equation(s) for the reaction(s) that took place.

13. Figure 2.15 shows the apparatus used for combustion of ammonia.
(a) Name
(i) gas Q
(ii) X
(b) Write the equation for the reaction that occurred in the test-tube.
(c) Name another substance that can be used instead of ammonium sulphate.
(d) State the role of
(i) The glass wool.
(ii) Calcium oxide.
(e) Explain why concentrated sulphuric acid is not used instead of calcium oxide.
(f) Write the equation for the combustion of ammonia.
(g) State one industrial use of ammonia.
14. Figure 2.16 shows an experimental setup for the laboratory preparation of nitrogen gas.

(a) What is the purpose of passing water into bottle X?


(b) Name liquid Y and state its role.
(c) (i) State the role of copper turnings.
(ii) State the conditions for the reaction taking place in the combustion tube.

(iii) Write the equation for the reaction taking place in the combustion tube.
(d) Explain why nitrogen gas collected in this experiment is not pure.
27. Chlorine and its compounds

27.1. Chlorine
Chlorine is element number 17 in the periodic table of elements. It belongs to group VII, the
halogens. Chlorine comes from the Greek word chloros, meaning green.

27.1.1. Laboratory preparation of chlorine


a) By oxidation of concentrated hydrochloric acid with manganese(IV) oxide

The experiment is set up as shown in figure 3.1. Concentrated hydrochloric acid is poured
into a flask containing manganese(IV) oxide and the flask shaken well. The mixture is heated
and chlorine gas formed is passed through a bottle containing water to dissolve any fumes of
hydrogen chloride, which are produced from concentrated hydrochloric acid. It is then dried
by passing it through concentrated sulphuric acid and collected in a gas-jar by downward
delivery because it is denser than air.

MnO2(s) + 4HCl(l) MnCl2(aq) + 2H2O(l) + Cl2(g)


b) By oxidation of concentrated hydrochloric acid with potassium permanganate solid
potassium permanganate is placed in a flask and concentrated hydrochloric acid is
dropped on to it from a tap funnel as shown in figure 3.2 Green/yellow gas is produced
which is collected over brine.

2KMnO4(s) + 16HCl(l) 2KCl(aq) + 2MnCl2(aq) + 8H2O(l) + 5Cl2(g)

The experiment need not be conducted in a fume-chamber, if the gas is collected over
brine.

c) Preparation of chlorine from bleaching powder


Dilute nitric acid or hydrochloric acid can be used together with bleaching powder. The
experiment is set up as shown in figure 3.2

CaOCl2(s) + 2HNO3(aq) Ca(NO3)2(aq) + H2O(l) + Cl2(g)

CaOCl2(s) + 2HCl(aq) CaCl2(aq) + H2O(l) + Cl2(g)

27.1.2. Industrial manufacture of chlorine


Chlorine is produced commercially by the electrolysis of sodium chloride solution (brine)
using a carbon anode and a mercury cathode. Sodium is deposited at the cathode and chlorine
gas liberated at the anode.

Reaction at cathode:

Na+(aq) + e- Na(s)
Reaction at anode:

2Cl-(aq) Cl2(g) + 2e-

Sodium ion is discharged because it requires less energy than the discharge of hydrogen ions
in case a mercury cathode is used.

27.1.3. Properties of chlorine


Chlorine bleaches damp blue and red litmus paper. Blue litmus paper is bleached after
turning red. It is a greenish yellow gas with a chocking irritating smell. It is fairly soluble in
water.

1. Reaction with phosphorus


When a piece of yellow phosphorus is lowered in a gas-jar full of chlorine, it burns
spontaneously (without application of heat) giving off white fumes of phosphorus trichloride
(phosphorus(III)chloride) and phosphorus pentachloride (phosphorus(V) chloride).

P4(s) + 6Cl2(g) 4PCl3(s)

P4(s) + 10Cl2(g) 4PCl5(s)

2. Affinity for hydrogen


Chlorine readily combines with substances, which contain hydrogen. When a piece of filter
paper is dipped in warm turpentine (C10H16) and then dropped in a gas-jar full of chlorine,
there is a red flash accompanied by a violent reaction and a black cloud of solid particles of
carbon is formed. Chlorine combines with hydrogen in turpentine leaving the black carbon
behind.

C10H16(l) + 8Cl2(g) 10C(s) + 16HCl(g)

3. Reaction with hydrogen


When a jet of burning hydrogen is lowered into a gas-jar full of chlorine, hydrogen continues
to burn with a white flame and clouds of steamy fumes of hydrogen chloride are formed
(figure 3.3). The greenish yellow colour of chlorine gradually disappears.

H2(g) + Cl2(g) 2HCl(g)


4. Reaction with hydrogen sulphide
Chlorine reacts with hydrogen sulphide forming a yellow particles of sulphur and white
fumes of hydrogen chloride.

H2S(g) + Cl2(g) 2HCl(g) + S(s)

5. Action of chlorine on iron


A stream of dry chlorine gas is passed over a coil of iron wire as shown in figure 3.4. On
heating the iron wire, it glows and continues to burn without further application of the flame.
It reacts with chlorine to form brown fumes of iron(III) chloride which condense in a bottle
placed at the end of the combustion tube as black crystals.

2Fe(s) + 3Cl2(g) 2FeCl3(s)

A tube containing the drying agent, anhydrous calcium chloride, is connected to the bottle to
prevent water from the atmosphere from entering the bottle as this would be absorbed by
iron(III) chloride which is very deliquescent. Excess chlorine, which is poisonous, escapes into
the fume chamber.

The formation of iron(III) chloride and not iron(II) chloride shows that chlorine is an oxidizing
agent. Iron(II) chloride is immediately oxidised by chlorine to iron(III) chloride.
Sodium chloride can be prepared in similar way.

2Na(s) + Cl2(g) 2NaCl(s)

Iron(II) chloride (white solid) is made in the same way, using dry hydrogen chloride
instead of chlorine.

2HCl(g) + Fe(s) FeCl2(s) + H2(g)

Sodium, potassium, calcium and magnesium burn in chlorine forming white fumes of the
chloride which settle to a white solid.

Note: When iron(III) chloride crystals are dissolved in water they give a deep yellow
solution from which yellow crystals of hydrated iron(III) chloride may be obtained by
evaporation to the point of crystallization.

6. Reaction with a dutch metal


When a thin strip of dutch metal (alloy of copper and zinc, mainly copper) is placed in a
gas-jar full of chlorine, it burns spontaneously with a green flame (due to the copper) to
form copper(II) chloride and a little of zinc chloride.

Cu(s) + Cl2(g) CuCl2(s)

Zn(s) + Cl2(g) ZnCl2(s)

7. Action of chlorine on iron(II) chloride solution


When a stream of chlorine is bubbled through a pale green solution of iron(II) chloride,
the colour of the solution changes to yellow. Chlorine oxides the iron(II) ions to iron(III)
ions.

2FeCl2(aq) + Cl2(g) 2FeCl3(aq)

Or 2Fe2+(aq) + Cl2(g) 2Fe3+(aq) + 2Cl-(aq)

8. Reaction with alkalis


(i) Cold dilute alkalis

When chlorine gas is bubbled through cold aqueous alkalis, the hypochlorite and the
chloride of the metal are formed.

Cl2(g) + 2NaOH(aq) NaOCl(aq) + NaCl(aq) + H2O(l)

Cl2(g) + 2KOH(aq) KOCl(aq) + KCl(aq) + H2O(l)

(ii) Hot concentrated alkalis

When chlorine is passed into hot concentrated alkalis, a mixture of the chlorate and
the chloride is formed.

6KOH(aq) + 3Cl2(g) KClO3(aq) + 5KCl(aq) + 3H2O(l)

6NaOH(aq) + 3Cl2(g) NaCl3(aq) + 5NaCl(aq) + 3H2O(l)

or 6OH-(aq) + 3Cl2(g) ClO3-(aq) + 5Cl-(aq) + 3H2O(l)

9. Bleaching action of chlorine


Chlorine reacts with water forming hypochlorous acid and hydrochloric acid.
Cl2(g) + H2O(l) HCl(aq) + HOCl(aq)

Hypochlorous acid is a very reactive compound and readily gives up its oxygen to the
dye, to form a colourless compound, that is, the dye is oxidised to a colourless
compound.

Dye + HOCl(aq) HCl(aq) + (dye + O)

Colourless

Dry litmus paper is not bleached by chlorine because there is no formation of


hypochlorous acid. Sodium hypochlorite and potassium hypochlorite have a bleaching
effect.

10. Effect of sunlight on chlorine water


Chlorine gas is passed into distilled water for sometime until the water becomes yellow-
green in colour. When an inverted tube containing this chlorine water is exposed to
sunlight as shown in figure 3.5, a colourless gas (oxygen) which re-lights a glowing splint
is formed.

2Cl2(g) + 2H2O(l) 4HCl(aq) + O2(g) (overall equation)

This occurs in two stages.

(i) Formation of hypochlorous acid and hydrochloric acid as a result of reaction

between water and chlorine.

H2O(l) + Cl2(g) HOCl(aq) + HCl(aq)

(ii) Decomposition of hypochlorous acid to liberate oxygen.

2HOCl(aq) 2HCl(aq) + O2(g)


11. Displacement reactions of chlorine
Chlorine is higher in the reactivity series than bromine and iodine and therefore can
displace them from solutions of their salts in water. This is because chlorine is more
reactive than bromine and iodine due to the fact the incoming electron is more strongly
attracted into the outer energy level of the smaller atom. The attraction force on the
electron will be greater for chlorine than for bromine and iodine, since the outer energy
level of chlorine is closer to the nucleus. As one goes down the group, the extra election
is further away from the nucleus. It will, therefore, be attracted less strongly thus the
reactivity of the halogens decreases down the group.

Order of reactivity of halogens

F Flourine most reactive

C1 Chlorine

Br Bromine

I Iodine least reactive

When chlorine gas is bubbled into a solution of potassium bromide in water, the
colourless solution immediately turns red due to formation of bromine water.

2KBr(aq) + Cl2(g) 2KCl(aq) + Br2(aq)

Chlorine displaces iodine from potassium iodine solution forming a dark brown solution
due to formation of iodine.

2KI(aq) + Cl2(g) 2KCl(aq) + I2(aq)

Bromine can displace iodine from iodides but cannot displace chlorine from chlorides.
Therefore on addition of few drops of bromine to a solution of potassium iodine in water,
the solution becomes brown due to the formation of iodine.
2KI(aq) + Br2(l) 2KBr(aq) + I2(aq)

27.1.4. Tests for chlorine


It is a greenish yellow gas, which turns damp blue litmus paper red, then bleaches it. Since
the gas is acidic, the damp blue litmus paper first turns red and then bleached.

27.1.5. Uses of chlorine


1. It is used as a bleaching agent. It is used in paper industry and textile industry to bleach
wood pulp and cotton respectively.
2. It is used in the manufacture of hydrogen chloride by burning hydrogen and chlorine.
H2(g) + Cl2(g) 2HCl(g)

3. Chlorine is used to kill micro-organisms during sewage and water treatment.


4. It is used in manufacture of polychloroethene or polyvinyl chloride.

27.2. Hydrogen chloride

27.2.1. Laboratory preparation of hydrogen chloride

Concentrated sulphuric acid is added to sodium chloride in the flask as shown in figure 3.6.
Effervescence occurs and misty fumes of a gas are formed. The gas is passed through a bottle
containing concentrated sulphuric acid to dry it and collected by upward displacement of air
since the gas is denser than air.

H2SO4(l) + NaCl(s) NaHSO4(aq) + HCl(g)


Sodium sulphate is not formed because it requires higher temperatures than is provided in
this experiment. Sodium chloride is the one most commonly used because it is cheap and
readily available.

27.2.2. Test for hydrogen chloride


It produces a white precipitate of silver chloride with silver nitrate solution.

AgNO3(aq) + HCl(g) AgC1ls) + HNO3(aq)

It also produces dense white fumes of ammonium chloride with ammonia.

NH3(g) + HCl(g) NH4Cl(g)

27.2.3. Properties of hydrogen chloride


1. It is a white fuming gas with a choking smell.
2. It turns damp blue litmus paper red.
3. It is very soluble in water. This can be shown by the fountain experiment.
4. It liberates hydrogen from certain metals. All metals above copper in the reactivity series
react with hydrogen chloride to form hydrogen and the corresponding chloride.
Fe(s) + 2HCl(g) FeCl2(s) + H2(g)

Mg(s) + 2HC2(g) MgCl2(s) + H2(g)

27.3. Hydrochloric acid

27.3.1. Preparation of hydrochloric acid


It can be prepared by dissolving hydrogen chloride gas in water using the setup shown in
figure 3.7. The filter funnel is used in passing hydrogen chloride into water in order to
prevent the sucking back of water from the beaker into the reaction flask. Hydrogen
chloride is so soluble in water that, in the process of dissolving, too much of it could dissolve
at one time creating a low gaseous pressure in the reaction flask as well as in the delivery
tube by the atmospheric pressure outside. The funnel is arranged with its rim only just
immersed in the water in order to ensure that when water is sucked into the funnel, contact
with the water is broken and the water falls back into the beaker rather than being sucked
back along the delivery tube.
27.3.2. Properties of hydrochloric acid
1. It turns blue litmus paper red.
2. It reacts with metals producing hydrogen gas. It reacts with metals above hydrogen in the
activity series.

Mg(s) + 2HCl(aq) MgCl2(aq) + H2(g)

Zn(s) + 2HCl(aq) ZnCl2(aq) + H2(g)

3. It liberates carbon dioxide from carbonates and hydrogencarbonates.


CaCO3(s) + 2HCl(aq) CaCl2(aq) + H2O(l) + CO2(g)

NaHCO3(s) + HCl(aq) NaCl(aq) + H2O(l) + CO2(g)

4. It reacts with alkalis and basic oxides producing salt and water only.
NaOH(aq) + HCl(aq) NaCl(aq) + H2O(l)

CuO(s) + 2HCl(aq) CuCl2(aq) + H2O(l)

5. Concentrated hydrochloric acid reacts with oxidizing agents such as potassium


permanganate liberating chlorine gas.
2KMnO4(s) + 16HCl(l) 2KCl(aq) + 2MnCl2(aq) + 8H2O(l) + 5Cl2(g)
27.3.3. Uses of hydrochloric acid
1. It is used in removal (de-scaling) of rust from iron before it is galvanized. It is also used
in cleaning metals before they are electroplated.
2. It is used in manufacture of plastics such as polychloroethene.

27.3.4. Properties of hydrogen chloride in methylbenzene


Dry hydrogen chloride is a covalent compound and therefore it dissolves in organic solvents
such as methylbenzene. In the solution, the dissolved hydrogen chloride does not ionize, and
remains in the molecular state because methibenzene is not a proton (H +) acceptor. Therefore
the solution contains no ions and does not conduct electricity. Water, however, is a proton
acceptor and in dilute aqueous solution, hydrogen chloride is fully ionized.

HCl(g) + H2O(l) H3O+(aq) + Cl-(aq)

These ions are responsible for conducting electricity in the solution. The oxonium ion
liberates hydrogen with the more electropositive metals and carbon dioxide with carbonates
and hydrogencarbonates.

Mg(s) + 2H3O+(aq) Mg2+(aq) + 2H2O(aq) + H2(g)

CO32-(aq) + 2H3O+(aq) 3H2O(l) + CO2(g)

HCO3-(aq) + H3O+(aq) 2H2O(l) + CO2(g)

Thus these properties are not shown by hydrogen chloride in methylbenzene, that is, the
solution does not contain hydrogen or oxonium ions responsible for acidic characteristics and
the solution contains no ions which carry an electric current.

27.4. Testing for soluble chloride


To the solution of the suspected chloride in water, add dilute nitric acid followed by silver
nitrate solution. A white precipitate of silver chloride will be observed.

Ag+(aq) + Cl-(aq) AgCl(s)

To the white precipitate add ammonia solution. The precipitate dissolves to form a colourless
solution.

AgCl(s) + 2NH3(aq) Ag(NH3)2+(aq) + Cl-(aq)


Nitric acid prevents the precipitation of other insoluble silver salts such as silver carbonate.
The only common insoluble chlorides are lead(II) chloride and silver chloride. Lead (II)
chloride is soluble in hot water.

Exercise

1. (a) Draw a well labeled diagram to show how a sample of dry hydrogen chloride can be
prepared in the laboratory.
(b) Dry hydrogen chloride gas was passed over heated iron filings. Write an equation for
the reaction that took place.

(c) The solid product in (b) was dissolved in water and aqueous sodium hydroxide added
to the resultant solution drop wise until in excess.

(i) State what was observed.

(ii) Write equation for the reaction.

(d) Chlorine gas was passed through a solution of the product in (b)

(i) State what was observed.

(ii) Write an ionic equation for the reaction.

(e) (i) Name one reagent that can be used to test for the anion formed in (d).

(ii) State what is observed when the reagent you have named is used.

2. (a) A mixture consists of sulphur and iron filings. Explain briefly how a sample of
sulphur can be obtained from the mixture.
(b) A sample of the mixture in (a) was heated in a porcelain dish.

(i) State what was observed.

(ii) Write equation for the reaction that took place.

3. State what would be observed and write ionic equation(s) for the reaction(s) that take
place when
(i) A solution of silver nitrate is added to potassium chloride solution.
(ii) A solution of barium chloride is added to sodium sulphate solution.
4. Chlorine can be prepared in the laboratory from hydrochloric acid.
(i) Name the other reagent used in the preparation of chlorine.
(ii) State the conditions for the reaction.
(iii) Write an equations for the reaction which takes place between hydrochloric acid and
the reagent you have named in (i).

5. During the preparation of chlorine in the laboratory, the gas may be passed through water
and concentrated sulphuric acid before collection.
(a) State the use of
(i) water
(ii) concentrated sulphuric acid

(b) Chlorine is a bleaching agent when in the presence of water.


(i) Write an equation for the reaction between chlorine and water.

(ii) Using equations, explain the bleaching action of chlorine.


(c) (i) State what would be observed if chlorine was bubbled through a solution of
iron(II) sulphate.
(ii) Write an equation for the reaction between chlorine and iron(II) chloride.

(d) Burning sodium was plunged into a jar of chlorine.


(i) State what was observed.
(ii) Write the equation for the reaction.

6. (a) Draw a labeled diagram of the apparatus you would use to prepare chlorine in the
laboratory, using potassium permanganate.
(b) State what is observed when

(i) A piece of yellow phosphorus is lowered in a jar of chlorine.

(ii) Burning turpentine (C10H16) is lowered in a jar of chlorine.

(iii) Chlorine is bubbled into a solution of potassium iodide.

7. (a) With the aid of a well labeled diagram describe an experiment in the laboratory to
show that hydrogen chloride gas is very soluble in water.

(b) State what is observed when hydrogen chloride is


(i) Bubbled in lead(II) nitrate solution.

(ii) Passed through a gas-jar of ammonia.

(c) Hydrogen chloride was dried and passed over heated iron filings.

(i) State what would be oserved.

(ii) Write equation of the reaction that took place.

8. (a) (i) Draw a well labeled diagram to show the preparation of iron(III) chloride
using chlorine.

(ii) State what would be observed during the reaction.

(iii) Write an equation leading to the formation of iron(III) chloride.

(b) (i) State what would be observed if aqueous ammonia was added to a solution

of iron(III) chloride.

(ii) Write an ionic equation for the reaction in (b) (i).

9. (a) State what would be observed if chlorine is passed through


(i) litmus solution
(ii) Aqueous potassium iodide.
(iii) Cold dilute sodium hydroxide solution.
(b) Write an equation for the reaction that takes place in (ii).

10. When a compound M is heated with concentrated sulphuric acid, a gas that forms dense
white fumes with ammonia is liberated.
(a) Identify the anion in M
(b) (i) State what would be observed when a solution of M is added to silver nitrate
solution.
(ii) Write an ionic equation for the reaction which occurs in (i) above.

(c) State what would be observed when lead(II) nitrate solution is added to a solution of
M and the mixture warmed.
11. (a) The substance Y reacts with solid chloride to produce hydrogen chloride.
(i) Identify Y.
(ii) State the conditions for the reaction.
(iii) Write the equation for the reaction.
(b) (i) Name the substance that is formed when hydrogen chloride is dissolved in
water.
(ii) Explain why an aqueous solution of hydrogen chloride is an electrolyte
whereas the solution of the gas in methylbenzene is a non-electrolyte.
28. Extraction of metals

28.1. Introduction
The major factor determining the method used for extraction of metals from their ores is the
position of the metals in the electrochemical series. An ore is a naturally occurring substance
from which a metal can be extracted.

Very reactive metals, that is, those higher in the activity series occur mainly as chlorides.
They are extracted by electrolysis of their fused salts. Such metals include potassium,
sodium, calcium, magnesium and aluminium.

Metals in the middle of the series such as zinc, iron, lead and copper mainly occur as oxides,
carbonates and sulphides. They are extracted by reduction of the ore. Chemical reduction
involves the extraction of a metal from its ore by heating the ore with a strong reducing agent
such as coke. This method is used in extraction of iron. Thermal reduction involves roasting
(heating directly in air). It is applicable in the extraction of metals such as zinc, copper and
lead. At some stage in this process, the method might be intergrated with chemical reduction.

Metals lower in activity series, that is, mercury, silver and gold mainly occur as free metals in
the earths crust. They are mainly dug up in the pure form.

28.2. Concentration of ores


Very often ores are found contaminated with earthly impurities. The following methods are
employed to pick out richer ores.

1. Magnetic separation

Many metals ores containing magnetic impurities are partially refined in this way. The
crushed ore is placed on a conveyer belt, which has a magnetic roller at one end. As the
ore passes over the magnetic roller, it is separated into two parts, one containing the
partially refined ore and the other containing all the magnetic impurities.
2. Froth flotation

Froth flotation is the process in which the ore is powdered, mixed with oil and water, and
air is brown through. The froth contains the ore which is skimmed off.

3. Hydrolic method

The rock containing the ore is blasted with a stream of water and earthly matter is washed
away, leaving the heavier ores.

4. Mechanical sorting

It is a physical method of separating an ore from earthly matter by hand picking.


However, this method has a limited application because some particles of the desired ore
can be thrown away.

5. Solvent extraction

The ore is dissolved in the solvent. Some components of the ore dissolve in the solvent
while others do not.

28.3. Sodium
Sodium occurs as sodium carbonate (soda ash), sodium nitrate and sodium chloride (rock
salt).

28.3.1. Extraction of sodium


Sodium metal is extracted by electrolysis of fused sodium chloride (obtained from rock salt
or seawater) to which calcium chloride has been added to lower its melting point from 800 oC
to about 600oC. This saves electrical energy and therefore makes the process more
economical. An iron cathode and a carbon (graphite) anode are used. At 600 oC, sodium and
chlorine would react violently together to reform sodium chloride. To prevent this reaction,
the Downs cell (figure 8.1) contains steel gauze around the graphite anode to keep them
apart. Chlorine gas produced as a by-product is collected in a cylinder. Molten sodium
collects in the inverted trough, placed over the cathode, rises up the pipe, and is tapped off
and collected under dry nitrogen. Nitrogen is inert under ordinary conditions and therefore
hardly reacts with sodium. This process is called Downs process.

Reaction at the cathode:

Na+(l) + e+ Na(s)

Reaction at the anode:

2C1-(l) Cl2(g) + 2e-

28.3.2. Uses of sodium metal


1. It is used in the manufacture of anti-knock compound, tetraethyl lead, added to petrol
(regular) to make it super which does not make the engine knock. Knocking in a car
engine is caused by the petrol and air mixture burning too rapidly in the cylinders, giving
a sudden blow or knock to the piston, which is not good for the engine. Tetraethyl lead
slows down the combustion process of a mixture of petrol and air.
2. It is used in manufacture of sodium cyanide which is used in extraction of gold.
3. Used in nuclear reactions to absorb some of the heat produced during the reactions.

28.4. Copper
The principal ores of copper are copper pyrites (CuFeS 2), cuprite (Cu2O), copper(I) sulphide
(Cu2S) and malachite (CuCO3.Cu(OH)2).
28.4.1. Extraction of copper
Copper pyrites is the ore usually used for the extraction of copper and there are three stages
involved.

(i) The concentration of the ore to remove impurities.

(ii) Roasting and reduction.

(iii) Refining of the impure copper.

28.4.2. Concentration of the ore


The ore is concentrated by a process of froth flotation. The ore is crushed into a fine powder
and poured into a tank containing water and a frothing agent such as pine oil. Compressed air
is blown into the tank to keep the mixture in constant agitation. As a result of agitation,
impurities sink to the bottom of the tank. The ore particles stick to the frothing agent and
float on the surface. The ore particles with the frothing agent are skimmed off and dried.
Sulphuric acid is added to separate the ore from the frothing agent.

28.4.3. Roasting and reduction


The ore is roasted in air to produce copper(I) sulphide.

2CuFeS2 (s) + 4O2(g) CuS(s) + 3SO2(g) + 2FeO(s)

Sulphur dioxide escapes from the top of the furnace. By adding silicon dioxide, SiO 2, and
heating in absence of air, the iron(II) oxide is converted into a slag of iron(II) silicate,
FeSiO3, which floats on top of the molten copper(I) sulphide and it is tapped off.

FeO(s) + SiO2(s) FeSiO3(1)

Copper(I) sulphide is reduced to the metal by heating a regulated supply of air.

Cu2S(s) + O2(g) 2Cu(s) + SO2(g)

This copper produced is impure and is called blister copper because of the blistered
appearance on the copper surface caused by the escaping gases on cooling.
28.4.4. Refining of the impure copper
The impure copper is purified (refined) by electrolytic process using copper(II) sulphate
solution as the electrolyte. The cathode is pure copper and the impure copper is made the
anode as shown in figure 8.2.

During electrolysis, the copper atoms of the anode lose electrons to form copper(II) ions
which dissolve in the solution.

Cu(s) Cu2+(aq) + 2e-

Then the copper(II) ions are attracted to the cathode where they gain electrons and become
copper atoms.

Cu2+(aq) + 2e- Cu(s)

The overall effect is that copper gradually dissolves from the anode and is deposited on the
cathode. Copper from the cathode is removed by stripping.

Impurities which are higher than copper in the activity series, such as iron, also dissolve from
the anode but are not deposited on the cathode. They accumulate in solution in the
electrolyte. Impurities which are lower than copper in the activity series do not dissolve at
all. They fall to the bottom of the container as sludge. The elements which were present in
the original copper ore.
28.4.5. Uses of copper
1. Copper is used as a conductor of electric power in wires and cables.
2. It is used for making bronze which is used for manufacturing ball bearings.
3. It is used for making kettles for brewing beer.
4. Used for making roofing sheets because it is corrosion resistant.

28.5. Iron
The main iron ores are haematite (Fe2O3), magnetite (Fe3O4), iron disulphide (pyrites, FeS2)
and spathic iron ore (FeCO3).

28.5.1. Extraction of iron


Haematite and magnetite are the most commonly used ores for the extraction of pure iron.
The ore is first mixed with coke and limestone and the mixture is fed into the blast furnace
from the top of the furnace (figure 8.3). Hot air is blown into the furnace at the bottom which
comes into contact with the red hot coke, producing carbon dioxide.

C(s) + O2(g) CO2(g)

Higher up the furnace, the source of oxygen is less and more coke combines with carbon
dioxide produced to form carbon monoxide.

C(s) + CO2(g) 2CO(g)

Carbon monoxide produced reduces the iron oxides to molten iron.

Fe3O4(s) + 4CO(g) 3Fe(l) + 4CO2(g)

Fe2O4(s) + 3CO(g) 2Fe(l) + 3CO2(g)

Molten iron runs to the bottom of the furnace and is tapped off into moulds where it is
solidified. The moulds are called Pigs and therefore this impure form of iron is called pi-
iron. Limestone is decomposed by heat to calcium oxide and carbon doxide.

CaCO3(s) CaO(s) + CO2(g)

The iron contains impurities such as silicon dioxide (sand), which combine with calcium
oxide to form a molten slag that floats on top of the molten iron and it is tapped off.

CaO(s) + SiO2(s) CaSiO3(l)


The wastes gases, mainly nitrogen and oxides of carbon, escape from the top of the furnace.
They are used in a heat exchange process of heat incoming air and so help to reduce the
energy costs of the process. Slag is used in making road foundations, phosphorus fertilizers
and cement.

28.5.2. Casting iron (pig-iron)


This is impure iron containing impurities such as carbon, sulphur, phosphorus and silicon. It
has a low melting point and it is brittle. It is used in places where it cannot be subjected to
great strain or stress, for example, in manufacture of Bunsen burner bases and domestic
boilers.

28.5.3. Wrought iron


This is the purest form of iron and is obtained from cast iron by heating it with iron(III) oxide
in a furnace lined with limestone. Iron(III) oxide supplies oxygen for the oxidation process.
When hot air is fed into the furnace, non-metallic impurities such as carbon and sulphur are
oxidized to their oxides and are removed as waste gases.

C(s) + O2(g) CO2(g)

S(s) + O2(g) SO2(g)

Phosphorus and silicon are oxidized to solid oxides.


4P(s) + 5O2(g) 2P2O5(s)

Si(s) + O2(g) SiO2(s)

Calcium oxide (from decomposition of limestone) reacts with these solid oxides, forming
calcium phosphate and calcium silicate, which float on the surface of the iron as slag.

3CaO(s) + P2O5(s) Ca3(PO4) 2(l)

CaO(s) + SiO2(s) CaSiO3(l)

Wrought iron is malleable. It is very tough and therefore can withstand some strain. It can be
used to make iron nails, iron sheets and agricultural implements.

28.5.4. Steel
Steel is an alloy of pure iron with a small percentage of carbon and other elements. Other
such as tungsten, chromium, nickel and manganese which are added to produce types of steel
(table 8.1) with different properties. Steel is hard, tough, strong and malleable.

Steel Composition Uses

Mild steel 99% iron, 0.5% carbon Car bodies, large structures

Hard steel 99% iron, 1% carbon Cutting tools, razor blades

Manganese steel 87% iron, 13% manganese Drill bits, springs

Stainless steel 74% iron, 18% chromium, Cutlery, kitchen sinks, surgical
8% nickel instruments

Tungsten steel 95% iron, 5% tungsten Edges of high-speed cutting


tools

Table 8.1 Different types of steel

28.5.5. Recycling of metals


With increasing use of metals, natural deposits of metal ores will eventually run out. The
metal ores will last longer if metals are recycled. For example, tin can be removed from scrap
food cans to be reused in making new food cans.
Metal ores can also be made to last longer by use of alternative materials. For example,
instead of using iron, plastic materials can be used in making dustbins. Many parts of cars are
now made of plastic materials. This is also advantageous in that plastic materials do not
corrode and can be moulded into much more complex shapes and textures than the metals
which they have replaced.

28.5.6. Alloy
An alloy is a metallic substance consisting of a mixture of two or more metals or a mixture of
a metal with a non-metal. Alloys have suitable properties when compared with pure metals.
Alloys are usually less malleable and ductile than pure metals. They also have low melting
points and electrical conductivity than pure metals. For example, solder has a lower melting
point than lead and tin. Because of its low melting point, solder can be used to join metals.
Examples of common alloys are given in table 8.2.

Alloy Composition Uses

Brass Copper and zinc Ornaments, buttons and


screws

Bronze Copper and tin Ornaments

Duralumin Aluminium with small amounts of Aircraft and bicycle parts


magnesium, manganese and
copper

Solder Tin and lead Joining metals

Steel Refer to table 8.1 Refer to table 8.1

Copper coinage Copper, tin and zinc Coins

Type metal Lead, antimony and tin Printing

Silver coinage Copper and nickel Coins

Table 8.2 Common alloys

Exercise

1. (a) Extraction of metals is essentially a reduction process. Explain the statement using
extraction of iron as an example. Write the equation to illustrate your answer.
(b) State the conditions under which iron may react with

(i) Oxygen.
(ii) Water.

(iii) Chlorine.

(c) Write an equation for the reaction in (b) (ii) and (iii).

(d) Steel is an alloy of iron.

(i) Explain what is meant by an alloy.

(ii) Name the elements which are used in making stainless steel.

(iii) State one use of stainless steel.

(iv) Suggest a reason why the use of stainless is preferred to that of pure iron.

2. (a) Sodium metal is extracted by electrolysis of fused sodium chloride to which calcium
chloride has been added.
(i) Give a reason for the addition of calcium chloride.

(ii) Name a material that can be used as the cathode and another that can be used as
the anode.

(iii) Write equations for the reaction that take place at each electrode.

(iv) Describe how the product at the cathode is collected.

(v) Name one other element that can be extracted by similar method.

(b) Name a place in Uganda where a plant for the extraction of sodium could be
constructed. Give a reason for your answer.

(c) Describe what would be observed if a small piece of sodium metal was heated and
quickly plunged into a gas-jar of oxygen. Write an equation for the reaction that takes
place.

3. Iron is extracted by a reduction process in the blast furnace.


(a) Name two raw materials used besides the iron ore.
(b) Write the equation leading to the production of iron from its ore.
(c) State one use of iron.
(d) State what is observed when iron nail, is dropped in beaker of copper(II) sulphate
solution.
4. (a) (i) What is an alloy?
(ii) Give an example of an alloy.

(b) State two uses of the alloy in a (ii)

5. (a) Name one ore of iron and write its formula.


(b) During the extraction of iron, limestone and coke are added into the blast furnace.
Explain why each of the following are added into the blast furnace.

(i) Coke

(ii) Limestone
29. Organic chemistry

29.1. Introduction
Organic chemistry is a branch of chemistry dealing with compounds of carbon except oxides
of carbon, carbonates, hydrogencarbonates and carbides of metals. Carbon has the ability to
form bounds to itself. These bounds are very strong and can be single, double or triple bonds.
Owing to this fact, chains of varying sizes can be formed which contribute to a wide range of
stable compounds. These compounds are known as organic compounds. Carbon forms four
covalent bonds making it possible to have different groups attached to the chains of carbon
atoms. This will also lead to a wide diversity of compounds being formed.

29.2. Hydrocarbons
Hydrocarbons are compounds containing hydrogen and carbon atoms. They have a molecular
formula, CxHy, where x and y are whole numbers. They are classified into several types
according to their structures. The main classes of hydrocarbons are alkanes, alkenes and
alkynes.

29.3. Homologous series


A series of compounds related to each other is called a homologous series e.g. alkanes,
alkenes and alkynes. Each member is called a homologue. A homologous series has the
following characteristics.

i) All members conform to a general molecular formula. For example, in case of alkanes,
the general molecular formula is CnH2n+2 where n1.
ii) Each member differs in molecular formula from the next by CH 2, for example members
of the alkanes are CH4, C2H6, C3H8 and so on.
iii) All members show similar chemical reactions though they vary in vigour.
iv) The physical properties of members change gradually in the same direction along the
series.

29.4. Functional groups


A functional group is the most reactive part of any organic compound. It determines the
chemical properties of the homologous series. For example, for alkanes, alkenes, alkynes and
alcohols, the functional groups are carbon-hydrogen bond, carbon-carbon double bond,
carbon-carbon triple bond and hydroxyl group (-OH) respectively.
29.5. Alkanes
Alkanes are hydrocarbons with the general molecular formula, C nH2n+2, where n 1, for
example methane (CH4), ethane (C2H6), propane (C3H8), butane (C4H10), pentane (C5H12),
hexane (C6H14), heptanes (C7H16), octane (C8H18), nonane (C9H20), decane (C9H22) et. Alkanes
are saturated hydrocarbons. This means that the molecules of alkanes consist of carbon and
hydrogen atoms and single covalent bonds only. In other words, all the atoms exert their
usual combining power with other atoms. That is each carbon atom is bonded to four other
atoms. This is illustrated below.

H H

H C C H Ethane

H H

Structural formula

The structural formula shows the sequence and arrangement of atoms in a molecule. For
example, the structural formula of propane is shown below.

H H H

H C C C H or CH3CH2CH3

H H

Exercise 9.1

Write the structural formula for each of the following hydrocarbons.

(a) Hexane
(b) Butane
Alkyl groups

Removal of one hydrogen atom from an alkane molecule leaves a monovalent group called
an alkyl group. Table 9.1 shows some groups and their molecular formulae.

Alkane Molecular Alkyl group Molecular


formula formula

Methane CH4 Methyl CH3-

Ethane C2H6 Ethyl C2H5-

Propane C3H8 Propyl C3H7-

Butane C4H10 Butyl C4H9-

Pentane C5H12 Pentyl C5H11-

Nomenclature of alkanes

The general rules of naming organic compounds were laid down by the International Union
of Pure and Applied Chemistry (I.U.P.A.C.)

Rules of naming alkanes:

(a) The first step is to choose the longest chain of carbon atoms which is called the parent
chain. In the structural formula below, the longest chain has eight carbon atoms and thus
it is taken as the parent chain and it is called octane.

CH3

4
3

CH3 CH CH CH2 CH2 CH2 CH2


2
CH3

It is possible at times to have longest chains of carbon atoms and the one which is taken
as a parent chain is that one which has a higher number of the chains. In the structural
formula below, the parent chain has three side chains and it is called heptanes.

(b) After identifying the parent chain, the carbon atoms are numbered from one end to the
other. The numbering should be in such a way that the carbon atoms carrying the side
chain gets the lowest number and the position of the side chain is indicated by the number
assigned to the carbon atom to which it is attached.

(c) If there are more than one side chain, then the numbering of the carbon atoms is done in
such a way that the sum of the numbers used to locate the side chains is lowest and this is
called the lowest sum rule. Consider the structural formula below.
From left to right, the sum of the numbers where the side chains are attached

= 2 + 3 + 6 = 11

From right to left, the sum of the numbers where the side chains are attached

= 2 + 5 + 6 = 13

Therefore, the numbering is from left to right and hence the name of the compound is
2,3,6-trimethylheptane.

(d) If there is more than one type of side chain, then the side chains are prefixed and should
be put in alphabetical order preceding the name of the parent chain.

In case of a particular side chain appearing twice of three times, then di, tri, tetra, pent
etc, are used. The locants (numbers used to locate a side chain) are written in increasing
order separated by commas and hyphens (-) separates the numbers from the prefix. Refer
to the example in (c), that is 2,3,6-trimethylheptane.

Exercise

1. Give the names of the following structural formulae.


(a) CH
(b) Ch

(c) Ch

(d) Ch

(e) Ch
2. Write the structural formula of the following compounds.
(a) 2,2-dimethylpentane
(b) 3-ethylhexane
(c) 2-methylpropane

29.5.1. General properties of alkanes

29.5.1.1. Physical properties


(i) The first four members are gases, the next twelve members are liquids and the rest

are waxy solids at room temperature.

(ii) They are insoluble in water but soluble in non-polar solvents like

trachloromethane.

(iii) They are less dense than water. Their densities rise gradually with increasing

molecular mass.

29.5.1.2. Chemical properties


(i) Alkanes burn in air forming carbon dioxide and water.

CH4(g) + 2O2(g) CO2(g) + 2H2O(g)

In a limited supply of air, carbon monoxide is formed.

2CH4(g) + 3O2(g) 2CO(g) + 4H2O(g)

During combustion, a great deal of heat is liberated. Owing to that fact, alkanes are used
as fuels for industrial and domestic purposes. For example, butane is used in gas cigarette
lighters. Methane is found in natural gas and bio gas. It is used in gas appliances. Butane
found in petrol is used to run petrol engines.
(ii) They undergo substitution reactions with the halogens, producing corresponding
compounds. A substitution reaction is a reaction in which one atom or group of atoms in a
molecule is replaced by another. For example, methane reacts with chlorine forming
chloromethane and hydrogen chloride, the reaction being catalysed by light (photo catalysis).

CH4(g) + Cl2(g) CH3Cl(g) + HCl(g)

One hydrogen atom of the methane molecule has been replaced by a chlorine atom. In a
similar way, excess of chlorine may produce dichloromethane (CH2Cl2), trichloromethane
(CHCl3) and tetrachloromethane (CCl4).

CH3Cl(g) + Cl2(g) CH2Cl2(l) + HCl(g)

Dichloromethane

CH2Cl2(g) + Cl2(g) CHCl3(l) + HCl(g)

Trichloromethane

CHCl3(g) + Cl2(g) CCl4(l) + HCl(g)

tetrachloromethane

Exercise

When a mixture of ethane and chlorine was exposed to sunlight, the colour of chlorine
disappears.

(a) Write the equation for the reaction that takes place when ethane is mixed with a limited
amount of chlorine.
(b) Name the type of reaction.
(c) What is the role of sunlight in this reaction?

Isomerism

Isomerism is the occurrence of two or more compounds with the same molecular formula but
different structural formulae. Compounds which have the same molecular formula but
different structural formulae are called isomers. All alkanes with more than four carbon
atoms have more than one structure for a given molecular formula, that is, they exhibit
isomerism. The greater the number of carbon atoms in an alkane, the greater the number of
possible isomers.
The easiest way of finding the isomers is to draw the longest chain of carbon atoms first and
reduce it by one carbon atom at a time.

Isomers of butane (molecular formula, C4H10)

Isomers of pentane (molecular formula, C5H12)


Exercise

Compound Q has a molecular formula, C6H14. Give the structural formulae and names of
possible isomers of compound Q.

29.6. Petroleum (crude oil)


Petroleum is an organic mineral. Petroleum was formed from the remains of plants and
animals which lived millions of years of ago. Their dead bodies sank to the bottom of seas
and were soon covered with mud and sand. The chemical effects of pressure, heat and
bacteria converted the remains into petroleum. Petroleum is composed mainly of
hydrocarbons although some of its constituents contain elements such as oxygen, sulphur and
nitrogen.

29.6.1. Fractional distillation of petroleum


Petroleum is a complex mixture of gases, liquids and solids. Its components can be separated
by fractional distillation. Petroleum is pumped through a furnace where it is heated by
electricity or gas in a furnace to about 400oC. The hot liquid crude oil is then passed into a
fractionating tower which consists of trays and bubble caps placed at different levels of
different temperatures. In other words, each tray is a little cooler than the one below because
it is a little further from the source of heat. As the hot mixture of gases and vapours passes up
the tower from tray to tray via bubble caps, the various vapours condense at different levels,
according to their boiling points. The products which are gases at room temperature leave at
the top of the tower as shown in figure 9.1. The fractions (groups of compounds of similar
size) collected in each tray are tapped off and redistilled. After fractional distillation,
impurities are removed. The commonest impurity is sulphur, which is removed and used to
manufacture sulphuric acid. The table 9.2 shows the different fractions of crude oil and their
uses.
Straight-chain alkanes obtained from the fractional distillation of crude oil, are purified and
mixed with an aqueous solution of yeast. The yeast acts on the alkanes and converts them to
proteins, which are separated out and dried. The end-product is a powder which is used as
animal feed.

Fraction Uses

1. Gas (propane and butane) Fuel for cooking and lighting

2. Petrol Motor and aviation fuel

3. Paraffin oil (kerosene) Lighting and heating

4. Gas oil To make petrol

5. Diesel oil Diesel engines

6. Lubricating oil Lubrication in machine parts

7. Waxes and bitumen Vaseline, greases and candles

Road and runway surfacing

Table 9.2 Fractions of crude oil and their uses


29.6.2. Cracking
Fractional distillation of crude oil yields only about 20 percent of petrol. With the demand for
petrol increasing, it has become necessary to devise a new process of obtaining it, that is, by
cracking of gas oil. Cracking is the process of breaking down the long chain hydrocarbons
into shorter-chain molecules. Large molecules of oils can be broken down into smaller
molecules of petrol and gases by cracking.

C10H22 C7H16 + C3H6(g)

oil petrol

The molecules of petrol contain 5-9 carbon atoms and gases are mainly alkenes containing 2
to 4 carbon atoms.

29.6.3. Bio gas


Bio gas is formed by the anaerobic action of bacteria on cellulose and other organic matter.
Bio gas contains 65-75% methane and other gases include carbon dioxide, ammonia and
hydrogen sulphide. The source of bio gas is animal wastes and to some extent plant wastes. A
simple bio gas generator consists of a container in which animal wastes or plant wastes are
mixed with a limited amount of water and then covered to exclude aerial oxidation. At
temperatures of 25-30oC, the anaerobic bacteria present decompose the wastes to form bio
gas. Bio gas is used for cooking and lighting purposes.

Advantages of bio gas production

(i) Bio gas is cheap to produce.

(ii) The solid by-product is used as fertilizers since it contains a high nitrogen content.
29.6.4. Disadvantages of bio gas production
Some of the gases contained in bio gas are air pollutants. When bio gas is burnt, sulphur
dioxide is formed by oxidation of hydrogen sulphide.

2H2S(g) + 3O2(g) 2H2O(g) + 2SO2(g)

Sulphur dioxide leads to the formation of acid rain which results in damage to plants and
aquatic organisms (refer to the effects of air pollution, chapter 12). Hydrogen sulphide reacts
with many metals. The tarnishing of silver objects is due to the reaction with hydrogen
sulphide to form silver sulphide, which is black. Paints which contain lead compounds are
also discoloured, due to the formation of black lead(II) sulphide. Hydrogen sulphide and
ammonia, which are contained in bio gas, cause eye irritation.

29.6.5. Alkenes
The alkenes are members of a homologous series of general molecular formula, CnH2n,
where 2. They are characterized by a carbon-carbon double bond as their functional group
and therefore alkenes are unsaturated compounds. An unsaturated compound is one in
which some atoms do not exert all their combining powers with other atoms. For example in
alkenes, not all the carbon atoms are bonded to four other atoms as illustrated below.

Nomenclature
In accordance with I.U.P.A.C. system, alkenes are named by dropping the ending ane from
the names of the corresponding alkenes and replacing it with the suffix ene for example
ethane (C2H4), propene (C2H6), butane (C4H8), pentene (C5H10) etc.

29.7. Ethene

29.7.1. Laboratory preparation of ethene


Ethene is obtained by dehydration of ethanol by excess hot concentrated sulphuric acid. The
experiement is set up as shown in figure 9.2. The mixture is heated with care to about 180 oC.
Ethene is evolved and is passed through a bottle containing sodium hydroxide solution to
remove sulphur dioxide, a by-product in small amount formed as the ethanol reduces
sulphuric acid slightly. Ethene is collected over water since it is insoluble in water.

CH3CH2OH(1) CH2=CH2(g) + H2O(g)

When the reaction is complete, the junction between the flask and the bottle should be
disconnected to avoid the possibility of sodium hydroxide solution sucking back into hot
concentrated sulphuric acid.

Properties of ethene

Ethene as the first alkene may be used to indicate some of the physical and chemical
properties of alkenes.
29.7.1.1. Physical properties
(i) Ethene is a colourless, sweet smelling and non-poisonous gas.

(ii) It is slightly less dense than air.

(iii) It is insoluble in water but soluble in organic solvents.

29.7.1.2. Chemical properties


1. Combustion
Ethene burns in air and the products of complete combustion are carbon dioxide and steam.
The flame tends to be smoky due to unburnt carbon because of the high proportion in ethen.

C2H4(g) + 3O2(g) 2CO2(g) + 2H2O(g)

2. Addition reactions
Alkenes are reactive compounds because the double bonds are readily converted to single bonds
by addition of other atoms. So ethene undergoes addition reactions. An addition reaction is a
reaction I which a molecule adds to an unsaturated molecule by breaking a double or a triple
bond.

(a) (i) Reaction with bromine


When ethene is passed through liquid bromine, the liquid becomes colourless. This acts as a
distinguishing test between saturated compounds such as alkanes and unsaturated ones such as
alkenes and alkyenes.

CH2=CH2(g) + Br2(l) BrCH2CH2Br(l)

1,2-dibromoethane

Addition of bromine across the double bond takes plae readily in the presence of an organic
solvent such as tetrachloro methane (CCl 4) or ether. The solvent dissolves the halogen to form a
solution such that when ethene is bubbled through the solution, the reaction takes place more
efficiently.

(ii) Reaction with bromine water


Bromine dissolves partially in water to form a solution called bromine water which contains
hypobromous acid and hydrobromic acid.

Br2(l) + H2O(l) HOBr(aq) + HBr(aq)

Since hydrobromic acid is a volatile liquid (can easily vaporize), hypobromous acid remains in
the solution to participate in the reaction with the alkene.

CH2=CH2(g) + HOBr(aq) Br-CH2CH2-OH(aq)

2-bromoethanol

Therefore, when ethene is bubbled through bromine water, the red colour of bromine water is
discharged.

(b) Reaction with hydrogen


Ethene combines with hydrogen if the two are passed over finely divided nickel catalyst at about
150oC. Platinum catalyst is used at room temperature. The reaction is called catalytic
hydrogenation.

CH2=CH2(g) + H2(g) CH3CH3 (g)

CH2=CH2(g) + H2(g) CH3CH3 (g)

This reaction is applied in changing double bonds in vegetable oils into single bonds. For
example in margarine production. The hardening of liquid vegetable oils into solid fats is called
addition hydrogenation. The oil is heated and mixed with a finely divided nickel catalyst, and
then hydrogen is blown through the mixture under pressure.
(c) Ethene decolourises acidified or alkaline potassium permanganate solution (from
purple to colourless). Also this reaction is a characteristic test for an unsaturated
compound.

29.7.2. Uses of ethene


1. In the manufacture of plastics such as polyethene.
2. In the manufacture of ethanol.
3. In the process of ripening of fruits.

Exercise

1. The compound with the molecular formula C10H22 can undergo the following reaction:
C10H22 C8H18 + W

(a) Name the process involved in this reaction.


(b) To which hydrocarbon series does W belong?
(c) Name gas W.
(d) Write the equation for complete combustion of gas W.
(e) Gas W was bubbled through
(i) Bromine.

(ii) Bromine water.

(f) Write the equation for the reaction that took place in e(i) and (ii).
(g) Name the product formed in e(i)

2. Two hydrocarbon compounds are represented by the molecular formulae, C3H6 and C3H8
(a) To which hydrocarbon series does each of them belong?
(b) Give the name of each compound.
(c) Describe any chemical test that can be used to distinguish between the two
compounds.
(d) Which one of the compounds is unsaturated?
(e) The unsaturated compounds named in (d) was reacted with hydrogen under certain
conditions.
(i) State the conditions necessary for the reaction to take place.

(ii) Name the type of reaction.

(iii) State one industrial application of the type of reaction named in (ii).
29.7.3. Alkynes
Alkynes are a homologous series of unsaturated hydrocarbons of the general formula, C nH2n-
2, where n 2. They contain carbon-carbon triple bond ( C C ) as the functional group.

They are unsaturated compounds.

Nomenclature

The alkynes are named by dropping the ending ane of the corresponding alkane and
replacing it with the suffix yne. The table 9.3 shows the molecular formulae and names of
the first five alkynes.

Alkyne Molecular formula

Ethyne C2H2

Propyne C3H4

Butyne C4H6

Pentyne C5H8

Hexyne C6H10

Table 9.3 Molecular formulae and names of first five alkynes


29.7.4. Ethyne (acetylene)
Ethyne as the first alkyne may be used to indicate some of the physical and chemical
properties of alkynes. It is prepared from the laboratory.

29.7.4.1. Physical properties of ethyne


1. Ethyne is a colourless, odourless and sweet smelling gas.
2. It is slightly soluble in water.

29.7.4.2. Chemical properties of ethyne


1. Combustion
Ethyne burns in air with a yellow sooty flame forming carbon dioxide and steam. The reaction is
exothermic. The soot is due to unburnt carbon because of its high content in ethyne.

2C2H2(g) + 5O2(g) 4CO2(g) + 2H2O(g)

2. Addition reactions
The triple bond in ethyne contributes a lot to its chemical properties. Ethyne undergoes
addition reactions with the substances that react with alkenes except that ethyne reacts more
slowly than alkenes. For example, it takes ethyne sometime to decolourize bromine unlike
ethene which does it almost instantaneously.
(i) Ethyne decolorizes bromine. The triple bond is first converted into a double bond, then
into a single bond.

HC CH + Br2 Br-CH = CH-Br

1,2-dibromoethene

Br-CH=CH-Br + Br2 Br2CHCHBr2

1,1,2,2-tetrabromoethane

(ii) A mixture of ethyne and hydrogen when passed over a nickel catalyst at about 150 oC
produces ethene.

HC CH + H2 CH2=CH2

Further hydrogenation can occur and the final product is ethane.

CH2=CH2 + H2 CH3CH3

29.7.4.3. Uses of ethyne


(i) Used in manufacture of polyvinyl chloride plastic which has a wide variety of uses.

(ii) Used in oxy-acetylene flame, used in welding and metal cutting.

Exercise

Two hydrocarbon compounds, Q ans Z, are represented by the molecular formula

C5H12 ans C3H4 respectively.

(a) Give the name of each hydrocarbon.


(b) To which homologous series does each of them belong?
(c) Which hydrocarbon contains multiple bonds?
(d) Write the equation for complete combustion of Z.
(e) Name the products of incomplete combustion of Z.

29.7.5. Alkanols (alcohol)


Alkanols are a homologous series of the general formula, CnH2n+1OH, where n 1. The
functional group of the alkanols is the hydroxyl group (-OH).

Nomenclature

Alcohols are systematically named as alkanols, that is, the name is a particular member is
obtained by dropping the ending e of the corresponding alkane and replacing it with the
suffix o1. The table 9.4 shows the molecular formulae and name of the first five alkanols.

Number of Molecular Name


carbon atoms formula

1 CH3OH Methanol

2 C2H5OH Ethanol

3 C3H7OH Propanol

4 C4H9OH Butanol

5 C5H11OH Pentanol

Table 9.4 Molecular formulae and names of first five alkanols


29.8. Ethanol (ethyl alcohol)

29.8.1. Manufacture of ethanol


Ethanol is prepared by fermentation of sugars in the presence of yeast. Fermentation is a
process in which sugars are decomposed by enzymes into alcohol. Cassava is crushed and
heated in steam under pressure to extract starch. Starch is treated with malt (partially spouted
barley) for an hour at 60oC. Malt supplies an enzyme, diastase, which hydrolyses starch to
sugar, maltose.

2C6H10O5(aq) + H2O(1) C12H22O11(aq)

starch maltose

Yeast is then added at room temperature and one of its enzymes, maltase, catalyses the
hydrolysis of maltose to glucose.

C12H22O1(aq) + H2O(1) 2C6H12O6(aq)

glucose

Another enzyme present in yeast, zymase, catalyses the decomposition of glucose to ethanol
and carbon dioxide.

C6H12O6(aq) 2C2H5OH(aq) + 2CO2(g)

The resulting solution is crude ethanol which is converted to pure ethanol by fractional
distillation.

In Uganda, locally crude ethanol (known as Tonto) is obtained from bananas. Juice is
extracted from ripe bananas by squeezing them using spear grass leaves or banana leaves.
The juice is filtered to remove any solid impurities. The filtrate (juice) is then poured into a
locally made wooden container where it is mixed with ground roasted sorghum. The
container is covered and the mixture is allowed to ferment for two days. The resulting
solution is crude ethanol, locally known as Tonto.
Alternatively, millet is ground to flour and the flour is mixed with water to form a paste
which is covered in a container or buried in the ground for a few days so that it can ferment.
The fermented paste is removed and roasted to obtain malt. Miller grains are soaked in water
for sometime and allowed to germinate. It is then dried to give yeast. Yeast is added to malt
in appropriate proportion. A carefully determined amount of water is then added to the
mixture of yeast and malt to form a liquid mixture. The mixture is covered and stored in a
warm place for about 3 to 4 days so that an alcoholic drink locally called Malwa is formed.

29.8.2. Properties of ethanol


Ethanol is a colourless liquid with a burning taste. It is soluble in water. It is a hygroscopic
substance.

29.8.2.1. Chemical properties


(i) Combustion

Ethanol burns in air with a blue flame to give carbon dioxide and water. The reaction is
exothermic.

C2H5OH(l) + 3O2(g) 2CO2(g) + 3H2O(l)

(ii) Dehydration of ethanol

When excess concentrated sulphuric acid is added to ethanol and the mixture heated to
180oC, ethanol is dehydrated to ethene.

C2H5OH(l) C2H4(g) + H2O(l)

It is sometimes called elimination reaction because a molecule of water is eliminated from


the alcohol to form an alkene.

(iii) Reducing properties

Ethanol can be oxidized to ethanoic acid by an acidified solution of potassium dischromate.


During the reaction, the orange solution of acidified potassium dichlomate turns green.

C2H5OH CH3COOH
ethanoic acid

Also ethanol decolourises acidified potassium permanganate solution.

C2H5OH CH3COOH

29.8.2.2. Uses of ethanol


(i) Ethanol is used as a solvent for perfumes, vanishes, paints and preparations.

(ii) It is used in manufacture of organic compounds such as ethanoic acid.

(iii) Used in alcoholic drinks such as beer, wines and spirits.

(iv) Ethanol is used as a fuel.

Exercise

1. When ethanol reacts with concentrated sulphuric acid, a hydrocarbon T is formed.


(a) Give the name of the hydrocarbon T.
(b) To which homologous series does the hydrocarbon T belong?
(c) State the conditions necessary for the reaction to take place.
(d) Name the type of reaction that occurs.
2. (a) State what is observed when methanol is added to acidified potassium dichromate and
the mixture heated.
(b) Name the product formed.

29.9. Carboxylic acids and esters


Carboxylic acids (Alkanoic acids)

Carboxylic acids are compounds of the homologous series of the general molecular formula
CnH2n+1COOH. In this series, CnH2n+1 represents the alkyl (R) groups. So carboxylic acids can
be written as RCOOH. The functional group of members in this series is the carboxyl group
(-COOH). They are sometimes called organic acids.

Nomenclature

Carboxylic acids are named as derivatives of alkanes by dropping the ending e of the
corresponding alkane and replacing it with the suffix oic and the functional group is always
at the end of the chain. The first three members in the series are methanoic acid (HCOOH),
ethanoic acid (CH3COOH) and propanoic cid (CH3CH2COOH).
29.9.1. Properties of carboxylic acids
1. Acidic character
When carboxylic acids dissolve in water, the solution formed turns blue litmus paper red.
When carboxylic acids dissolve in water, they are slightly ionized, that is they are weak
acids.

CH3COOH(aq) CH3COO-(aq) + H+(aq)

ethanoic acid ethanoate ion

(i) Carboxylic acid react with strongly electropositive metals liberating hydrogen gas.

CH3COOH + 2Na 2CH3COONa + H2

sodium ethanoate

(ii) Carboxylic acids react with bases to form salt and water.

CH3COOH + NaOH CH3COONa + H2O

CH3COOH + ZnO (CH3COO) 2Zn + H2O

zinc ethanoate

(iii) Carboxylic acid reacts with carbonates and hydrogencarbonates liberating carbon
dioxide.

CH3COOH + NaCO3 2CH3COONa + H2O + CO2

2. Formation of esters
When a carboxylic acid reacts with an alcohol, a sweet smelling compound called ester is
formed.

CH3COOH + C2H5OH CH3COOC2H5 + H2O

Ethylethanoate (ester)

29.10. Esters
Esters have a formula, RCOOR1, where R and R1 are alkyl groups. Long-chain carboxylic
acids are very often referred to as fatty acids because their chief source is from esters in
animal fats and vegetable oils. Fats and oils are esters occurring naturally in plants and
animals. Esters produced from saturated fatty acids are usually solids at room temperature
and are called fats. Esters obtained from unsaturated fatty acids are usually liquids at room
temperature and are called oils.

When a mixture of sodium hydroxide or potassium hydroxide solution and an ester is heated,
the ester is hydrolysed to form sodium salt. The alkaline hydrolysis of any ester is called
saponification.

RCOOR1+ NaOH(aq) RCOONa + R1OH.

29.10.1. Uses of esters


1. Used for making soap.
2. Used as a solvent for drugs and antibiotics.
3. Used for artificial flavouring.
4. Higher esters such as wax can be used for making candle.

29.11. Soap
Soap is a sodium or potassium salt of a long-chain carboxylic acid, that is, potassium stearate
(C17H35COOK) and sodium stearate (C17H35COONa).

29.11.1. Manufacture of soap


Soap is manufactured by a process called saponification, that is, alkaline hydrolysis of an
ester. Vegetable oil for fat is boiled with concentrated sodium hydroxide solution or
potassium hydroxide solution, for some time until frothing stops.

RCOOR1 + NaOH RCOONa + R1OH

The soap is precipitated by addition of concentrated sodium chloride solution. This process is
called salting out of soap. Sodium chloride lowers the solubility of soap in the mixture and
causes the precipitation of soap which floats on top of the liquid. It is then removed and
compressed into a continuous block which is cut into bars.

Hard soap consists of sodium salts of carboxylic acids. Soft soap consists of potassium salts
for example potassium stearate. Liquid soap is a mixture of soft soap and coconut oil whereas
toilet soaps are produced by adding dyes and perfumes to the purified soap.
29.11.2. The cleansing action of soap
When soap is added to a cloth in water, the polar end containing COO -Na+ group dissolves in
water and non-polar organic end containing the hydrocarbon chain dissolves in non-polar
grease or oil deposits in a cloth. Agitation causes emulsification of grease or oil and then
dirty tiny droplets are carried away by clean water. If water is hard, soap first reacts with
dissolved calcium and magnesium ions to form insoluble salts which are precipitated as
scum.

Ca2+(aq) + 2St-(aq) CaSt2(s)

Mg2+(aq) + 2St-(aq) MgSt2(s)

29.11.3. Soapless (synthetic detergents)


Detergents are substances that are used to improve the cleaning properties of water by
facilitating the emulsification and removal of grease. Soap is a detergent but the name is
usually used to for synthetic substitutes for soap. Soapless detergents have a similar structure
to soap molecules, that is, a long hydrocarbon portion and a short ionic portion.

Soapless detergents are more effective than soap in hard water since they do not form a scum.
This is because the calcium and magnesium salts of the hydrocarbon sulphonic acids of
which the detergents are composed, are soluble in water, so there is no precipitate (scum)
formed.

In the laboratory, soapless detergents can be prepared by boiling a vegetable oil with
concentrated sulphuric acid. The hydrogen sulphate compound formed is then neutralized by
adding sodim hydroxide solution. A precipitate forms and on evaporation, a white solid
soapless detergent is formed.

In industries, soapless detergents are made by boiling long-chain hydrocarbons (obtained


from petroleum) with concentrated sulphuric acid to form hydrocarbon sulphonic acid. The
acid is then neutralized with dilute sodium hydroxide or potassium hydroxide solution to
produce a salt which has similar properties to the sodium slats of organic acids used as soap.

Soapless detergents can be manufactured in solid form (for example washing powders) or in
liquid form (for example washing-up liquids and shampoos). Washing powders contain a
number of other components. Phosphates are added to prevent scum formation. Sodium
perborate gives the washing powder a mild bleaching action. Sodium sulphate and silicate
help to keep the powder dry and free flowing. Some powders also contain enzymes to digest
organic dirt like food stains and blood. Common detergents include Omo, Noimi, Fab and
Axion.
29.11.4. Advantages of synthetic detergents over soap
(i) Synthetic detergents are more soluble in water than soap.

(ii) Synthetic detergents dont form scum with hard water unlike soap.

29.11.5. Advantages of soap over synthetic detergents


(i) Soap is biodegradable, that is, it can be decomposed by micro-organisms while some
detergents are non-biodegrable. Therefore synthetic detergents can pollute rivers and soil
more than soap.

(ii) Soap is cheaper than detergents.

29.12. Polymerisation
Polymerization is the process by which many small molecules are combined to form a single
complex molecule. The small molecules that come together are called monomers and the
complex molecule formed is called a polymer. Polymers are long-chain molecules with
repeated units produced by the process of polymerization. There are two types of
polymerization, that is, addition polymerization and condensation polymerization.

29.13. Addition polymerization


This is the combination of many molecules of the same compound to form one complex
molecule without loss of molecules. It occurs between molecules (monomers) containing
double or triple bonds which undergo addition reactions. During the reaction, the double or
triple bonds break allowing these molecules to join to one another forming a long chain
(polymer). Addition polymers can be easily moulded into different shapes hence production
of a variety of items is possible. Polymers formed by addition polymerization include
polyethene, polypropene, polyvinyl chloride (P.V.C) and synthetic rubber.

29.14. Polyethene
It is a polymer formed by addition polymerization. Polyethene contains ethene as the
monomer. When many (n) ethene molecules combine, they form the polymer, polythene.
nCH2 = CH2 CH2 CH2 where n = about 600

There are two common types of polyethene depending on the conditions provided during
their manufacture.

(i) Low-density polyethene

Here the reaction involves heating ethene at about 200 oC and pressure of 1200
atmospheres in the presence of a small amount of oxygen. The polymer formed under
these conditions is soft because these conditions prevent close packing of chains. This
type of polyethene is used in making film and sheet material for plastic bags and
wrapping polyethene. It is also used as a film in solar heaters and driers instead of glass.

(ii) High-density polyethene

This is produced through the use of zieglar catalyst at a temperature of 60 oC and low
pressure of 1 atmoshpere. The polymer formed has fewer branched chains that are closely
packed. This is used for making moulds for rigid articles such as crates for milk and beer
bottles, toys, water pipes and electric cable pipes.

29.15. Polypropene
The monomer for this polymer is propene. Propene polymerises in presence of a Ziegler
catalyst to form a polymer, polypropene.

It is used to make beer bottle crates and ropes such as those used for drying clothes.

29.16. Polyvinyl chloride (P.V.C)


Another name for P.V.C is polychloroethene. The monomer chloroethene (commonly called
vinyl chloride), polymerises to form polyvinyl chloride.

Polychloroethene is used to make water pipes, gramophone records and light fittings such as
sockets, plugs and bulb-holders.
29.17. Synthetic rubber
Synthetic rubber is made up of two monomers, that is, but-1,3-diene and phenylethene.
Therefore synthetic rubber is a co-polymer because it is made up of two different monomers.
Polymerization occurs in presence of a peroxide.

n(CH2=CHCH=CH2) + n(CH2=CH) (CH2CH=CHCH2CH2CH)n

C6H5 C6H5

The elasticity of rubber is caused by the coiling of the rubber molecules. When rubber is
stretched, the molecules straighten out and when it is released, the molecules coil up again.

29.18. Condensation polymerization


This is the combination of many molecules to form a complex molecule with loss of a
molecule such as water. Polymers formed by condensation polymerization include nylon,
melamine, Bakelite and polyesters such as terylene. Terylene is mixed with wool to form
terylene wool used for making suits and fishing lines. Melamine isused to make trays, plates
and many households utensils. Bakelite is used to make electric plugs and radio cases. Starch
is also formed from glucose molecules by condensation polymerization.

n(C6H12O6) (C6H10O5)n + nH2O

glucose starch

29.19. Types of polymers


There are three types of polymers.

1. Natural polymers

These are polymers that exist in nature and are mainly manufactured by plants. Table 9.5
shows some of the natural polymers, their monomers and the type of polymerization they
undergo.

Polymer Monomer Type of polymerization

Proteins Amino acids Condensation


Starch Glucose Condensation

Silk Amino acids Condensation

Cellulose Glucose Condensation

Rubber 2-methylbut-1,3-diene Addition

Table 9.5 Natural polymers

Natural rubber

Natural rubber is got from latex which slowly extrudes from the bark of rubber trees
when it is cut. It is coagulated by addition of ethanoic acid

Vulcanization of rubber

Natural rubber in its raw form is soft and sticky when warm and therefore it is unsuitable
for most of the intended uses. It can be made stronger, harder and more durable by
heating it with sulphur at about 140oC. The sulphur atoms are added across the carbon-
carbon double bonds to form cross-linkages between the polymeric chains of rubber. This
process is called vulcanization of rubber. Table 9.6 shows the properties of raw rubber
compared with vulcanized rubber. Vulcanized rubber is used for making toys, tyres, tyre
inner tubes and foot wear such as soles and gum boots.

Raw rubber Vulcanized rubber

- It is soft and sticky. - It is hard and non-sticky.


- Soluble in organic solvents. - Insoluble in organic solvents.
- Easily melts. - Resistant to heat.
- Thermoplastic. - Thermosetting.

Table 9.6 Properties of raw and vulcanized rubber

2. Synthetic polymers
These are man-made polymers for example polyethene, nylon, terylene etc. All synthetic
polymers are plastic in nature hence are plastics. Plastics are man-made materials
composed of giant molecules based on carbon atoms. Plastics are classified into groups
according to the changes which occur on heating.

(i) Thermoplastics (thermosoftening plastics)

These are plastics which become soft and mouldable on heating without undergoing
any significant changes and on cooling they harden, for example polyethene, nylon
and polyvinyl chloride. There are no cross links between the chains in thermoplastics.
On heating, the chains move freely over each other thus the plastic melts.

(ii) Thermosetting plastics (Thermosets)

These are plastics which decompose on heating and cannot be reshaped after
manufacture. They are rigid, hard and brittle, for example Bakelite. In thermosetting
plastics, there are strong cross links between the chains which gives a rigid structure.
Heating has no effect until a temperature high enough to break some of the cross links
is reached. Thus the plastic decomposes. There are cross-linking agents that convert
thermoplastics into thermosetting plastics, for example sulphur in vulcanization of
rubber.

3. Semi-synthetic polymers
Semi-synthetic polymers exist. Rayon is a semi-synthetic polymer because it is made
chemically from cellusoe in form of wood pulp. The wood pulp is treated with sodium
hydroxide solution and carbon disulphide. This converts the cellulose into a syrup
substance called viscose, which is then forced through small holes in a metal plate into a
bath of dilute sulphuric acid. The acid converts the viscose solution into glossy
transparent filaments which can be twisted to form rayon threads.

29.20. Advantages of synthetic polymers over natural polymers


1. Synthetic polymers are relatively cheaper than natural polymers.
2. They are easy to manufacture.
3. They are usually stronger than the corresponding natural material.
29.21. Disadvantages of synthetic polymers
1. People working in factories producing these polymers are exposed to health harzards
as a result of inhaling the fumes from the chemicals used.
2. These polymers are non-biodegradable and therefore they can accumulate damaging
the soil.

Exercise

1. Under certain conditions ethene undergoes a reaction that can be represented by the
following equation.
nCH2 = CH2 CH2 CH2

(a) Name the type of reaction.


(b) Name the product of the reaction.
(c) Give any one use of the product.
(d) Explain one major environmental problem associated with the use of the product
named in (b).
2. (a) Explain what is meant by polymerization.
(b) State one natural polymer formed by condensation polymerization and state its
monomer.

3. The structure of a polymer is as shown below

(a) Write the structural formula for the monomer of the polymer.
(b) Name the
(i) Monomer.

(ii) Polymer.

4. (a) In the manufacture of soap, oil or fat is heated with sodium hydroxide solution.
(i) Name the process of making soap.

(ii) What is the purpose of adding saturated sodium chloride solution?

(iii) State the chemical nature of soap.

(b) Sometimes when soap is used for washing clothes, a scum is formed.
(i) What is a scum?

(ii) What causes the formation of scum?

(iii) Give the general name given to water which forms scum with soap.

(iv) Describe a chemical method by which the type of water you named in b(iii) can
be treated to avoid formation of scum. Write equations for the reactions that are
involved.

(a) (i) Name one soapless detergent that can be used instead of soap.
(ii) What is the advantage of using soapless detergents rather than soap?

(iii) What are the disadvantages of using soapless detergents?

5. (a) Crude ethanol is manufactured by the process known as fermentation.


(i) Explain what is meant by the term fermentation.

(ii) Write equation for the reaction that takes place during fermentation.

(b) Write equation to show how ethanol can be converted to ehtene and indicate the
conditions for the reaction.

(c) (i) State what would be observed when ethene is reacted with bromine

(ii) Write an equation for the reaction.

6. (a) explain what is meant by addition polymerization


(b) Name one synthetic polymer formed by addition polymerization and state two uses of
the named polymer.

(c) State two natural polymers.

(d) State two advantages of natural polymers over synthetic polymers.

7. (a) State the differences between fats and oils. Give one example of each.
(b) Briefly describe how soap can be prepared.

(c) State what would be observed if soap solution was shaken with a solution containing
magnesium hydrogencarbonate.

(d) Explain your answer in (c)

(e) State what would be observed if a solution of soapless detergent was used instead of
soap solution.

8. The molecular mass of gas x is 28 and its empirical formula is CH2.


(a) Determine the molecular formula of x.
(b) write the
i) Structural formula of x
ii) Equation for the reaction between x and bromine.
(c) (i) Name one reagent that could be used to identify x.
(ii) State what would be observed if the reagent named in c(i) was reacted with x.

9. (a) Ethene can be prepared by reacting ethanol and sulphuric acid.


(i) State the conditions for the reaction.

(ii) Write the equation for the reaction.

(b) Explain what is meant by the term polymerization.

(c) Write an equation for the formation of polyethene from ethene.

(d) State two uses of polyethene.

10. Glucose, C6H12O6, can be converted to ethanol by a catalytic reaction caused by an


enzyme produced from yeast.
(a) Name the
(i) Reaction in which yeast converts glucose into alcohol.

(ii) Enzyme produced by yeast during the reaction.

(b) Write the equation for the reaction that leads to the formation of ethanol.
(c) Briefly describe how ethanol produced can be concentrated.
11. (a) (i) What is a polymer?
(ii) Distinguish between a natural and artificial polymer. In each case give two
examples.

(b) Describe the process of vulcanization of rubber. In your description include:

(i) The importance of vulcanization of rubber industry.

(ii) Two useful items of vulcanized rubber.

12. Ethanol can be converted to substances P and Q according to the reaction scheme shown
below.
C2H5OH C2H4 C2H6

step1 P step 2 Q

(a) Name substances P and Q.


(b) write the structural formula of P
(c) Name the reagent used in step 1.
(d) State the products for complete combustion of Q in excess air.
(e) Name the catalyst used in step 2.
30. Energy changes

30.1. Introduction

Chemical changes are normally accompanied by energy changes. Energy is neither created
nor destroyed but it can be transformed from one form to another form. Chemical energy is
transformed into chemical energy in electrochemical cells. Fuels such as coal, store chemical
energy which is transformed into heat energy when it is burnt. During metabolism, the
chemical energy of food such as carbohydrates is converted to heat energy to keep the body
warm, to mechanical energy in muscles ans to electrical energy in the impulses within our
nerve fibres. The most common form of energy change in chemical reactions is the heat
change, and is our major concern.

30.2. Enthalpy
Enthalpy is the energy (heat) content of a substance which is stored in its bonds. Energy is
released when bonds are formed and to break bonds, energy must be supplied. The enthalpy
of a substance is denoted by H. Changes in enthalpy are denoted by H (delta H). Enthalpy
changes occur in a reaction when some old bonds in the reactants break and new bonds are
formed in the products.

30.3. Exothermic and endothermic reactions


An exothermic reaction is one during which heat is liberated to the surroundings. In this kind
of reaction heat is given out and therefore, the temperature of the products rises above room
temperature but with time it drops to room temperature as heat is losts to the surroundings. At
the end of the reaction, the heat content of the products is less than that of the reactants.
Therefore the enthalpy (heat) change of the reaction is negative.

i.e. H = H products - H reactants

H = H2 H1 where H1 = initial enthalpy and H2 = final enthalpy.

Since H2 < H1, the H is negative. This can be illustrated using an energy level diagram
figure 10.1.
For example, when carbon reacts with oxygen, heat is evolved.

C(s) + O2(g) CO2(g) H = - 393kJ

The chemical energy in carbon and oxygen is partly transferred to chemical energy in carbon
dioxide and partly evolved as heat. Thus carbon dioxide has less energy than the starting
materials, carbon and oxygen (figure 10.2). Therefore the value of enthalpy change is
negative.

An endothermic reaction is one during which heat is absorbed from the surroundings. When
an endothermic reaction occurs, the heat required for the reaction is taken from the reacting
materials and the temperature of the products falls below the initial temperature. Eventually,
the temperature of the products raises to room temperature again as heat is absorbed from the
surroundings. In this case, the heat content of the products is greater than that of the reactants
and the enthalpy change is positive.

i.e. H = H products - H reactants

H = H2 H1

Since H2> H1, then H is positive. Figure 10.3 illustrates an energy level diagram for an
endothermic reaction. The units for enthalpy change are kilo joules per mole (kJ mol-1)

For example, when hydrogen reacts with iodine, heat is absorbed from the surroundings.

H2 (g) + I2(g) 2HI(g) H = +52 kJ


Hydrogen iodide has more energy than the starting materials, hydrogen and iodine (figure
10.4). Therefore the value of enthalpy change is positive.

Exercise

1. State which of the following processes is endothermic and which is exothermic.


(a) The breaking of a chemical bond.
(b) The formation of a chemical bond.
2. State whether the following reactions are endothermic or exothermic.
(a) H+(aq) + OH-(aq) H2O(l) H = -57.3 kJ mol-1
(b) C(s) + 2S(l) CS2 (l) H = + 177 kJ mol-1

30.4. Types of enthalpy changes


1. Enthalpy of combustion
Enthalpy of combustion is the enthalpy change that occurs when one mole of a substance
burns completely in oxygen. The H valves are always negative because heat is always
evolved. For example, the enthalpy of combustion of carbon is -393 kJ mol-1.

C(s) + O2(g) CO2(g) H = -393 kJ mol-1

Example: Enthalpy of combustion

Carbon undergoes complete combustion according to the equation.

C(s) + O2(g) CO2(g) H = -393 kJ mol-1

How much heat is given out when 20 g of carbon are completely burnt?

Solution:

12 g of carbon burn to give 393 kJ.

1 g of carbon burns to give 393 kJ.

12

20 g of carbon burn to give 393 x 20

12
= 655 kJ

30.4.1. Determination of enthalpy (heat) of combustion of ethanol

A thin walled tin can is filled with a known volume of water. Ethanol is added to a specimen
bottle and a wick is filled in a cork through the bottles mouth. The specimen bottle with its
contents is weighed and its mass is recorded. The apparatus is set up as shown in figure 10.5.
The initial temperature of the water is recorded and the wick is lighted to heat the water. The
water is stired carefully with the thermometer. When the thermometer shows a convenient
temperature rise, the flame is blown off and the highest temperature reached is recorded. The
bottle and its contents is reweighed after cooling. A tin can is a good conductor of heat and
transmits most of the heat directly to the water.

Specimen results:

Initial temperature of water = 24oC

Final temperature of water = 40oC

Volume of water in the can = 100 cm3

Mass of bottle + ethanol before burning = 32 g

Mass of bottle + ethanol after burning = 30 g


Calculations:

Heat energy absorbed by the water = mc

Where m = mass of the water.

c = specific heat capacity of the water.

= temperature rise.

Mass of water, m = density x volume

But density of water = 1 g/cm3

m = 1 x 100 = 100 g

c = 4.2 J g-1 oC-1

= 40 24 = 16oC

Energy absorbed by water = 100 x 4.2 x 16 = 6720 J

Mass of ethanol burnt = 32 30 = 2 g

Molar mass of ethanol (C2H5OH) = (12 X 2) + (5 X 1) + 16 + 1 = 46 g

46 g of ethanol contain 1 mol.

2 g of ethanol contain 1 x 2mol.

46

= 0.044 mol.

0.044 mol of ethanol completely burn to give 6720 J.

1 mol of ethanol would burn to give 6720 J.

0.044

= 152727 J or 152.7 kJ

The enthalpy of combustion of ethabol is -152.7 kJ mol-1.

The value obtained in this experiment is not very accurate because of heat losse to
surroundings.
Exercise

1. The following reaction takes place when methanol is burnt in oxygen.


2CH3OH(l) + 3O2(g) 2CO2(g) + 4H2O(g) H = -1452 Kj mol-1

(a) Is this reaction endothermic or exothermic?


(b) How much heat energy would be liberated by burning
(i) 4 mol of methanol?

(ii) 4 g methanol?

2. (a) Write an equation for the combustion of methane in


(i) Excess oxygen.

(ii) Limited oxygen supply.

(b) When 11.5 g of methane are burnt in excess oxygen, 640 kJ of heat are produced.
Calculate the

(i) Heat of combustion of methane.

(ii) Volume of methane at room temperature that will burn to produce 1560kJ.

(iii) Mass of methane that will burn to produce 1060 kJ.

(iv) Heat evolved when 4 mol of methane are burnt.

(v) Heat evolved when 32 g of methane are burnt.

3. When 23.6 g of butane were burnt, the heat produced raised the temperature of 50 g of
water from 30oC to 40oC.
(a) Write an equation for the complete combustion of butane.
(b) Calculate the heat of combustion of butane.

2. Enthalpy of solution
The enthalpy of solution is the enthalpy change that occurs when one mole of a substance
is dissolved in sufficient amount of water such that no further heat change occurs on
dilution. When one mole of sulphuric acid dissolves in water, heat is evolved. The
reaction is exothermic and therefore the enthalpy of solution is negative. Some reactions
are endothermic. For example, when sodium chloride dissolves in water, heat is absorbed
and therefore the enthalpy of solution is positive.
30.5. Determination of the enthalpy (heat) of solution o f sodium chloride
A known volume of water is palced in a plastic cup and its temperature recorded. A known
mass of sodium chloride is added to the water. Carefully water is stirred with a stirrer and the
lowest temperature of the solution is recorded.

Specimen results:

Volume of water = 100 cm3

Mass of sodium chloride = 31 g

Initial temperature = 24 oC

Final temperature = 19.27 oC

Calculations:

Heat evolved =

Mass = density x volume

Mass of water = 1 x 100 = 100 g

Total mass of solution = 100 + 31 = 131 g

Temperature change, = 24 19.27 = 4.73 oC

Assuming the specific heat capacity of the solution is 4.2 J g-1 oC-1

Heat evolved = 131 x 4.2 x 4.73 = 2602 J or 2.602 kJ

Molar mass of NaCl = 23 + 35.5 = 58.5 g

58.5 g of sodium chloride contain 1 mol.

31 g of sodium chloride contain 1 x 31mol

58.5

= 0.53 mol

0.53 mol of sodium chloride absorbed 2.602 kJ

1 mol of sodium chloride would absorb 2.602 kJ


0.53

Enthalpy of solution of sodium chloride is +4.91 kJ/mol.

This value is less than the accurate valuw, +4.97, because of the experimental errors.

Exercise

1. The heat of solution of sulphuric acid is -70kJmol -1. Calculate the mass of sulphuric acid
that will evolve 350 kJ of heat when sulphuric acid is dissolved in water.
2. When 231 g of ammonium nitrate were dissolved in water, 75 kJ of heat were absorbed.
Calculate the heat of solution of ammonium nitrate.

30.6. Enthalpy of neutralization


Enthalpy (heat) of neutralization is the enthalpy change that occurs when one mole of
aqueous hydrogen ions reacts with one mole of aqueous hydroxide ions to form one mole of
water.

H+(aq) + OH-(aq) H2O(l)

The enthalpy of neutralization of any strong acid and strong alkali is -57.3kJ mol -1 and is
constant. This is because the acids and alkalis and the salt produced in the reaction are
completely ionized in solution and the net reaction is the formation of water molecules. The
enthalpy of neutralization of a weak acid or alkali is less than 57.3 kJ mol -1 and is not constant.
This is because weak acids or alkalis are partly ionized in aqueous solution. Some heat is
absorbed for complete ionization of the acid or alkalis in order for neutralization to occur. The
enthalpy (heat) of ionization affects the overall enthrapy change.

30.7. Determination of heat of neutralization


A known volume of 1 M sodium hydroxide is placed into a placed into a plastic cup. It is stired
with a stirrer and its temperature, 1, recorded. An equal volume of 1 M hydrochloric acid is
added to the sodium hydroxide after taking its temperature, 2. The mixture is stirred well with a
stirrer and the highest temperature, 3, attained by the mixture is recorded.

Specimen Results:

Volume of 1 M hydrochloric acid = 50 cm3


Volume of 1 M sodium hydroxide = 50 cm3

Temperature of 1 M sodium hydroxide 1 = 25 oC

Temperature of 1 M hydrochloric acid, 2 = 24 oC

Temperature after mixing, 3 = 30.83 oC

Calculations:

Heat evolved (enthalpy change) = mc

Total volume of solution after mixing = 50 + 50 = 100 cm3

Mass = density x volume

Assuming the density of the solution is 1 g/cm3

Mass of the solution = 1 x 100 = 100 g

Initial temperature (average temperature) = 1 +2 = 24 + 25

2 2

= 24.5 oC

Temperature rise = 30.83 24.5 = 6.33oC

Assuming the specific heat capacity of a mixture is 4.2 J g-1oC-1

heat evolved = 100 x 4.2 x 6.33

= 2658.6 J or 2.66 kJ

1000 cm3 of sodium hydroxide contain 1 mol

50 cm3 of sodium hydroxide contain = 1 x 50

1000

= 0.05 mol

0.05 mol of sodium hydroxide evolve 2.66 kJ

1 mol of sodium hydroxide would evolve 2.66 kJ

0.05
Heat of neutralization is -53.2kJ/mol

The valve obtained is not very accurate because of experimental errors.

Exercise

1. 80 cm3 of 1 M nitric acid and 80 cm3 of 1 M sodium hydroxide, both at 25 oC were mixed
in a plastic beaker. The mixture was stirred and its maximum temperature was 31.34 oC.
(specific heat capacity of the solution = 4.2J/g/oC, density of the solution = 1 g/cm3)
(a) Write the ionic equation for the reaction which took place.
(b) Calculate the
(i) Number of moles contained in 80 cm3 of 1 M sodium hydroxide.

(ii) Heat evolved when 80 cm3 of 1 M sodium hydroxide react with 80 cm 3 of 1 M


nitric acid.

(iii) Heat neutralization.

2. Aqueous hydrogen ions react with aqueous hydroxide ions according to the equation.
H+(aq) + OH-(aq) H2O(l) H = 57 kJ mol-1

How much energy is released if

(i) 4 mol of hydrogen ions are neutralized?

(ii) 0.25 mol of hydrogen ions are neutralized?

(iii) 2 mol of sulphuric acid are completely neutralized.

30.8. Enthalpy of displacement


When zince is added to copper(II) sulphate solution, a displacement reaction occurs. Usually
during such reactions heat is evolved.

Zn(s) + CuSO4(aq) Cu(s) + ZnSO4(aq)

or Zn(s) + Cu2+(aq) Cu(s) + Zn2+(aq)

30.8.1. Determination of the enthalpy (heat) of displacement of the reaction between


copper(II) sulphate solution and zinc
50 cm3 of 0.5 M copper(II) sulphate solution is placed in a plastic beaker. The initial
temperature of the solution is recorded. About 2 g of zinc powder is added to copper(II)
sulphate solution in the plastic beaker. The mixture is stirred carefully with a thermometer
and the highest temperature attained is recorded.
Specimen Results:

Volume of copper(II) sulphate solution = 50 cm3

Intial temperature = 25.0 oC

Final temperature = 28.5 oC

Calculations:

Heat evolved = mc

Mass = density x volume

Assuming the density of the solution is 1 g/cm3.

Mass of the solution = 1 x 50 = 50 g

Assuming the specific heat capacity of the solution is 4.2 Jg-1oC-1

Heat evolved = 50 X 4.2 X (28.5 25) = 735 J

1000 cm3 of copper(II) sulphate solution contain 0.5 moles

50 cm3 of copper(II) sulphate = 0.5 x 50

1000

= 0.025 mol

Zn(s) + CuSO4(aq) ZnSO4(aq) + Cu(s)

The reaction mole ratio of zinc to copper(II) sulphate is 1:1

Moles of zinc that reacted = 0.025 mol

0.025 mol of zinc evolve 735 J

1 mol of zinc would evolve 735 J

0.025

= 29400 J or 29.4 kJ

Heat of displacement is -29.4 kJ/mol.


Exercise 10.5

1. The amount of ehat evolved when 16 g of copper was displaced from the solution by 2.4
g metal, Q, was 720 kJ. Calculate the heat of displacement. (Q = 24)
2. Iron reacts with copper(II) ions according to the equation.
Cu2+(aq) + Fe(s) Cu(s) + Fe2+(aq) H = -151 kJ mol-1

Calculate the mass of iron that will cause a heat change of -170 kJ.

3. (a) When methane burns in oxygen, heat is produced. Write an equation for the
combustion of methane in excess oxygen.
(b) The heat of combustion of methane is -890 kJ mol -1. calculate the volume of methane
gas at s.t.p that when burned in excess oxygen would raise the temperature of 178 g of
water by 10 oC.

4. (a) Bio gas contains mainly methane. Name two raw materials that can be used to
produce biogas.
(b) Methane burns in oxygen according to the equation:

CH4(g) + 2O2 (g) CO2 (g) + 2H2O(g) H = -890 kJ/mol

Calculate the volume of methane at s.t.p. that will burn in excess oxygen to produce
2670kJ.

5. The enthalpy of combustion of carbon is -393 kJ mol-1.


(a) Write an equation for the complete combustion of carbon.
(b) 80kg of charcoal cost 4,000/=. Calculate the cost of charcoal required to produce
16735kJ.
6. (a) Describe an experiment that can be carried out to determine the heat of combustion of
ethanol. Draw a diagram to illustrate your answer.
(b) Would you expect the heat of combustion as determined in the experiment in (a) to be
greater than, lower than or equal to the theoretical value. Give a reason for your answer.

(c) When 0.382 g of ethanol was burnt, the heat evolved raised the temperature of 100 g
of water from 16.0 oC to 43.0 oC. Calculate the heat of combustion of ethanol.

(d) Name two products, other than water of incomplete combustion of ethanol.

7. (a) 50 cm3 of 2 M hydrochloric acid and 50 cm3 of 2 M sodium hydroxide, both at 22 oC,
were mixed in a plastic beaker. The mixture was stirred and its maximum temperature
was 35oC. (specific heat capacity of the solution = 4.2 J/g oC, density of the solution = 1
g/cm3)
(i) Write an ionic equation for the reaction which took place.
(ii) Calculate the heat of the reaction.

(b) 50 cm3 of 2 M ammonia solution was used instead of sodium hydroxide solution in
(a). State whether the heat of the reaction was greater than, smaller than or equal to the
value you have calculated in (a) (ii). Give a reason for your answer.

8. (a) Write an equation to show how ethanol can be prepared form glucose.
(b) State how a sample of ethanol obtained from the product of the reaction in (a) can be
purified.

(c) When 23 g of ethanol was completely burnt, 13600 J of heat was produced. Calculate
the molar heat of combustion of ethanol.

9. 7.5 g of methane, CH4 was completely burnt in air. Methane burns in air according to the
following equation:
CH4(g) + 2O2(g) CO2(g) + 2H2O(g) H = 890 kJ mol-1

Calculate the

(i) Mass of carbon dioxide formed.

(ii) Heat evolved.

10. When 6.4 g of zinc powder were added to 250 cm 3 of a 0.1 M copper(II) sulphate
solution in a plastic cup, 5.45 kJ of heat was liberated.
(a) Explain why a plastic cup was used instead of a metallic cup.
(b) Write an equation for the reaction between zinc powder and copper(II) sulphate.
(c) Calculate the
(i) Number of moles of zinc in 6.5 g of zinc powder.

(ii) Number of moles of zinc which reacted with copper(II) sulphate.

(iii) Heat energy produced when 1 mole of zinc reacts with 1 mole of copper(II)
sulphate.

11. The formation of methanol from hydrogen and carbon monoxide is represented by the
equation.
2H2(g) + CO(g) CH3OH(g) H = -92 kJ/mol

What would be the energy released in kJ mol-1, when 3.2 g of methanol is formed?

12. An experiment was carried out to determine the molar heat of combustion of methanol. A
small lamp containing methanol was weighed and then lit. The heat produced by the
combustion of methanol was used to raise the temperature of 100 g of water in a metal
can (ignore the heat required to raise the temperature of the metal can). The spirit lamp
was weighed again after the experiment.
Results:
Mass of the spirit lamp + contents before heating = 36.17 g.

Mass of the spirit lamp + content after heating = 34.07 g.

Temperature of the water before the ecperiment = 20 oC.

Temperature of the water after the experiment = 80 oC.

(a) What was the rise in temperature of water during the experiment?
(b) Calculate the amount of heat obtained by the water during the experiment.
(c) What mass of methanol was burnt during the experiment?
(d) Calculate the heat produced when
(i) 1 g of methanol was burnt.

(ii) 1 mol of methanol (CH3OH) molecules is burnt.

13. The following pairs of compounds were reacted together and the maximium temperature
rise was recorded for each reaction.
A. 200 cm3 of 2 M sodium hydroxide and 200 cm3 of 2 M ethanoic acid.
B. 200 cm3 of 2 M ammonia solution and 200 cm3 of 2 M ethanoic acid.
C. 200 cm3 of 2 M sodium hydroxide and 200 cm3 of 2 M hydrochloric acid.
(a) State the pair which showed the
(i) Highest temperature.

(ii) Lowest temperature.

(b) Explain your answer in (a) (ii).


31. Rate of reaction and equilibrium

31.1. Rate of reaction


Some reactions proceed very fast. For example, when aqueous ammonia is added to a
solution of lead(II) salts, a white precipitate forms immediately. Some reactions proceed at
moderate speed. For example, it takes sometime for the reaction between calcium carbonate
and dilute hydrochloric acid to come to completion. Other reactions are slow. For example, it
takes iron a few days to rust in moist air. The above mentioned reactions proceed at different
rates.

The rate of a chemical reaction is the progress of the reaction in unit time. In other words, the
rate of a chemical reaction is the rate at which products are formed or the rate at which
reactants are used up in the reaction.

Rate of reaction = concentration in moles per litre

time in seconds

Units for the rate of reaction are moles/litre/second i.e mol/l/s.

31.1.1. Determination of rate of reaction


Let us consider the reaction between magnesium and dilute hydrochloric acid.

Mg(s) + 2HCl (aq) MgCl2 (aq) + H2(g)

The determination of the rate of this reaction can be done by either measuring the volume of
hydrogen evolved with time or by measuring the time a given length of magnesium ribbon
takes to dissolve in varying concentrations of the acid.

31.1.2. Determination of rates of reaction by measuring the volume of the gas


evolved with time
A known mass of magnesium and a known volume of dilute hydrochloric acid in a test-tube
tied with a thread, are placed in a conical flask and the experiment is set up as shown in
figure 11.1. The stopper is opened for a moment so that thread is free. The test-tube drops
pouring hydrochloric acid into the conical flask. At the same time, the clock is started. The
volume of hydrogen in the syringe is recorded at regular intervals until the reaction is
complete.
A graph of volume of hydrogen evolved against time is plotted. A typical graph has the form
of figure 11.2.

To determine the rate of reaction at a given time, say t 1, the tangent to the curve is drawn at
that time as shown in figure 11.3. The gradient of the tangent is the rate of reaction at that
time, that is y/x. the units are cm3/s.
Exercise

In determination of the rate of reaction, 10 g of calcium carbonate were reacted with dilute
hydrochloric acid. The mass of the flask and its contents was weighed with time.

(a) Write the equation for the reaction that took place.
(b) Sketch a graph of mass of flask and its contents against time.

Factors affecting the rate of chemical reactions

Factors which affect the rate of a chemical reaction are concentration, temperature, surface
are (particle size), pressure, catalyst and light. You are required to perform various
experiments to investigate the effect of these factors on the rate of chemical reactions.

1. Effect of concentration of reactants on the rate of reaction

The rate of the reaction depends on the frequency with which reacting particles collide,
which frequency depends on the concentration of the reactants. The higher the
concentration, the higher the frequency of collision and therefore the higher the rate of
the chemical reaction.

Experiment:

Investigation of the effect of concentration on the rate of the reaction

Make a mark with blue or black ink on a piece of paper. Place 50 cm 3 of 0.05 M sodium
thiosulphate solution into a beaker. Add 10 cm 3 of 1 M hydrochloric acid to the sodium
thiosulphate and at the same time start the stop clock. Gently shake the mixture to mix
the solution well and place the beaker on the paper over the mark. Watch the mark
through the solution from above the beaker. Stop the clock when the mark just
disappears. Vary the concentration of the thiosulphate solution by taking 40, 30, 20 and
10 cm3 each time by adding distilled water. Tabulate your results including 1/time. Plot
graphys of volume of sodium thiodulphate solution against 1/time (time -1) and against
time. The rate of reaction is proportional to the reciprocal of time (time -1). Your graphs
should appear as shown in figure 11.4a and 11.4b.

The mark disappears becaue the reaction between hydrochloric acid and sodium thiosulphate
forms a precipitate of sulphur which renders the mixture opaque.

NaS2O3(aq) + 2HCl(aq) 2NaCl(aq) + S(s) + H2O(l) + SO2(g)

Figure 11.4a shows that the higher the volume of the sodium thiosilphate, the less the time
taken to form a precipitate. Figure 11.4b shows that the rate of the reaction increases with
increase in volume of sodium thiosulphate solution.

Exercise

Magnesium was reacted with excess dilute sulphuric acid.

(a) Write an equation for the reaction that took place.


(b) On the same axis, sketch a graph of volume of hydrogen evolved against time when equal
volumes the following are reacted with the same mass of magnesium.
(i) 0.5 M sulphuric acid.

(ii) 2 M sulphuric acid.

(c) On the same axis, sketch a graph of volume of hydrogen evolved against time when the
following are reacted with the same concentration and volume of sulphuric acid.
(i) 10 g of magnesium.

(ii) 50 g of magnesium.

31.1.3. Effects of temperature on the rate of reaction


When the temperature is increased, the reacting particles gain more kinetic energy and
move at a greater speed. The frequency at which the reacting particles collide increases
and thus the rate of the reaction increases. Therefore, the higher the temperature, the
higher the rate of reaction.

31.1.3.1. Investigation of the effect of temperature on the rate of reaction


The previous experiment can be repeated by reacting sodium thiosulphate solution and
hydrochloric acid at varying temperatures, using the same concentration of the
thiosulphate. Put a test-tube containing 1 M hydrochloric acid into a beaker of water
maintained at 30 oC. After sometime, add at the same time start of 0.05 M sodium
thiosulphate solution in a beaker and at the same time start mark. Note the time taken for
the mark to disappear. The experiment is repeated using different temperatures. Tabulate
your results including 1/time. Plot graphs of temperature against 1/time. The shapes of
typical graphs are shown in figure 11.5a and 11.5b.
Figure 11.5a shows that the higher the temperature the less the time taken to form a
precipitate. Figure 11.5b shows that the rate of the reaction increases with increase in
temperature.

Exercise

Sodium sulphite solution was added to dilute sulphuric acid.

(a) Write an equation for the reaction that took place.


(b) On the same axis, sketch a graph of volume of sulphur dioxide against time when the
reaction was carried out at
(i) 24 oC.

(ii) 40 oC.

31.1.4. Effect of a catalyst on the rate of reaction


A catalyst is a substance which alters the rate of chemical reactions without undergoing
any overall chemical change itself. Most catalysts speed up the rate of reaction. The
greater the amount of the catalyst but within the limits, the higher the rate of reaction.
Powdered catalysts offer a larger surface area over which the reaction takes place and
therefore are more effective than one in lump form. Catalysts remain unchanged
chemically after a reaction has taken place. Catalysts are very specific to a particular
chemical reaction. A catalyst which slows down a reaction is called a negative catalyst.

31.1.4.1. Investigation of the effect of catalyst on the rate of reaction


Place 100 cm3 of 0.1 M hydrogen peroxide in a conical flask. Add 0.5 g of
manganese(IV) oxide to the hydrogen peroxide in conical flask. Then set up the
experiment as shown in figure 11.1. record the volume of oxygen in the syringe at regular
intervals unitl the reaction is complete. Repeat the experiment using 1 g of
manganese(IV) oxide. When the graphs of volume of oxygen against time are plotted
using the same axes, they appear as shown in figure 11.6.

Exercise

1. Name the catalysts used in the following processes/reactions.


(i) The contact process.

(ii) The Haber process.

(iii) The decomposition of hydrogen peroxide.

(iv) Hydrogenation of oils to form fats.

2. Potassium chlorate decomposes according to the following equation.


2KClO3(s) 2KCl(s) + 3O2(g)

(a) Name the catalyst that may be used in this reaction.

(b) On the same axis, sketch a graph of volume of oxygen evolved against time when

(i) The catalyst is not used.

(ii) 0.5 g of the catalyst are used.


31.1.5. Effect of surface area on the rate of reaction

Solid react much more rapidly when powdered than when in large lumps. This is because
reactions with solids take place at the surface. Powdered solids present a large surface
area over which the reaction occurs than solids in lump form.

31.1.5.1. Investigation of the effect of surface area on the rate of reaction


Pour 20 cm3 of 1 M hydrochloric acid in a test-tube. To the conical flask add 10 g of
calcium carbonate lumps and then set up the experiment as shown in figure 11.1. Record
the volume of carbon dioxide in the syringe at regular intervals until the reaction is
complete. Repeat the experiment using the same mass of powdered calcium carbonate.
When the graphs of volume of carbon dioxide against time for both powdered calcium
carbonate and calcium carbonate lumps, are plotted using the same axes, they appear as
shown in figure 11.7.

Exercise

1. Calcium carbonate lumps were mixed with dilute nitric acid in a conical flask. The mass
of the flask and its contents was weighed with time.
(a) Write the equation for the reaction that took place.
(b) (i) Sketch a graph of mass of flask and its contents against time. Label the graph A.
(ii) On the same axis, sketch the graph that would be obtained when powdered
calcium carbonate is used instead of calcium carbonate lumps. Label the graph B.

2. (a) Which one of the following reaction mixtures will produce hydrogen more quickly at
room temperature?
(i) Magnesium ribbon + dilute sulphuric acid.

(ii) Magnesium powder + dilute sulphuric acid.

(b) Give a reason for your answer in (a).

(c) Suggest two other methods by which the rate of this reaction can be altered.

31.1.6. Effect of light on the rate of reaction


Some reactions are photosensitive, that is, their rates are affected by light.

31.1.6.1. Investigation of the effect of light on the rate of reaction

Add 1 cm3 of sodium chloride solution to two test-tubes. To each test-tube add a few drops of
silver nitrate solution. Immediately, a white precipitate forms. Put one test-tube in a dark cup
board and the other in sunlight for about 4 minutes. Record your observations.

Sodium chloride solution forms a white precipitate with silver nitrate solution according to
the equation.

Ag+(aq) + Cl-(aq) AgCl(s)

In presence of light, the precipitate darkens because of the decomposition of silver chloride
to solver and chlorine. In absence of light, the precipitate remains white.

2AgCl(s) 2Ag(s) + Cl2(g)

The effects of light on hydrogen peroxide and concentrated nitric acid explain why they are
stored in dark-glass bottles.
31.1.7. Effect of pressure on the rate of reaction

A change in pressure only affects reactions which occur in the gas phase. When pressure
of a gaseous mixture is increased, the gases are compressed. This brings the reacting
particles together and thus increases the frequency at which the reacting particles together
and thus increases the frequency at which the reacting particles collide hence increased
rate of reaction.

32. Equilibrium

Equilibrium is the point in a reversible reaction when the rate at which the reactants are
forming the products is equal to the rate at which the products are dissociating to the
reactants. Therefore, at equilibrium, both the products and the reactants are present. A
reversible reaction is one which proceeds in both directions, that is, forward and backward.

32.1. Factors affecting equilibrium

The factors that affect equilibrium are temperature, pressure, concentration and catalyst. The
effect of these factors on equilibrium was first investigated by Louise Le Chatelier who came
up with a principle known as Le chateliers principle. The principle states that when a
chemical equilibrium reaction is distributed externally by a change in one of the factors upon
which it depends, the equilibrium shifts in a direction so as to reduce the effects of the
change.

(a) Temperature

Consider the Haber process where the reaction is exothermic.

3H2(g) + N2(g) 2NH3(g) H = 93.63 kJ mol-1


The forward reaction is exothermic and therefore the backward reaction is endothermic.
If heat is supplied, the equilibrium shifts in the direction which requires more heat, that
is, the backward reaction which uses up the excess heat occurs. However, if the
equilibrium vessel is cooled, the equilibrium shifts to the right, producing more ammonia.

(b) Pressure

In a gaseous system, an increase in pressure leads to a decrease in the volume of the gases
involved and the reverse is true. Let us again consider the Haber process.

N2(g) + 3H2(g) 2NH3(g)

One volume of nitrogen combines with three volumes of hydrogen to produce two
volumes of ammonia. The forward reaction occurs with a decrease in volume from four
to two volumes. If additional pressure is applied to the system, the equilibrium shifts in
the direction of a reduction in volume, that is, the forward reaction is favoured and more
ammonia is produced. If pressure of the system is decreased, the equilibrium shifts in the
direction of an increase in volume, that is, the backward reaction occurs and more of the
reactants are produced. Gaseous equilibrium reactions which are not accompanied by a
change in volume, are not affected by pressure changes e.g

N2(g) + O2(g) 2NO(g)

Exercise

Consider the following reaction equations:

H2(g) + I2(g) 2HI(g) H = +52 kJ mol-1

3H2(g) + N2(g) 2NH3(g) H = 93.63 kJ mol-1


What will be the effect on the concentration of hydrogen iodide and ammonia in the
equilibrium mixture of

(i) Increasing the temperature?

(ii) Decreasing the temperature?

(iii) Increasing the pressure?

(iv) Decreasing the pressure?

(c) Concentration

If the concentration of one of the substances present in an equilibrium reaction is


changed, the equilibrium shifts in the direction of a decrease in the concentration of the
added substance. Consider the following reaction.

N2(g) + O2(g) 2NO(g)

If extra oxygen is pumped into the reaction vessel, the equilibrium shifts in the direction
that results in a decrease in oxygen concentration, that is, the forward reaction occurs and
more nitrogen monoxide is produced. If the concentration of one of the reactants
decreases instead of increasing, the equilibrium will shift to cancel this decrease and the
backward reaction will occur to restore the balance. If more of nitrogen monoxide is
added to the equilibrium mixture, the backward reaction will occur producing more
reactants, that is, the equilibrium shifts to the left in order to offset the effect of the
increase in concentration of nitrogen monoxide. If there is a decrease in the concentration
of nitrogen monoxide, the forward reaction is favoured and the equilibrium shifts to the
right producing more nitrogen monoxide.

(d) Catalyst

Catalysts do not have any effect on the position of the equilibrium. In an equilibrium
reaction, a catalyst increases the rate of both the forward and backward reactions, that is,
a catalyst enables an equilibrium to be attained much more quickly than when there is no
catalyst.

Industrial applications of chemical equilibrium

This idea of equilibrium is applied in some industrial process. In the Haber process,
ammonia is synthesized from nitrogen and hydrogen according to the equation.

N2(g) + 3H2(g) 2NH3(g) H = -93.63 kJ mol-1

Ammonia is produced with a decrease in volume and therefore high pressure will
increase the yield of ammonia. The reaction is exothermic therefore low temperature will
favour the production of ammonia. However, by lowering the temperature, the rate of the
reaction is reduced. The presence of a catalyst will give a sufficient reaction rate despite
the relatively low temperature. In general, a maximum yield of ammonia is obtained by
using the following conditions.

(i) Very high temperature of 250 to 500 atmospheres.

(ii) Temperature of about 450 oC.

(iii) Catalyst of finely divided iron.

Aluminium oxide is added to make the catalyst more porous hence promoting its
effectiveness.

Exercise

1. The graph below shows the effect of temperature on the rate the reaction between
calcium carbonate of the same mass and excess 2 M hydrochloric acid.
(a) If curve B is for the reaction at 40 oC, which curve shows the reaction taking place at
(i) 20 oC

(ii) 60 oC

(b) Explain why the curves eventually end at the same level.
(c) State one other method that can be used to measure the rate of the reaction between
calcium carbonate and hydrochloric acid.

2. A certain mass of zinc powder was reacted with dilute hydrochloric acid at room
temperature.
(a) (i) Write an equation for the reaction.
(ii) Draw a graph to show how the volume of the gaseous product varied with time.

(b) What would be the effect of


(i) Adding copper(II) sulphate solution to the reaction mixture at room temperature?

(ii) Using the same mass of zinc granules instead of the zinc powder?

(c) Give a reason for your answer in (b) (ii).

3. 12.0 g of clean magnesium ribbon were added to 50 cm 3 of 1 M sulphuric acid. The


volume of the gas evolved was measured at fixed time interval until the reaction stopped.
(a) Write the equation of the reaction that took place.
(b) (i) sketch a graph of volume of the gas (on y-axis) against time (on-axis). Label the
graph G1.
(ii) On the same axis sketch the graph that would be obtained if 12.0 g of magnesium
powder were used instead of magnesium ribbon. Label this graph G2.

(c) (i) Give a brief explanation of the cause of difference in the graphs G1 and G2.
(ii) Name one other factor that can cause similar results as in b(i) above.
4. (a) 12 g of large pieces of calcium carbonate were reacted with 50 cm 3 of 2 M
hydrochloric acid at room temperature. The decrease in mass was measured at regular
intervals.
(i) Write an equation for the reaction.

(ii) Sketch a graph to show variations of decrease in mass with time.

(b) State what would be observed if the same mass of calcium carbonate powder was
used instead of the large pieces. Give a reason for your answer.

(c) State what would be observed if the same mass of large pieces of calcium carbonate
was used at 40 oC. Give a reason for your answer.

(d) Which of the two reactants was in excess?

5. The figure 11.11 shows the set up of the experiment used to study the rate of evolution of
a gas when 1.0g of powdered calcium carbonate was reacted with 50cm3 of 2 M
hydrochloric acid at 25 oC.

(a) Sketch a graph to show the variation of the volume of the gas evolved in the reaction
with time. Describe the shape of the graph.
(b) On the same diagram in (a) above sketch a graph to show the results obtained when
(i) 1.0 g of powdered calcium carbonate was reacted with 100 cm 3 of 1 M
hydrochloric acid at 25 oC.
(ii) 1.0 g of powdered calcium carbonate was reacted with 50 cm3 of 2 M
hydrochloric acid at 25 oC.

(c) Explain the shapes of the graphs you have sketched in (b) (i) and (ii) above.
(d) 1.0 g of powdered calcium carbonate was reacted with 20 cm 3 of 2 M hydrochloric
acid. Which one of the reactants was in excess?

6. When a certain volume of 0.1 M hydrochloric acid was reacted at room temperature with
excess of iron filings, a gas was produced.
(a) Draw a labeled diagram to show how the rate of reaction was determined.
(b) Write equation for the reaction that took place.
(c) Draw a sketch graph of the volume of the gas evolved against time.
(d) State how the rate of reaction would change if the reactions was carried out at a
temperature above room temperature.
TRIAL QUESTIONS FOR PAPER 1

- Attempt all numbers.

- Choose the correct answer from the alternatives A, B, C and D and write it
on the answer sheet provided.

Molar gas volume at s.t.p = 22.4 dm3 and at room temperature is 24 dm3.

1 Faraday = 96500

Use simple scientific calculators where applicable.

1. How many electrons are contained in the ion of atom with atomic number
15?
A. 10 B. 15 C. 18 D. 13

2. The best method of separating a mixture of sodium chloride and


ammonium chloride would be
A. Sublimation B. Distillation C. Filtration D. Crystallization

3. The most economic substance used to speed up the oxidation of sulphur


(IV) oxide to sulphur (VI) oxide during manufactured of sulphuric acid is
A. Platinised asbestos C. Manganese (IV) oxide

B. Vanadium (V) oxide D. Iron fillings


4. The mass of magnesium atoms which contains the same number of atoms
as 62 g of phosphorus is
A. 4.8 g B. 0.48 g C. 48 g D. 480 g

5. To substance X, was added barium nitrate solution followed by little dilute


nitric acid. A white precipitate formed and dissolved in the acid. The most
likely anion in X is:
A. SO42- B. SO32- C. Cl- D. NO3-

6. Which one of the following will conduct electricity in solid state?


A. Copper C. Lead (II) bromide
B. Sodium chloride D. Diamond

7. The volume occupied at room temperature by 7.7 g of Propane is:


(1 mole of Propane gas occupies 24 dm3 at room temperature)

A. 0.42 dm3 B. 4.2 dm3 C. 0.0238 dm3 D. 0.238 dm3

8. Concentrated Copper (II) Chloride solution was electrolysed using platinum


electrodes. The substance formed at the anode is
A. Copper metal B. Hydrogen C. Chlorine gas D. Oxygen

9. Solid ammonium carbonate when heated decomposes as follows:

NH4)2 CO3 (s) 2NH3 (g) + CO2 (g) + H2O (l)

The volume of ammonia gas produced at S.T.P when 6.5 g of ammonium


carbonate are heated is
(N = 14, H = 1, C = 12, O = 16)

A. 44.8 x 6.5 dm3 B. 34 x 6.5 dm3

96 96
C. 2 x 22.4 x 96 dm3 D. 2 x 17 x 96 dm3

6.5 6.5

10. Which one of the following salt will not give oxygen when heated?
A. Potassium chlorate (V) B. Copper (II) nitrate

C. Ammonium nitrate D. Silver nitrate

11. Substance P was separately treated in two test tubes using sodium
hydroxide and ammonia solution. In both cases it formed precipitate
soluble in excess. The cation in P is
A, Mg2+ B. Zn2+ C. Al3+ D. Pb2+

12. The formula of the molecule formed by W (2:8:2) and D (2:8:5) is


A. W 2 D3 B. W2D5 C. W3D6 D. W3D2

13. 3.65 of sodium nitrate dissolved is 63 g of water at room temperature. The


solubility of sodium nitrate is:
A. 63 x 100 g/100g of water B. 3.65 x 63 g/100g water

3 100

C. 63 x 1000 g/63 g of water D. 3.65 x 100 g/100g of water


3.65 63

14. Water gas is mixture of


A. Carbon monoxide and carbon dioxide.

B. Carbon monoxide and nitrogen.


C. Carbon monoxide and water vapour.

D. Carbon monoxide and hydrogen.

15. Strontium carbonate (SrCO3) relative formular mass = 148 reacts with
hydrochloric acid according to the equation.

SrCO3 (s) + 2HCl (aq) Sr cl2 (aq) + CO2(g) + H2O (l)

The volume of 0.4 M hydrochloric acid required to just dissolve 8.3 g of


strontium carbonate is

A. 148 x 16.6 cm3 B. 0.4 x 148 cm3

2 x 0.4 2 x 16.6

C. 2 x 04 cm3 D. 2 x 148 cm3

16.6x148 0.4 x 16.6

16. The mass in grams of potassium hydroxide used to prepare 500 cm3 of
0.2M solution for volumetric analysis is (K = 39 O = 16 H = 1)
A. 56 B. 5.6 C. 1.20 D. 11.2

17. Which of the following pairs of oxide will react with both dilute sulphuric
acid and sodium hydroxide solution?
A. CuO and ZnO B. PbO and MgO
C. Al2O3 and CuO D. ZnO and Al2O3

18. Which one of the following salt dissolves in water to give an alkaline
solution?

A. Na2CO3 B. NaCl C. NH4Cl D. (NH4)2 CO3

19. The percentage composition of nitrogen in one mole of solid ammonium


phosphate; (NH4)3 PO4 is (P = 31, O = 16 N = 14 H - 1)

A. 14 x 100 B. 54 x 100

149 149

C. 18 x 100 D. 42 x 100

149 149

20. Which one of the following oxide is reduced by carbon?


A. Magnesium oxide B. Calcium oxide

C Iron (III) Oxide D. Sodium oxide

21. Magnesium was burnt in air. When was water added to the cold products,
a gas was given off. The gas was

A. Hydrogen B. Nitrogen C. Oxygen D. Ammonia

22. Which one of the following is not natural fibre?


A. Silk B. Nylon C. Sisal D. Cotton
23. Which one of the following is a synthesis process?
A. Fe(s) + 2H Cl (g) NH3 (g) + HCl (g)

B. NH4 (Cl) (s) NH3 (g) + HCl (g)

C. 4NH3 (g) + 3O2 (g) 2N2(g) + 6H2O (g)

D. 2 Fe(s) + 3Cl2 (g) 2Fe Cl3 (s)

24. A hydro carbon Z when completely burnt in excess oxygen produce 220g
of carbon dioxide and 45 g of water. The empirical fomular of z in
A. CH B.. CH2 C CH3 D. C2H3

25. During sewage treatment process the stage at which sludge is formed is
A. filtration B. sedimentation

C. biological treatment D. Chlorination

26. Which one of the following is not a description of oxidation process?

A. Addition of oxygen B. Increase in oxidation waste

C. Gain of electrons D. Loss of electrons

27. Which one of the following substances is not a polymer?


A. Nylon B. Sucrose C. Starch D. Natural rubber

29. The concentration of moles per litre of 37.9 q of anhydrous sodium


carbonate in 250 cm3 of solution is (Na = 23, C = 14 O = 16)

A. 0.158 M B. 1.580 M C. 15.80 M D. 0.0158 M


30. Catalytic oxidation of ammonia produces nitrogen monoxide according to
the equation
4NH3 (g) + 5 O2 (g) 4NO (g) + 6H2O (l)
The volume of nitrogen monoxide produced when ammonia reacts completely with 60
cm3 of oxygen is

A. 180 cm3 B. 30 cm3 C. 48 cm3 D. 240 cm3

31. 30 cm3 of 2.6M hydrochloric acid completely reacts with 5.0 g of impure
Zinc carbonate. The percentage purity of zinc carbonate in the mixture is
(Zn = 65, C = 12, O = 16, H = 1, Cl = 35.5)

A. 60% B. 75% C. 80% D. 97.5%

32. A bottle of sulphuric acid is labelled 10M H2SO4. This means in every one
litre of solution there are

A. 10 moles of acid B. 10 molecules of acid

C. 10 molecules of hydrogen D. 10 moles of H+ ions

33. A beaker containing lime water is left exposed to the atmosphere for some
time. The white solid that formed at the bottom of the beaker is
A. Calcium oxide B. Calcium hydroxide

C. calcium carbonate D. Calcium hydrogen carbonate.

34. The gas collected when chlorine water is exposed to sun light is
A. Chlorine B. Oxygen C. Hydrogen chloride D. Hydrogen
35. Which one of the following salts is prepared by double decomposition?
A. Sodium sulphate B. Lead (II) nitrate

C. Sodium carbonate D. Lead (II) sulphate

37. 50 cm3 of a mixture of Ammonia and Carbon dioxide was passed over
heated copper (II) Oxide, and nitrogen formed occupied 20 cm3 under the
same condition as a mixture. The percentage of ammonia in the mixture is
A. 20% B. 80% C. 40% D. 60%

38. When dilute solution of sodium chloride is electrolysed using carbon


electrode. The equation for the reaction at the anode is represented by
A. 2H+ (aq) + 2e H2(g)

B. 2 Cl- - 2C Cl2 (g)

C. Na+ + e Na (s)

D. 4 OH (g) 2 H2O (l) + O2 (g)

39. the mass of copper deposited from a solution of Copper (II) chloride when
a current of 1.2A is passed for 3000Sec (Cu = 63.5)

A. 63.5 x 1.2 x 3000 B. 63.5 x 2 x 96,500

2 x 96,500 1.2 x 3000

C. 3000 x 1.2 x 96500 D. 96,500 x 2

2 x 63.5 63.5 x 1.2 x 3000

40. Graphite burns in air according to the equation.


C (s) + O2 (g) CO2 (g) H = -390 Kj
When 48 g of graphite is burnt in excess oxygen the heat produced is
A. -97.5 Kj B. -195Kj C. -780 Kj D. -1560Kj

41. In each of the questions 41 45 there is assertion (left and reason (right).
The correct answer will be as follows:
A. if both assertion and reason are true statements, and reason is correct
explanation of assertion.

B. if both assertion and reason are true statements but reason is not correct
explanation of the assertion.

C. if the assertion is true but reason is not correct statement.

D. if assertion is not correct statement but reason is true.

Instruction summarised

Assertion Reason
A True True and reason is a correct explanation
B True True but reason not a correct explanation.
C True Incorrect
D Incorrect True

41. Concentrated sulphuric acid BECAUSE concentrated sulphuric


removes water from ethanol acid is oxidizing agent.

42. Methanol is a hydrocarbon BECAUSE it is made up of carbon,


hydrogen and oxygen

43. Calcium hydrogen carbonate BECAUSE calcium hydrogen


carbonate is an acid salt.

44. Graphite does not conduct BECAUSE Graphite is non-metal.

45. Concentrated sulphuric acid BECAUSE Ammonia gas is alkaline


cannot be used to dry in nature.
ammonia gas

In each of the questions 36 40 one or more of the answers given may be


correct. Read each question carefully and then indicate on your answer sheet
according to the following:

A. if 1,2,3 only are correct.


B. If 1,3 only are correct.
C. If 2,4 only are correct.
D. If 4 only is correct.

INSTRUCTION SUMMARISED
A B C D
1,2,3 only 1,3, only 2,4 only 4 only Correct
Correct Correct Correct

46. Which one of the following is responsible for forming fur in kettles used

for boiling water?

1. Calcium sulphate
2. Calcium carbonate
3. Magnesium sulphate
4. Magnesium carbonate.

47. When sulphur dioxide is passed through sodium hydroxide solution for a
long time, which one of the following product(s) are formed?
1. Sodium sulphate 2. Sodium sulphite

3. Sodium hydrogen sulphate 4. Sodium hydrogen sulphite

48. Which one of the following is/are extracted by chemical reduction?


1. Iron 2. Sodium 3. Copper 4. Calcium

49. Which of the following solutions contains the same number of hydrogen
ions?
1. 1 l of 1M H2SO4 2. 1 l of 2M H Ol

3. 2 l of 1 M HNO3 4. 4.2 l of 2M HBr.

50. Which of the following affect the rate of reaction of gases?


1. Size of the molecule

2. Temperature

3. Surface area

4. Pressure.
NameIndex No

P545/1

CHEMISTRY MOCK 2008

Paper 1

1 hours.

Instructions:

The paper consists of fifty compulsory objective type of questions. For each of the
questions 1 50, four alternatives lettered A D are given. Indicate the letter which
corresponds to the best alternative.

1. The solvent used to extract esters from rose flowers is

A. Water

B. Ethanol

C. Concentrated sulphuric acid

D. Ethanoic acid.

2. Which of the following would give off carbon dioxide acid rapidest? 2 molar

A. Ethan dioic acid and powdered marble

B. sulphuric acid and powdered marble

C. hydrochloric acid and powdered marble.

D. Ethanoic acid and marble chips.

3. Ammonia gas was passed over a copper foil, the foil glowed red hot a
colourless gas which turned reddish brown on exposure to air. The role of
copper toil was to.

A. reduce ammonia to nitrogen (II) oxide.


B. oxidize ammonia to water and nitrogen.
C. catalyze oxidation of ammonia to nitrogen (II) oxide.
D. absorbs the excess ammonia to from a complete ion.
4. A homologous series is a group of compounds which.

A. have the same formula

B. display similar properties.

C. can be prepared using a common method

D. can be expressed in more than on way.

5. The following are catalysts except.

A. concentrated sulphuric acid

B. sunlight

C. hydrogen peroxide

D. bromine

6. Which of the following would form a white precipitate in excess sodium


hydroxide?

A. Ca2+ B. Zn2+ C. Al3+ D. NH4+

7. The solubility of a salt 75cm3 of a situated solution containing 30g 100-1 of


water

30 x 100 30 x 75 45 x 100 30 x 100


75 45 30 45
A. B. C. D.

8. Which of the following is the reason why zinc is not a typical transition metal?

It (does not)

A. form coloured ions


B. catalyse any reaction
C. have variable oxidation states
D. forms amphoteric oxide.
9. Which of the following is correct about stalactites and stalagmites?

A. vertical pillars of pure calcium carbonate growing from the floor and
root a cave.

B. decomposing gypsum rocks

C. milky suspension oozing out of breast like lime stone rocks.

D. impure calcium carbonate from chemical weather of limestone rocks.

10. Which of the following is the major atmospheric pollution?

A. radiation from nuclear power stations.

B. smog from active engines

C. carbon dioxide from carbon compounds.

D. sulphur (IV) oxide from industrial waste gases.

11. 25.00cm3 of phosphoric acid (H3PO4) completely reacted with 18.7 cm3 of
0.5M sodium

carbonate (Na2 CO3). The molarity of phosphoric acid is;

25 x 0.5
3 x 18.7 x 0.5 2 x 18.7 x 0.5
18 2 25 3 25
A. B. C. D.
3 x 2 x 25 x 0.5
18.7

12. The atomic mass of chlorine is 35.5. The mass of the isotopes of chlorine is
35 and 37: Which of the following is correct?
35 37
A. Cl is more abundant than Cl
37 35
B. Cl is heavier than Cl
37
C. Cl is radioactive
35
D. Cl is lighter and passes through the mass spectrometer fast.

13. Which of the following defines polymers?

A. synthetic or natural macromolecules which are multipurpose.

B. synthetic macromolecules which are non-biodegradable

C. macromolecules used as food.

D. macromolecules which foams in water.

14. Which of the following gases decolorizes acidified potassium manganate


(VII)?

A. Methane B. Sulphur (VI) oxide.

C. Ammonia D. Ethene

15. The last products of petroleum distillation are:

A. Bitumen and gas oil

B. Waxes and diesel

C. Kerosene and gasoline

D. Bitumen and waxes.

H H

16. Ethene; C=C is converted into polyethene. The number of hydrogen


atoms in

H H

polyethene when n= 50 is

A. 100 B. 200 C. 300 D. 400


17. To confirm sulphate ions one adds;followed
by.

A. barium chloride, dilute sulphuric acid

B. iron (II) sulphate, concentrated sulphuric acid.

C. silver nitrate; dilute nitric acid.

D. barium chloride; dilute hydrochloric acid.

18. Scum is.

A. soluble salt formed between acid and Ca2+

B. insoluble salt formed between Mg2+ and soap.

C. soluble salt formed between soap and NO +

D. insoluble salt formed between SO-3 and Ca2+

19. Which of the following is hardest to decompose?

A. NaHCO3 B. Ca(HCO3)2 C. KHCO3D. CaCO3

20. Which of the following is suitable to separate a mixture of sulphur and


ammonium chloride?

A. Dissolve in carbon disulphide, filter, crystallize


B. Dissolve in acid filter, evaporate to dryness.
C. Heat to melt, filter, cool
D. Dissolve in water, add silver nitrate, and filter.

21. Calcium carbonate reacts with hydrochloric acid according to the equation.

CaCO3(s) + 2HCl(aq) CaCl2(aq) + H2O(1) + CO2(g).

The volume of carbon dioxide formed at room temperature when 5.00g of


calcium carbonate completely reacts with hydrochloric acid is
(Ca=40, C=12, O=16, I mole of a gas occupies 24dm 3).

A. 1.2dm3. B. 1.12dm3. C. 3.0dm3. D. 2.8dm3

22. Which of the following is not a commercial use of ammonia?

Manufacture of ;

A. Nitric acid B. Smelling salt


B. Explosives C. fertilizers.

23. 20.00cm3 of 0.25 molar solution of an acid Hn X completely reacts with 35.00
cm3 of 0.5M sodium hydroxide.

The stoichiometry of the reaction between the acid and alkali is.

A. 1 : 3.5 B. 4 : 1 C. 1 : 1 D. 1 : 4

24. The formula of the ion formed when excess ammonia reaction with copper (II)
sulphate is.

A. [Cu (NH4)3]2- B. [Cu(NH3)]2+

C. [Cu (NH3)4]2- D. [Cu (NH3)4]2+

25. The gas formed when chlorine water is exposed is

A. hydrogen chloride. B. Oxygen.

C. hydrogen D. hydrogen chlorate(I)

26. Which of the following would not form a precipitate?

A. calcium hydroxide and carbon dioxide.

B. lead (II) nitrate and sodium sulphate

C. copper (II) nitrate and sodium chloride


D. silver nitrate and potassium bromide.

27. Undecane (C11H24) is converted to nonane (C9H20) by the process of

A. decomposition. B. cracking

C. decarboxylation D. dehydration

28. Nitrogen reacts with hydrogen according to the equation.

N2(g) + 3H2(g) 2 NH3(g)

The volume of the gaseous products formed when 50cm 3 of nitrogen is mixed
with 120cm3 of hydrogen is

A. 170cm3 B. 90cm3

C. 80cm3 D. 70cm3

29. The mass of a substance discharged during electrolysis depends upon.

A. concentration of the ions in solution

B. charge on the ion.

C. position in the electrochemical species.

D. nature of the electrode.

30. Which of the following pairs of metals would make the most efficient chemical
cell?

A. potassium and gold B. Zinc and copper

C. mercury and silver D. Sodium and calcium

31. Which of the following salts dissolves in water to make a solution of pH less
than 7 is

A. Ammonia chloride B. Calcium nitrate


C. Sodium sulphide D. ammonium ethanoate

32. 5.70g of hydrated salt X was strongly heat and weighed to constant mass
2.12g of the anhydrous salt remained. The number of molecules of water of
crystallization is

[X = 106, H2O = 18]

A. 2.2 B 15.8 C. 10.0 D. 6.0

33. Steel saucepans are coated with copper in order to

A. improve the beauty B. increase conductivity.

C. improve its tensile strength. D. prevent rusting.

34. The mass of nitrogen formed when 240 coulombs of electricity is used in
electrolysis of molten magnesium nitride [Mg=24, N=14, 1F= 96,500
coulombs]

A. 240 x 28 B. 240 x 74

96500 x 6 96500 x 2

C. 240 x 14 D. 6 x 240 x 28

6 x 96,500 2 x 96,500

35. Which of the following renders iron passive?

A. HNO3(1) B. H2SO4(aq)

C. NaCl(aq) D. HCl(1)
36. Which of the following processes take place naturally?

A. Fractional crystallization in salt lakes

B. Dry ice in the atmosphere

C. Neutralization in sea water

D. Displacement of iodine by chlorine in sea water

37. The oxidation stat of manganese in K2 Mn O4 [K= + 1 O = -2] is

A. +4 B. +6 C. +2 D. -6

38. Which of the following gases is given off in Crater Lake; (L. Nyamunuuka) W.
Uganda?

A. Sulphur (IV) oxide. B. ammonia

C. Carbon disulphide D. Hydrogen sulphide.

39. The bleaching action of chlorine is different from sulphur (IV) oxide in that
chlorine (forms)

A. choric (I) acid donates oxygen to the colored substance

B. hydrogen chloride adds hydrogen to the dye.

C. substances bleached turn brown on exposure.

D. makes substance stronger and whiter.

40. 1.00g of hydrocarbon X was found to contain 0.827g of carbon. It vapour


density is 29.5. Which of the following is the molecular formula of X is.

A. C2 H 2 B. C4H10

C. C6H12 D. C2 H 6

41. The number of moles of ammonium ions contained in 20cm 3 of 0.1M


ammonium sulphate (NH4)2SO4 is
A. 20 x 1 B. 20 x 1 x 2

1000x 10 1000 x 10

C. 20 x 1 x1 D. 1000 x 0.1

1000 x 10 2 20

In each of the question 41-45 one or more of the statement is/are


correct. Read the statements carefully then answer the questions
according to the following instructions. Choose

A. if statements 1, 2, 3, are correct.

B. if statements 1, 3, are correct

C. if statements 2, 4 are correct.

D. if statement 4 only is correct.

A B C D

1, 2, 3 1, 3 2, 4 4 only
correct correct correct correct

41. The reagent(s) most suitable to distinguish between Pb 2+ and Al3+ is (are)

1. Potassium iodide

2. Sodium carbonate

3. Dilute hydrochloric acid

4. Ammonium hydroxide.

42. Which of the following elements forms more than one oxide?

1. Nitrogen

2. Sulphur
3. Carbon

4. Calcium.

43. During electrolysis of concentrated hydrochloric acid using carbon electrodes

1. Oxygen is given off at the anode.

2. Chlorine is given off at the anode.

3. Hydrogen is given off at the cathode

4. Water molecules dissociate to restore the equilibrium.

44. Which of the following gas (es) is given at in marshes?

1. Carbon dioxide

2. Sulphur (IV) oxide

3. Ammonia

4. Methane.

45. Which of the following compounds contains multiple bonds?

1. C2 H 2

2. C3 H 3

3. N2

4. C10H16

In each of the questions 46-50 there is an assertion (statement) on


the left hand side and reason (statement) on the right hand side.
Read the assertion and reason carefully and then answer the
question according to the instructions. Choose.

A. if the assertion is true reason is true and correct explanation.


B. if the assertion is true, reason is true, not correct explanation.
C. if the assertion is true, reason is false
D. if the assertion is false, reason is true.

Summarized instructions.

A B C D

Asserti True True True False


on

Reason True True not false


correct correct
explanati explanation
on

46.

Carbon dioxide turns blue Because It contains low


litmus red hydrogen
concentration.

47.

Duralumin is used in Because It is lighter and


construction of air crafts. resistant to corrosion

Potash alum is added to water Because It removes colouring


48.
supplies matter.

Sulphur (IV) oxide turns Because It is an acidic anhydride


acidified potassium dichromate
green
49.

50. Ethane decolorizes acidified Because It is a saturated


potassium manganate (VII). hydrocarbon.

NameIndex No

P545/1

CHEMISTRY PRE MOCK08

Paper 1

1 hours.

Instructions:

The paper consists of fifty compulsory objective type of questions.


For each of the questions 1 50, four alternatives lettered A D
are given. Indicate the letter which corresponds to the best
alternative.

1. Which of the following processes would be most suitable for


separating sulphur and water?

A. Sublimation B. Chromatography

C. Filtration D. Crystallization

2. The action of heat on copper II carbonate is represented by


the equation
CuCO3(s) CuO(s) + CO2(g)

What will be the mass in gram of the oxide formed when


2.48 g of the carbonate is completely decomposed?

(Cu = 64, O = 16, C = 12)

A. 0.80 B. 0.88 C. 1.24 D. 1.60

3. The role of limestone in the extraction of iron in the blast


furnace is to

A. produce quicklime which removes the impurities


present.

B. raise the heat in the furnace to the required


temperature.

C. produce carbon dioxide that reduces the oxide.

D. lower the density of the molten iron so that it sinks to


the bottom.

4. The fermentation of sugar is brought about by adding to the


sugar solution

A. glucose B. water C. hydrochloric acid D.


enzyme

5. The mass in grams of potassium hydroxide used to prepare


500 cm3 of 0.2M solution for volumetric analysis is (K = 39
O = 16 H = 1)

A. 56 B. 5.6 C. 1.20 D. 11.2


6. The equation for the reaction of dilute sulphuric acid with
magnesium metal is Mg(s) + H2SO4 (aq) H2(g) +
MgSO4(aq).

The mass of magnesium metal that could give about 50 cm 3


of hydrogen gas at room temperature is (Mg = 24, l mole of
a gas occupies about 24000 cm3 at room temperature).

A. 50.0 g B. 5.0 g C. 0.5 g D. 0.05


g

7. Which of the following pairs of elements will combine


together to form a compound of simplest formula XY 2?

Atomic No. of X Atomic No. of Y

A. 18 9

B. 11 17

C. 9 19

D. 20 9

8. Which of the following changes will not take place

A. Mg(s) + 2H+(aq) H2(g) + Mg2+(aq)

B. Cu2+(aq) + 2OH(aq) Cu(OH)2(s)

C. Cu(s) + 2H+(aq) H2(g) + Cu2+(aq)

D. Zn(s) + Cu2+ (aq) Cu(s) + Zn2+(aq).

9. The number of moles of ammonium ions contained in 20 cm 3


of 0.1M ammonium sulphate, (NH4)SO4 is
A. 20 x 1 B. 20 x1 x2

1000 10 1000 10

C. 20 x 1 x D 1000 x 0.1

1000 10 20

10. Which of the following could be used to distinguish between


hydrogen chloride and chlorine gas?

A. Litmus paper B. Lime water

C. A burning splint D. Aqueous potassium


dichromate

11. The equation for the reaction between carbon dioxide and
lime water is

CO2(g) + Ca(OH)2(aq) CaCO3(s) + H2O(l)

One mole of a gas occupies 24 litres at room temperature.

What is the minimum volume of carbon dioxide required to


precipitate

1 g of calcium carbonate? ( C = 12, 0 = 16, Ca = 40).

A. 24 cm3 B. 100 cm3 C. 240 cm3 D. 353 cm3

12. X, Y and Z are elements in the same short period of the


periodic Table. The oxide of x is amphoteric, the oxide of Y is
basic, and the oxide of Z is acidic. Arranged in order of
increasing atomic number they are:
A. YXZ B. XYZ C. ZYX D. YZX

13. 1.0 g of hydrocarbon T was found to contain 0.827 g of


carbon, and vapour density of 29.5. What is the molecular
formular of T?

A. C2H2 B. C4H10 C. C4H12 D. C2H6

14. Element Y has atomic mass 27 and 14 neutrons. The


electronic configuration of Y is

A. 2: 8: 3 B. 2: 8: 4 C. 2: 8: 7 D. 2: 8:
18: 8: 5

15. Which of the following ions reacts with sodium chloride


solution to form a white precipitate soluble on heating?

A. Mg2+ B. Ca2+ C. Pb2+ D. Al3+

16. Ammonia is oxidized by copper (II) oxide according to the


following equation.

2NH3(g) + 3CuO(s) 3Cu(s) + N2(g) + 3H2O(l)

The volume of ammonia which will be oxidized by 6.0 g of


copper (II) oxide at s.t.p (Cu = 64, 0 = 16, 1 mole of a gas
occupies 22.4 dm3) is

A. 2240 cm3 B. 560 cm3 C. 1120 cm3 D. 120


cm3

17. Which one of the following is used to separate sodium and


chlorine in Downs cell?
A. Iron B. Mercury C. Steel D. Copper

18. The mass of copper deposited when 240 coloumbs of


electricity is used in the electrolysis of copper (II) sulphate is

A. 240 x 64 B. 64 x 96,500

2 x 96,500 2 x 240

C. 240 x 2 x 64 D. 2 x 96,500

96,500 240 x 64

19. The simplest formula of aluminium oxide is Al 2O3 and the


formula of aluminium acetate is (CH 3COO)3 Al. Deduce the
charge on the acetate (CH3COO) ion

A. +1 B. +3 C. -1 D. -3.

20. In the manufacture of sulphuric acid, sulphur dioxide and


oxygen are passed over a heated catalyst. This is necessary
because a catalyst

A. alters the rate of a chemical reaction.

B. helps to produce more sulphuric acid.

C. helps to purify the gases before they react.

D. absorbs the heat produced when the gases react.


21. The arrangement in the two columns below show elements
in group I and group VII in the Periodic Table; in their correct
order.

Group I Group VII

W top of group E

X F

Y G

Z H

Which pair of elements will produce the most vigorous


reaction, when combined together?

A. W+E B. Z+E C. Z + H D. W+
H

22. Which of the following suggests that in many solids particles


may be packed in a regular manner?

A. Shape of crystals B. Density of crystals

C. Size of crystals D. Solubility of solids in water

23. During the analysis of copper II oxide by reduction, the


following figures were obtained.

Mass of test tube = 18.12g

Mass of test tube and copper (II) oxide = 20.64g

Mass of test tube and copper = 20.12g


The percentage of oxygen in copper (II) oxide is

A. 2.00 x 100 B. 0.52 x 100

20.64 2.52

C. 2.00 x 100 D. 2.52 x 100

2.52 20.64

24. Which of the following is NOT true about metallic bonding?

A. There are many positive ions in the structure.

B. The delocalised electrons experience no attraction from


the atoms.

C. In the structure the atoms are closely packed.

D. The delocalised electrons do not belong to any


particular atom.

25. If you wished to make a sample of the acid salt Na 2HPO4


using sodium hydroxide (NaOH) and phosphoric acid (H 3PO4);
which of the following quantities would you mix?

A. 10 cm3 of 1M NaOH and 5 cm3 of 1M H3PO4

B. 10 cm3 of 1M NaOH and 10 cm3 of 1M H3PO4

C. 10 cm3 of 1M NaOH and 15 cm3 of 1M H3PO4

D. 10 cm3 of 1M NaOH and 20 cm3 of 1M H3PO4


26. Metals which are very reactive are usually extracted from
their ores by

A. electrolysing the molten ore.

B. heating the ore in open air.

C. heating the ore with carbon.

D. electrolysing its aqueous solution.

27. The chief disadvantage in using propanone as a solvent for


the extraction of oil from groundnuts is because of
propanones

A. solubility in water B. low boiling point

C. low density D. inflammability

28. 60 cm3 of hydrogen and 20 cm3 of oxygen were made to


react in a closed container according to the equation

2H2(g) + O2(g) 2H2O(l).

At the end of the reaction:

A. only 20 cm3 of hydrogen would have reacted.


B. only 80 cm3 of water would have been formed.
C. only 20 cm3 of hydrogen would remain.
D. all 60 cm3 of hydrogen would have reacted.

29. An aqueous solution of nickel chloride was electrolysed


between platinum electrodes. The mass of nickel deposited
during the electrolysis depends on

A. the number of coloumbs passed.


B. the atomic mass of nickel.

C. the number of electrons flowing.

D. the number of charges in the nucleus.

30. Chlorine is passed through sodium bromide solution. Which


of these statements/observations is NOT correct about the
reaction?

A. Sodium and bromine are formed.

B. The solution turns brown.

C. Chlorine is the oxidising agent.

D. Sodium chloride and bromine are formed.

31. Which of the following decompose on heating?

A. ZnCO3 and LiCO3 B. K2CO3 and CaCO3

C. Na2CO3 and CaCO3 D. CuCO3 and PbCO3

32. Which is soluble in water?

A. PbSO4 B. ZnSO4 C. PbCO3 D. ZnCO3

33. 1.22 g phosphorous combined with 0.95 g of oxygen.. The


simplest formula of the oxide is (P = 31, O = 16)

A. P2O3 B. P3O2 C. P4O D. P5O10


34. When ethene gas was passed through bromine water, the
red colour of the bromine water turned colourless. The
formula of the colourless compound formed is

H H H H

A. Br C C Br B. Br C C H

H H Br H

H Br

C Br C = C Br D. Br C = C Br

H Br

35. Which of the following is an alkene?

A. C2H2 B. C3H6 C. C4H10 D. C5H12

36. 560 cm3 of a gaseous substance G has mass of 1.20 g at


s.t.p. The molecular mass of G is (1 mole of gas occupies
22.4 dm3 at s.t.p).

A. 49.29 B. 48.00 C. 26.64 D.


12.32

37. The formula of the iron formed when excess ammonia reacts
with copper (II) sulphate solution is
A. [Cu(NH4)4]2- B. [CuNH4)3]2+

C. [Cu(NH3)4]2- D. [Cu(NH3)4]2+

38. In the petroleum industry the hydrocarbons of higher


molecular masses are converted to hydrocarbons of lower
molecular masses by

A. Fractional distillation B. Cracking

C. Decomposition D. Synthesis

39. Which of the following metals would form the best pair for
construction of the chemical cell?

A. Potassium and Gold B. Magnesium and Zinc

C. Aluminium and Sodium D. Zinc and Copper

40. The structure below represents a polymer

CH3 H CH3 H

C C C C
H H H H

The monomer for the polymer is

A. H2C = CH2 B. CH3CH2CH3 C. CH3 C = CH2 D.


CH3CH3

In each of the question 41 45 one or more of the statements is


(are) correct. Read the statements carefully then choose the
answer according to the following instructions: Choose

A. if statements 1, 2, 3 are correct.


B. If statements 1, 3 are correct.
C. If statements 2, 4 are correct.
D. If statement 4 only is correct.

SUMMARIZED INSTRUCTIONS.

A B C D
1,2,3 correct 1,3 correct 2,4 correct 4 only correct

41. Which of the following decolourize acidified potassium


manganate (VII)?

1. Ethene

2. Chlorine

3. Sulphur dioxide

4. Hydrogen sulphide
42. When sodium hydroxide was added to separate test tubes
containing cations, there was no observable change. Which
of the following were the cations?

1. Ca2+

2 Mg2+

3. Al3+

4. NH4+

43. Which of the following statements is true about the zinc


copper cell?

1. The zinc rod is negatively charged.

2. The copper rod dissolves to form copper ions.

3. The copper ions are discharged at copper rod.

4. The zinc ions are discharged at zinc rod.

44. On combustion of ethane, water and carbon dioxide are the


only products.

This shows that ethane contains

1. Oxygen 2. Carbon 3. Water vapour 4.


Hydrogen

45. Which of the following substance(s) is/are raw materials used


during the manufacture of sodium carbonate in the solvay
process?
1. Concentrated sodium chloride (Brine)
2. Ammonia

3. Carbon dioxide

4. Calcium oxide

In each of the questions 46 50 there is an assertion (statement)


on the left hand side and a reason on the right hand side. Read
the questions then answer the questions according to the
instructions.

Choose:

A if the assertion is true, Reason is true and is correct


explanation of the assertion.

B if the assertion is true, Reason is true but is not correct


explanation for the assertion.

C if the assertion is true, Reason is a false statement.

D if the assertion is false, Reason correct.

SUMMERISED INSTRUCTIONS.

Assertio A B C D
n
True True True False
Reason True and True not False True
correct correct
explanation
explanation
46. In the contact process sulphur BECAUSEthe oxide is lost in
mists dioxide is dissolved in 98% when dissolved
in water.

Concentrated sulphuric acid

47. Carbon dioxide turns lime BECAUSEcarbon dioxide is


acidic gas.

Water milky

48. When concentrated nitric acid BECAUSEnitric acid oxidises,


Iron (II) is added to Iron (II) sulphate ions to
Iron (II) ions.

solution, the changes from

green to brown.

49. Sodium chloride is added BECAUSE sodium


chloride is a

to a mixture of fats and normal salt.

sodium hydroxide solution

during saponification.
50. A solution of hydrogen chloride BECAUSE Methyl
benzene is a in methyl benzene liberates
non-ionising solvent.

carbon dioxide from a carbonate.

Name: -------------------------------------------------------------------------- Index No.


--------------

Class --------------------------- Signature ---------------------


545/1

S. 4 CHEMISTRY PAPER ONE

MOCK EXAMINATIONS, 2009

TIME: 1 HOUR 30 MINUTES.

The paper consists of 50 compulsory objective type of questions. For each


of the questions 1-50, four alternatives lettered A, B, C and D are given.
Indicate the letter which corresponds to the best alternative in the box on
the right hand side of each question. Do not use a pencil. If you change
your mind cross the letter using a single line, indicate the letter of your new
choice besides the box.

FOR OFFICIAL USE:


1. A chemical garden was constructed by adding sodium silicate to a mixture
of coloured salts in water, green, white, blue, yellow pillars (flowers) rose in
the beaker. Which of the following ions were in the mixture?

Cu 2 Mg 2 Ca 2 Fe 2 Zn 2 Cu 2 K NH +4
A. B.

Fe 2 Fe3 Cu 2+ Ca 2+ Cu 2 Pb2+ Na + Fe2+


C. D.

14
6 C
2. Isotopic carbon twelve ( ) is used to

A. Calibrate the mass spectrometer

B. Trace metabolic path ways

C. Carbon dating fossils

D. Cure cancerous out growths.

3. Which of the following pairs of metals is suitable for construction of a


chemical cell?

A. Mg/Na B. Fe/Cu C. K/Au D. Cu/Zn


4. Which of the following would least affect the rate of reaction between
marble and hydrochloric acid?

A. Temperature B. Surface area

C. Catalyst D. Proximity

Read the account below to answer question 5. Barium hydroxide was


electrolyzed using carbon electrodes. Dilute sulphuric acid was added to
the solution a drop at a time. The bulb shone brightly at first became
dimmer until it stopped completely.

5. Which of the following was the reason?

A. preferential discharge of hydroxyl ions.

B. precipitation of barium sulphate

C. decomposition of barium hydroxide

D. formation of water from hydrogen and hydroxyl ions.

6. An ore is a (an)

A. native rock containing a metal

B. pure compound from which a metal is extracted.

C. precious mineral contained in a rock.

D. mixture of compounds from which a metal is extracted.

7. An organic compound is represented by the formula.


CH3

H 3C C CH3

H
The name of the compound is

A. butane B. propane

C. 2 methyl butane B. 2 methyl propane

8. Which of the following is used as a catalyst in the manufacture of sulphur


(VI) oxide?

A. Iron B. Bromine

C. Platinum D. Rhodium

9. Colourless crystalline sodium carbonate forms a white powder. Which of


the following is the formula of the white powder?

A. Na2 CO3.10H2O B. Na2 CO3 .H2O

C. Na2CO3.9H2O D. Na2 CO3.H2O

10. 20.00cm3 of oxygen was exploded with 50.00cm3 of hydrogen. The


products cooled to room temperature. The total volume of the gaseous
product is;

A. 50cm3 B. 70cm3

C. 40cm3 D. 30cm3

11. The mass of copper deposited when 240 coulombs of electricity is used in
electrolysis of copper sulphate is (Cu=63.5, IF- 96,500 coulombs).
63.5x240 63.5 x 240 240 x63.5 x 2
96500 2 x96500 96500
A. B. C. D.
2 x96500
240 x63.5

12. A glass rod was dipped in a solution X and held in a stream of hydrogen
chloride gas. The drop on the end of the glass rod turned milky. The cation
present in the solution X is;

4
A. Ca2+ B. NH C. Ag+ D. Ba2+

13. The volume of 0.25 molar hydrochloric acid required to react with 20.00cm3
of 0.1 molar sodium carbonate is given by;
20 x0.1 20 x 0.25 2 x 20 x 0.25 2 x 20 x0.1
2 x0.25 2 x 0.1 0.1 0.25
A. B. C. D.

14. The atomic number of elements X and Y is 7 and 9 respectively. The


formula of the compound formed between X and Y is

A. XY3 B. X3 Y2 C. XY D. X 3Y

15. Which of the following gases does not reduce copper (II) oxide to copper?

A. Hydrogen B. Carbon monoxide

C. ammonia D. Carbon dioxide


16. Which of the following hydroxides forms a yellow solid on cooling?

A. Pb(OH)2 B. Zn(OH)2

C. Fe(OH)3 D. Cu(OH)2

17. Hydrogen peroxide decomposes in the presence of the Fe3+ the results of
the experiment are known in the sketch Y. To obtain the sketch X one has
to.

A. use Fe2+ in place of Fe 3+ B. elevate the temperature


C. omit Fe3+ D. use excess Fe3+

18. Which of the following reactions is endothermic?

A. combustion of carbon fuels

B. dehydration of copper (II) sulphate

C. neutralization of sodium hydroxide by hydrochloric acid.

D. dehydration of sugar by concentrated sulphuric acid.

heat
19. Carbon burns in air according to the equation (C(s) + O2(g) CO2(g)
H 390KJ. The heat produced when 48g of carbon completely burns in air
is;

[C=12, ]

A. 97KJ B. 195KJ C. 780KJ D. 1560KJ

20. Part of the reactivity series is given below Cu, Mg, C, K, Na, Al. The
elements are not in the correct order. Which of the following equations
represents a possible reaction?
heat
A. 2Na2O(s) + C 4Na(s) + CO2(g)

heat
B. 2MgO(s) + C 2Mg(s) + CO2(g)

heat
C. 2 Al + 3CuO Al2O3(s) +3Cu(s)

D. 2K2O(s) +4Na(s) 2Na2O(s) +4K

21. Which of the following acids will yield the least volume of carbon dioxide
when reacted with a fixed mass of marble?

A. Nitric acid B. Sulphuric acid

C. Hydrochloric acid D. Ethanoic acid.


22. Which of the following is an example of diene (Elastaner= substance which
stretches)

A. Polyethene B. Rubber

C. Cellulose D. Bakelite

23. The diagram below is used to illustrate the solubility of a gas. The delivery
tube on the left is to.

A. Equalize pressure
B. prevent suck back

C. allow in little water

D. allow entry of carbon dioxide which turned blue litmus red

24. Which of the following is a commercial use of alkanes? Manufacture of

A. fuel for combustion B. lower alkanes by cracking

C. polymers D. Degreasing agents.

25. Which of the following has the highest oxidizing power?

A. Carbon B. Nitrogen

C. Oxygen D. Fluorine

26. Which of the following is carried out during the extraction of sulphur using
the frash method?

A. natural gas is compressed and heated under controlled air.

B. steam and super heated water are sent to sulphur beds

C. super heated water and hot air are sent to the sulphur deposits

D. carbon disulphide and hot water are sent to the sulphur beds.

27. Gas X has a repulsive odour. The gas X is


A. SO2 B. NH3

C. H2 S D. CH4

28. Isomers are;

A. elements which are alike

B. compounds with same molecular mass.

C. compound which form a homologous series

D. different forms of an element.

29. Element X decays into element Y according to the equation.


210
82
X 210
83
Y

The particle which was used is.



A. Alpha ( ) B. Beta ( )

C. Gamma ( ) D. neutron ( )

30. Electrovalent compounds are usually

A. solids which sublime on heating

B. liquids which have a high boiling point.

C. ions which migrate to poles during electrolysis

D. solids which have a high melting point.

31. Rhombic and prismatic sulphur are allotropes because

A. each can be converted into the other by altering the conditions


B. both dissolves in carbon disulphide

C. each combust in air to form sulphur (IV) oxide

D. both are yellow in colour.

32. Which of the following is incorrect about ammonia? (Manufacture of)

A. explosives

B. restore breath

C. dry (Leclanch) cells

D. burn in air to form reddish brown fumes.

33. Iron reacts with steam according to the equation

3Fe(s) + 4H2O(g) Fe3O4(s) +4H2(g)

The mass of iron required to form 662cm3 of hydrogen at room


temperature is

56 x 662 x1000 3 x56 x662 56 x3x662 x 4


4 x 24 x3 4 x 24 x1000 1000 x 24
A. B. C. D.
4 x 24 x1000
3 x662 x56
34. Which of the following is incorrect about ethene gas?

A. accelerates rotting of fruits

B. decolorizes acidified potassium manganate (VII)

C. is less dense than air.

D. burns with a yellow sooty flame.

35. When 0.1 Foradays were passed through a solution containing a metal ion
X .

5.96g of the metal was formed. The charge on the ion X is 1 Foraday =
96,500 coulombs, X= 179.

A. + B. 2+ C. 3+ D. 3-

36. Which of the following will form a green precipitate when sodium carbonate
solution is added?

A. Ca2+ B. Fe2+ C. Pb2+ D. Fe3+

37. A student soaked a white material in sodium chlorate I solution (Jik). The
material became crumpled and turned cream (dirty white) instead of
becoming white

Which of the following was the material made of?

A. cotton B. sisal C. wool D. nylon

38. When testing for Cl- dilute nitric acid is added to

A. silver sulphate B. lead chloride

C. barium nitrate D. silver nitrate


39. Which of the following mixtures would produce chlorine gas?

A. sodium chloride and concentrated sulphuric acid.

B. trilead tetroxide and concentrated hydrochloric acid.

C. potassium manganate (VII) and concentrated nitric acid.

D. sodium chloride and iodine.

40. Sulphuric acid reacts with sodium hydroxide according to the equation.

H2SO4(aq)+ 2NaOH Na2SO4(aq) + 2H2O(1)

What volume of 2 molar sulphuric acid is required to completely react with


10cm3 of 2 molar sodium hydroxide?

A. 5cm3 B. 10cm3 C. 20cm3 D. 30cm3

In each of the questions 41-45, one or more of the statements is (are)


correct. Read the statement carefully and indicate the correct answer
according to the instructions. Choose

A. if statements 1, 2, 3 are correct


B. if statement 1, 3 are correct
C. if statement 2, 4 are correct
D. if statement 4 only is correct.

Summarized instructions:

A B C D
1, 2, 3 correct 1, 3 correct 2, 4 correct 4 only correct

41. Which of the following is an example of a polymer (s).

1. polyethene 2. bakelite

3. rubber 4. decane
42. Which of the following is (are) radioactive?
266 3
109 Une 1 H
1. 2.

18 15
8 O 7 N
3. 4.

43. Which of the following is an alkene?

1. CH2 2. C3 H6

3. C5H10 4. C4 H8

44. Which of the following would form a yellow precipitate when potassium
iodide solution is added?

1. Pb2+ 2. Al3+

3. Ag+ 4. Cu2+

45. During electrolysis of concentrated hydrochloric acid using carbon


electrodes.

1. oxygen is given off at the anode

2. chlorine is given off at the anode

3. hydroxyl ions accumulate at the cathode

4. hydrogen is given off at the cathode


In each of the questions 46-50, there is an assertion (statement) on the left
hand side and a reason (statement) on the right hand side. Read the
statements carefully then indicate the correct answer according to the
instructions.

Choose;

A. If the assertion is a true statement, reason true and correct


explanation of the assertion.
B. If the assertion is a true statement, reason is true not correct
explanation of the assertion.
C. If the assertion is true, reason is false
D. If the assertion is false, reason is true.

Summarized instructions:

A B C D
Assertion True True True False
Reason True, correct True not False true
explanation explanation

46. An hydrous cobalt (II) chloride forms Because It is deliquescent


a blue solution

47. Aluminium reacts explosively with Because Two moles of aluminium react with 3
copper (II) oxide moles of copper
(II) oxide
48. Sodium vapour is used in street Because It burns with a yellow flame to
lights light the dark streets.

49. Fluoro carbons are pollutants Because They deplete the ozone layer which
prevents ultra violet radition direct
contact.

50. Hydrogen chloride is an acid an Because Hydrochloric acid is formed on


hydride dissolving in water.

Wish you success


aName: ----------------------------------------------------------------------------- Index No.
---------------------------

Signature
--------------------------

545/1

Chemistry

End of term one 2010

Paper 1

Tuesday 20th April, 2010

Time: 1 hours

INSTRUCTIONS TO CANDIDATES.

This paper is made up of 50 objective type questions. Answer all questions. You are
required to write the correct answer, A, B, C or D in the box provided on the right hand
side of each question.
Do not use pencil.

FOR EXAMINERS USE ONLY.


1. A Solid substance S was heated ,and turned to gas. What is the name given to this
physical change?

A. gas to solid B. sublimation C. evaporation D. Melting

2. Which of the following hydroxide is soluble?

A. Calcium hydroxide B. magnesium hydroxide

C. copper (II) hydroxide D. Iron (II) hydroxide

3. Phosphoric acid H3PO4 dissociates into two hydrogen ions, the 3rd hydrogen ion is not
ionisable. From this, conclusion can be made that phosphoric acid is;

A. tribasic B. monobasic C. dibasic D. weak

4. A gasless dense than air, soluble in water and alkaline in nature is best collected by;

A. water B. Down ward delivery

C. acidic drying agent D. basic drying agent

5. Which one of the following would cause a basic change in sodium hydroxide when
bubbled in excess?

A. CO2 B. SO2 C. H2S D. Cl2

6. Which one of the following is the reason why sodium extracted by Downs process is
kept under nitrogen?

A. prevent sodium from reacting with chlorine

B. form a nitride that would make sodium passive

C. Create inert atmosphere

D. eliminate oxygen that would react with sodium

7. Which one of the following is used to prepare a sample of soap in the laboratory?

A. precipitation B. neutralization C. synthesis D. displacement

8. In the manufacture of sugar from sugar cane, the syrup is passed through a sulphuriser.
The role of the sulpuriser is to;

A. add sulphur nutrient to sugar

B. remove the brown colour of sugar

C. produce sulphur (II) oxide to kill the germ


D. enable sugar crystals to form

9. Which one of the following is observed when two drops of phenolphthalein indicator is
added to dilute hydrochloric acid>?

A. remains colourless B. turns yellow

C. remains orange. D. turns pink

10. Which one of the following is observed at the anode when molten lead (II) bromide is
electrolysed?

A. dull grey solid B. greenished yellow vapour

C. colourless vapour D. reddish brown vapour.

11. Percentage of carbon in sucrose, C12H22O11(Relative molecular mass,342) is

12 100 12 100 144 100 342 100


A. 342 B 144 C 342 D 144

12. Which one of the following is observed when at the anode when copper (II) chromate is
electrolysed?(Cu2+ is blue,CrO42- is yellow)

A. yellow B. blue C.orange D. green

13. The loss in mass when Iron (II) sulphate -7-water is strongly heated is heated is due to;

A. Change of iron (II) sulphate to iron (III) sulphate

B. change of water to water vapour

C. decomposition of sulphate ions to oxides of sulphur

D. change from hydrated to anhydrous salt.

14. Which one of the following is the reason why tree are planted in cities ?

A. improve the beauty B. provide oxygen to clean the environment

C. provide CO2 for photo synthesis D. provide shelter for people and animals.

15. Which one of the following decolourises litmus paper?

A. Cl- B. ClO- B. SO32- D. HSO4-

16. Which one of the following salt would have PH greater than 7?

A. NH4Cl B. (NH4)2SO4 C. (NH4)2CO3 D. NH4NO3

17. In industrial areas, wall painted with lead compounds blacken. The ions responsible for
tarnishing the walls is;
A. S2- B. Cl- C. C4- D. O2-

18. The mass in grams of OH ions in 0.25M NaOH solution is (H=1, O=6)

0.25 4
0.25 17 0.25
0.25 17 17
A. 17 B. C. D. 4

19. The minimum volume of 1M HCl(aq) required to produce 0.25g of Hydrogen with excess
Magnesium is

A. 24cm3 B. 100cm3 C. 250cm3 D. 1000cm3

20. When ammonium nitrate is heated, it produces

A. nitrogen dioxide B. ammonia C. dinitrogen oxide D. nitrogen


monoxide

21. The reaction of iron and chlorine gas

A occurs at ordinary temperature. C. requires platinum catalyst

B Produces iron (II) chloride. D. Produces iron (III) chloride.

22. The Fountain experiment is used to determine the solubility of gases except

A. NH3 B. HCl C. SO2 D. SO3

23. Iron (III) chloride reacts with Sodium hydroxide according to the equation

FeCl3 (aq) + 3NaOH(aq) Fe(OH)3 (s) + 3NaCl(aq)

The mass in grams of iron (II) hydroxide precipitated when 6cm3 of 0.1M sodium
hydroxide reacts with iron (II) chloride is;

[Fe(OH)3 =107]

6 107 0.1 6 107 1


0.1 6 107 3 0.1 107
0.1 1000 3 1000
A 1000 B. C. D. 1000

24. 20.00cm3 of 0.1M sodium carbonate reacted with 25.00cm3 of XM phosphoric acid. The
morality of the acid is;

25 0.1100 2 0.1 20 1000 3 0.1 20 1000 20 0.11000


A. 20 1000 B. 3 1000 25 C. 2 1000 25 D. 1000 25

25. In which of the following is a catalyst not used?

A. Ostwalds B. Harber C. contact D. solvay


26. Which of the following is used to test for the nitrate ions?

A. Iron metal B. silver nitrate

C. iron (II)sulphate D. Concentrated nitric acid.

27. Which one of the following is used to determine the formular of water?

A. electolysis using Hoffmans Voltmeter B. titration of alkali and hydrated


acid

C. reduction of water using a metal D. synthesis of water from its


elements

28. The atomic number of elements in the periodic table is given,

(K=3 L=7 M=9 N=10)

Which of them have the largest atomic volume?

A. K B. L C. N D. M

29. Which one of the following is the reason why sulphur IV oxide is used in fruit juices?

It is (has)

A. reducing agent B. oxidizing C. pleasant taste D. bleaching agent

30. Which one of the following causes death in coal mines/

A. carbon particles from coal B. carbon dioxide and


chlorine

C. hydrogen sulphide and methane D. Carbon monoxide

31. Which one of the following does not decompose on heating?

A. sodium chloride B. sodium nitrate

C. sodium chlorate(V) D. sodium hydrogen carbonate

32. When 40g of the oxide of element X was reduced,32.00g of the element was obtained.
The simplest formula of the oxide is; (X=65, O=16)

A. X2O B. XO C. XO2 D. X2O3

33. Which one of the following contain the same number of particles as 8.00g of oxygen?

(O=16 Cl=35.50 S=32.00 H=1.00 Mg=24.00 N=6.00, NA=6.02X1023 particles)

A 1.00g of hydrogen B. 35.00g of chlorine

C. 16.00g of sulphur D. 6.00g of magnesium.


34. Iron nails were dipped in liquid X. the nails covered with brown coating. The liquid X was;

A. copper II sulphate B. concentrated nitric acid

C. molten grease D. brown paint

35. A pure sample of water is obtained by;

A .harvesting rain water B. combustion of hydrogen

C. reducing copper II oxide D. distilling copper II sulphate

36. Which of the following may be used to identify chloride ions?

A. damp blue litmus paper held above the solution

B. lead II nitrate followed by dilute nitric acid and warmed.

C. barium chloride followed by dilute hydrochloric acid.

D. Silver chloride followed by dilute nitric acid.

37. An atom Y has 20 nuetrons and mass number 37,the electronic configuration of X is

A 2:8:7 B 2:7 C 2:8:8:7 D 2:8:8:1

38. Which one of the following hydrocarbons contains 20% by mass of hydrogen.

A. C2H2 B. CH4 C. C2H6 D. C4H10

39. Which one of the following would bring about the change Fe2+ Fe3+ ?

A. H2S B. H2O2 C. HNO3 D. SO2

40. Calcium carbonate decomposes according to the following equation;

CaCO3(s) CaO(s) + CO2( g)

The volume of carbondioxide produced at s.t.p when 5.00g of calcium carbonate is


completely decomposed is;(Ca =40 C=12 molar gas volume at s.t.p. is 22.4dm3

A. 112cm3B. 224dm3 C. 1120cm3 D. 2240dm3

In each of the following questions 41-45 one or more of the statement(s) is ( are) correct

Indicate the letter which corresponds to the best alternative according to the instructions.
Choose;

A If statements 1, 2,3 are correct.

B If statements 1,3 are correct

C If statement 2,4 are correct

D If statements 4 is correct

A B C D

1,2,3 correct 1,3 correct 2,4 correct 4 is correct


41. Which of the following turns blue litmus red?

1. chlorine 2. sulphur IV oxide

3. silicon IV oxide 4. nitrogen IV oxide.

42. Which one of the following forms a black residue on strong heating?

1 Fe(NO3)2 2. Cu(NO3)2 3. CuCO3 4 FeCO3

43. Which of the following sublime(s)

1.ammonium nitrate 2.iron (III) sulphate

3.sulphur (VII) oxide 4.iodine

44. The property of sulphur which is used in the Frasch process is (are);

1.insolubility 2.low density

3.strong intermolecular forces 4.low melting point

45. Which of the following is reduced by hydrogen gas?

1.mercury (II) oxide 2.iron (III) oxide

3.lead (II) oxide 4.potasium oxide

In each of the questions 46-50, there is a statement on the left hand side (assertion) and
reason on the right hand side. Read the statements carefully then, choose the letter
which corresponds to the best alternative. The instructions are as in the table below;

A B C D

Assertion True True False True

Reason True, correct Not correct True False


explanation explanation
46. Water is a mixture of hydrogen and oxygen BECAUSE the two gases are given off
during

electrolysis

47. Ethanoic acid,CH3COOH is a tetra basic acid BECAUSE it contains 4 hydrogen

Atoms

48. Concentrated sulphuric acid is not used to BECAUSE the acid has low affinity for
water

dry ammonia

49. A candle burns with a yellow flame BECAUSE it contains hydrogen

50. Helium is filled in balloon in preference BECAUSE Helium is denser but in


combustible.

to hydrogen
Name.Centre/Index No/

Signature..

545/1
Chemistry
Paper 1
July/August, 2010
1 hours

Uganda Certificate of Education


RESOURCEFUL MOCK EXAMINATIONS
CHEMISTRY
PAPER 1

Instructions
This paper consists of 50 objective type questions
Answer the questions by writing the correct alternative in the box on the right hand side
of the paper
1. A dilute aqueous solution og potassium iodide was electrolysed. The product at
the cathode is likely to be
A. iodine B. potassium
C. oxygen D. hydrogen

2. In which one of the following pairs are both oxides acidic?


A) Na2SO4 , Li2O
B) MgO , Fe2O3
C) CaO, No2
D) SO2 , P2O5
3. Which one of the underlined substances in the given equations is acting as an
oxidizing agent?
A) CuO (s) + H2 (g) Cu (s) + H2O (l)
B) 2Al (s) + Fe2O3 (s) Al2O3 (s) + 2Fe (s)
C) Mg (s) + 2HCl (g) MgCl2 (s) + H2 (g)
2+ 2-
D) Ba (aq) + CO 3 (aq) Ba2+ . CO32-(s)
4. Water is said to be an oxide. Which one of the following reactions best demonstrates
that fact?
A) water with anhydrous copper (II) sulphate
B) steam with magnesium
C) pure, recently boiled water with an iron nail
D) water with calcium oxide
5. 240g of magnesium react completely with excess hydrochloric acid according to the
equation.
Mg (s) + 2HCl (aq) MgCl2 (aq) + H2 (g)
What is the maximum decrease in mass due to escape of hydrogen in this reaction?
(H=1, Mg=24)
A) 2.24g
B) 2.40g
C) 0.20g
D) 0.02g
6. The element most likely to remove oxygen from zinc oxide when a mixture of the
element is heated is;
A)iodine
B) magnesium
C) copper
D) iron
7. Hydrogen sulphide gas burns in oxygen according to the equation.
2H2S (s) + 3O2 (g) 2H2O (g) + 2SO2 (g)
The volume of hydrogen at room temperature and pressure, used up when 2.4litres of
hydrogen sulphide are completely burned is (1mol of gas occupies 24litres at room
temperature and pressure)
A) 7.2litres
B) 4.8litres
C) 3.6litres
D)2.4litres
8. Duralumine is an alloy of
A) copper and zinc
B) magnesium and aluminium
C) lead and aluminium
D) copper and lead
9. Which one of the following atoms contains the most neutrons
A) 137 N
B) 147 N
C) 136 C
D) 146 C
10. Carbon dioxide was bubbled in the test tubes containing the solutions shown. In
which of the test tubes is a white precipitate likely to form?

11. Concentrated nitric acid and concentrated sulphuric acid may be separated by their
action on
A) copper (II) oxide
B) litmus paper
C) iorn (II) sulpahte
D) sodium hydroxide solution
12. The element R reacts with the element T to form a compound R 2T3. The ion formed
by T is
A)T2+
B) T3+
C) T3-
D) T2-
13. When 2.02g of potassium nitrate was heated strongly 1.70g of a solid remained.
2KNO3 (s) 2KNO2 (s) + O2 (g)
What volume of oxygen measured at room temperature was evolved?
A) carbon dioxide
B) calcium oxide
C) steam
D) copper (II) oxide
14. Which one of the following substances does not react with red hot carbon?
A)carbon dioxide
B) calcium oxide
C) steam
D) copper (II) oxide
15. The yield of ammonia in the reaction;
N2 (g) + 3H2 (g) 2NH3 (g) + heat
A) raising the temperature
B) increasing the pressure
C) employing a suitable catalyst
D) adding an inert gas
16. Which one of the following is NOT a property of ethene
A) it turns potassium manganate (VII) colourless
B) it is an unsaturated hydrocarbon
C) it is a saturated hydrocarbon
D) it decolourises bromine water
17. Which one of the following combinations would produce oxygen at the fastest rate?
A) 100cm3 of 2mH2O2 heated at 30C
B) a mixture of 1.0g of MnO2 and 100cm3 of 2mH2O2 at room temperature
C) 100cm3 of 1MH2O2 heated at 30C
D) a mixture of 0.5g of MnO2 and 100cm3 of 2MH2O2 heated at 30C
18. Which one of the following is not an application of electrolysis?
A) synthesis of elements
B) purification of metals
C) anodizing aluminium
D) electroplating
19. The atomic number of magnesium is 12 and its relative atomic mass is 24. Which
one of the following represents the magnesium ion, Mg 2+
Protons Neutrons Electrons
A) 12 12 12
B) 12 10 12
C) 10 12 10
D) 12 12 10

20. When steam was passed over heated magnesium, the mass of the dry residue was
0.4g more
than that of the magnesium. How many moles of steam molecules were decomposed?
(H=1 O=16)
A) 0.40
B) 0.25
C) 0.025
D) 0.022
21. Most carbonates decompose readily on heating to form
A) an oxide and carbon dioxide
B) a metal and carbon dioxide
C) an oxide and oxygen
D) an oxide and carbon
22. A piece of metal foil was wrapped around an iron nail which was then put in a test
tube containing water. The foil that would best protect the iron nail form corrosion is;
A) lead
B) zinc
C) copper
D) tin
23. In which one of the following pairs would the substances be likely to react together if
their powders were mixed and heated?
A) carbon and lead (II) oxide
B) zinc and aluminium oxide
C) copper and lead (II) oxide
D) iron and magnesium oxide
24. Which one of the following is the best reason why zinc is able to displace copper
from solutions of copper salts?
A) zinc is more electronegative than copper
B) zinc loses electrons more easily than copper
C) zinc is a stronger oxidizing agent than copper
D) zinc has few electrons than copper
25. Methanol burns in oxygen according to the following equation
CH3OH + 3/2 O2 (g) CO2 (g) + 2H2O H=-730kJ
The heat given off when 6.4g of methanol burns in oxygen is
A) 32 X 730 kJ
6.4

B) 6.4 x 730 kJ
32

C) 6.4 X 730 kJ
32 16

D) 32 X 730 kJ
6.4 16
26. Which one of the following salts would give the highest volume of carbon dioxide
when reacted with

Name..Centre/Index No
Signature.

545/1

Chemistry

Paper 1

August 2011

1 hours

Uganda Certificate of Education

UNASE MOCK EXAMINATIONS

CHEMISTRY

PAPER 1

TIME: 1 hours

Instructions

This paper consists of 50 objective type questions

Answer the questions by writing the correct alternative in the box on the right
hand side of the paper.
1. Fused calcium chloride when exposed to air changes from solids to liquids.
This is because the salt is:

A. Deliquescent
A
B. hygroscopic

C efflorescent

D. hydrated
2. Which of the following compounds is decomposed by heat leaving a black
residue?

A. copper (II) oxide

B. lead (II) nitrate

C. potassium manganate (VII) A

D. zinc carbonate

3. The gas obtained when chlorine reacts with excess ammonia is

A. hydrogen

B. nitrogen
C
C. hydrogen chloride

D. nitrogen dioxide

4. A hydrogen ion contains

A. an electron

B. a neutron C

C. a proton

D. a neutron

5. Which one of the following oxides will react with magnesium most readily
when both are heated together?

A. zinc oxide B
B. copper (II) oxide
C. iron (II) oxide
D. lead (II) oxide
6. Sodium sulphite reacts with dilute hydrochloric acid according to the
equation
Na2SO3 (aq) + 2HCl (aq) 2NaCl (aq) + SO2 (g) + H2O (l)
The maximum volume in litres of the sulphur dioxide gas formed,
measured at room temperature when 6.3g of sodium sulphite reacts with
dilute hydrochloric acid is
A. 6.3 x 24 B. 126 x 24
126 6.3

C. 6.3 x 126 D. 6.3 X 24 A


24 2.52

7. Which one of the following statements best defines the term electrolysis?
A. conduction of electricity through a solution
B. decomposition of an electrolyte by electricity
C. ionization of an electrolyte B
D. migration of ions to the anode

8. The heat liberated when 1.0g of methanol CH3OH is burnt in air is


22.6kJ. The heat liberated when 1 mole of methanol is burnt in air is
(H=1, C=12, O=16)
A. 1x 22.6kJ
B. 61x 22.6kJ
C. 31 x 22.6kJ C
D. 32 x 22.6kJ

9. Which one of the following pairs of substances consists of isotopes?


A. diamond and graphite
B. ethane and propane
C. calcium hydrogen carbonate
A
D. calcium sulphate

10. Rain water dissolves limestone because of formation of


A. calcium carbonate
B. calcium hydroxide
C. calcium hydrogen carbonate
D. calcium sulphate

11.Which one of the following reacts with water with evolution of heat?
A. calcium oxide
B. zinc oxide
C. magnesium oxide
D. lead (II) oxide

12. 20cm3 of hydrochloric acid of unknown molarity reacted with 25cm3 of


0.05m sodium carbonate solution. The molarity of the acid is;
A. 2 x 25 x 0.05
20

B. 25 x 0.5
2 x 20

C. 20 x 2 x 0.05
25 x 2

D. 20 x 2
25 x 0.05
13. The nuclear composition of our atoms W, X, Y and Z are shown in the
table below

Atom W X Y Z
Number of protons + neutrons 12 23 14 24
Number of protons 6 12 8 12

Which of the atoms are isotopes?


A. W and X
B. W and Y
C. X and Y
D. X and Z

14. Which one of the following hydroxides is soluble in excess sodium


hydroxide solution?
A. copper (II) hydroxide
B. calcium hydroxide
C. lead (II) hydroxide
D. magnesium hydroxide

15. Steel is a form of iron which contains


A. sulphur
B. magnesium
C. carbon
D oxygen
16. The number of moles of hydrogen ions in 500cm 3 of 0.1M sulphuric acid
is
A. 0.25
B. 0.5
C. 0.1
D. 2.0

17. Compounds which have identical molecular formulae but different


structural formulae are called?
A. allotropes
B. isotopes
C. isomers
D. homologues

18. Which one of the following compounds does not produce an oxide when
heated moderately?
A. sodium hydroxide
B. copper (II) nitrate
C. calcium hydroxide
D. lead (II) nitrate
19. Which one of the following nitrates does not give off oxygen when
heated?
A. NaNO3
B. NH4NO3
C. Mg(NO3)2
D. Zn(NO3)2\

20. Which one of the following statements is correct about the electrolysis
of dilute sulphuric acid?
A. Hydrogen is liberated from the anode
B. acidity decreases at the cathode
C. sulphate ions are discharged ate the anode
D. the total amount of acid decreases

21. Zinc oxide is


A. an acidic oxide
B. a basic oxide
C. a neutral oxide
D. an amphoteric oxide
22. The boiling points of nitrogen, oxygen, hydrogen and helium are 77k,
90k, 50k and 3k respectively. Which one of the gases has got the
strongest inter molecular forces?
A. hydrogen
B. nitrogen
C. helium
D. oxygen

23. Hydrogen gas burns in chlorine in chlorine to form white misty fumes.
The misty fumes are;
A. smoke from the burning hydrogen
B. droplets of hydrochloric
C. chlorine water formed
D. unburnt hydrogen and chlorine

24. Which one of the following salts is best prepared by neutralization?


A. sodium sulphate
B. barium sulphate
C. calcium carbonate
D. lead (II) chloride

25. What mass of pure sodium hydroxide is required to make 5 litres of 0.9M
solution?
A. 0.9 x 40 x 5000
1000

B. 0.9 x 40 x 100
5000

C. 0.9 x 100 x 500


40

D. 40 x 100 x 5000
0.9

26. Elements P,Q,R and S have the following electronic configurations


P 2:4
Q 2:8:2
R 2:8
S 2:8:7
The pair of elements that will form ionic compound is
A. P and R
B. P and S
C. R and S
D. Q and S

27. Sulphur dioxide reacts with oxygen to give sulphur trioxide according to
the following equation.
2SO2(g) + O2(g) 2SO3(g) H =-98 kJ/ mol

Which of the following statements is true abouut the reaction?


A. increase the pressure increases the rate of forward reaction
B. increase inthe temperature decrease the rate of forward reaction.
C. decrease inthe temperature decrease the yeild of sulpur trioxide
D. decrease in pressure decrease the yeild of sulphur trioxide

28. A colourles solution ,Z, reacted with aqueous sodium hydroxide to give
a white precipitate soluble in excess alkali. When Z was treated with
aqueous potassium iodide, a yellow precipitate was observed.Z
contains
A. aluminium ions
B. lead ions
C. zinc ions
D. calcium ions

29. Which one of the following elements reacts with chlorine to form a
covalent compound?
A. calcium
B. hydrogen
C. copper
D. zinc

30. Which one of the following is the most ractive towards magnessium?
A. flourine
B. chlorine
C. bromine
D. iodine
31. 100cm3 of hydrochloric acid dissolves 3g of magnessium ribbon . The
molarity of the acid is
(Mg= 24)
A. 0.025
B. 0.25
C. 2.5
D. 25

32. A certain hydrocarbon has 82.2% by mass carbon.


Its molecular mass is 58. Its empirical formula is?
(C=12,H=1)
A. C2 H2
B. C2 H5
C. C2 H8
D. C4H10
33. Magnessium reacts with silver nitrate solution according to the following
equation.
Mg(s)+2AgNO3(aq) Mg(NO3)2(aq) + 2Ag (s)
The mass of silver deposited when 2.4g of magnessuim powder is
added to silver nitrate solution is ?
A. 2.16g
B. 1.08g
C. 21.6g
D. 10.8g
(Mg=24,Ag=108)

34. Alumimium and sulpur are in group 3 and 6 of the periodic table
respectively. The formula of alumium sulphide is
A. AlS
B. AlS3
C. Al2S3
D. Al3S2
35. In the petroleum industry the hydrocarbons of higher molecular mass
are converted to hydrocarbons of lower molecular mass by
A. distillation
B. cracking
C. evaporation
D. cooling

36. Which of the following sets of elements are in the same group of the
periodic table? (atomic numbers; ( C=6, O=8, S=16,Al=13)
A. oxygen and carbon
B. oxygen and sulphur
C. oxygen and aluminium
D. carbon and sulphur

37. Which of the follwing cations NOT form a carbonate when reacted with
aqueous sodium carbonate.
A. Al3+(aq)
B. Fe2+ (aq)
C. Ca2+ (aq)
D. Mg2+(aq)

38. When sodium hydroxide was added to an aqueous solution of salt X, a


green gelatinous precipitate was obtained. On standing, the green
precipitate turned reddish-brown. What ion did X contain?
A. copper (II) hydroxide
B. aluminium ion
C. iron (II) ions
D. iron (II) ion

39. Which of the following halogens has the highest oxidizing power?
A. iodine
B. fluorine
C. chlorine
D. bromine

40. Which of the following reagents would produce a visible reaction when
added to aqueous sodium chloride?
A. aqueous barium chloride
B. concentrated hydrochloric acid
C. aqueous silver nitrate
D. dilute sulphuric acid

In each of the following questions 41 to 45, one or more answers given


may be correct. Read each question carefully and then choose.
(A) If 1, 2, 3 only are correct
(B) If 1, 3 only are correct
(C) If 2, 4 only are correct
(D) If only 4 is correct
INSTRUCTIONS SUMMARIZED

A B C D
1, 2, 3 1, 3 2,4 4
Only correct Only correct Only correct Only correct

41. Which of the following solutions contains the same mass of sodium
hydroxide?
1. 200cm3 of 2M sodium hydroxide
2. 400cm3 of 1M sodium hydroxide
3. 100cm3 of 4M sodium hydroxide
4. 400cm3 of 2M sodium hydroxide

42. Chlorine, bromine and iodine belong to the halogen family.


1. they are all gases at room temperature
2. they all form ions with a single negative charge
3. they are all very soluble in water C
4. they all react vigorously with potassium

43. Ethene
1. burns with a smoky flame
2. is an alkene
3. is a hydrocarbon
A
4. is an alkane

44. Which of the following will conduct electricity?


1. solid copper (II) sulphate
2. copper
3. ethanol C
4. zinc chloride solution

45. When magnesium looses two electrons, the particle formed


1. is an inert gas
2. is an allotrope of magnesium
3. carries a double negative charge D
4. carries a double positive charge

Each of the questions 46 to 50 consist of an assertion (statement) on


the left hand side and a reason on the right hand side.
Select
(A) If both assertion and reason are true statements and the reason
is a correct explanation of the assertion
(B) If both assertion and reason are true statements but the reason is
not a correc6 explanation of the assertion.
(C) If the assertion is true but the reason is an incorrect statement
(D) If the assertion is incorrect but the reason is a true statement

INSTRUCTIONS SUMMARIZED

A B C D
Assertio True True True False
n
Reason True correct True not correct false
explanation explanation

46. The charge on an ion of lead is because the formula of


positive lead (II) sulphate is Pb(SO4)2

47. When a fixed volume of because manganese (IV) oxide B


2M hydrogen peroxide is catalyses the reaction
heated with some
manganese (IV) oxide, the total
volume of oxygen produced is A
greater than when no
manganese (iv) is used

48. Sodium chloride does not because the ions in the solid are not
A
conduct electricity in the solid free to move.
state
because nitrogen has a lower
49. During the industrial molecular mass than oxygen.
preparation of nitrogen and
oxygen from liquid air, nitrogen A
comes off first
50 Magnesium can displace because the ions of both magnesium
copper from aqueous and copper carry a positive
copper (II) sulphate charge of two
B

Name..Centre/Index No
Signature.

545/1 Chemistry
Paper 1
April 2014
1 hours

Instructions
This paper consists of 50 objective type questions
Answer the questions by writing the correct alternative in the answer sheet
provide below.

1 11 21 31 41
2 12 22 32 42
3 13 23 33 43
4 14 224 34 44
5 15 25 35 45
6 16 26 36 46
7 17 27 37 47
8 18 28 38 48
9 19 29 39 49
10 20 30 40 50
1. An alloy of solder consists of
A. Zinc and Lead
B. Copper and Lead
C. Copper and Tin
D. Tin and Lead

2. Evaporation is a process which occurs


A. through out the liquid at a fixed temperature
B. through out the liquid at any temperature
C. at the surface of the liquid at a fixed temperature
D. at the surface of the liquid at any given temperature

3. Which of the following method removes only temporary hard water?


A. addition of sodium carbonate
B. addition of ammonia solution
C. distillation
D. ionic exchange

4. A pure sample of zinc carbonate is prepared in the laboratory by reacting


an aqueous solution of zinc sulphate with an aqueous solution of
A. sodium carbonate
B. potassium hydrogen carbonate
C. carbonic acid
D. calcium carbonate

5. An ion is best defined as


A. a charged particle
B. a charged atom or a charged group of atoms
C. an atom or group of atoms that has lost or gained electron (s)
D. a cation or anion

6. In the Daniell chemical cell the reaction at the positive terminal (electrode)
is
A. Cu2+(aq)+ 2e Cu (s)
B. Cu (s) Cu2+(aq)+ 2e
C. Zn2+(aq)+ 2e Zn (s)
D. Zn (s) Zn2+(aq)+ 2e

7. Which of the following is a natural polymer?


A. rubber B. rayon
C. nylon D. polyethene.
8. Sodium hydrogen carbonate dissolves in water as shown in the equation
2NaHCO3(s) + H2O(l) 2NaOH (aq) + H2CO3(aq)
The pH of the solution
A. 1 B. 6 C. 7 D. 9

9. Which of the following can be used to increase the rate at which


fermentation occurs?
A. roasted millet
B. germinated millet
C finely ground millet
D. paste of millet

10. To an aqueous solution of X was added an aqueous solution of barium


nitrate followed by nitric acid; a white precipitate that dissolved in dilute
acid was formed. The anion in X was
A. sulphate B. chloride
C. carbonate D. hydrogen sulphate

11. Astatine belongs to group VII of the Periodic Table. Which of the following
is true about astatine? It
A. forms an ion with a positive.
B. forms a diatomic molecule.
C. is a bleaching agent.
D. is used in the treatment of water.

12. Calculate the molar heat of neutralization given off; 50cm3 of 1M sulphuric
acid was mixed with 50cm3 of 2M sodium hydroxide solution and the
temperature of the solution changed by 130C.
(Specific heat capacity of solution 4.2 Jg-1/0C, density of solution is 1gcm-1)

A. 100 x 4.2 x 13 B. 100 x 4.2 x 13


1.5 0.1

C. 100 x 4.2 x 13 D. 100 x 4.2 x 13


0.15 0.2
13. Which of the following compounds contains the least percentage of
sulphur?
[H =1, S =32, O = 16]
A. H2S2O7 B.
H2SO4
C. SO2 D.
H2 S
14. Which of the following is not a redox reaction?
A. ZnO(s) + C(s) Zn(s) + CO (g)
B. ZnO(s) + H2SO4(aq) ZnSO4(aq) + H2O (l)
C. Zn (s) + 2H2SO4(aq) ZnSO4(aq) + 2H2O (l) + SO2(g)
D. Zn (s) + H2SO4(aq) ZnSO4(aq) + H2 (g)

15. Carbon monoxide was passed over 2.32g of an oxide of iron the residue
remaining weighed 1.68g. The formula of the oxide is
[Fe =56, O = 16]
A. FeO B. Fe2O3
C. Fe3O4 D. Fe4O3

16. X, Y and Z are elements in the same short period of the periodic Table.
The oxide of X is amphoteric, the oxide of Y is basic, and the oxide of Z is
acidic. Arranged in order of increasing atomic number they are:
A. YXZ B. XYZ C. ZYX D. YZX

17. Catalytic combustion of ammonia in air produces


A. nitrogen and water
B. Nitrogen monoxide and water
C. Nitrogen (IV) oxide and water
D. Dinitrogen oxide and water

18. Which of the following compounds undergo polymerization reaction?


A. CH4 B. C3 H6
C. C3 H8 D. C4H10

19. Sodium hydrogen carbonate reacts with sulphuric acid according to the
equation
2NaHCO3(s) + H2SO4 (aq) Na2SO4 (aq) + CO2 (g) +2H2O (l)
The volume of 0.4M sulphuric acid required to dissolve 4.2g of the sodium
hydrogen carbonate is
[Na=23,H=1,C=12,S=32,O=16]
A. 84 x 2 x 4.2 cm3 B. 1000 x4.2 x 0.4 cm3
1000 x 0.4 84 x 2

C. 84 x 2 x0.4 cm3 D. 1000 x4.2 cm3


1000 x 4.2 84 x 2 x 0.4

The table below shows the number of electrons, neutrons and protons in
particles P, Q, R, S, T and U.
Study the table below and answer questions 20 22
Particle Electron neutron protons
s s
P 19 20 19
Q 18 22 18
R 19 22 19
S 10 8 8
T 18 14 20
U 2 2 2

20. Which atoms of elements belong to the same group of the Periodic Table?
A. P and R B. Q and T
C. Q and U D. Q and R

21. Identify the letters that represent a pair of isotopes


A P and R B. Q and T
C. Q and U D. Q and R

22 Identify the letter that represents a anion


A. R B. S C. T D U

23 Graphite differs from diamond in that


A they have different densities
B. they have different arrangement of atoms
C. graphite has delocalized electrons whereas diamond does not.
D. graphite is soft and is used as a lubricant whereas diamond is not.
24. When concentrated sulphuric acid is heated with ethanol ethene is formed
because sulphuric acid is a strong
A. dibasic acid B. oxidizing agent
C. drying agent D. dehydrating agent

25. Gas X was passed over heated copper (II) oxide. A brown substance and
water vapour were formed. Gas X could be
A. carbon monoxide
B. ammonia
C. sulphur (IV) oxide
D. hydrogen chloride
26. Sodium thiosulphate reacts with dilute hydrochloric acid according to the
equation
S2O32-(aq) + 2H+(aq) S(s)+ SO2(aq) + H2O(l)
A graph of the reciprocal of time (1/t) was plotted against volume of
thiosulphate that was used.

The straight line graph shows that


A. rate is inversely proportion to volume of thiosulphate
B rate is directly proportion to volume of thiosulphate
C the rate increases with increase in temperature
D. the rate increases in presence of a catalyst.

27. Magnesium was burnt in air. When was water added to the cold products,
a gas was given off. The gas was
A. hydrogen B. nitrogen
C. oxygen D. ammonia
28. A hydrocarbon Z when completely burnt in excess oxygen produced 220g
of carbon dioxide and 45 g of water. The empirical formula of Z is
A. CH B. C2 H
C. C5H2.5 D. C10H5

29. A beaker containing lime water is left exposed to the atmosphere for some
time. The white solid that formed at the bottom of the beaker is
A. calcium oxide B. calcium hydroxide
C. calcium carbonate D. calcium hydrogen carbonate.

30. The gas collected when chlorine water is exposed to sun light is
A. chlorine B. oxygen
C. hydrogen chloride D. hydrogen
31. A filter paper was soaked in a saturated solution of sodium sulphate and
placed on a slide as shown below. A crystal of copper (II) chromate was
placed onto the filter paper and a current was allowed to flow for some
time. Which of the following was observed?

A. The blue color moved towards Y


B. The yellow color moved towards Y
C. Bubbles of a colorless gas at Y
D. Brown deposit at Y

32. 6.44g of hydrated salt X was strongly heat and weighed to constant mass
2.84g of the anhydrous salt remained. The number of molecules of water
of crystallization is
[X = 142, H2O = 18]
A. 3.6 B 0.2
C. 10.0 D. 6.0
33. 50 cm3 of a mixture of ammonia and carbon dioxide was passed over
heated copper (II) Oxide, and nitrogen formed occupied 20 cm3 under the
same condition as a mixture. The percentage of ammonia in the mixture
was
A. 20 B. 80
C. 40 D. 60

34 Which of the following will give off hydrogen acid fastest?


A. 100cm3 0.1M ethanoic acid and zinc powder
B. 100cm3 0.1M sulphuric acid and zinc powder
C. 100cm3 0.1M hydrochloric acid and zinc powder.
D. 100cm3 0.1M hydrochloric acid and zinc granules

35. Soap is by the following reaction


A. sodium hydroxide and CH3COOH
B. sodium hydroxide and CH3 (CH2)8 COOH
C. calcium hydroxide and CH3COOH
D. calcium hydroxide and CH3 (CH2)8 COOH
36. Which of the following salts dissolves in water to make a solution of pH
less
than 7 is
A. Ammonia chloride
B. Calcium nitrate
C. Sodium sulphide
D. ammonium ethanoate

37. Which of the following acids renders iron passive? Concentrated


A. nitric acid
B. sulphuric acid
C. hydrochloric acid
D. phosphoric acid

38. The bleaching action of chlorine is different from sulphur (IV) oxide in that
chlorine (forms)
A. choric (I) acid donates oxygen to the colored substance
B. chloric (I)acid removes oxygen from the colored substance
C. substances bleached turn brown on exposure.
D. hydrogen chloride adds hydrogen to the dye.

39 Which of the following is NOT true about metallic bonding?


A. There are many positive ions in the structure.
B. The delocalized electrons experience no attraction from the atoms.
C. In the structure the atoms are closely packed.
D. The delocalized electrons do not belong to any particular atom.

40. Compounds which have identical molecular formulae but different


structural
formulae are called
A. allotropes
B. isotopes
C. isomers
D. homologues

In each of the following questions 41 to 45, one or more answers given may
be31 correct. Read each question carefully and then choose.
A. If 1, 2, 3 only are correct
B. If 1, 3 only are correct
C. If 2, 4 only are correct
D. If only 4 is correct
INSTRUCTIONS SUMMARIZED
A B C D
1, 2, 3 1, 3 2,4 4
Only correct Only correct Only correct Only correct

41. Which of the following decompose(s) on strong heating to form an oxide?


1. sodium hydrogen carbonate
2. calcium hydrogen carbonate
3. ammonium hydrogen carbonate
4. copper (II) carbonate

42. Which of the following is/are property (ies) of concentrated sulphuric acid?
It
1. is reduced to sulphur (IV) oxide
2. gives off hydrogen when reacted with magnesium
3. is hygroscopic
4. is deliquescent
43. The gas given off when bananas ripen decolorizes acidified potassium
manganate (VII). The gas is probably
1. hydrogen sulphide
2. ethane
3. sulphur (IV) oxide
4. ethene

44. Which of the following usually cause(s) water pollution?


1. sodium carbonate
2. sodium phosphate
3. calcium hydrogen carbonate
4. sewage

45. In which of the following equations is sulphur (IV) oxide behaving as a


reducing agent?
1. Cl2(g) + 2H2O(l)+ SO2(g) H2SO4(aq)+ 2 HCl(g)
2. 2Mg(s) + SO2(g) 2MgO(s)+ S(s)
3. 2HNO3(l) + SO2(g) H2SO4(aq)+ 2 NO2l(g)
4. H2S(gl )+ SO2(g) S (s)+ H2O(l)

Each of the questions 46 to 50 consist of an assertion (statement) on the


left hand side and a reason on the right hand side.
Select
A. If both assertion and reason are true statements and the reason
is a correct explanation of the assertion
B. If both assertion and reason are true statements but the reason is
not a correct explanation of the assertion.
C. If the assertion is true but the reason is an incorrect statement
D. If the assertion is incorrect but the reason is a true statement

INSTRUCTIONS SUMMARIZED
Assertion Reason
A. True True and correct explanation
B. True True not correct explanation
C. True False
D. False True

46. Ethane decolorizes acidified because It is a saturated


potassium manganate (VII). hydrocarbon.

47. Duralumin is used in because It is lighter and resistant to


construction of air crafts. corrosion

48. Oxygen is liberated at the anode because The hydroxide ions are
when brine is electrolyzed with lower in the electrochemical
carbon electrode series than chloride ions

49. A piece of magnesium continues because magnesium is higher in the


to burn in carbon dioxide reactivity series than carbon

50 When a piece of phosphorous because Hydrogen chloride is formed


is lowered in a jar of chlorine during the reaction
white fumes are observed

END.

Marking Guide 2012


1. D 11. B 21A 31. A 41.C
2. D 12. B 22.B 32.C 42. B
3. B 13.B 23. B 33.B 43. D
4. B 14.B 24. D 34.B 44. C
5. C 15. D 25.B 35.B 45. B
6. D 16. A 26.B 36.A 46.D
7. A 17. B 27. D 37. A 47. A
8. C 18. B 28. A 38. A 48. D
9. B 19. D 29. C 39.B 49.B
10.C 20. C 30.B 40.C 50.C

Marking Guide 2012


1. D 11. B 21A 31. A 41.C
2. D 12. B 22.B 32.C 42. B
3B 13.B 23. B 33.B 43. D
4B 14.B 24. D 34.B 44. C
5C 15. D 25.B 35.B 45. B
6D 16. A 26.B 36.A 46.D
7A 17. B 27. D 37. A 47. A
8C 18. B 28. A 38. A 48. D
9B 19. D 29. C 39.B 49.B
10.C 20. C 30.B 40.C 50.C

Name..Centre/Index No/..

Signature.

545/1 Chemistry
Paper 1
August 2013
1 hours

Uganda Certificate of Education


MOCK EXAMINATIONS
CHEMISTRY
PAPER 1
TIME: 1 hours

Instructions
This paper consists of 50 objective type questions
Answer the questions by writing the correct alternative in the box on the right hand side
of the paper.
FOR EXAMINERS USE ONLY
1. Which of the following pairs of compounds is used to prepare lead (II) carbonate?
A. Lead (II) nitrate and sodium carbonate
B. Lead (II) nitrate and sodium hydrogen carbonate
C. Lead and carbonic acid
D. Lead (II) hydroxide and carbonic acid.

2. Which of the following solution are acidic? A mixture of a solution of


A. 25cm3 of 2M sodium hydroxide and 25 cm3 of 2M hydrochloric acid.
B. Excess carbon dioxide bubbled in sodium hydroxide solution
C. 25cm3 of 2M ammonia solution and 25 cm3 of 2M ethanoic acid.
D. 25cm3 of 2M ammonia solution and 25 cm3 of 2M hydrochloric acid.

3. Which of the following form a precipitate when mixed with aqueous sodium
hydroxide?
A. NO2 B. CO2 C. NO2 D. Cl2

4. Which oxide is not stable when heated in air?


A. silicon (IV) oxide. B. zinc oxide
C. mercury (II)oxide D. copper(II) oxide

5. For which reason is water considered a compound? It


A. contains hydrogen and oxygen
B. has weight
C. is a liquid at room temperature
D. is a different from hydrogen and oxygen.

6. Which of the following forms a colorless solution? A combination of


A. aqueous ammonia and aqueous zinc nitrate.
B. aqueous ammonia and aqueous lead (II) nitrate.
C. aqueous sodium hydroxide and aqueous calcium nitrate .
D. aqueous sodium hydroxide and aqueous magnesium nitrate.

7. Which of the following pairs of compounds can be used to prepare nitric acid?
A. calcium nitrate and concentrated sulphuric acid
B. sodium nitrate and concentrated sulphuric acid
C. lead (II) nitrate hydroxide and concentrated sulphuric acid
D. barium nitrate and concentrated sulphuric acid.

8. Which gas decolorizes acidified potassium manganate (VII)?


A. ethane. B. hydrogen sulphide
C. sulphur (VI) oxide D. sulphur (IV) oxide

9. What is the purpose of using a thermometer in fractional distillation? It


determines
A. the temperature of the boiling point of the substance..
B. the time to stop collecting a particular fraction of the distillate.
C. the temperature to collecting a particular fraction of the distillate
D. the temperature of decomposition of a particular fraction of the distillate

10. Ammonia is formed according to the equation below;


N2(g)+ 3H2(g) 2NH3(g) H= -290kJmol-1

Which of the following conditions would favour maximum yield of ammonia?


A. low temperature and low pressure
B. low temperature and high pressure
C. high temperature and low pressure
D. high temperature and high pressure

11 when

In each of the following questions 41 to 45, one or more answers given may
be31 correct. Read each question carefully and then choose.
A. If 1, 2, 3 only are correct
B. If 1, 3 only are correct
C. If 2, 4 only are correct
D. If only 4 is correct
INSTRUCTIONS SUMMARIZED
A B C D
1, 2, 3 1, 3 2,4 4
Only correct Only correct Only correct Only correct

41. Which of the following decompose(s) on strong heating to form an oxide?


1. sodium hydrogen carbonate
2. calcium hydrogen carbonate
3. ammonium hydrogen carbonate
4. copper (II) carbonate

42. Which of the following is/are property (ies) of concentrated sulphuric acid?
It
1. is reduced to sulphur (IV) oxide
2. gives off hydrogen when reacted with magnesium
3. is hygroscopic
4. is deliquescent

43. The gas given off when bananas ripen decolorizes acidified potassium
manganate (VII). The gas is probably
1. hydrogen sulphide
2. ethane
3. sulphur (IV) oxide
4. ethene

44. Which of the following usually cause(s) water pollution?


1. sodium carbonate
2. sodium phosphate
3. calcium hydrogen carbonate
4. sewage

45. In which of the following equations is sulphur (IV) oxide behaving as a


reducing agent?
1. Cl2(g) + 2H2O(l)+ SO2(g) H2SO4(aq)+ 2 HCl(g)
2. 2Mg(s) + SO2(g) 2MgO(s)+ S(s)
3. 2HNO3(l) + SO2(g) H2SO4(aq)+ 2 NO2l(g)
4. H2S(gl )+ SO2(g) S (s)+ H2O(l)
Each of the questions 46 to 50 consist of an assertion (statement) on the
left hand side and a reason on the right hand side.
Select
A. If both assertion and reason are true statements and the reason
is a correct explanation of the assertion
B. If both assertion and reason are true statements but the reason is
not a correct explanation of the assertion.
C. If the assertion is true but the reason is an incorrect statement
D. If the assertion is incorrect but the reason is a true statement

INSTRUCTIONS SUMMARIZED
Assertion Reason
A. True True and correct explanation
B. True True not correct explanation
C. True False
D. False True

46. Graphite is a good conductor of because it is soft.


electricity

47. The molar heats of because they form neutral solutions


neutralization of strong acids
with strong bases are
approximately all equal

48. Nitrogen reacts with burning because nitrogen is highly reactive.


Magnesium to form a nitride.

49. Chlorine water bleaches dyes because the solution contains


hypochlorous acid.

50 Sodium metal is store under because it reacts with water to form


water liberating hydrogen gas.
Name..Centre/Index No

Signature.

545/1
Chemistry
Paper 1
August 2013
1 hours

WAKISSHA JOINT MOCK EXAMINATIONS

Uganda Certificate of Education


CHEMISTRY

PAPER 1

1hour 30 minutes

Instructions

This paper consists of 50 objective type questions

Answer all questions

Write the correct alternative in the box on the right hand side of the paper.

Use pen and write the letters clearly

For Examiners use only

1. The ability of liquids and gas to flow depends on


A. high intermolecular forces
B. low intermolecular forces
C high kinetic energy
D. low kinetic energy

2. Which of the following is a neutral oxide?


A. Fe3O4 B. Fe2O3
C. NO D. N2O4
3. The element most likely to remove oxygen from zinc oxide when a mixture
of the oxide and the element is heated is
A. lead B. magnesium
C. copper D. iron

4. Which of the following anions forms a white precipitate with barium nitrate
solution followed by dilute nitric acid?
A. SO42- B. CO32-
C. Cl- D. NO3-

5. Atomic number of an element R is 13, the electronic configuration of the


ion of R is
A. 2:8:8 B. 2:8:5
C. 2:8:3 D. 2:8

6. The reaction between two substances is exothermic. Which of the


following is most likely to slow down the rate of reaction?
A. increasing the temperature of the surroundings
B. placing the reagents in an ice bath.
C. using an excess of one of the reagents
D. removing the products as fast as they are formed.

7. Carbon dioxide shows the following properties. It is


(i) Denser than air
(ii) A sublimate
(iii) Non-combustible
(iv) Does not support combustion
The use of carbon dioxide as a coolant depends on

A. (i) and (ii) B. (ii) and (iii)


C. (iii) and (iv) D. (ii) only
8. 75g of a saturated solution contained 30g of a salt. What is the solubilty of
the salt?
A. 40g/100g of water
B. 66.7g/100g of water
C 6.67g/100g of water
D. 250g/100g of water

9. In which of the following reactions is chlorine acting as an acidic gas?


A. 3Cl2(g) + 2Fe(s) 2FeCl3(s)
B. Cl2(g) + 2Na(s) 2NaCl(s)
C. 3Cl2(g) + NaOH(aq) NaOCl(aq)+NaCl(aq)+H2O(l)
D. 3Cl2(g) + H2(g) 2HCl(g)

10. Which of the following substances sublime when heated?


A. ammonium chloride B. hydrated sodium carbonate
C. copper (II) oxide D. ammonium sulphate

11. The atomic numbers of the elements X, Y, W and Z are 9, 11, 16 and12
respectively. Which one of the following pairs of the elements will form a
covalent compound?
C. X and W B. X and Y
C. Z and W. D. Y and W

12. The elements that can be extracted from their oxides by chemical
reduction using carbon (coke) are.
A. Al and Zn B. Zn and Fe
C. Mg and Cu. D. Ca and Cu

13. Which of the following hydroxides is soluble in excess aqueous ammonia


solution but insoluble in excess sodium hydroxide?
A. Fe(OH)2 B. Pb(OH)2
C. Cu(OH)2 D. Ca(OH)2

14. Which of the following processes is not used to remove permanent


hardness in water?
A. treatment with sodium carbonate
B. distillation
C ion exchange
D. addition of calcium hydroxide

15. An element A forms a covalent compound with oxygen of formula


A2O3.Given that A belongs to Period 3 of the Periodic Table, the possible
electronic configuration of the atom A is
A. 2:8:5 B. 2:8:7
C. 2:8:3 D. 2:8:6
16. Graphite is used as an electrode in electrolysis because it
A. is soft and slippery
B. has hexagonal layers
C has mobile electrons
D. is black and translucent

17. Which of the following is a weak acid?


A. citric acid B. phosphoric acid
C. carbonic acid D. hydrochloric acid

18. 20cm3 of 0.1M sodium carbonate solution react with 10cm3 of dilute
hydrochloric acid according to the equation below
Na2CO3(aq) + 2HCl (aq) 2 NaCl (aq) + CO2 (g) +H2O (l)
The molarity of the acid is
A. 0.1M B. 0.4M
C. 0.8M D. 0.2M

19. Which of the following catalysts is used in the manufacture of sulphuric


acid by the Contact process
A. manganese (IV) oxide
B. finely divided iron
C. vanadium (V) oxide
D. platinized asbestos

20. Which of the following would dissolve in dilute hydrochloric acid?


A. lead (II) chloride B. lead (II0 sulphate
C. barium sulphite. D. barium sulphate

21. Nitrogen reacts with hydrogen according to the equation


N2(g)+3H2(g) 2NH3(g)
The total volume of gaseous products formed at the end of the reaction
when 120cm3 of hydrogen were mixed with 50cm3 of nitrogen is
A. 170cm3 B. 90cm3
C. 80cm3 D. 70cm3

22. Which of the following reagents can be used to oxidize iron (II) ions?
A. sulphur dioxide B. hydrogen sulphide
C. chlorine. D. hydrogen chloride

23 the number of protons, neutrons and electrons in an atom Z represented

by protons neutrons electrons


A 129 213 84
B. 84 129 213
C. 84 129 129
D. 84 129 84

24. When 4 g of an oxide of the element X were reduced.3.2g of Z were


obtained. The simplest formula of the oxide of X
(X= 64, O = 16)
A. XO B. X 2O
C. XO2 D. X2O3

25. A solid N dissolves in water to form a colourless gas that fumes with
hydrogen chloride gas. The solid M is likely to be
A. Na2O2 B. Mg3N2
C. Na2NO3 D. Mg(NO3) 2

26. Vulcanization of rubber is aimed at increasing its


B. strength and reactivity
B stability and viscosity
C elasticity and reactivity
D. strength and elasticity

27. Which of the following conduct electricity in electrolytes?


A. neutrons B. electrons
C. protons D. ions

28. Which of the following pairs of salts can be separated by filtration?


E. Sodium carbonate and ammonium carbonate
F. Ammonium sulphate and magnesium sulphate
G. Barium chloride and barium nitrate
H. zinc carbonate and zinc sulphate
29. Glucose burns in oxygen according to the equation below.
C6H12O6(s) + 6O2(g) 6CO2 (g) +6H2O (l) H= -28022KJmol-1
The amount of heat produced when 18.0 g of glucose is completely burnt
in oxygen at the same temperature is
[H=1, C=12, O=16]
A. 2802 B. 180
18.0 x180 2802 x18.0

C. 180 x18.0 D. 2802 x180


2802 18.0

30. The melting and boiling points of substances W, X, Y and Z are


summarized below.
Substance Melting point 0C boiling point 0C
W -135 -0.5
X -38.9 356.6
Y -145 -10
Z -112 75.5
Which of the following pairs of substances are liquids at room
temperature?
A. W and X B X and Y
C. X and Z D. Y and Z

31. Which of the following diffuses fastest under similar conditions of


temperature and pressure?
[C=12, O=16, Cl=35.5, N=14, H=1]
A. NH3 B CO
C. SO2 D. Cl2

32. -------------------------is a process by which starch is formed from glucose


molecules
A. neutralization B polymerization
C. hydrolysis D. fermentation
33. which of the following hydrocarbons decolourises bromine water when
bubbles through it?
A. ethene B. ethane
C. butane D. methane

34 Sodium nitrate decomposes according to the following equation.


2NaNO3(s) 2NaNO2(s)) +O2 (g)
The volume of oxygen at stp produced when 10.0 g of sodium nitrate is
decomposed is. (1mole of a gas occupies 22.4dm3 at stp., Na=23, N=14,
O=16)
A. 10x22.4 B. 10 x170
170 22.4

C. 22.4x10 D. 10x85
85 22.4

35. 0.98g of gas M at stp occupies 8.4 dm3. The relative molecular mass of gas
M is (1mole of a gas occupies 22.4dm3 at stp)
A. 0.98 x 22.4 B. 0.98 x 8.4
8.4 22.4

C. 22.4 x 8.4 D. 0.98


0.98 22.4 x 8.4

36. Which of the following anions when in solution will react with lead (II)
nitrate to form a white precipitate which dissolves on heating and reappears
on cooling?
A. iodide ions B. chloride ions
C. sulphate ions D. carbonate ions

37. Which of the following salt does NOT produce ammonia on heating?
A. ammonium chloride B. ammonium carbonate

C. ammonium nitrate D. ammonium sulphate


38. Which of the following is observed when solid lead (II) nitrate is strongly
heated?
A. a white residue
B. a residue which is reddish brown when hot and yellow on cooling
C. a residue which is yellow when hot and white on cooling.
D. a grey residue

39 What mass of sodium hydroxide would be needed to neutralize exactly


200cm3 of solution containing 49g of sulphuric acid?
NaOH(aq+ H2SO4(aq) Na2SO4(aq) (aq) 2H2O (l)
A. 4g B. 8g C. 16g D. 32g

40. The structure below is of a polymer

The name of the monomer is


A. ethene B. ethane
C. butane D. butene

In each of the following questions 41 to 45, one or more answers given may
be correct. Read each question carefully and then choose.
A. If 1, 2, 3 only are correct
B. If 1, 3 only are correct
C. If 2, 4 only are correct
D. If only 4 is correct

INSTRUCTIONS SUMMARIZED

A B C D
1, 2, 3 1, 3 2,4 4
Only correct Only correct Only correct Only correct

41. Which of the following element(s) react directly with oxygen to form an
acidic oxide?
1. magnesium 2. silver
3. copper 4. gold

42. When dry hydrogen gas is passed over heated copper (II) oxide
1. copper (II) oxide is reduced. 2. Hydrogen is reduced.
3. hydrogen is oxidised 4. Copper is oxidised

43. The salt(s) that can be prepared by precipitation is/are


1. lead (II) nitrate 2. Lead (II) sulphate

3. barium nitrate 4. Barium sulphate

44. Which of the following contains the same number of particles as 2.4 dm3 of
argon at room temperature?
[Mg = 24, Ca= 40, C= 12, O = 16, Al = 27, molar volume =24dm3 at room
temperature]
1. 2.4g of magnesium 2. 4.0g of calcium
3. 4.4g of carbon dioxide 4. 3.4g of aluminium

45. Carbon and phosphorus are similar in that both


1. are non metallic elements
2. exist in allotropic forms
3. form covalent oxides
4. form neutral oxides

Each of the questions 46 to 50 consist of an assertion (statement) on the


left hand side and a reason on the right hand side.
Select
A. If both assertion and reason are true statements and the reason
is a correct explanation of the assertion
B. If both assertion and reason are true statements but the reason is
not a correct explanation of the assertion.
C. If the assertion is true but the reason is an incorrect statement
D. If the assertion is incorrect but the reason is a true statement

INSTRUCTIONS SUMMARIZED

Assertion Reason
A. True True and correct explanation
B. True True not correct explanation
C. True False
D. False True
46. Hydrogen gas is collected by because it can explode when mixed
upward delivery. with air.

47. the reactivity of alkali metals because The atomic radius of alkali
decreases down the group metals increases down the
group.

48. Hydrogen chloride gas can be because hydrogen chloride gas and
used instead of ammonia gas ammonia gas have the
in the fountain experiment. same solubility in water.

49. Concentrated sulphuric acid is because it ionizes completely to


strong oxidizing agent. produce H+ ions in solution.

50 In the electrolysis of dilute because sulphate ions carry more


sulphuric acid. Hydroxyl ions charge than hydroxyl ions.
are discharged in preference to
sulphate ions.
TRIAL QUESTIONS FOR CHEMISTRY PAPER 2.

SENIOR FOUR

FIRST TERM EXAMS 2007

CHEMISTRY PAPER II

Time: 2 hours.

Instructions:

Section A consists of 10 structured questions. Attempt all questions in


Section A.

Section B consists of 4 semi-structured questions. Attempt any two in this


section.

1 2 3 4 5 6 7 8 9 10 11 12 13 14 Total
SECTION A (Answer all questions in this section)

1. 12.5 g of zinc carbonate was strongly heated until there was no


further change. (Zn = 65, C = 12, O = 16)

(a)(i) State what was observed. (1 mks)

__________________________________________________________________

__

__________________________________________________________________

__

(ii) Write equation for the reaction in (a)(i) (1 mks)

__________________________________________________________________

__

__________________________________________________________________

__
(b) Determine the loss in mass (2 mks)

__________________________________________________________________

__

__________________________________________________________________

__

__________________________________________________________________

__

__________________________________________________________________

__

__________________________________________________________________

__

__________________________________________________________________

__

2. (a) Write an equation for the reaction by which ammonia is


prepared in laboratory. (1 mks)
__________________________________________________________________

__

__________________________________________________________________

__

(b)(i) State what is observed when dry ammonia is passed over


heated lead (II) oxide. (1 mks)

__________________________________________________________________

__

__________________________________________________________________

__

(b)(ii) Write equation for the reaction in b(i) (1 mks)

__________________________________________________________________

__

__________________________________________________________________

__

(c) Give one use of ammonia ( mk )


__________________________________________________________________

__

__________________________________________________________________

__

3. X cm3 of 0.1 M hydrochloric acid neutralizes 15 cm3 of 0.2 M


sodium carbonate solution.

(a) Write equation for the reaction. (1 mks)

__________________________________________________________________

__

__________________________________________________________________

__

(b) Determine the value of x (3 mks)

__________________________________________________________________

__

__________________________________________________________________

__
__________________________________________________________________

__

__________________________________________________________________

__

__________________________________________________________________

__

__________________________________________________________________

__

__________________________________________________________________

__

__________________________________________________________________

__

4. State what is observed and in each case write ionic equation for the
reaction.

(a) Chlorine gas is bubbled through a solution of Iron (II)


chloride.

Observation (1
mk)
__________________________________________________________________

__

__________________________________________________________________

__

Ionic equation (1 )

__________________________________________________________________

__

__________________________________________________________________

__

(b) When lead (II) nitrate solution is added to a solution of


sodium iodide

Observation (1
mk)

__________________________________________________________________

__

__________________________________________________________________

__

Ionic equation (1 mks)


__________________________________________________________________

__

__________________________________________________________________

__

5. Magnesium nitrate was heated until there was no further change.

(a) Write equation for the reaction. (1 mks)

__________________________________________________________________

__

__________________________________________________________________

__

(b) When 2.4 g of magnesium was heated, calculate the volume of


gaseous mixture produced. (3 mks)

__________________________________________________________________

__

__________________________________________________________________

__
__________________________________________________________________

__

__________________________________________________________________

__

__________________________________________________________________

__

__________________________________________________________________

__

__________________________________________________________________

__

__________________________________________________________________

__

__________________________________________________________________

__

__________________________________________________________________

__
6. (a)(i) Define allotropy (1
mk)

__________________________________________________________________
__

(ii) Name two major allotropies of carbon. (4


mks)

__________________________________________________________________

__

__________________________________________________________________

__

(b) Give one use of each allotrope named in a(ii) and in each case
give a property that make them suitable for the use. (2 mks)

__________________________________________________________________

__

__________________________________________________________________

__

__________________________________________________________________

__
__________________________________________________________________

__

__________________________________________________________________

__

__________________________________________________________________

__

(c) Name one other element that exhibits allotropy. (1


mk)

__________________________________________________________________
__

7.
The set-up above represents electrolysis of dilute sulphuric acid.

(a) Name the gases M and N (1 mk)

__________________________________________________________________

__

__________________________________________________________________

__

(b)(i) Write ionic equation for product of gas M. (1 )

__________________________________________________________________

__

__________________________________________________________________

__

(ii) What volume of gas produced at room temperature when


current of 10A is allowed to pass through the solution for 90
minutes.

(1 mole gas occupies 24 dm3 at room temperature) (1F = 96500 C)


__________________________________________________________________

__

__________________________________________________________________

__

__________________________________________________________________

__

__________________________________________________________________

__

__________________________________________________________________

__

__________________________________________________________________

__

__________________________________________________________________

__

__________________________________________________________________

__
__________________________________________________________________

__

__________________________________________________________________

__

8. Study the diagram below and answer the questions that follow.

HCl

Salt
Y

MnO2 Gas X Cold dilute


NaOH

NaCl

(a) Name gas X and Salt Y (2)

Gas X _______________________________________________

Salt Y _______________________________________________

(b) Write equation for the reaction leading to the formation of gas
X (1
mk)
____________________________________________________________
_

(c) State what is observed when burning sodium metal is lowered


in a gas jar containing gas X. (1
mk)

__________________________________________________________________

__

__________________________________________________________________

__

(ii) Write equation for reaction in b(i) (1


mk)

__________________________________________________________________

__

__________________________________________________________________

__

9. The figure below represents a section of the periodic table. Use it to


answer the questions below. The letters used are not the actual
symbols of the elements.

I
VIII
II III IV V VI VII

U S R

P Q T

(a) Write the formular of a compound formed between pairs of


elements and in each case state the type bond formed.

(i) Q and S (1
)

__________________________________________________________________

__

__________________________________________________________________

__

(ii) T and R (1
)

__________________________________________________________________

__

__________________________________________________________________

__
(b)(i) Arrange the elements above in order of increasing size
beginning with smallest. (1
mk)

__________________________________________________________________

__

__________________________________________________________________

__

(ii) Draw the structure of compound formed between T and S using


their outermost electrons. (1
mk)

__________________________________________________________________

__

__________________________________________________________________

__
__________________________________________________________________

__

__________________________________________________________________

__

10. A gaseous hydrocarbon is made up of 85% carbon by mass. 10


dm3 of the hydrocarbon weighs 12.5 g at s.t.p.

(C = 12, H = 1, One mole of a gas occupies 22.4 dm 3 at s.t.p)

(a) Calculate

(i) the empirical formula of the hydrocarbon. (2)

__________________________________________________________________

__

__________________________________________________________________

__

__________________________________________________________________

__

__________________________________________________________________

__
__________________________________________________________________

__

__________________________________________________________________

__

__________________________________________________________________

__

__________________________________________________________________

__

(ii) the relative molecular mass of the hydrocarbon. (2)

__________________________________________________________________

__

__________________________________________________________________

__

__________________________________________________________________

__
__________________________________________________________________

__

__________________________________________________________________

__

__________________________________________________________________

__

(b) Determine the molecular formular of the hydrocarbon. (1)

__________________________________________________________________

__

__________________________________________________________________

__

__________________________________________________________________

__

__________________________________________________________________

__
SECTION B. (Answer any two questions)

11. Explain the following observations and in each case write


equation(s) where possible.

(a) When carbon dioxide is bubbled through a solution of


calcium hydroxide, the solution turns milky and later turns clear.
(4 mks)

(b) When blue litmus paper is dipped in Chlorine water it turns


red and then bleached. (4
mks)

(c) When chlorine is bubbled through a solution of Potassium


bromide, the solution changes from colourless to brown. (3 mks)

(d) When hydrochloric acid is dissolved in water, the solution


formed conducts electricity but when dissolved in methylbenzene it
does not. (4 mks)

12. (a) Starting Zinc granules, describe how zinc sulphate crystals
can be prepared in laboratory. (6
mks)
(b) Ammonia solution was added to zinc sulphate solution dropwise
until in excess.
(i) State what was observed. (1
mk)

(ii) Write ionic equation for the reaction that took place. (1 )

(c) (i) Name the reagent that can be used to identify sulphate ions
in zinc sulphate solution. (1
mk)

(ii) State what is observed when the reagent is added to the


solution and write ionic equation for the reaction. (2 mks)

(d) Zinc reacts with sulphuric acid according to the reaction


below.

Zn(s) + H2SO4 (aq) ZnSO4 (aq) + H2 (g)

Calculate the mass of zinc required to produce 0.224 dm3 of


hydrogen at s.t.p (3 mks)

(1 mole of gas occupies 22.4 dm3 at s.t.p, Zn = 65, H = 1, S = 32

O = 16)

13. (a) Define the following terms:

(i) Solute (2)


(ii) Saturated solution (2)

(iii) Solubility of salt (2)

(b) The table below shows the solubilities of Salt A and Salt B at
different temperatures.

Temperature 0C 0 10 20 30 40 50 60
Solubility Salt A 13 20 32 45 63 85 110
g per 100g of water
Salt B 32.5 34 35 36 37 38 39

(i) Plot the graph of solubility against temperature for Salt A and Salt
B using the same axes. (4
mks)

(ii) A saturated solution of Salt A was cooled from 450C to 250C.


Determine the mass of Salt A deposited. (2
mks)

(iii) If salt A is potassium nitrate calculate the numbers of moles of


potassium nitrate obtained in b(ii) above. (2 mks)

(c) Which salts, A and B dissolve endothermically and why? (1 mk)

14. (a)(i) Define the term ore (1 mk)


(ii) Name the chief ore of sodium and iron metals. (1 mk)

(b)(i) Briefly describe how molten iron can be obtained from the ore
named in a(ii) (no diagram required). Include all the necessary
equations, uses of coke and calcium carbonate.

(c) Describe how impure copper can be purified. Include equation


and diagrams,

END.

Name__________________________________ Index /No_______/____

Signature_______________________________

SENIOR FOUR

END OF TERM I EXAMS 2008

CHEMISTRY PAPER II
Time: 2 hours.

Instructions:

Section A consists of 10 structured questions. Attempt all questions in


Section A.

Section B consists of 4 semi-structured questions. Attempt any two in


this section.

1 2 3 4 5 6 7 8 9 10 11 12 13 14 Total

Cu =64 O=16 C=12 Na= 23 H=1 N=14 K=19

Molar volume at s.t.p. is 22.4dm3

SECTION A (Answer all questions in this section)

1. Sulphur (IV) oxide can be prepared and collected in the laboratory by the
action of sulphuric acid on copper.

(a) Write the equation for the reaction


(2marks)

_______________________________________________________________________

(b) (i) State the conditions for the reaction


(1mark)

________________________________________________________________________

(ii) What method is used to collect sulphur (IV) oxide . Explain your answer
(1mark)

____________________________________________________________________________

____________________________________________________________________________
(c) State what would be observed if sulphur (IV) oxide were bubbled into an
aqueous solution of Iron (III) oxide.
(1mark)

____________________________________________________________________________

2. Carbon dioxide was bubbled through 100 cm3 of 2M sodium hydroxide


solution until no further change.

(a) State what was observed


(1mark)

____________________________________________________________________________

(b) (i) Write equation(s) for the reaction


(2marks)

____________________________________________________________________________

(ii) Calculate the volume of carbon dioxide that would completely react with
the sodium hydroxide.
(2marks)

3. A 2M solution of potassium iodide was electrolysed using carbon


electrodes.
(a) State what was observed at (1mark)

(i) cathode____________________________________________________________
_______

(ii)
anode_____________________________________________________________________

(iii) Write the equation for reaction that took place at the anode
(1mark)

(b) The pH of the solution at the cathode was tested using the universal
indicator.

(i) State what was observed


(1mark)

____________________________________________________________________________

(ii) Explain the observation


(2marks)

__________________________________________________________________________

4. Iron is often galvanized.

(a) Explain how galvanization protects the iron


(2marks)

____________________________________________________________________________

____________________________________________________________________________

(b) State the conditions that causes rusting


(1mark)
(c) Carbon monoxide was passed over dry rust at about 600o C.

Write the equation for the reaction


(2marks)

5. Magnesium burnt in air to form magnesium nitride.


(a) Write the equation for the reaction.
(2marks)

(b)Calculate the volume of air necessary to completely react with 6.0g of


magnesium if the percentage of nitrogen in air is 78 by volume.
(3marks)

6. 6.3 g of sodium carbonate contaminated with sodium chloride was


dissolved in water and the solution made up to1 dm3. 25.0 cm3 of this
solution reacted completely with 24.8 cm3 of 0.1M hydrochloric acid.

(a) Write the equation for the reaction.


(1marks)

(b) Determine the percentage purity of the sodium carbonate


(3marks)
7. Chlorine gas is prepared by the action of hydrochloric acid on substance Q

(a)(i) Name substance Q ________________________________________ (marks)

(ii) State the condition(s) for the reaction


(1mark)

____________________________________________________________________________

(ii) Write the equation for the reaction


(1marks)

____________________________________________________________________________

(b) Chlorine was bubbled through litmus solution.

(i) State what was observed


(1marks)

___________________________________________________________________________

(ii) Write the equation for the reaction


(1marks)

8. The table below shows the melting points and electrical conductivities of
substances A, B and C. Use this information to answer the questions that
follow

Electrical conductivity
Substance Melting Aqueous Molten state Solid state
s point oC solution

A 850 Not soluble conducts conducts


B -100 conducts Does not Does not conduct
conduct
C 780 conducts conducts Does not conduct

(a)Name the particles that are responsible for the electrical conductivity of

(i) Substance A_______________________________________________________

(ii) Aqueous B________________________________________________________

(1mark)

(b)Explain why substance C conducts electricity in the molten state but not
in the solid

state
(2marks)

__________________________________________________________________________

(c) State and explain the type of bonding in


(2marks)

(i)B__________________________________because_______________________________
__

___________________________________________________________________________

(ii)C__________________________________because_______________________________
_

___________________________________________________________________________
9.(a) What is a fertilizer?
(2marks)

(b) Ammonium nitrate, potassium nitrate and calcium hydroxide are usually
added to soil. (i) Name one of the above salts that would be added to an
alkaline soil and explain your answer?
(2mark)

____________________________________________________________________________

____________________________________________________________________________

____________________________________________________________________________

____________________________________________________________________________

10. The table below shows the products formed when nitrates P, Q and R
are heated.

Nitrate Products
P Metal oxide, nitrogen (IV) oxide and
oxygen
Q Metal, nitrogen (IV) oxide and oxygen
R Metal nitrite and oxygen
(a)Arrange the metals in order of reactivity stating with the most reactive
(1mark)

_______________________________________________________________________

(b)Concentrated sulphuric acid was added to R and the test tube gently
heated.

(i) State what was observed


(1mark)

____________________________________________________________________________

(ii)Write the equation for the reaction


(2marks)
____________________________________________________________________________

(c)Nitrogen compounds are pollutants. State two examples


(1mark)

______________________________________________________________________________
______________________________________________________________________________
________________________________________________________________________

SECTION B

Attempt any two questions in this section

11. Explain the following observations. Write equations where applicable

(a) Graphite is used as electrodes in electrolysis and as a lubricant.


(3marks)

(b) Electrolysis of dilute sulphuric acid is equivalent to electrolysis of water


(6marks)

(c) When zinc is dipped in copper (II) sulphate, the solution turns from blue
to colourless and a brown deposit is formed.
(3marks)

(d) When sulphur (IV) oxide is bubbled in barium nitrate solution a white
precipitate is observed.
(3marks)

12.(a) Name and write the formulae of one ore from which sodium metal is
extracted (2mark)

(b) Outline the manufacture of sodium metal (diagram not required)


(6marks)

(c) What differences, if any, are there in the products when a current is
passed through molten sodium chloride and molten sodium metal. Explain
your answer. (5marks)

(d) State two applications of electrolysis.


(2marks)

13. Sodium carbonate 10 water contains water of crystallization.


(a) What do understand by the term water of crystallisation?
(2marks)

(b) Sodium carbonate 10 water was exposed to the atmosphere.


(i) State what was observed
(2marks) (ii) Name the process taking place
(1mark) (ii) Write an equation for the reaction
(2marks)

(c)(i) Briefly describe how a sample of sodium carbonate is prepared in the


laboratory.
(6m
arks)
(ii) State two uses of sodium carbonate
(2marks)

14. The diagram bellow shows the extraction of sulphur by the Fraschs
process

Tank D
bed deposit

A B C

(a)(i) Name the substances that pass through tubes, A, B and C


(3marks)
(ii) State the conditions for the process.
(2marks) (iii) State one property of sulphur that makes it possible to be
extracted using the above process.
(1mark) (iv) How is the sulphur separated from tank D?
(1mark)

(b) Sulphur exhibits allotropy. The transition temperature of sulphur is 96oC.


(i) What do you understand by the term allotropy?
(2marks)
(ii) Using the two allotropes of sulphur explain the term transition
temperature. (3marks)

(c) Sulphur was burnt in excess oxygen.


(i) State what was observed.
(1mark)
(ii) Write the equation for the reaction.
(2marks)

Name____________________________________ Index /No___________________

Signature_______________________________

SENIOR FOUR

MID TERM EXAMS 2008

CHEMISTRY PAPER II

Time: 2 hours.

Instructions:

Section A consists of 10 structured questions. Attempt all questions in


Section A.

Section B consists of 4 semi-structured questions. Attempt any two in this


section.

1 2 3 4 5 6 7 8 9 10 11 12 13 14 Total
Cu =64 O=16 C=12 Na= 23 H=1 N=14 K=19

Molar volume at s.t.p. is 22.4dm3

SECTION A (Answer all questions in this section)

1. A mixture of zinc oxide and magnesium oxide can be separated by


filtration. Name the
(a) (i) solvent used__________________________________(1mark)

(ii) filtrate contains _______________________________________

(iii) residue ________________________________________(1mark)

(b)Explain why it is possible to separate the mixture. (3marks)

___________________________________________________________________________

2. 0.3g of copper(II) oxide was dissolved in X cm3 of a 0.1M hydrochloric


acid.

(a ) (i) State what was observed. (1mark)

(ii) Write the equation for the reaction (1mark)

(c) Calculate the number of


(i) moles of copper (II) oxide used

(ii) moles of hydrochloric acid


(iii) hence the volume X of 0.1M Hydrochloric acid
(3marks)

3. On analysis, a mixture was found to contain the following three ions;


chloride, sulphate and carbonate. Using the table below describe a test
that can be carried out to confirm the presence of the ions. Write in the
spaces provided, the expected observation(s) you would make. (4.5
mks)

ION TEST OBSERVATION

Chloride

Cl-

Sulphate

SO42-(aq)

Carbonate

CO32-(aq)

4. 3.1g of a carbonate XCO3 was heated to constant mass and 2.0g of the
metal oxide was formed.
(a) Write the equations for reactions.
(1.5marks)

______________________________________________________________________________

(b) Calculate:

(i) the atomic mass of X


(2marks)

(ii)The volume of carbon dioxide (at stp) produced when the carbonate is
completely decomposed (1.5
marks)

5. Concentrated sulphuric acid was added to copper turnings and the mixture
heated. A white solid and a colourless gas were observed.
(a) Name the
(i) white solid___________________________________________________(1mark)

(ii) colourless gas _______________________________________________ (1mark)

(iii)reagent used to identify the colourless gas and state what is observed
when the gas is reacted with the reagent
(2marks)
______________________________________________________________________________
______________________________________________________________________________

(b) 2-3 drops of water were added to the white solid.


(i) State what was observed
(0.5marks)
____________________________________________________________________________

6. 5.72g of sodium carbonate decahydrate was exposed to air for some


time.
(a) (i) State what was observed (1mark)

(ii) Write the equation for the reaction. (1mark)

_____________________________________________________________________________

(b) Calculate the percentage change in mass after several days


(3.5marks)

7. Part of the Periodic Table is shown in the figure below. The letters do not
represent the actual symbols of the elements.

I II III IV V VI VII VIII


G M R
Q L
T U

(a)(i) State an element that can form an ion with a charge of 2- (1mark)

(ii) State the type of bond that exists in the compound formed when it reacts
with G.
(1mark)

______________________________________________________________________________

2.5g of Q reacts completely with 1200 cm3 of gas R at s.t.p.


(i)Write a balanced equation for the reaction between Q and R
(2marks)

(ii)Determine the atomic mass of Q.


(2marks)

8. (a) What is an indicator?


(1mark)

______________________________________________________________________________

(b)State and explain what would be observed when the universal indicator is
added to the following solutions.

(i) Potassium sulphate solution (2marks)

______________________________________________________________________________
______________________________________________________________________________

(ii) Potassium carbonate solution (2marks)

______________________________________________________________________________

9. (a) Air is a gaseous mixture; carbon dioxide is a gaseous compound. State


two major differences between them
(2marks)

Air Carbon dioxide


(i)

(ii)
(b)(i) State what would be observed when carbon dioxide is bubbled into
calcium hydroxide solution for a long time
(1.5marks)
______________________________________________________________________________

______________________________________________________________________________

(ii)Write equation(s) for the reaction(s)


(3marks)

______________________________________________________________________________

10. (a) What do you understand by the term hard water?


(1mark)

(b) In an experiment to find out which ions dissolved in water, are


responsible for the hardness, the following results were obtained:

Test solution Rain NaCl FeCl3 KCl CaCl2 MgCl2


water
Volume of soap 5 5 10 5 20 15
needed to make a
lather/cm3
Precipitate observed NO NO YES NO YES YES
(i)Which particular ions are responsible for the hardness?
(1.5marks)

______________________________________________________________________________

(ii)Explain why the precipitate is not observed with all salts assuming the
soap was sodium stearate
(2marks)

______________________________________________________________________________
SECTION B

Attempt any two questions in this section

11. (a)(i) Describe how you would prepare pure crystals of lead (II) nitrate in
the laboratory. (ii) Write the equation for the reaction.
(6marks)

(b) (i) State what would be observed when lead (II) nitrate crystals are
strongly heated.
(ii) Write the equation for the reaction.
(4marks)

(c) 0.15M hydrochloric acid was added to 25cm3 of 0.1M lead (II) nitrate.
(i) State what was observed
(ii) Write an ionic equation for the reaction
(2marks)
(iii) Calculate the volume of 0.15M hydrochloric acid needed to completely
react with the lead (II) nitrate solution.
(2marks)

12. Explain the following observations. Write equations to explain your


answer where possible.

(a) When sodium hydroxide pellets were exposed to air, a colourless


solution was formed and later a white crystalline solid developed.
(4marks)

(b) When a jar of hydrogen sulphide is inverted over a jar containing moist
sulphur (IV) oxide a yellow solid is observed.
(3marks)

(c) Concentrated sulphuric acid can not be used to dry ammonia gas
(3marks)

(d) When chlorine gas was passed through a solution containing potassium
bromide, a red colouration was observed. (3marks)

(e) Concentrated nitric acid turns brown on standing (2marks)

13.(a) (i)Describe and explain how a dry sample of hydrogen gas may be
prepared in the laboratory. (N.B No diagram is required) (4marks)
(ii) Write the equation for the reaction
(2marks)

(b) Dry hydrogen were passed over heated 8.0g copper (II) oxide.
(i) State what was observed.
(ii) Explain the observation
(3marks)
(iii) Write the equation for the reaction (2marks)
(iv) Calculate the volume of hydrogen at room temperature that reacted
completely with copper (II) oxide (2marks)

(c) State two commercial uses of hydrogen (2marks)

14. (a)Define the following terms:


(i) Solute
(2marks) (ii) Saturated solution
(2marks) (iii) Solubility of salt
(2mark)

(b) The table below shows the solubilities of Salt A and Salt B at different
temperatures.

Temperature 0C 0 10 20 30 40 50 60
Solubility Salt A 13 20 32 45 63 85 110
g per 100g of Salt B 32.5 34 35 36 37 38 39
water

(i)Plot the graph of solubility against temperature for Salt A and Salt B using
the same axes. (4
marks) (ii)A saturated solution of
0 0
Salt A was cooled from 45 C to 25 C. Determine the mass of Salt A
deposited. (2 marks)
(iii)If salt A is potassium nitrate calculate the numbers of moles of potassium
nitrate obtained in b(ii) above. (2
marks)

(c) Which salts, A and B dissolve endothermically and why? (1


mark)
Name.............................................................. Index /No..................................

Signature........................................................................
SENIOR FOUR
END OF TERM 1 EXAMINATION 2011
CHEMISTRY PAPER II APRIL
Time: 2 hours.

Instructions:
Section A consists of 10 structured questions. Attempt all questions. Answers to
these questions must be written in the spaces provided.
Section B consists of 4 semi-structured questions. Attempt any two any
questions in this section. Answers to these questions must be written in the
answer booklets provided.
In both sections all working must be clearly shown.

FOR EXAMINERS USE ONLY


1 2 3 4 5 6 7 8 9 10 11 12 13 14 TOTAL

SECTION A (50 MARKS)


1. Complete the table below by stating one method by which components
of the mixtures described can be separated.

COMPONENT OF MIXTURE METHOD OF SEPARATION


(I mark each)
(a) Two miscible liquids with boiling
points close together.

(b) A soluble solid and a solvent.

(c) A soluble solid containing a soluble


impurity.

(d) Complex solids in ink solution

2.(a) Burning magnesium was lowered into a gas jar of sulphur (IV)
oxide gas.
(i) State what was observed. (1marks)
------------------------------------------------------------------------------------------------------------
------------------------------------------------------------------------------------------------------------
------------------------------------------------------------------------------------------------
(ii) Write equation for the reaction that took place. (1marks)
------------------------------------------------------------------------------------------------------------
----------------------------------------------------------------------------------------------------
(iii) State the property of sulphur (IV) oxide shown in the reaction. (marks)
------------------------------------------------------------------------------------------------------------
----------------------------------------------------------------------------------------------------
(b) Write equation for one other reaction in which sulphur (IV) oxide shows
property you have stated in a (iii). (1marks)
------------------------------------------------------------------------------------------------------------
----------------------------------------------------------------------------------------------------

3. The diagram below shows the set up of the apparatus that was used to
investigate the effect of hot manganese (IV) oxide on hydrogen chloride
gas.

(a) Write equation for the reaction in that took place in the glass tube
(1marks)
------------------------------------------------------------------------------------------------------------
----------------------------------------------------------------------------------------------------(b)
State
(i) the role of manganese (IV) oxide in the reaction in (a) (mark)

--------------------------------------------------------------------------------------------------- ----

(ii) what was observed in the beaker containing potassium iodide solution.
(1marks)
------------------------------------------------------------------------------------------------------------
------------------------------------------------------------------------------------------------------------
------------------------------------------------------------------------------------------------
(c) Write an ionic equation for the reaction to illustrate your answer in (b)(ii).
(1marks)
------------------------------------------------------------------------------------------------------------
------------------------------------------------------------------------------------------------------------
------------------------------------------------------------------------------------------------
4.(a)(i) Define the term salt. (1mark)
------------------------------------------------------------------------------------------------------------
------------------------------------------------------------------------------------------------------------
------------------------------------------------------------------------------------------------
------------------------------------------------------------------------------------------------------
(ii) Write the name and formula of an acid salt. (1mark)
------------------------------------------------------------------------------------------------------------
-----------------------------------------------------------------------------------------------
(b) When 2.4g of crystals of salt W were heated, 1.5g of anhydrous salt was
obtained. Determine the number of moles of water of crystallisation in one
mole of crystalline W.
(The molar mass of W =158, H =1, O= 16) (3marks)
------------------------------------------------------------------------------------------------------------
------------------------------------------------------------------------------------------------------------
------------------------------------------------------------------------------------------------------------
------------------------------------------------------------------------------------------------------------
------------------------------------------------------------------------------------------------------------
------------------------------------------------------------------------------------------------------------
------------------------------------------------------------------------------------------------------------
------------------------------------------------------------------------------------------------------------
------------------------------------------------------------------------------------------------------------
--------------------------------------------------------------------
5. 25.0cm3 of 0.1M sodium carbonate solution reacted completely with a
solution containing 5.0g of sulphuric acid in 250cm3 of solution.
Calculate
(i) the molarity of sulphuric acid in moles per dm3. (3marks)

------------------------------------------------------------------------------------------------------------
------------------------------------------------------------------------------------------------------------
------------------------------------------------------------------------------------------------------------
------------------------------------------------------------------------------------------------------------
------------------------------------------------------------------------------------------------------------
-----------------------------------------------------------------------------------
(ii) the volume of acid used. (2marks)
------------------------------------------------------------------------------------------------------------
------------------------------------------------------------------------------------------------------------
------------------------------------------------------------------------------------------------------------
------------------------------------------------------------------------------------------------------------
------------------------------------------------------------------------------------------------------------
-----------------------------------------------------------------------------------
------------------------------------------------------------------------------------------------------------
--------------------------------------------------------------------------------------------------
-------------------------------------------------------------------------------------------------------
------------------------------------------------------------------------------------------------------------
----------------------------------------------------------------------------------------------------
6. The elements carbon, magnesium and aluminium can combine with
oxygen to form oxides.
(a) State the class of the oxides of
(i) Carbon ( mark)
--------------------------------------------------------------------------------------------------------
(ii) Magnesium ( mark)
--------------------------------------------------------------------------------------------------------
(iii) Aluminium ( mark)
--------------------------------------------------------------------------------------------------------
(b) Write equation for the reaction that would take place in each case if the
oxides of carbon, magnesium were separately treated with water.
(i) oxide of carbon (1 mark)
------------------------------------------------------------------------------------------------------------
----------------------------------------------------------------------------------------------------
(ii) oxide of magnesium (1 mark)
------------------------------------------------------------------------------------------------------------
----------------------------------------------------------------------------------------------------

(c) Name one other element that can combine with oxygen to form an oxide
in the same class as that of aluminium. (mark)

--------------------------------------------------------------------------------------------------------

7.(a) During the laboratory preparation of hydrogen gas, dilute sulphuric acid
was added to zinc metal followed by a little amount of cooper (II)
sulphate solution.

(i) Write equation for the reaction leading to the formation of hydrogen gas.
(1 mark)
------------------------------------------------------------------------------------------------------------
----------------------------------------------------------------------------------------------------
(ii) State the role of copper (II) sulphate. ( mark)
--------------------------------------------------------------------------------------------------------
(b) State the conditions under which hydrogen may be displaced from water
by:
(i) sodium metal ( mark)
------------------------------------------------------------------------------------------------------
(ii) iron filings ( mark)
------------------------------------------------------------------------------------------------------
------------------------------------------------------------------------------------------------------
(c) Write the equation for the reaction in (b)(ii) (1 mark)
------------------------------------------------------------------------------------------------------
8.(a) A mixture of magnesium powder and lead (II) oxide was heated strongly
until no further change.
(i) State what was observed. (1 mark)
------------------------------------------------------------------------------------------------------------
----------------------------------------------------------------------------------------------------
(ii) Write equation for the reaction that took place. (1 mark)
------------------------------------------------------------------------------------------------------------
----------------------------------------------------------------------------------------------------
(b) The experiment in (a) was repeated using a mixture of copper turnings
and magnesium oxide.
State what was observed, ( mark)

-------------------------------------------------------------------------------------------------------

(c) Explain your observations in (a) and (b) (1 mark)


--------------------------------------------------------------------------------------------------------
-------------------------------------------------------------------------------------------------------
------------------------------------------------------------------------------------------------------------
------------------------------------------------------------------------------------------------------------
-----------------------------------------------------------------------------------------------
9. During the manufacture of sodium hydroxide, a concentrated sodium
chloride solution (brine) is electrolysed using mercury cathode as shown
in the diagram below.
(a) Identify the products collected at

(i) X................................................................................................ ( mark)

(ii) Y................................................................................................ ( mark)

(b) Write equation for the reaction leading to the formation of the product
collected at Y (1mark)

------------------------------------------------------------------------------------------------------------
----------------------------------------------------------------------------------------------------(c)
Name the substance used as the anode. (1mark)

--------------------------------------------------------------------------------------------------------
(d) State;
(i) one industrial use of sodium hydroxide. ( mark)

(ii) what is observed if chlorine gas is bubbled through a cold dilute solution
of sodium hydroxide? (1mark)

--------------------------------------------------------------------------------------------------------

10. Under suitable conditions ammonium chloride reacts to produce


ammonia according to the equation.
Ca(OH)2(s) + 2NH4Cl(s) CaCl2(s) + 2NH3 (g) + 2H2O(l)
(a) State;
(i) the condition(s) for the reaction leading to the formation of ammonia.

------------------------------------------------------------------------------------- ( mark)

(ii) how ammonia is collected and give a reason for your answer. (1mark)

------------------------------------------------------------------------------------------------------------
---------------------------------------------------------------------------------------------------

(iii) why ammonia is not dried using fused calcium chloride or concentrated
sulphuric acid. (no equation required). (1 marks)

------------------------------------------------------------------------------------------------------------
----------------------------------------------------------------------------------------------------
(c) When X g of ammonium chloride was used in preparation of ammonia
as shown in the equation in (a0, 4.0g of pure and dry calcium chloride
was obtained. Determine the value of X
(H =1, N= 14, Cl = 35.5, Ca = 40)

------------------------------------------------------------------------------------------------------------
------------------------------------------------------------------------------------------------------------
------------------------------------------------------------------------------------------------------------
--------------------------------------------------------------------------------------------
------------------------------------------------------------------------------------------------------------
------------------------------------------------------------------------------------------------------------
------------------------------------------------------------------------------------------------------------
--------------------------------------------------------------------------------------------

SECTION B (30 MARKS)

Answer any two questions from this section.


11.(a) In an experiment to analyse the composition of air. 100.0 cm3 of dry air
was carefully analyzed.
State the approximate volume of dry air that was occupied by:
(i) carbon dioxide ( marks)
(ii) oxygen ( marks)
(iii) nitrogen ( marks)

(b) The analysis in (a) was repeated on a day that had rained.
(i) State how the volume of carbon dioxide would compare with that
obtained in (a) ( marks)
(ii) Explain your answer in (b) (i) (1 marks)

(c) Name one reagent that can be used to remove carbon dioxide from a
sample of air and write the equation that would take place when the
reagent is used. (2marks)

(d) Name and describe the industrial process by which a mixture of


nitrogen and oxygen can be separated. (No diagram requires).
(5marks)

(e)(i) Write the formula and chemical name of limestone (1mark)


(ii) Briefly describe the reactions starting from carbon dioxide that lead to
cause of hardness in water from rivers and lakes in limestone area.
(No equation or diagram required.) (4marks)

12. The atomic numbers of some elements are given below.

Element hydrogen carbon sodium chlorine


Atomic number 1 6 11 17

(a) Write the electronic configuration of; (1 marks)


(i) carbon
(ii) sodium
(iii) chlorine
(b) State how sodium and chlorine form ions and explain why the ions
formed are stable. (3marks)

(c) Write the structure of the compound formed when carbon combines with
chlorine. (1mark)

(d) Briefly explain the following;


(i) Sodium chloride in solid state does not conduct electricity but it
conducts electricity when molten and is decomposed by the electric
current. (6marks)
(ii) Molecules of hydrogen chloride dissolve in methylbenzene to form a
solution that forms a white precipitate when ammonia is passed into it.
(3marks)

13(a) Outline how a reasonably pure and dry sample of copper (II) sulphate
crystals can be prepared in the laboratory, starting from copper (II) oxide.
Write equation(s) to illustrate your answer. (7marks)

(b) State what would be observed, and write equation for the reaction that
would take place if;
(i) a crystal of copper (II) sulphate was dropped in concentrated sulphuric
acid and the mixture warmed. (2marks)
(ii) to an aqueous solution of copper (II) sulphate was added dilute
sodium hydroxide solution drop-wise until the alkali was in excess.
(2marks)

(c) Lead (II) ions can react with sulphate ions according to the following
equation.
Pb 2+(aq) + SO4(aq) PbSO4(s)

(i) State what would be observed if lead (II) nitrate was added to aqueous
copper (II) sulphate solution. (1mark)
(ii) Excess aqueous lead (II) nitrate solution was added to a solution
containing 2.50g anhydrous copper (II) sulphate. The mixture was
stirred, filtered and the residue dried and weighed.
Calculate the mass of the dry residue that was obtained. (3marks)
(Pb = 207, Cu = 64, S = 32, O = 16)
14. Iron (III) oxide (haematite) is one of the common ores of iron from which
iron can be extracted in the blast furnace.
(a) Name
(i) one common ore of iron other than iron (III) oxide. (1mark)
(ii) one major impurity that can be found in the ore you have named in (a)(i)
(1mark)

(b) Outline the reactions which occur in the blast furnaces during the
extraction of iron from iron (III) oxide ore. (7marks)

(c)(i) Name the major components of stainless steel. (1mark)


(ii) State one use of stainless steel. (1mark)
(iii) Give a reason why stainless steel is more commonly used than pure
iron. (1mark)

(d) Most common compounds of iron are either those of iron (II) or iron (III).
Write the formula of one compound of
(i) iron (II) (mark)
(iii) iron (III) (mark)

(e) Name one reagent that can be used to distinguish between iron (II) and
iron (III) compounds and in each case state the observations that would be
made if the reagent you have named was used. (2marks)
Name.............................................................. Index /No..................................

Signature........................................................................

SENIOR FOUR
MID TERM EXAMINATION 2011
CHEMISTRY PAPER II
Time: 2 hours.

Instructions:
Section A consists of 10 structured questions. Attempt all questions.
Section B consists of 4 semi-structured questions. Attempt any two any
questions n this section.

1 2 3 4 5 6 7 8 9 10 11 12 13 14 Total

Relative atomic masses; O=16, C=12, Na= 23, H=1, 0=16, S=32, P=31
Molar volume at room temperature 24dm3
SECTION A (Answer all questions in this section)

1. (a) A mixture of sodium chloride and ammonium chloride were


heated strongly until no further change.
(i) State what was observed. (1mark)
..................................................................................................................
(ii) Write the equation for the reaction. (1marks)
..................................................................................................................
(b) The mixture above was dissolved in water. Lead (II) nitrate
solution was added to the aqueous mixture.
(i) State what was observed. (1mark)
..................................................................................................................
(ii) Write the ionic equation for the reaction. (1marks)
.................................................................................................................

2 The following table shows the atomic numbers and mass numbers of
elements A, B and C.

Element Atomic number Mass number


A 11 23
B 8 16
C 20 40

(a) State the number of electrons and neutrons present in atom of


element A. (1mark)
..................................................................................................................
..................................................................................................................
(b) Write the electron configuration of the atom of C (1mark)
.................................................................................................................
(c) To which group of the Periodic Table does element B belong?
Give a reason for your answer. (2marks)
..................................................................................................................
..................................................................................................................
(d) Write the formula of the ion formed from the atom of element B.
.................................................................................................... (1mark)
3. Lead and molten lead (II) bromide are both conductors of electricity.
(a) Name the conducting species in
(i) lead..................................................................................
(ii) molten lead (II) bromide. (1marks)
.................................................................................................................
(b) Aqueous copper (II) sulphate solution was electrolysed using the
set up shown in the diagram below.

(i) Describe what happens at each electrode after


electrolysis. (1marks)
..................................................................................................................
..................................................................................................................
..................................................................................................................
..................................................................................................................
................................................................................................................
(ii) Write the equation for the reaction that took place at A
(1mark)
..................................................................................................................
(iii) State two industrial applications of electrolysis (1mark)
..................................................................................................................
..................................................................................................................

4. 3.60 g of impure sodium carbonate was dissolved in water to make up


500 cm3 of solution. 25 cm3 of the solution required exactly
24.0 cm3 of 0.1M hydrochloric acid for complete reaction.
Calculate
(i) the molarity of sodium carbonate (3marks)
..................................................................................................................
..................................................................................................................
..................................................................................................................
..................................................................................................................
..................................................................................................................
................................................................................................................
..................................................................................................................
..................................................................................................................
..................................................................................................................
................................................................................................................

(ii) the percentage purity of the original sample of the carbonate


(2marks).
..................................................................................................................
..................................................................................................................
..................................................................................................................
..................................................................................................................
................................................................................................................
..................................................................................................................

5. (a) Sulphuric acid reacts with copper turnings under suitable


condition(s) to produce gas X

(i) State the conditions for the reaction (1mark)


..................................................................................................................
(ii) Identify gas X (mark)
..................................................................................................................
(iii) Write the equation for the reaction (1marks)
.................................................................................................................

(b) Briefly describe a test you would carry out to identify gas X.
(2marks)
..................................................................................................................
..................................................................................................................
..................................................................................................................

6. (a). State what would be observed if into aqueous potassium iodide


was;
(i). bubbled chlorine. (1marks)
..................................................................................................................
..................................................................................................................
(ii). added 2-3 drops of lead (II) nitrate solution. (mark)
................................................................................................................
(b). (i). Give a reason for the reaction in (a) (i). (1mark)
..................................................................................................................
..................................................................................................................
..................................................................................................................

(ii). Write an ionic equation for the reaction in (a) (ii). (1mark)
..................................................................................................................

7. (a). Magnesium was burnt in air.


Write equation (s) for the reaction (s) that took place. (3marks)
..................................................................................................................
..................................................................................................................
................................................................................................................
(b). When water was added to the residue from (a) a gas R, that gave
dense white fumes with concentrated hydrochloric acid was evolved.
(i) Identify R. (1mark)
.............................................................................................................
(ii) Write equation for the reaction leading to the formation of
R. (1 marks)

.................................................................................................................
8. A compound X of formula mass 90 contains 26.67% carbon and 2.22%
hydrogen by mass, the rest being oxygen.
(a). Calculate
(i). the empirical formula of X (3marks)
..................................................................................................................
..................................................................................................................
..................................................................................................................
..................................................................................................................
..................................................................................................................
..................................................................................................................
..................................................................................................................
..................................................................................................................
..................................................................................................................
(ii). the molecular formula of X (1mark)
..................................................................................................................
..................................................................................................................
..................................................................................................................

(b). X dissolves in aqueous sodium hydrogen carbonate with


effervescence.
(i). Suggest the chemical nature of X (mark)
..................................................................................................................
(ii). Write an ionic equation for the reaction between X and
aqueous sodium hydrogen carbonate. (1marks)

.................................................................................................................

9. (a) When an iron nail is exposed to the atmosphere it rusts.


(i) Define the term rusting. (2mark)
..................................................................................................................
..................................................................................................................
(ii) Write the name and formula of rust. (1mark)
..................................................................................................................
..................................................................................................................
(b) Rust was dissolved in dilute hydrochloric acid.
(i) State what was observed. ( mark)
..................................................................................................................
(ii) Write the equation for the reaction in b (i) (1 mark)
..................................................................................................................

10. Study the diagram below and answer the questions that follow.

Concentrated
Hydrochloric
acid
Salt Y

Substanc Chlorine Cold dilute


eX gas NaOH
NaCl

(a) Name substance X and Salt Y (2marks)


(i) Substance X ...
(ii) Salt Y .
(iii)
Write the equation for the reaction leading to the formation
of chlorine gas. (1mark)
..
(b) White phosphorous is lowered in a gas jar containing chlorine gas (i)
State what is observed (1mark)


..
(ii) Write equation for reaction in b (i) above (1mark)

SECTION B
Answer two questions only in this section. Start each question on a fresh page.

11. (a) Copper (II) sulphate-5- water contains water of crystallization.


What do understand by the term water of crystallisation? (1mark)

(b) (i) Briefly describe how a pure sample of hydrated copper (II)
sulphate is prepared in the laboratory. (7marks)
(i) Write the equation for the reaction. (1mark)
(c) Copper (II) sulphate-5- water was heated strongly until no further
change.
(i) State what was observed (1mark)
(ii) Write the equation for the reaction (1mark)

(c) A strip of zinc metal was dipped into aqueous copper (II)
sulphate solution.
(i) State what was observed (1mark)
(ii) Write an ionic equation for the reaction. (1mark)

12. (a) Oxygen is prepared in the laboratory by catalytic decomposition


of hydrogen peroxide
(i) Name the catalyst used (1mark)
(ii) State the condition(s) for the reaction in (a) (mark)
(iii) Write the equation for the reaction. (1mark)

(b) Briefly describe how oxygen is manufactured on large scale.


(6marks)
(c) 3.2 g of Sulphur were burnt in excess oxygen.
(i) Write the equation for the reaction (1mark)
(ii) Calculate the volume of oxygen at room temperature that
reacted completely with sulphur. (3marks)

(d) State three uses commercial of oxygen (1mark)

13. Fertilizers are made by mixing ammonium nitrate, diammonium


hydrogen phosphate [(NH4)2HPO4] and potassium chloride. The flow chart
below shows the process by which N.P.K. fertilizer can be manufactured.

NITROGEN HYDROGEN ROCK POTASSIUM


PHOSPHATE CHLORIDE

AMMONIA NITRIC ACID PHOSPHORIC


ACID
AMMONIUM NITRATE AND
DIAMMONIUM HYDROGEN
PHOSPHATE

N. P. K. FERTILIZER

(a) Briefly explain what is meant by a fertilizer. (2marks)

(b) State the industrial processes by which the ammonia and nitric
acid used for making the N.P.K. fertilizer are produced (2marks)

(c) Diammonium hydrogen phosphate is an acid salt. Explain


(2marks)

(d) Diammonium hydrogen phosphate and ammonium nitrate are


prepared from ammonia.
(i) Describe how the two compounds are prepared. (2marks)
(ii) Write the equations for the reactions leading to the
formation of the two compounds (3marks)
(iii) Which one of the elements in the N. P. K. fertilizer is
introduced into the fertilizer by direct addition of its
compound? (1mark)

(e) Calculate the percentage of nitrogen and phosphorous in


diammonium hydrogen phosphate fertilizer. (3marks)

14. Explain the following observations, give equations where necessary.


(a) Diamond is a hard substance with a high melting point (4marks)

(b) When exposed to the atmosphere for some days, sodium


hydroxide pellets turn into a liquid, which later forms white
crystals. (5marks)
(c) When aqueous sodium hydrogen carbonate solution is added to
magnesium chloride solution, there is no observable change.
When the resultant solution is heated a white precipitate is
formed. (4marks)

(d) When sodium hydroxide is heated with ammonium chloride


a gas that turns litmus blue is given off. (2marks)

Name.............................................................. Index /No..................................

Signature........................................................................
SENIOR FOUR
BEGINNING OF TERM EXAMINATION 2012
CHEMISTRY PAPER II
Time: 2 hours.

Instructions:
Section A consists of 10 structured questions. Attempt all questions.
Section B consists of 4 semi-structured questions. Attempt any two any
questions n this section.

1 2 3 4 5 6 7 8 9 10 11 12 13 14 Total

Relative atomic masses; O=16, C=12, Mg= 24, H=1,


Molar volume at room temperature 24dm3
Molar volume at standard temperature and pressure 22.4dm3

SECTION A (50 marks)


Attempt all questions
1.(a) A hydrocarbon Y contains 85.7% carbon. Calculate the simplest
formula. (2marks)
..................................................................................................................
..................................................................................................................
..................................................................................................................
..................................................................................................................
..................................................................................................................
..................................................................................................................

(b) 1 mole of hydrocarbon Y on complete combustion in air produced 96 dm3


of carbon dioxide at room temperature.

(i) Calculate the molecular formula of Y (2marks)


..................................................................................................................
..................................................................................................................
..................................................................................................................
..................................................................................................................

(iii) Write down at least two structure formulae of Y (1mark)


..................................................................................................................

.................................................................................................................

2. Name one pair of substance in each case, which when mixed together
can be separated by
(a) Use of a separating funnel (1 marks)

(b) Fractional crystallization (1 mark)

(c) Filtration (1 mark)

(d) Chromatography (1 mark)

(e) Fractional distillation (1 mark)

3.(a) Excess carbon monoxide gas was passed over a heated sample of an
oxide of iron as shown in the diagram below. Study the diagram and the
data below to answer the questions that follow.

Mass of empty dish =10.98g


Mass of empty dish + oxide of iron =13.30g
Mass of empty dish + residue =12.66g
(i) Determine the formula of the oxide of iron. (Fe=56, O=16) (3 marks)









.
(ii) Write an equation for the reaction which took place in the dish(1 marks)

..
(b) Rusting is a destructive process in which iron is converted into hydrated
iron (III) oxide.
(i) State two conditions necessary for rusting to occur
(1
mark)

(ii) One method to protect iron from rusting ( mark)


4. The table below shows the products formed when nitrates P, Q and R are
heated.

Nitrate Products
P Metal oxide, nitrogen (IV) oxide and oxygen
Q Metal, nitrogen (IV) oxide and oxygen
R Metal nitrite and oxygen

(a) Arrange the metals in order of reactivity stating with the most reactive.
(1mark)

(b) Concentrated sulphuric acid was added to R and the test tube gently
heated.
(i) State what was observed (1mark)


.
(iii) Write the equation for the reaction (1marks)


(c) Nitrogen compounds are pollutants. State two examples (1mark)


5. Magnesium burnt in air to form magnesium nitride.
(a) Write the equation for the reaction.
(1marks)


(b) Calculate the volume of air necessary to completely react with 6.0g of
magnesium if the percentage of nitrogen in air is 78 by volume.
(3marks)










..
..

6. Sulphur (IV) oxide can be prepared and collected in the laboratory by the
action of sulphuric acid on copper.
(a) (i) State the conditions for the reaction (1mark)


(ii) Write the equation for the reaction (1marks)


(b) Name one reagent that can be used for
(i) identifying sulphur (IV) oxide and ethene (1 mark)
..
(ii) distinguishing between sulphur (IV) oxide and ethene (1 mark)
..
(iii) State what would be observed if the reagent you have named in (a) (ii)
was treated with each ( mark @)
sulphur (IV) oxide .
and ethene.

7. During the manufacture of sodium hydroxide, a concentrated sodium


chloride solution (brine) is electrolyzed using mercury cathode as shown
in the diagram below.

(a) Identify the products collected at

(i) X................................................................................................ ( mark)

(ii) Y................................................................................................ ( mark)

(b) Write equation for the reaction leading to the formation of the product
collected at Y (1marks


(c) Name the substance used as the anode ( mark)

(d) State
(i) One industrial use of sodium hydroxide ( mark)

(ii) Write equation for the reaction when chlorine gas is bubbled through a
cold dilute solution of sodium hydroxide. (1 marks)

8. The figure below represents a section of the periodic table. Use it to


answer the questions below. The letters used are not the actual symbols of
the elements.

I VIII

II III IV V VI VII
U S R
P Q T

(a) Write the formula of a compound formed between pairs of elements and in
each case state the type bond formed.

(i) Q and S (1marks)



(ii) T and R (1marks)

(iv) State two physical properties of compound formed between T and R
(2marks)

....................
(b) Arrange the elements above in order of increasing size beginning with
smallest. (1 mark)

9. 1.00g of a metal carbonate XCO3 was dissolved in 50.00 cm3 of 1M hydrochloric


acid. The excess acid required 60.00 cm3 of 0.5M sodium hydroxide for
complete neutralization.
(a) Write an equation for the reaction between hydrochloric acid and the metal
carbonate. (1mark)


(b) Calculate the number of moles of
(i) excess hydrochloric acid. (1mark)

..

..

..
(ii) hydrochloric acid that reacted with the metal carbonate. (1mark)



..
(iii) XCO3 that reacted. (1 mark)

..

..
(iv) atomic mass X in XCO3. (1 mark)
[C = 12, O = 16 ]



10. The full symbol of the atom of an element is .


(i) Write the electronic configuration of element X (1mark)
..................................................................................................................
(ii) Draw a diagram to show how X and hydrogen forms a compound.
( H=1) (2marks)

(b) The hydride of element X conducts electricity in the molten state.


Write the equation for the reaction occurring at the
(i) anode. (1mark)

(ii) cathode. (1mark)


SECTION B
Attempt only two questions

11 Explain the following observations. Write equations where applicable.


(a) An aqueous solution of ammonium chloride is acidic. (4marks)

(b) When sodium hydroxide is heated with ammonium chloride a gas that
turns litmus blue is given off. (3marks)
(c) When sodium hydroxide is added drop-wise until in excess to zinc
sulphate solution, a white precipitate is formed. The precipitate
dissolves in excess sodium hydroxide solution to form a colourless
solution. (4marks)

(d) Diamond is a hard substance with a high melting point (4marks)

12(a)(i) Describe how you would obtain a sample of sugar crystals from sugar cane
(7marks)
(ii) State two uses of sugar in the world of the sick (2marks)

(b) Glucose, C6H12O6, can be converted to ethanol by a process known as


fermentation.
(i) Explain the term fermentation (2marks)
(ii) Write the equation that takes place during fermentation (1mark)
(iii) Briefly describe how ethanol produce is concentrated (2marks)

13(a) Using suitable examples explain what is meant by the term


(i) basicity of an acid. (1marks)
(ii) an acid salt. (1marks)

(c) Outline how a pure dry sample of sodium chloride can be prepared in the
laboratory. (6marks)

(c). An aqueous solution of lead (II) nitrate was added to a solution of sodium
chloride and the resultant product was heated.
(i) State what was observed. (1mark)
(ii) Write an ionic equation for the reaction that took place. (1marks)
(d) Iron (III) chloride dissolves in water according to the following equation:
FeCl3(s) + 3H2O (l) Fe(OH)3(s) +3 HCl (aq)
(i) State what would be observed when a solution of sodium hydrogen
carbonate is added to iron (III) chloride solution. (1mark)
(ii) Write an ionic equation for the reaction that took place. (1mark)

14(a). Hydrogen can react with nitrogen in the presence of a catalyst to produce
ammonia on an industrial scale.
(i). State the sources of hydrogen and nitrogen which are used in the reaction
(1
marks)
(ii). Name the catalyst used in the reaction. (1mark)
(iii). Write equation for the reaction leading to the formation of ammonia. (1marks)

(b). In order for maximum yield of ammonia to be achieved during the industrial
preparation of ammonia as stated in (a) the reaction is carried out at a low
temperature.
(i). Give a reason for carrying out the reaction at low temperature. (1mark)
(ii). State one condition for the reaction other than low temperature and use of the
catalyst. (1mark)
(iii). Briefly describe and explain how the condition you have stated in (b)(ii) affects
the reaction. (2marks)
(c). The ammonia obtained by the reaction in (a) above can be oxidized in the
presence of a catalyst to manufacture nitric acid.
(i). Name the catalyst which is used in the reaction. (1mark)
(ii). State two conditions for the oxidation of ammonia during the manufacture of
nitric acid other than the catalyst which you have named in (c) (i).
(iii). Using equation only show how nitric acid can be produced by the catalytic
oxidation of ammonia. (3marks)

(d). Name one ammonium compound that is commonly used in agricultural industry
and state the purpose for which the compound is used in agriculture. (1mark)

Name.............................................................. Index /No..................................

Signature........................................................................

SENIOR FOUR
MID TERM EXAMINATION 2012
CHEMISTRY PAPER II
Time: 2 hours.
Instructions:
Section A consists of 10 structured questions. Attempt all questions.
Section B consists of 4 semi-structured questions. Attempt any two any
questions n this section.

1 2 3 4 5 6 7 8 9 10 11 12 13 14 Total

Relative atomic masses; O=16, C=12, Na= 23, H=1, 0=16, S=32, P=31
Molar volume at room temperature 24dm3
Molar volume at standard temperature and pressure 22.4dm3

Attempt all questions

1(a) A teabag was each suspended separately in a beaker containing of hot water
and another containing cold water and allowed to stand for 10 minutes

State what was observed after 10 minutes (2 marks)

(b) Explain your observation (1 marks)

(c) Name the process that occurred (1mark)

2. The experiment below was set up and left to stand for 5days

(a) State what was observed after 5 days. (1 marks)

(b) Galvanization is one method used to prevent rusting. Explain why galvanized iron
does not rust. (1 marks)

(c) Write the


(i) name and formula of rust. (1mark)

(ii) the equation between rust and hydrogen gas (1mark )

3. The graphs below show the boiling curve of pure water and an aqueous
solution of sodium chloride A
Temperature (0C)

Time in minutes

(a) Define the term boiling point (1marks)

............................................................................................................................
............................................................................................................................
............................................................................................................................

(b) Which boiling curve represents an aqueous solution of sodium chloride? Explain
your answer (2marks)

............................................................................................................................
............................................................................................................................
............................................................................................................................
(c) State one other factor that affects the boiling point of a liquid (1mark)
...............................................................................................................................
(d) Boiling points are used to determine the criteria of purity of a liquid.
State one other method .................................................................. (1mark)

4. Study the diagram below and answer the questions that follow

(a)(i) State what is observed in the beaker (1mark)


.................................................................................................................................
.................................................................................................................................
(ii) Write the equation(s) for the reaction (2marks)
.................................................................................................................................
................................................................................................................................
.................................................................................................................................
(b) State the role of
(i) platinum wire (1mark)
..........................................................................................................................
(ii) the function of oxygen (1mark)
.................................................................................................................................

5. 3.1g of a carbonate XCO3 was heated to constant mass and 2.0g of the metal
oxide was formed.
(a) Write equation for the reactions. (1 marks)

.
(b) Calculate:
(i) the volume of carbon dioxide at stp produced when the carbonate is completely
decomposed (molar volume at stp is 22.4dm3) (2marks)
.................................................................................................................................
.................................................................................................................................
.................................................................................................................................
.................................................................................................................................
.................................................................................................................................
.................................................................................................................................
.................................................................................................................................
.................................................................................................................................
(ii) the atomic mass of X (0 = 16, C=12)

.................................................................................................................................
.................................................................................................................................
.................................................................................................................................
.................................................................................................................................
.................................................................................................................................
.................................................................................................................................
.................................................................................................................................
.................................................................................................................................
.
6.(a) (i) Name a substance which when reacted with sodium peroxide can produce
oxygen (mark)
.

(ii) Write an equation for the reaction leading to the formation of oxygen.
(1 marks)

(b) Write an equation for the reaction that takes place when
(i) Hydrogen is burnt in excess oxygen (1 marks)

..
(ii) Sodium is burnt in limited supply of oxygen (1 marks)

..

7(a) A hydrocarbon Y contains 85.7% carbon. Calculate the simplest


formula. (2marks)
.............................................................................................................................................
.............................................................................................................................................
.............................................................................................................................................
.............................................................................................................................................
.............................................................................................................................................
.............................................................................................................................................
.............................................................................................................................................
.............................................................................................................................................

(b) 0.224g of hydrocarbon Y occupied 96 cm3 at room temperature.


(i) Calculate its molecular mass (1mark)
.............................................................................................................................................
.............................................................................................................................................
.............................................................................................................................................
............................................................................................................................................

(ii) Hence its molecular formula


(1mark) .............................................................................................................................
.............................................................................................................................................
.............. ..............................................................................................................................
...............

8. State what is observed and in each case write ionic equation for the reaction.
(a) Chlorine gas is bubbled through a solution of Iron (II) chloride.
Observation (1mark)



Ionic equation (1 marks)



(b) When lead (II) nitrate solution is added to a solution of sodium iodide
Observation (1mark)



Ionic equation (1 marks)

9. 1.00g of pure ammonium chloride was boiled with 20.0cm 3 of a solution of


sodium hydroxide until evolution of ammonia had ceased. If the resulting
solution required 11.0 cm3 of 0.1M hydrochloric acid for neutralization.
(a). Write equation for the reaction that took place between ammonium chloride and
sodium hydroxide (1 marks)

(b). Calculate
(i) volume of ammonia evolved at stp
(N =14, Cl=35.5, H = 1) (2marks)








(ii) number of moles of sodium hydroxide that was in excess (1mark)




(iii) the molar concentration of sodium hydroxide. (1 marks)






.

10. The following chart below shows the contact process that is used to manufacture
sulphuric acid.

(a) Identify substances X and Y


X: .. ( mark)
Y: . ( mark)

(b)(i) Name the catalyst used in Chamber A. (1mark)


(ii) Write equation for the reaction taking place in Chamber A. (1mark)

...
(c) State what is observed when concentrated sulphuric acid is added to hydrated
(i) copper (II) sulphate. (1mark)

(ii) potassium chloride (mark)

SECTION B
Attempt any two questions in this section. Start each question on a fresh page.

11(a)(i) Describe how you would prepare pure crystals of lead (II) nitrate in the
laboratory.
(ii) Write the equation for the reaction. (7marks)

(b) Lead (II) nitrate crystals were heated strongly


(i) State what was observed. (2marks)
(ii) Write the equation for the reaction. (6marks)

(c) 0.15M hydrochloric acid was added to 25cm 3 of 0.1M lead (II) nitrate.
(i) State what was observed
(ii) Write an ionic equation for the reaction (2marks)
(iii) Calculate the volume of 0.15M hydrochloric acid needed to completely react
with the lead (II) nitrate solution. (2marks)

12. Explain the following observations. Write equations to explain your answer
where applicable.
(a) Sodium conducts electricity both the solid and liquid state whereas sodium
chloride does not conduct electricity in the solid state but conducts electricity in
the molten state. (4marks)

(b) A solution of hydrogen chloride gas in methyl benzene has no effect on dry
litmus paper but an aqueous solution of hydrogen chloride gas turns moist
litmus paper red. (3marks)

(c) When a jar of hydrogen sulphide is inverted over a jar containing moist sulphur
(IV) oxide a yellow solid is observed. (3marks)

(d) When sodium hydrogen carbonate is added to magnesium chloride


solution, there is no observable change. When the resultant solution
is heated a white precipitate is formed. (4marks)
13(a) With the help of equations, outline how a dry sample of hydrogen chloride gas
can be prepared in the laboratory staring from sodium chloride.
N.B. Diagram not required (4marks)

(b) Draw a labeled diagram of the set up of the apparatus to show that hydrogen
chloride gas very soluble in water (3marks)

(c) Hydrogen chloride gas reacts with iron wool.


(i) State the conditions for the reaction (1mark)
(ii) Write the equation for the reaction (1marks)
(iii) Calculate the volume of hydrogen chloride gas at room temperature required to
form 5.35g of the iron chloride. (2marks)
( Cl =35.5 Fe = 56 H=1)

(b). State what would be observed and write equation for the reaction that would
take place if aqueous hydrogen chloride was reacted with solid calcium
carbonate. (3marks)

14(a) Distinguish between empirical formula and molecular formula (2marks)

(b) 5.6g of a hydrocarbon X occupying 2.24 dm 3 at s.t.p. was heated in excess


oxygen; 17.6g of carbon dioxide was produced. Calculate
(i) the empirical formula of X (3marks)
(ii) the molecular formula of X (2marks)

a. Describe an experiment you would carry out in the laboratory to determine the
molecular formula of magnesium oxide.

Name.............................................................. Index /No..................................

Signature........................................................................
SENIOR FOUR
END OF TERM 1 EXAMINATION
APRIL 2014
CHEMISTRY PAPER II
Time: 2 hours.

Instructions:
Section A consists of 10 structured questions. Attempt all questions.
Section B consists of 4 semi-structured questions. Attempt any two any questions n this
section.

1 2 3 4 5 6 7 8 9 10 11 12 13 14 Total

Attempt all questions


1(a) Steel is an alloy of iron and is used widely than iron itself.
(i) Define the term alloy
(1mark)

---------------------------------------------------------------------------------------------------------------------
-------------- ---------------------------------------------------------------------------------------------------
--------------------------------- ---------------------------------------------------------------------------------
---------------------------------------------------(i) State two property of steel which is the reason
for its being used widely than iron

(2mark)
------------------------------------------------------------------------------------------------------------
------------------------ ------------------------------------------------------------------------------------------
------------------------------------------(b) State the constituents of each of the following
alloys
(i) Duralumin
(1mark)
------------------------------------------------------------------------------------------------------------
--- (ii) Solder
(1mark)
------------------------------------------------------------------------------------------------------------
---

2.(a) Graphite and lampblack are allotropes of carbon.


(i) Define the term allotropy
(1mark)
------------------------------------------------------------------------------------------------------------
------------------------ ------------------------------------------------------------------------------------------
------------------------------------------ ------------------------------------------------------------------------
-----------------------------------------------------------
(ii) State one physical difference between graphite and lampblack.
(1mark)
------------------------------------------------------------------------------------------------------------
------------------------ ------------------------------------------------------------------------------------------
------------------------------------------
(b) State one use of
(2marks)
(i) graphite
--------------------------------------------------------------------------------------------------
(ii) lamp black
-----------------------------------------------------------------------------------------------
(iii) Write an equation to show the incomplete combustion of lampblack
(1mark)
------------------------------------------------------------------------------------------------------------
---

3. Copper and aqueous copper (II) sulphate both conductors of electricity.


(a) Name the conducting species in
(i) Copper.
--------------------------------------------------------------------------------------------------(ii)
aqueous copper (II)
sulphate------------------------------------------------------------------------ ---------------------------
------------------------------------------------------------------ (1marks)
(b) Aqueous copper (II) sulphate solution was electrolyzed using the set up shown in
the diagram below.

(i) Describe what happens at each electrode after electrolysis.


(1marks)
------------------------------------------------------------------------------------------------------------
--- ------------------------------------------------------------------------------------------------------------
--- ------------------------------------------------------------------------------------------------------------
--- ------------------------------------------------------------------------------------------------------------
---
(ii) Write the equation for the reaction that took place at A
(1mark)
. .-----------------------------------------------------------------------------------------------------------
---
(iii) State two industrial applications of electrolysis
(1mark)
------------------------------------------------------------------------------------------------------------
--- ------------------------------------------------------------------------------------------------------------
---

4. Iron (II) sulphate 7 water and copper (II) sulphate 5 water are prepared in
the laboratory by action of sulphuric acid on iron and copper respectively.
(a) Name the conditions for the reaction of sulphuric acid and
(i) Iron
(mark)
. ---------------------------------------------------------------------------------------------------
---(ii) Copper
(mark)
---------------------------------------------------------------------------------------------------
---
(b). Write equation to show the reaction in which copper reacts with sulphuric
acid under the conditions stated to form copper (II) sulphate.
(1marks)
---------------------------------------------------------------------------------------------------
--- ---------------------------------------------------------------------------------------------------
---
(c) Write equations to show the effect of heat on the
(i) Iron (II) sulphate 7 water (1marks)
---------------------------------------------------------------------------------------------------
--- --------------------------------------------------------------------------------------------------
(ii) Copper (II) sulphate 5 water
(1mark)
---------------------------------------------------------------------------------------------------
---

5. The following experiment was set up to investigate the elements contained in a


candle

(a) State what was observed during the experiment. (1 marks)

---------------------------------------------------------------------------------------------------
--- --------------------------------------------------------------------------------------------------
(b)(i) From the observation in (a) what elements are present in a candle wax.
Explain your answer
(2marks)
---------------------------------------------------------------------------------------------------
--- ---------------------------------------------------------------------------------------------------
--- --------------------------------------------------------------------------------------------------
--------------------------------------------------------------------------------------------------
(ii) Write an equation to represent the combustion of a candle (1
marks)
---------------------------------------------------------------------------------------------------
---

6. Manganese (IV) oxide is used in the laboratory preparation of chlorine and


oxygen.
(a) Name one substance that when treated with manganese (IV) oxide can be used
in the laboratory preparation of
(i) Chlorine(
marks)

(ii) oxygen (
marks)

(a) Write equation to show the reaction in which manganese (IV) oxide together with
the substance that you have named in (a) produce
(i) Chlorine (1 marks)
----------------------------------------------------------------------------------------------------------------
----------------------------------------------------------------------------------------------------------------
----
(ii) Oxygen
(1marks)
---------------------------------------------------------------------------------------------------------------
---------------------------------------------------------------------------------------------------------------
----
(b) Write an equation for the oxidation of iron by oxygen (1mark)

-----------------------------------------------------------------------------------------------------------------

----------------------------------------------------------------------------------------------------------------

7(a). 20.0cm3 of a solution containing 6.3g per dm3 of a dibasic acid H2X. nH2O
required exactly 20.15cm3 of a 0.1M sodium hydroxide solution for complete
neutralization. (i). Calculate the
concentration of the acid in moles per dm3 of the solution. (3marks)
------------------------------------------------------------------------------------------------------------
-- ------------------------------------------------------------------------------------------------------------
-- ------------------------------------------------------------------------------------------------------------
-- ------------------------------------------------------------------------------------------------------------
-- ------------------------------------------------------------------------------------------------------------
-- ------------------------------------------------------------------------------------------------------------
-- ------------------------------------------------------------------------------------------------------------
-- ------------------------------------------------------------------------------------------------------------
--- ------------------------------------------------------------------------------------------------------------
---

(ii). Calculate the formula mass of the hydrated acid


(1mark)
--------------------------------------------------------------------------------------------------------------
------------------------------------------------------------------------------------------------------------
--
(iii) Deduce the value of n. (X=88) (1marks)

------------------------------------------------------------------------------------------------------------
--- ------------------------------------------------------------------------------------------------------------
--- ------------------------------------------------------------------------------------------------------------
---

8. The apparatus below shows combustion of ammonia in oxygen.

(a)(i) Name the reagents used to produce ammonia in the laboratory


(1mark)
------------------------------------------------------------------------------------------------------------
-
(ii) State the conditions for the reaction
(mark)
------------------------------------------------------------------------------------------------------------
---
(iii) Write the equation for the reaction
(1mark)
------------------------------------------------------------------------------------------------------------
---
(b) What is the role of cotton wool?
(mark)
------------------------------------------------------------------------------------------------------------
--- ------------------------------------------------------------------------------------------------------------
---
(c) Write the equation for the combustion of ammonia in oxygen
(1mark)
------------------------------------------------------------------------------------------------------------
--- ------------------------------------------------------------------------------------------------------------
---

9. Atom X has atomic mass 31 and 15 protons.


(a) State the
(i) Number of neutrons in one atom of X.
(1mark)
------------------------------------------------------------------------------------------------------------

(ii) Write the electronic configuration of X.


(1mark)
------------------------------------------------------------------------------------------------------------
---
(iii) Write the formula of the compound formed between X and chlorine
(atomic number of chlorine is 17).
(1mark)
------------------------------------------------------------------------------------------------------------
---
(iv) State the type of bond formed between X and chlorine.
(1mark)
------------------------------------------------------------------------------------------------------------
--- (v) State two property of the compound formed between X and chlorine.
(1mark)
---------------------------------------------------------------------------------------------------------------------
---

10. Study the table below and answer the following questions.

Substanc Melting boiling Solubility Electrical conductance Density at


e 0
point C point C 0
in water Solid Molten form room temp V=
A 714 1418 V none
form good 2.3 g/cm3 very
B 95 56 V none none 0.8 g/cm3
C 1083 2580 I good good 8.9 g/cm3
D 101 34 V none none 2.55 g/dm3
E 23 77 I none none 1.6 g/cm3
F 219 183 S none none 1.33 g/dm3
soluble; S= slightly soluble; I = insoluble
(a)(i) Name two substances that are liquids at room temperature.
(marks@)
-----------------------------------------------------------------------------------------------------------
--
(ii) Which of them is more volatile?
-------------------------------------------------------------------

(b) Which substance(s) would dissolve in water and can be separated from the
solution by (marks@)
(i) fractional distillation
----------------------------------------------------------------------------------
(ii) evaporation of the water
----------------------------------------------------------------------------

(c) Which substance A to F


(marks@)
(i) have a structure consisting of ions
----------------------------------------------------------------
(ii) is a
metal .---------------------------------------------------------------------------------------------
(iii) is a liquid which would form separate layer with water?
-------------------------------------
Would the water be above or
below? .------------------------------------------------------------

(d) Which substance is a gas that


(marks@)
(i) would not be collected effectively over water
--------------------------------------------------
(ii) would be collected effectively over water
--------------------------------------------------

SECTION B (30 MARKS)


Answer any two questions from this section

11(a) Describe how dry crystals of copper (II) nitrate can be prepared in the laboratory
starting from copper (II) oxide
(4marks)

(b) State what would be observed when an aqueous solution of copper (II) nitrate is
added
(i) sodium hydrogen carbonate solution (
mark)
(ii) zinc powder (1
mark)

(c)(i) Write ionic equations for the reactions that took place in (b)(i) and (b)(ii)
(2mark)
(ii) Explain the observation in(b)(i)
(2marks)

(b) Copper (II) nitrate crystals were heated strongly until no further change.
State what was observed and write the equation.
(3marks)

12(a)i).Define the term solubility of a solute


(2marks) (ii). State two factors that affect the solubility of a solute.
Explain how each fact affects solubility.
(4marks)
(b). The solubilities of a salt W at various temperatures are shown in the table below:
Temperature (o C) 0 20 40 60 80 100
Solubility(grams/100grams of solvent 4.0 5.0 12.0 21.5 39.5 60.0

Plot the graph of solubility of salt W against temperature.


(3marks)

(c). Use the graph in (b) to determine


(i). the solubilities of W and 15oC and 75oC.
(2marks)
(ii). the mass of W which would crystallize out if a solution of W was cooled from
750C to 150C
(1mark)

(d). State
(i) how you would distinguish between hand water and soft water. (1
marks)
(ii). one use of hard water. (
mark)

(e). Name one compound which when dissolved in water will cause.
(i). permanent hardness of water
(mark)
(ii). temporary hardness of water (
mark)

13. Explain each of the following observations. Write equations where appropriate

(a) Sodium metal conducts electricity in the solid state whereas sodium chloride
conducts electricity in the molten state.
(5marks)
(b) Hard water wastes soap.
(5marks)

(c) Sulphuric acid is not used to dry ammonia gas


(5marks)

(d) When a mixture of magnesium powder and copper (II) oxide are heated together
a mixture of a white and brown solids are formed.
(5marks)

14. In recent years water pollution has become a serious problem; some of the
main sources of pollution being sewage and use of detergents

(a) What is meant by the term sewage


(3marks)

(b) The flow diagram below shows what takes place in the sewage works.

Sedimentation Anaerobic Aeration tank


tank digester

Water Chlorination
bodies

(i) Describe briefly what takes place in the sedimentation tank.


(1mark)
(ii) State the conditions in the anaerobic digester. (1
marks)
(iii) Explain why the undigested sewage is mixed with air in the Aeration tank
(1
marks)
(iv) The effluent is chlorinated before released into water bodies. Explain.
(1mark)
(v) Name three uses of sewage work products.

(3marks)

(b) Define the term detergent


(1mark)
(ii) Name one commercial detergent
(1mark)
(iii) Explain how detergents pollute water
(2marks)

MARKING GUIDE TERM 2 2014


Que Answer mar
k
1(a) (i) An alloy is a uniform mixture of one metal with one or more substance usually a 5
metal or carbon. (1)
(ii) Steel is resistant to corrosion (1) stronger than pure iron (1)
EQUIVALENT; appearance, more malleability,

(i) Duralumin; aluminium, magnesium and copper (1)


(b)
(ii) Solder; lead and tin (1)

TOTAL 5
2(a) (i) Allotropy is the existence of an element in two or more forms due to different 1
arrangement of atoms in the same physical state. (1)

(ii) Graphite is crystalline () lampblack is amorphous () 1

(b) (i) Graphite; lubricant,(1) or pencil leads, or electrodes 2


(ii) lamp black; ink, (1) or shoe polish

(c) 1
2C(s)+O2(g) 2CO(g)
TOTAL 5
3(a)(i) Copper; electrons()
(ii) aqueous copper (II) sulphate; ions ()

(b) (i) at electrode A (+ve) the electrode dissolves ()and decreases in mass()
at electrode B (-ve) a brown coating ()and increases in mass()
(ii) Cu(s) Cu2+ (aq) + 2e (1)
(ii) purification of metals ()
electroplating ()
extraction of metals

TOTAL 5
4(a) (i) Iron; dilute ()sulphuric acid
ii) Copper; hot concentrated() sulphuric acid

(b) Cu(s) + 2H2SO4(aq) CuSO4(aq) + SO2(g)+2H2O(l) (1)

(c) (i) 2FeSO4.7H2O(s) Fe 2O3(s) + SO2(g) +SO3(g) + 14H2O(l) (1)


CuSO4.5H2O(s) CuO (s) +SO3(g) + 5H2O(l) (1)

TOTAL 5
5(a) A white precipitate () colourless liquid on the watch glass, () the level of
calcium hydroxide rose ()

(b) (i) Carbon ()because carbon dioxide ()is given off


Hydrogen() because water() is formed
CxHy + x + y/4 O2(g) xCO2(g)+ y/2 H2O(l) (1)

TOTAL 5
6(a) (i) hydrochloric acid ()
(ii) hydrogen peroxide ()

(b) (i) MnO2(s) + 4HCl(aq) MnCl2(aq) + 2H2O (i) + Cl2(g) (1)


(ii)2H2O2(aq) 2H2O (i) + O2(l) (1)

(c) 3Fe(s) + 2O2(g) Fe3O4(aq) (1)

TOTAL 5
3
7(a) (i) In 20.15 cm of sodium hydroxide there are (20.15 x 0.1)/1000 (1) 5
= 2.015 x 10-3
Reaction ratio acid to base 1: 2
In 20.00 cm3 of acid there are (2.015 x 10-3)/2 = 1.0075 x 10-3 (1)
In 1000cm3 of acid there are (1.0075 x 10-3 x1000)/20 (1)
= 0.05

(ii) 0.05 moles of acid are in 6.3 g()


1 mole of acid is in (6.3)/0.05 = 126 ()

(iii) H2X. nH2O =126 ()


2 + 88 + 18n = 126
n = (126 90)/18
=2 ()

TOTAL 5
8(a) (i) Ammonium chloride ()and calcium hydroxide ()
(ii) heat()
(iii) Ca(OH)2(s) + 2NH4Cl(s) CaCl 2(s) + 2NH3(g) + 2H2O(l) (1)

To spread the oxygen evenly ()


(b)
4NH3(g) + 3O2(g) 2N2(g) + 6H2O(l) (1)
(c)
TOTAL 5
9(a) (i) Number of neutrons in one atom of X.=16 (1)
(ii) 2:8:5 (1)
(iii) XCl3 (1)
(iv) Covalent (1)
(v) Has low mpt and bpt()
Insoluble in water () or EQUIVALENT
TOTAL 5
10(a (i) liquids at room temperature. = B (), E ()
) (ii) more volatile? = B ()

(i) fractional distillation = B ()


(b) (ii) evaporation of the water = A ()

(i) have a structure consisting of ions =A ()


(c) (ii) is a metal = C ()
(iii) is a liquid which would form separate layer with water=E ()
Would the water be above or below= ABOVE ()

(i) would not be collected effectively over water D ()


(d) (ii) would be collected effectively over water F ()

TOTAL 5
10(a o Add small spatula ends full of copper (II) oxide powder to dilute nitric acid () in 6
) a beaker until the oxide can dissolve no more.()
o Warm the mixture to increase the rate of reaction.
o Filter () the mixture to remove the unreacted copper (II) oxide powder.
o Place the filtrate in an evaporating basin and heat () until the solution is
saturated. ()
o Allow the solution to cool () and crystallise. ()
o Filter off the crystals () wash () in little water () and dry () between filter
papers.
o CuO(s) +2HNO3(aq) Cu(NO3)2(aq) + H2O(l) (1)

(i) A green precipitate ()


(ii) A brown precipitate, () solution changes from blue () to colourless ()

(b) 2
(i) Cu2+(aq) + 2HCO3-(aq) CuCO3(s) + CO2(g) +H2O(l) (1)
2+
(ii) Cu (aq) + Zn(s) Cu (s) + Zn2+ (1)
(iii) The green precipitate is copper (II) carbonate ()that is insoluble()
(c) 3

The blue ()crystals turn to black, ()a brown gas given off(),( a colourless liquid
given off)( three observations)
2Cu(NO3)2(s) 2CuO (s) + 4NO2 (g) + O2(g) (1)
(d) 3

TOTAL 15
12(a (i) No of grams of a solute that dissolves in 100g of solvent to form a saturated 2
) solution (until no more solute can dissolve) at that temperature. (2)
(ii) Factors : temperature(1) nature of solvent (1)
Explanation: as temperature rises solubility off most solutes increase(1) 4
(iii) the higher the temperature the higher the solubility (1)
Covalent solutes dissolve in organic solvents
Ionic solutes dissolve in polar solvents(1)

labelled axes ( 1) scale (1) shape (1)

(b) 70
3
60

50

40
Solubility(grams/100gra
solubility g/100g solvent ms of solvent
30 Exponential
(Solubility(grams/100gra
ms of solvent )
20

10

0
0 100 200

Temperature
(c)

(d)

Solubilit
y at 150C 4.75g (1)
0
At 75 C 32.5 g (1)
(ii) mass that crystallises out 32.5 4.75 = 27.75g (1)

(i) add soap solution(); soft water lathers easily(), hard water does not lather
easily()
(ii) brewing beer()

(e)(i) permanent hardness ... calcium sulphate()


Temporary hardness ... calcium hydrogen carbonate()

TOTAL 15
13(a Sodium metal has delocalized () electrons () that are responsible for carrying the 3
) current.
In the solid state sodium chloride have ions () that are held together by strong
electrostatic forces, ()the ions are not free to move to carry the current.
In the molten state the forces of attraction between the ions are weakened (), and
the ions are mobile () and free to can carry the current.

Hard water contains calcium (1) or magnesium ions. On addition of soap, scum ()is
(b) formed which is an insoluble salt ()of magnesium or calcium stearate as follows: 4
2+ -
Ca (aq) + 2St (aq) Ca St2(s) (1)
It is only when all the calcium and magnesium ions have been removed() from water
that a lather will form()

Sulphuric acid is an acid ()ammonia is alkaline ()and the two react forming a salt() 3
(c) H2SO4(aq) + 2NH3(g) (NH4)2SO4(aq) (1)

Magnesium is higher in the reactivity series than copper(1), it reduces (1), copper
(d) (II) oxide to copper (),; the brown solid and itself is oxidised (1), to magnesium
oxide (),; the white solid. 5
Mg(s) + CuO(s) MgO (s) + Cu(s) (1)
TOTAL 15
14(a Waste water refuse containing human excreta and other waste (1), from factories, 3
) gutters, toilets, ,bathrooms and kitchens (mentions at least two sources) (1) that
enter sewers() and flow to sewage works ()

(i) Sedimentation; involves the clustering together of suspended particles into big
(b) lumps () which sink and settle at the bottom. Potassium aluminium sulphate 1
(Alum) (1)is added
(ii) Anaerobic digester; there is no oxygen here () appropriate temperature () is 1
and right pH() are required
(iii) Aeration tank; To enable the an aerobic micro organism grows here () and 1
breaks down any remaining organic material()
(iv) Chlorination; To kill germs(1)
1

Uses of sewage by products;


Sludge used as fertilizers (1)
(c) 3
Making tar for road constructions (1)
Methane gas used as a fuel (1)

(i) A detergent is a sodium or potassium salt of a long chained sulphonate or


4
hydrogen sulphate (1)
(d)
(ii) (ii) commercial detergent OMO (1) or EQUIVALENT

(iv) detergents contain phosphates (1) that promote the growth of algae that
deprive oxygen (1)from aquatic life
TOTAL 15
Name.............................................................. Index /No..................................
Signature........................................................................

SENIOR FOUR
MID TERM EXAMINATION
MARCH 2014
CHEMISTRY PAPER II
Time: 2 hours.

Instructions:
Section A consists of 10 structured questions. Attempt all questions.
Section B consists of 4 semi-structured questions. Attempt any two any questions n this
section.

1 2 3 4 5 6 7 8 9 10 11 12 13 14 Total
Attempt all questions
1 Air is a mixture of gases
(a) State
(i) two reasons why air is regarded as a mixture and not a compound.
(1mark)

.
(ii) the method by which the major components of air are separated industrially.
(1mark)

.
(iii) Give a reason for your answer in (a)(ii)
(marks)

.
(b) Write equation to show the reaction that take place between the most abundant
components of air with magnesium. (1
marks)

..
(c) Name one major pollutant of the atmosphere and state its effect
(1mark)
Pollutant.
Effect

2(a)(i) State what is observed when sodium carbonate solution is added to copper (II)
sulphate solution.
(mark)

(ii) Write an ionic equation for the reaction


(1marks)

(b)(i) State what was observed when dilute nitric acid was added to to the product of
the raction in (b)(i)
(1marks)

(ii) Write an equation for the reaction.


(1mark)

3. The table below shows the physical properties of elements W,X,Y and Z.
Subastance Melting point 0C boiling point 0C Solubility in water Density at 250C
W -117 78.5 very soluble 0.8
X -78 -33 very soluble 0.77 x 10-3
Y -23 77 insoluble 0.16
Z -219 -183 slightly soluble 1.33 x 10-3

(a) (i)Identify the substance that would dissolve in water and could be separated from
solution by fractional distillation. .
(1mark)
(ii) Give a reason for you answer.
(1mark)


(b) Which substances is a liquid at room temperature and when mixed with
water forms two separate layers?
(1mark)

Identify a substances that is a gas at room temperature and when can be


collected;
(i) over water ..
(1mark)
(ii) by downwards displacement of air ( density of air is1.29 x 10 -3g cm-3 at room
temperature) .
(1mark)

4. A gaseous hydrocarbon P which is an alkene was analyzed.


(a) 0.224g of this gas occupied 96 cm 3 at room temperature and pressure
Calculate the
(i) relative molecular mass of P(molar volume is 24 dm3)
(2marks)
...

(ii) molecular formula of P


(1mark)

(b) Write down the 2 possible structural formulae of P and give their systematic
names
(2marks)
...

...

..

..

5(a) Name the reagents that are neded to prepare sodium hydrogen carbonate and
write the equation for the reaction.
(1mark)

Reagents..

.......
Equation
(1marks)

...
...
(b) Iron (III) chloride dissolves in water according to the following equation :
FeCl3(s)+3H2O(l) Fe(OH)3(s)+3HCl(aq)
(i) State what is observed when a solution of iron (III) chloride is added to sodium
hydrogen carbonate solution.
(1mark)



(ii) Explain your answer.
(1mark)

.


..
6. Part of the Periodic Table is shown below, with some elements labelled Q to Z.
The letters are not the usual symbols of these elements.
I II II IV V VI VII VII
Q W Z
X Y
R T

(a) State which one of the elements is;


(i) the most reactive metal
...
(mark)
(ii) the most reactive non-metal
..
(mark)
(iii) A monoatomic gaseous element
... ..
(mark)
(b) Write the formula of a compound formed between pairs of elements and in each
case state the type bond formed.
(i) T and W
(1mark)


(ii) W and X
(1mark)




(c) Arrange the elements above in order of increasing size beginning with smallest.


(1mark)

7. (a) Define the term oxidation in terms of electron.


(mark)



..

(b) Write the equation of the following half equations and in each case state whether
the reaction is reduction or oxidation reaction.
(i) The conversion of hydrogen ions (H+) to hydrogen molecules (H2)
(1mark)
............................................................................................................................
...........................................................................................................................
(ii) The conversion of iron (II) ions (Fe2+) to iron (III) ions (Fe3+)
(1mark)
............................................................................................................................
...........................................................................................................................
(iii) The conversion of chlorine molecules to chloride ions (Cl --)
(1mark)
......................................................................................................................
...... ......................................................................................................................
....
.8. Name one reagent that can be used to distinguish between each of the following
pairs of ions. In each case state what would be observed when the reagent was
treated separately with each pairs of ions.
(a). chloride and iodide ions.
(i). reagent
(1mark)
.......
.....

(ii). Observation
(1mark)

Chloride

iodide

(b). hydrogen carbonate and carbonate


(i). reagent (1
mark)
.......
.....

(ii). Observation
(1mark)
Hydrogen
carbonate.

Carbonate

(c) Sulphate and sulphite


(i). reagent (1
mark)
.......
.....

(ii). Observation
(1mark)

Sulphate

Sulphite

9. The diagram below shows the set up of apparatus that was used to investigate
the effect of hot platinum on a mixture of sulphur dioxide and oxygen.
(a)(i) State what was observed. (1mark)

..
(ii). Give a reason (s) for your observation in (a) (i).
(1mark)

..
(iii). Write equation for the reaction that took place. (1mark
)

(b) (i).State the purpose of using the hydrogen peroxide mixed with manganese (IV)
oxide.
(1mark)

(ii). Write equation of reaction to illustrate your answer in (b) (i).


(1mark)

10. 1.00g of pure ammonium chloride was boiled with 20.0cm 3 of a solution of
sodium hydroxide until evolution of ammonia had ceased. If the resulting
solution required 11.0 cm3 of 0.1M hydrochloric acid for neutralization.
(a). Write equation for the reaction that took place between ammonium chloride and
sodium hydroxide (1 marks)


(b). Calculate
(i) Volume of ammonia evolved at stp
(N =14, Cl=35.5, H = 1) (2marks)







(ii) number of moles of sodium hydroxide that was in excess (1mark)







(iii) the molar concentration of sodium hydroxide. (1 marks)




.

SECTION B
Attempt any two questions in this section. Start each question on a fresh page.
11. Soap is prepared in laboratory by reacting vegetable oil or fat and an alkali
(a) State
(i) the process by which soap is prepared.
(1mark) (ii) the differences between fats and oils
(2marks)
(iii) one source each of a fat and oil
(1mark )

(c) Starting from the named oil or fat; describe how soap can be prepared in
laboratory.
(5marks)
(d) Explain the following; illustrate your answer using equations.
When soap was added to aqueous calcium hydrogen carbonate solution; a white
precipitate was formed. But when the soap solution was added to aqueous
calcium hydrogen carbonate solution that had been boiled no precipitate way
formed.
(6marks)

12(a)(i) With the aid of equations, outline how a dry sample of hydrogen chloride gas
can be prepared in the laboratory staring from sodium chloride.
N.B. Diagram not required
(5marks)
(ii) State two uses of hydrochloric acid.
(1mark)

(b) Explain the following


When ammonia gas is bubbled through a solution of hydrogen chloride gas in
methyl benzene a white precipitate is formed but when ammonia gas is bubbled
through an aqueous solution of hydrogen chloride there is no observable change.
(4marks)

(c) Hydrogen chloride gas reacts with iron wool.


iState the conditions for the reaction (1mark)
ii Write the equation for the reaction
(1marks)
iii Calculate the volume of hydrogen chloride gas at room temperature required to
form 5.35g of the iron chloride.
(2marks) ( Cl =35.5 Fe = 56 H=1)
.
13(a) Crude oil is a mixture of various hydrocarbons.
(i) What is a hydrocarbon?
(1mark) (ii) Name the method used to
separate the hydrocarbons in petroleum and explain the principle used.
(2mark)

(b) The hydrocarbons undergo a process called catalytic cracking.

(i) Explain the term catalytic cracking


(2marks) (ii) State one the importance of cracking
(1mark)

(c) Ethene is prepared in the laboratory by the dehydration of ethanol.


(i) Briefly describe and explain how ethanol is prepared from a named local material

(4marks)
(ii) Name the reagent used to dehydrate ethanol
(1mark) (iii) State the conditions for the reaction
(1mark)
(iv) Write the equation for the dehydration of ethanol.
(1mark)

(d) Ethene is a starting material of polyethene.


(i) State two uses of polyethene
(1mark) (ii) Polyethene is an environmental problem. Explain
(2marks)

14(a). Hydrogen can react with nitrogen in the presence of a catalyst to produce
ammonia on an industrial scale.
(i). State the sources of hydrogen and nitrogen which are used in the reaction
(1marks)
(ii). Name the catalyst used in the reaction.
(1mark)
(iii). Write equation for the reaction leading to the formation of ammonia. (1
marks)

(b). In order for maximum yield of ammonia to be achieved during the industrial
preparation of ammonia as stated in (a) the reaction is carried out at a low
temperature.
(i). Give a reason for carrying out the reaction at low temperature.
(1mark)
(ii). State one condition for the reaction other than low temperature and use of the
catalyst
(1mark)
(iii). Briefly describe and explain how the condition you have stated in (b)(ii) affects
the reaction.
(2marks)

(c). The ammonia obtained by the reaction in (a) above can be oxidized in the
presence of a catalyst to manufacture nitric acid.
(i). Name the catalyst which is used in the reaction.
(1mark)
(ii). State two conditions for the oxidation of ammonia during the manufacture of
nitric acid other than the catalyst which you have named in (c) (i).
(1mark)
(iii). Using equation only show how nitric acid can be produced by the catalytic
oxidation of ammonia.
(3marks)

(d). Name one ammonium compound that is commonly used in agricultural industry
and state the purpose for which the compound is used in agriculture.
(2mark)

Marking Guide Form 4 Mid term 1 2014

Que. Answer marks


1(a) (i)Air is a mixture 1
o it can be separated by physical methods ()
o its composition vary from place to place ()
o on mixing the components of air there is no heat given out or absorbed
o the properties of air is the average propertied of all the components in air
or EQUIVALENT

(ii)fractional distillation of liquid air (1)


1
(iii) the components of air have different boiling points. ()

(b) 1
3Mg(s) + N2(g) Mg3N2(s) (1)

(c) 1
Pollutant; carbon dioxide ()
Effect ; global warming () or EQUIVALENT

TOTAL 5
2.(a) (i) A green precipitate() is formed

(ii) Cu2+(aq)+ CO32-(aq) Cu CO3(s) (1)

(b) (i) the green solid dissolves () with effervescence () to form a green/blue
solution ()

(ii) CuCO3(s) + 2HNO3(aq) Cu(NO3)2(aq) + CO2(g) + H2O (l) (1)

TOTAL 5
3(a) (i) W (1)
(iii) Difference boiling points(1)

(b) (i) Y forms two separate layers with water(1)


(ii) Z collected over water(1)
(iii) X downward displacement of air(1)

TOTAL 5
3
4.(a) (i) 24,000 cm of a gas contain 1mole 2
96 cm3 of a gas contain (1 x 96)/ 24,000 = 0.004 moles ()
0.004 moles are contained in 0.224 g of alkene ()
1 mole will contain 0.224/0.004 () = 56 ()
(iii) CnH2n = 56
14n = 56 n=4
C4H8 ()
1

(b)

TOTAL 5
5(a) Reagents ; sodium hydroxide solution () and carbon dioxide () 1

Equation; CO2(g) + NaOH(aq) NaHCO3(s) (1)


1

(i) Bubbles () of a colourless gas () and (brown solid)


(b) 1

(ii) Iron (III) chloride undergoes hydrolysis forming hydrochloric acid () which
decomposes () the hydrogen carbonate into carbon dioxide; the colourless
1
gas()

TOTAL 5
6.(a) (i) the most reactive metal is R () 1
(ii) the most reactive non-metal Y ()
(iii) A monoatomic gaseous element Z ()
(b) 2

Z, W, Q, Y, X, T, R (1)
(c) 1
TOTAL 4
7(a) (i) oxidation is loss of electrons ()
(i) The conversion of hydrogen ions (H+) to hydrogen molecules (H2)
(b) 2H+(aq) + 2e H2(g) (1) reduction ()

(ii (ii) The conversion of iron (II) ions (Fe2+) to iron (III) ions (Fe3+)
Fe2+(aq) Fe3+(aq)+e (1) oxidation ()

(iii) The conversion of chlorine (Cl2) molecules to chloride ions(Cl--)


Cl2(aq) + 2e Cl - (1) reduction ()
TOTAL 5
8.(a) (i) the wire continues to glow () and dense white fumes () 5
(ii) the reaction is exothermic () and sulphur dioxide undergoes oxidation ()
2SO(g)+O2(g) 2SO3(g) (1)

(b) (i) Hydrogen peroxide is decomposed to oxygen() in the presence of a catalyst()


2H2O2(aq) O2(g)+ 2H2O (l) (1)

TOTAL 5
9.(a) chloride and iodide ions. 2
(i) reagent; lead (II) nitrate solution (1)
(ii) Observation
Chloride white precipitate()
Iodide yellow precipitate()

hydrogen carbonate and carbonate


(b) (i) reagent magnesium sulphate solution (1)
(ii) Observation
Hydrogen carbonate; no observable change() 2
Carbonate white precipitate()
sulphate and sulphite
(i). reagent ; dilute nitric acid (1)
(c) (ii). Observation
Sulphate, no observable change()
Sulphite; bubbles of a colourless gas. ()
2
TOTAL 6
10(a) NaOH(s) + NH4 Cl(s) Na Cl(s)+ NH3(g) +H2O(g) (1)

(b) (i)Rfm of NH4 Cl =14+4+35.5 =53.5()


53.5g of ammonium chloride contain 1 mole
1g of ammonium chloride contain 1/53.5 = 0.0187 moles()
Reaction ratio of NH4Cl : NH3 is 1:1
No of moles of ammonia produced is 0.0187 moles()
1mole of ammonia occupy 22.4dm3
0.0187moles of ammonia 22.4 x 0.0187 = 0.4187 dm3()

(ii) In 1000cm3 of hydrochloric acid there are 0.1 moles


In 11.0cm3 of hydrochloric acid there are(0.1 X 11.0)/1000 =1.1 x 10 -3moles()
Reaction ratio of HCl : NaOH is 1:1
no of moles of sodium hydroxide that was in excess = 1.1 x 10 -3 ()

(iii) total number of moles of sodium hydroxide in 20 cm 3 = 1.1 x 10-3 +0.0187 ()


= 0.0198
3
In 1000cm there are (0.0198 x 1000)/20 = 0.99 ()
The molarity of sodium hydroxide is 0.99M()
TOTAL 5
11(a) (i) saponification (1) 4
(ii) Fats are solids () at room temperature whereas oils are liquids.()
Fats are saturated organic compounds () oils are unsaturated ()
ii Fats; pigs, () oil; simsim seeds ()

(b) o Vegetable oil and dilute sodium hydroxide (1)are boiled()together in a beaker
using a low Bunsen flame()
o The mixture is stirred continuously () 5
o The mixture is shaken with water to see whether it forms lather. ()
o When the mixture begins to form lather, a saturated solution of sodium chloride (1)
is added to precipitate () the soap.(separate the soap from the reaction mixture)
o The soap is removed from the reaction mixture (), additives added and shaped.

Soap forms an insoluble salt with calcium ions ().


(c)
The white precipitate formed is scum ()or calcium stearate
Ca 2+(aq)+ 2St-(aq) Ca (St-)2(aq) (1)
On boiling the aqueous solution the calcium hydrogen carbonate decomposes () to give
insoluble () calcium carbonate (), thus the calcium ions are removed () from the
6
aqueous solution
Ca(HCO3)2(aq) CaCO3 (s) + CO2(g)+H2O(l) (1)

TOTAL 15
12(a) o Concentrated sulphuric acid (1) is placed in a funnel since it is a liquid.
o Solid () sodium chloride is placed in a flask (and the mixture is warmed gently.)
o The gas is passed through a wash bottle containing concentrated sulphuric acid ()
to dry it ()
o The gas is denser than air () and is collected by down delivery. ()
o H2SO4 (aq) + NaCl(s) NaHSO4(s) + HCl(g) (1)

o In methyl benzene hydrogen chloride reacts with ammonia to form ammonium


(b)
chloride (1) the white precipitate. Ammonium chloride is ionic (1) and does not
dissolve in organic solvents (1) like methyl benzene.
o An aqueous solution of hydrogen chloride reacts with ammonia forming ammonium
chloride which is ionic and dissolves in polar solvents (1) like water

o dry hydrogen chloride gas () and heat()


o Fe(s)+ 2HCl(g) FeCl2(s)+ H2(g) (1)
(c) o Rfm of FeCl2 =56 +(35.5x2)=127 ()
No of moles of iron (II) chloride =5.35/127 ()
Reaction ratio FeCl2:HCl is 1: 2
No of moles of hydrogen chloride that needed = (2 x 5.35/127) ()
1 mole of gas occupies 24 dm3
(2 x 5.35/127) moles occupy 24 x(2 x 5.35/127) ()
3
=2.022dm ()
o Used for reomving rust fro iron (descaling) ()
o Prickling metals before they are elctroplated. ()
o Manufacuter of PVC
o Used in preparation of soluble chloride e.g. calcium chloride OR EQUIVALENT
(d)
TOTAL 15
13.(a) (i) A hydrocarbon is a compound 3
that contains hydrogen and
oxygen only. (1)

(b) (ii) fractional distillation (1)


difference in boiling points (1) 3

(i) catalytic cracking: long


chained hydrocarbon() are
(c) heated in the presence of a
catalyst ()and lower
temperature ()into shorter
chains of petrol and alkenes.()

(ii) cracking gives high quality


petrol (1)
7
(i) Preparation of ethanol:
o Raw bananas () or millet
flour contain starch() which
undergoes hydrolysis() and
is converted to glucose
(sugar) ()
(d)
o In the presence of
2
germinated millet () or
sorghum which contains an
enzyme () the glucose
ferments and is broken down
into ethanol () and carbon
dioxide
o The crude mixture is distilled
()to obtain alcohol
(ii) sulphuric acid (1)

(iii) the acid must be


concentrated, ()temperature of
1800C, () or excess acid

(iv) CH3CH2OH Conc H2SO4


CH2 CH2 + H2O(l) (1)
170 oC-
180oC
(i) Carrier bags () electric
cable insulators () crates of
soda or milk toys
(ii) Polyethene is not
biodegradable (1)and when burnt
they produce toxic gases (1)

TOTAL 15
14(a) (i) Nitrogen is obtained from air () by fractional distillation of liquid air. Hydrogen 4
is obtained from water gas(1)
(ii) Catalyst used is finely ground iron (1)

(iii) N2(g) + 3H2(g) 2NH3(g) (1)

(b) (i) The reaction is exothermic (1)


(ii) High pressure (1) 4
(iii) High pressures increase the yield of ammonia () because the reactant
molecules are brought close together ()(concentration of the reactants
increases) the reaction is faster()
The production of ammonia is accompanied with a decrease in volume or occupy
less volume in comparison to the reactants. () the forward reaction is favored

(i) Catalyst platinum (1)


(c)
(ii) Heat () (at 7000C) and air ()
5
(iii) 4NH3(g) + 5O2(g) 4NO(g)+6H2O(l) (1)
2 NO(g) + O2(g) 2NO2(g) (1)
4NO2(g) + 2H2O(l) + O2(g) 4HNO3(aq) (1)
(d) (iv) Ammonium nitrate () increase crop yield () or decreases the soils alkalinity
2

TOTAL 15

Name: ................................................................Centre/Index No. ............./.......


Signature..............................................................................................................

545/2
Chemistry
Paper 2
June/July 2015
2 hours.

UGANDA CERTIFICATE OF EDUCATION


CHEMISTRY
PAPER 2
2 HOURS

INSTRUCTIONS TO CANDIDATES:
Section A consists of 10 structured questions. Attempt all questions in this
section. Answers to these questions must be written in the spaces provided.
Section B consists of 4 semi-structured questions. Attempt any two questions
from this section. Answers to the questions must be written in the answer
booklets provided.
In both sections, all working must be shown clearly.

1 2 3 4 5 6 7 8 9 10 11 12 13 14 Total

SECTION A (50 marks)


Attempt all questions in this section
1. Complete the table below by indicating the correct method of separating of
each of the five mixtures. (1mark@)

MIXTURE METHOD OF SEPARATION


(a) Iodine + sodium chloride
(b) Palm oil + water
(c) Potassium chloride + potassium nitrate
(d) Sodium chloride + sand
(e) Green chlorophyll + yellow chlorophyll

2(a) Complete the equations below, showing the effect of heat. (4marks)

(i) NaNO3(s)
(ii) AgNO3 (s)
(iii) Ca(NO3)2 (s)

(b) The equation below shows the reaction that should take place if potassium
carbonate was heated.

K2CO3(s) K2O(s) + CO2(s)

(i) Comment on the equation.


(1mark)


..
(ii) Give a reason for your answer (
mark)

3(a) Twoequallengths of burning magnesium ribbons were introduces


separately into a gas jar; one containing nitrogen and another oxygen.
(i) State the jar in which the burning of magnesium ribbon took a shorter
time to be completed. (1mark)

(ii) Give a reason for your answer in (i) (1mark)


(b) Write equation for the reaction that took place if water was added onto
the product in the gas jar containing; (1marks @)
(i) Oxygen


(ii) Nitrogen

..
(c) Universal indicator was added to the solution in each of the gas jars.
State what was observed. (1mark)
..

4. Part of the Periodic Table is shown below indicating positions of some


elements
I II III IV V VI VII VIII
Na Mg Al Si P S
Ca

The basic strength of the oxides of the elements shown in the Periodic
Table increase in the order
Na2O CaOMgO Al2O3 SiO2 P2O3 SO2
(a) State the trend in the metallic character of the elements
(i) Across the Periodic Table. (1mark)
.
(ii) Down the group (1mark)

(b)(i) Identify the oxide which is amphoteric. (1mark)

(ii) Write equation for the reaction of the oxide you have identified in (b)(i) with
nitric acid. (1mark)

5.(a) Write equation for the reaction that would take place if, over strongly
heated iron filings was passed a current of
(i) dry hydrogen chloride (1mark)



(iii) dry chlorine gas (1mark)

..
(b)(i) State what would be done in order to convert the product in reaction (a)(i)
in its aqueous solution to the product in reaction (a)(ii) (mark)



(iii) Write equation to illustrate your answer in (b)(i) (1mark)

..
(iii) Name one reagent that would be used to confirm that the reaction in (a)(ii)
had taken place. (mark)


State what would be observed if the reagent you have named in (b)(iii) was
treated with the product of the reaction in (b)(ii) (1mark)

6. The diagram below shows the set up of apparatus that was used to make
an electrochemical cell for comparing the reactivities of copper and silver.
(a) Draw an arrow on the diagram to show the direction in which electrons are
flowing.(mark)
(b)(i) Name the substance X(mark)

(iii) Identify on e substance that was used to make the electrolyte containing
silver ions.
(1mark)
(c)write equations for the reaction taking place at
(i) The copper electrode
(1mark)

.
(ii) The electrode labeled X
(1mark)

.
7.(a) Name one reagent that can be used to distinguish between;
(i) Zn2+(aq) andPb2+(aq) (mark)

.
(ii) SO42-(aq) andCO32-(aq)
(1mark)

.
(b) State what would be observed if the reagent you have named in (a) was used to
separately treat (1mark)
2+ 2+
(i) Zn (aq) and Pb (aq)

(2mark)

(ii) SO42-(aq) and CO32-(aq)


(2mark)

8.(a) Define the term standard solution.


(1mark)

(b) A solution was made by transferring exactly 20.0 cm 3 of 1M sulphuric acid into a
250cm3 volumetric flask, then carefully adding distilled water to bring the volume of
thenew solution to the mark. Determine the molar concentration of the new solution
with respect to hydrogen ions. (3marks)

9. The atomic numbers of elements X and Y are 17 and 20 respectively.


(a) Write the electronic configuration of
(i) X .
(1mark)
(ii) Y
(1mark)
(b)State ;
(i) the group in the Periodic Table to which X belongs
(mark)


(ii) the period in the Periodic Table to which Y belongs
(mark)

..
(c) X reacts with Y to form a compound Z
(i) State whether at room temperature, Z would be a gas or a volatile liquid or
a solid with a high melting point or a solid with a low melting point
(mark)


(ii) Give a reason for your answer in (c)(i)
(mark)


.

10. When 40.0 cm3 of a 2M hydrochloric acid was mixed with 40.0 cm3 of a
2M sodium hydroxide both at initial temperature of 25.80C the temperature
of
the solution rose to t0C. Determine t
(specific heat capacity of water =4.2Jg-1 0C-1, density of water=1g cm-1 and
enthalpy of neutralization of hydrochloric acid by sodium hydroxide
=56.5kJ mol-1)
(4marks)

SECTION B (30 MARKS)


Answer any two questions only in this section
11(a) Sodium ethanoate dissolves in water forming a solution which contains both
ethanoic acid and sodium hydroxide according to the equation.
CH3COONa(s) + H2O(l) CH3COOH(aq) + NaOH(aq)
When tested; an aqueous sodium ethanoate solution turned red litmus paper
blue. Explain the observation.
(3marks)

(b)(i) State two reactions which can be used to show that water contains hydrogen.

(2marks)
(iii) Write the equation to illustrate your statements in (b)(i)
(3marks)
(iv) Write an ionic equation for the reaction leading to the production of hydrogen
starting from zinc metal. (1marks)

(d) State what would be observed and write equation for the reaction that would take
place if
(i) Dry hydrogen was passes over heated copper (II) oxide (3marks)
(iii) Burning sodium was lowered into a gas jar containing pure dry hydrogen.
(2marks)

12(a) Describe how a sample of dry chlorine can be prepared in the laboratory starting
from potassium manganate (VII). (No diagram is required, but youranswer should
include conditions, equation, the drying agent and method of collection).
(5marks)

(b) Moist blue litmus paper was introduced into a test tube containing dry chlorine
State
(i) what was observed. (1mark)
(ii) the industrial application of the reaction in (b) (i)
(1mark)

(c) Write the equations for the reaction that would take place if chlorine was bubbled
through
(i) a dilute solution of potassium hydroxide. (1marks)
(ii) aqueous sodium bromide. (1marks)

(d) A colourless gas, T bubbles out of aqueous solution of chlorine that was left
exposed to sunlight.
(i) Identify T
(1mark)
(ii) Explain how T is produced and write equation to illustrate your explanation
(2marks)
(e) Write an equation to show the reaction that takes place when phosphorous
is
treated with chlorine.
(1mark)

13(a)(i) State how sulphur dioxide which is used for the production of sulphuric
acid
is obtained. (
mark)
(ii) Write the equation for the reaction that leads to the formation of sulphur
dioxide described in (a) (i) (1mark)
(iii) Name one reagent that can be used to confirm the presence of sulphur
dioxide and state what would be observed if sulphur dioxide was treated
with the reagent.
(2marks)
(iv) Give a reason for your observation in (a) (iii)
(1mark)

(b) In the contact process, sulphur dioxide is converted to sulphur trioxide


using a
catalyst at a relatively low temperature.
(i) Name the catalyst used. (
mark)
(ii) Write the equation for the conversion of sulphur dioxide to sulphur trioxide
(1mark)
(iii) Give a reason why the converted of sulphur dioxide to sulphur trioxide is
done at a low temperature in the contact process.
(1mark)

(c) One of the uses of sulphuric acid as a laboratory reagent is preparation of


ethene from ethanol. State
(i) The conditions under which sulphuric acid reacts with ethanol to produce
ethene.
(1mark)
(ii) The property of sulphuric acid which enables formation of ethene from
ethanol.
(1mark)

(d) Ethene can undergo a reaction leading tom the formation of polyethene.
(i) Name the reaction that leads to the formation of polyethene from ethene
and
statewhat the reaction you have named means in general terms. (1marks)
(ii) Write the equation for the reaction of ethene to produce polyethene
(1mark)

(e)Polyethene is a good example of synthetic polymer.


Name one synthetic polymer other than Polyethene and state one use of it

(1marks)

(f) Some other polymers exist, which are not synthetically produced.
(i) state what such polymers are called. (
mark)
(ii) Give one example of such polymers that you have stated in (f)(i) and give
one
use of it. (1mark)

14. Dilute nitric acid reacts with marble chips at room temperature according to
the equation
CaCO3(s) + 2HNO3 (aq) Ca(NO3)2(aq)+CO2(g)+H2O(l)
The of this reaction can be affected by a number of factors

(a) State how the reaction would be affected if;


(i) marble chips was completely ground
(mark)
(ii) nitric acid was diluted
(mark)

(b) Exlain in each case the effect of the action taken in (a) on the rate of reaction
as stated by you (4marks)

(c) The table below shows time in minutes for the complete reaction when
various
volumes of 2M nitric acid were added separately to equal mass of marble
chips contained in six different beakers.

Volume of 2M nitric acid cm3 35.0 20.0 10.0 6.0 3.5 2.5
Time of completion of reaction 0 10 20 30 40 50
(min)

(i) Plot a graph of volume of nitric acid (vertical) against time (horizontal axis)
(4marks)
(ii) Using your graph, describe the effect of concentration of nitric acid on the
rate of reaction.
(4marks)
(iii) State how the graph you have plotted in (c)(i) can be use to determine the
rate of reaction at a given time
(2marks)
END

MOCK 2015 MARKING GUIDE PAPER 2


Ques Answer Mark
t
1 (i) Iodine & sodium chloride Sublimation (1)
(ii) Palm oil & water separating funnel (1)
(iii) Potassium chloride & potassium nitrate fraction crystallization (1)
(iv) Sodium chloride & sand filtration (1)
(v) Green chlorophyll & yellow chlorophyll paper chromatography (1)

Total 5
2(a) (i) 2NaNO3(s) 2NaNO2(s )+ O2(g) (1)
(ii) 2AgNO3(s)2Ag2(s)+ O2(g) + 2NO2(g) (1)2Ca(NO3)2(s)2CaO(s)+ O2(g)
+ 4NO2(g) (1)

(b) (i) The reaction does not take place or The reaction is hypothetical(1)
(ii) Potassium carbonate is stable to heat. ()

Total 6
3(a) (i) Oxygen jar(1)
(ii) Nitrogen contains a triple bond that requires more heat to break. ()It
takes more time to break compared to oxygen double bond ()

(i) MgO(s)+ H2O(l)Mg(OH)2(aq)(1)


(b) Mg3N2(s)+ 6H2O(l) 3Mg(OH)2(aq) +2NH3(g)(1)

(c) The universal indicator turned blue in each of the jars. (1)

Total 6
4(a) (i) Metallic character decreases across the period (1)
(ii) Metallic character increases across the period (1)

(b) (i) Amphoteric aluminium oxide(1)


(ii) Al2O3(s0+6HNO3(aq) 2Al(NO3)3(aq)+3 H2O(l) (1)

Total 4
5(a) (i) Fe(s)+2HCl(aq) FeCl2(aq)+H2 (g)(1)
(ii) 2Fe(s)+3Cl2(aq) 2FeCl3(aq)(1)

(b) (i) Bubble chlorine gas through the aqueous solution ()


(ii) 2FeCl2(aq)+Cl2 (g) 2FeCl3(aq)(1)
(iii) Sodium hydroxide solution ()
(iv) Brown precipitate (1)
Total 6
6(a) Arrow from copper to silver ()

(b) (i) Silver ()


(ii) Silver nitrate (1)

(c) (i) Cu(s) Cu 2+(aq)+ 2e (1)


(ii) Ag+(aq) + eAg(s) (1)
Total 5
7(a) (i) Zn2+& Pb2+
Reagent: potassium iodide solution,() ammonia, sulphuric acid, hydrochloric acid
(ii) SO42+& CO32-
Reagent: sulphuric acid(1)or hydrochloric acid or acidified() barium nitrate()

(b) (i) Zn2+& Pb2+


Observation:Zn2+ solution remains colorless(1) no observable change
Pb2+ yellow precipitate (1)
(ii) SO42+& CO32-
Observation:SO42- solution remains colorless(1) no observable change
CO32-effervescence bubbles of a colourless gas(1)
Total 4
8.(a) Standard solution is a solution of known concentration ; in moles per dm 3 or grams
per dm3(1)

(b) In 20cm3of sulphuric acid there are (20 x 1)/1000 =2.0 x 10 -2()
In 250cm3 of diluted solution thee are 2.0 x 10-2()
In 1000 cm3 of diluted solution thee are [ 2.0 x 10-2 x 1000]/250() =0.08()
No of moles of hydrogen ions 0.08 x2 = 0.16()
+
The new solution is 0.16M H ions()

Total 4
9(a) (i) X =17 2: 8: 7(1)
(ii) Y = 20 2: 8:8:2 (1)

(b) (i) X is in group VII ()


(ii) Y is in period 4 ()

(c) The compound is a solid with high melting point ()


The bond between X and Y is ionic () with strong electrostatic forces between
the ions
Total 4
10 Total volume of solution 1s 40 + 40 = 80 cm3
Density of solution is 80 x1 =80 g()
Heat evolved by solution is (80 x 4.2 x t)/1000 kJ ()

No of moles of acid or alkali or water formed = (40 x 2)/1000 =8x10 - 2 ()


When 1mole of water is formed the heat given off 56.5 kJ
8x10-2 moles of water is formed the heat given off 56.5 x 8x10 -2 =4.52 ()

4.52 kJ = (80 x 4.2 x t)/1000 ()


t =(4.52 X 1000)/( 80x4.2) =13.5 ()

13.5 = t -25.8()
t =39.3()

Total 15
14(a) (i) Marble chips ground into a powder ; the rate of reaction increases() 1
(ii) Nitric acid diluted; the rate of reaction decreases ()

(b) Ground marble chips have a large () surface area (). This provides a greater
chance ()for more particles to participate in the reaction()

Dilute nitric acid; there are fewer hydrogen ions () per unit volume of the acid
solution. The average distance between the hydrogen ions and carbonate ions in 4
the marble chips increases (). The number () and frequency of collisions
()between the hydrogen and carbonate ions decrease resulting in a slower rate of
reaction

(c) Axes (1) scale (1) shape (1) accurate plot (1) 4

Name: ................................................................Centre/Index No. ............./.......


Signature:.........................................................
545/2
Chemistry
Paper 2
July 2013
2 hours.

UGANDA CERTIFICATE OF EDUCATION


CHEMISTRY
PAPER 2
2 HOURS

INSTRUCTIONS TO CANDIDATES:
Section A consists of 10 structured questions. Attempt all questions in this
section. Answers to these questions must be written in the spaces provided.
Section B consists of 4 semi-structured questions. Attempt any two questions
from this section. Answers to the questions must be written in the answer
booklets provided.
In both sections, all working must be shown clearly.

1 2 3 4 5 6 7 8 9 10 11 12 13 14 Total

(N =14, Cl=35.5, H = 1)

SECTION A (50 marks)


1. A mixture of silicon (IV) oxide and zinc oxide was separated by filtration.
(a) Name the
(i) solvent used .................................................................................. (1mark)
(ii) filtrate contains .............................................................................. (1mark)
(iii) residue...................................................................................... . (1mark)
(b) Explain why it is possible to separate the mixture. (2marks)
.................................................................................................................................
.................................................................................................................................
............................................................................................................... .

2(a) The full symbol of the atom of an element is .


(i) Write the electronic configuration of element X (1mark)
.......................................................................................................................
(ii) Draw a diagram to show how X and hydrogen forms a compound.
(H=1) (2marks)

(b) The hydride of element X conducts electricity in the molten state.


Write the equation for the reaction occurring at the
(i) anode. (1mark)


(ii) cathode. (1mark)


3.(a) A hydrocarbon Y contains 85.7% carbon. Calculate the simplest formula.
(2marks)
......................................................................................................................
.....................................................................................................................
.....................................................................................................................
..................................................................................................................
..................................................................................................................
..................................................................................................................
..................................................................................................................

(b) 0.224g of hydrocarbon Y occupied 96 cm3 at room temperature.


(i) Calculate its molecular mass (1mark)

.....................................................................................................................
.....................................................................................................................
.....................................................................................................................
.....................................................................................................................

.................................................................................................................

(ii) Hence its molecular formula (1mark)

.....................................................................................................................
.....................................................................................................................
.....................................................................................................................
..................................................................................................................

(iii) Write down at least two structure formulae of Y (1mark)


.....................................................................................................................

....................................................................................................................

.....................................................................................................................

4. A mixture of ethanol and substance A was heated as shown in the diagram


below. A colourless gas B was evolved.
(a)(i) Identify substances A and B (1mark)

B
(ii) Write an equation for the reaction (1marks)

(b) State what is observed in the test tube containing bromine water. (1mark)

(ii) Write an equation for the reaction (1marks)

5(a) Write equation for the reaction that would take place if, strongly heated
iron filings was passed a current of
(i) dry hydrogen chloride (1mark)

(ii)dry chlorine gas (1mark)

(b)(i) State what would be done in order to convert the product in reaction (a)(i)
in its aqueous solution to the product in reaction (a)(ii) (mark)

(iv) Write equation to illustrate your answer in (b)(i) (1mark)

(iv) Name one reagent that would be used to confirm that the reaction in (a)(ii)
had taken place. (mark)

State what would be observed if the reagent you have named in (b)(iii) was
treated with the product of the reaction in (b)(ii) (1mark)

6. The following chart below shows the contact process that is used to
manufacture sulphuric acid.

(a) Identify substances X and Y


X: .. ( mark)
Y: . ( mark)

(b)(i) Name the catalyst used in Chamber A. (1mark)



(iii) Write equation for the reaction taking place in Chamber A.(1mark)

(c) State what is observed when concentrated sulphuric acid is added to
hydrated
(i) copper (II) sulphate. (1mark)

(ii) potassium chloride(mark)

7. 1.00g of pure ammonium chloride was boiled with 20.0cm3 of a solution of


sodium hydroxide until evolution of ammonia had ceased. The resulting
solution required 11.0 cm3 of 0.1M hydrochloric acid for neutralization.
(a). Write equation for the reaction that took place between ammonium
chloride and sodium hydroxide (1marks)

(b). Calculate
(i) volume of ammonia evolved at stp (2marks)




(ii) number of moles of sodium hydroxide that was in excess (1mark)




(iii) the molar concentration of sodium hydroxide. (1 marks)




..





.

8. The following experiment was set up.


(a)(i) State what was observed (1 mark)

(ii) Name gas X and briefly describe how the gas can be identified in the
laboratory (1mark)

(b) Carbon dioxide was bubbled into the solution in the beaker.
(i) State what was observed ( mark)

(ii) Write the equation for the reaction (1marks)

9.(a) What is an indicator? (1mark)


.................................................................................................................................
.................................................................................................................................
...............................................................................................................................
b) State and explain what would be observed when the universal indicator is
added to the following solutions.
(iii) Potassium sulphate solution (2marks)
Observation:.............................................................................................................
Explanation..............................................................................................................
.................................................................................................................................

(iv) Potassium carbonate solution (2marks)


Observation:.............................................................................................................
.Explanation.............................................................................................................
.................................................................................................................................
.................................................................................................................................
.............................................................................................................................
10. The table below shows some properties of some elements. Study the table
and answer the following questions

Element Melting Boiling Electrical Volume that Solubility


point 0C point 0C conductivity at contains 1 mole of in cold
room atoms under room water
temperature conditions in cm3
A 659 1997 Yes 10.0 Insoluble
B -101 -34 No 12,000 Soluble
C -39 357 Yes 1408 1nsoluble
D Sublimes above 3700 Yes 5.4 1nsoluble
E -249 -246 No 24,000 1nsoluble
F -7.2 58 no 25.6 soluble

(a) Name the particles that are responsible for the electrical conductivity of
Substance A ...................................................................................... (1 mark)

(b Which letter represents


mercury?.............................................................................................................. (1
mark)

(c) Which letter represents a noble gas? (1marks)

.................................................................................................................................

(d) Suggest the identity of elements F. (1mark)

.................................................................................................................................

(e) Comment on the nature of element B. Give reasons(2marks)


.................................................................................................................................
.................................................................................................................................
.................................................................................................................................

SECTION B
Answertwoquestions only in this section. Start each question on a fresh page.

11. Sodium carbonate 10 water contains water of crystallization.


(a) What do understand by the term water of crystallisation? (2marks)

(b) Sodium carbonate 10 water was exposed to the atmosphere.


(i) State what was observed (1mark) (ii) Name the
process taking place (1mark) (iii) Write
an equation for the reaction (1 marks)

(c)(i) Briefly describe how a sample of sodium carbonate is prepared in the


laboratory. (7 marks)
(ii) State two uses of sodium carbonate (2marks)

12. Explain each of the following observations. Write equations where


appropriate
(a) Sodium metal conducts electricity in the solid state whereas sodium
chloride conducts electricity in the molten state. (5marks)

(b) Hard water wastes soap. (5marks)

(c) Sulphuric acid is not used to dry ammonia gas (5marks)

(d) When a mixture of magnesium powder and copper (II) oxide are heated
together a mixture of a white and brown solid are formed. (5marks)

13.(a) Nitric acid is manufactured by catalytic oxidation of ammonia


(i) Name the catalyst which is used in the reaction. (1mark)
(ii) State two conditions for the oxidation of ammonia during the
manufacture of nitric acid other than the catalyst which you have named
in (a) (i). (1mark)
(iii) Using equation only show how nitric acid can be produced by the
catalytic oxidation of ammonia. (3marks)
(b) Concentrated nitric acid was reacted with copper turnings.
(i) State what was observed. (1mark)
(ii) Explain the observation. (2marks)
(ii) Write the equation for the reaction. (1mark)
3
(c) 1.0g of a metal carbonate, XCO3 was dissolved in 50cm 1M nitric acid.
The excess acid required 30.0 cm3 of 1M sodium hydroxide solution for
complete neutralization .Calculate the atomic mass of X. (5marks)

14(a).State the conditions under which oxygen can be produced from


(i) hydrogen peroxide
(ii) potassium nitrate
(iii) Write equations for the reaction leading to the formation of oxygen in
each case (4marks)
(b )(i) What is meant by the term rate of formation of oxygen? (1mark)
(ii) With reference to hydrogen peroxide state three ways in which the rate
of formation of oxygen is increased. (3marks)
(iii) Name the natural process by which oxygen is increased in the
atmosphere. (marks)

(b). State what would be observed and in each case, write an equation for
the reaction. (5marks)
(i) sodium burnt in excess oxygen.
(ii) phosphorous in excess oxygen
MAKING GUIDE 2013

QUE ANSWERS MAR


K
1. (a) (i) Solvent used is dilute hydrochloric acid (1) (any dilute mineral acid)
(ii) filtrate contains zinc ions (1) 3
(iii) residue silicon (IV) oxide (1)

Silicon (IV) oxide is acidic (); will not dissolve in the acidic (). Zinc oxide is
(b) amphoteric (), it acts as a base and dissolves in the acid() 2

TOTAL 5
2 (a) (i) X 2.8.2 (1)

(ii)

(i) anode
(b)
2H-(l) H2(g) + 2e (1)

(ii) cathode
X2+ (l) +2e X(l) (1)

TOTAL 5
3 % of hydrogen = 100 - 85.7 = 14.3 ()
Element C H

% 85.7 14.3

Mole ratio 85.7/12 =7.141 14.3/1 = 14.3 ()

Divide by smallest 7.141/7.141 = 1 14.3/7.141 = 2 ()

Simplest formula is CH2 ()


3
(i) no of moles in 96 cm 96/2400 ()
96/2400moles are contained in 0.224 g of Y
1 mole of Y is contained in 0.224 x 24000/ 96 = 56 g ()
The molecular mass is 56

(ii) (CH2)n = 56
14n = 56
n = 56/14 = 4 ()
The formula is C4H8 ()

(iii) CH3 CH CH CH3 () CH2 CH CH2 CH3() CH2 C (CH3)CH3

TOTAL 5
4(a) (i)A is sulphuric acid () B is ethene ()

CH3CH2OH Concentrated H2SO4CH2 =CH2 + H2O(1)

(b) (i) The bromine water turns from brown ()to colourless ()
(ii) CH2 =CH2 + Br2CH2BrCH2Br(1)

TOTAL 5
5(a) (iii) Fe(s)+2HCl(aq) FeCl2(aq)+H2 (g) (1)
(iv) 2Fe(s)+3Cl2(aq) 2FeCl3(aq)(1)

(b) (v) Bubble chlorine gas through the aqueous solution ()


(vi) 2FeCl2(aq)+Cl2 (g) 2FeCl 3(aq)(1)
(vii) Sodium hydroxide solution ()
(viii) Brown precipitate (1)

TOTAL 6
6(a) X is air ()
Y is 98% sulphuric acid ()

(b) (i) Catalyst; vanadium (V) oxide (1)


(ii) 2SO2(g) + O2(g) 2SO 3(g) (1)

(c) (i) The blue solid() changes to white()


(ii) White fumes ()
TOTAL 5
7(a) NaOH(s) + NH4Cl(s) NaCl(s)+ NH3(g) +H2O(g)(1)
(i)Rfm of NH4Cl =14+4+35.5 =53.5()
53.5g of ammonium chloride contain 1 mole
(b) 1g of ammonium chloride contain 1/53.5 = 0.0187 moles()
Reaction ratio of NH4Cl : NH3 is 1:1
No of moles of ammonia produced is 0.0187 moles()
1mole of ammonia occupy 22.4dm3
0.0187moles of ammonia 22.4 x 0.0187 = 0.4187 dm3()

(ii) In 1000cm3 of hydrochloric acid there are 0.1 moles


In 11.0cm3 of hydrochloric acid there are(0.1 X 11.0)/1000 =1.1 x 10 -3moles()
Reaction ratio of HCl : NaOH is 1:1
no of moles of sodium hydroxide that was in excess = 1.1 x 10 -3 ()

(iii) total number of moles of sodium hydroxide in 20 cm 3 = 1.1 x 10-3 +0.0187 ()


= 0.0198
3
In 1000cm there are (0.0198 x 1000)/20 = 0.99 ()
The molarity of sodium hydroxide is 0.99M()

TOTAL 5
8(a) (i) bubbles() of a colourless gas()
(ii) The gas is hydrogen (1) a burning splint ()is introduced into the test tube, the
gas explodes() or pops

(b) (i) A white precipitate is formed. ()


(ii) Ca(OH)2(aq) + CO2(g) CaCO3(s) + H2O(l) (1)

TOTAL 5
9.(a) An indicator is substance that changes colour according to the hydrogen ion
concentration or hydroxide concentration. (1)

(b) (i) Potassium sulphate;


Observation: The colourless solution changes to green.()
Explanation: The solution is neutral because the salt is made from both a strong
acid ()and a strong alkali, ()which are fully dissociated the number of hydroxide
ions and hydrogen ions are equal()

(ii) potassium carbonate


Observation: The colourless solution changes to blue.()
Explanation: The solution is alkaline because the salt is made from both a weak
acid which is partially ionised ()and a strong alkali which is fully dissociated()
The number of hydroxide ions is greater than hydrogen ions()
TOTAL 5
10(a Substance A; electrons (1)
)
Mercury is C (1)
(b)
Noble gas is E (1)
(c) 1 mole of E occupy 24000 cm3(1)

Element F is a liquid it could be bromine (1)


(d)
TOTAL 5
11(a) Water of crystallization is a definite amount of water that a substance is
chemically combined when it crystallizes from an aqueous solution (2)

(b) (i) The white crystals () turn to a white powder ()


(ii) Efflorescence (1)
(iii) Na2CO3.10 H2O (s) Na2CO3.1H2O (s) + 9 H2O(l)(1)

(i) Carbon dioxide () free from acid () is bubbledinto a fairly concentrated


solution () of sodium hydroxide ()until a white precipitate is formed.()
(c) The precipitate is filtered off (), washed in a little water () and dried between
filter paper. ()
The precipitate is then heated () to constant mass ()
CO2 (g) + NaOH(aq) NaHCO3(s)(1)
2NaHCO3(s) Na2CO3(s) + CO2(g) + H2O(l)(1)

(ii) sodium carbonate is used to


soften hard water(1),
make glass (1)

TOTAL 15
12(a Sodium metal has delocalized electrons (1)that are responsible for carrying the 5
) current.
In the solid state sodium ions () and chloride ions () are firmly held by strong
electrostatic forces (1) therefore t are not mobile () hence inability to conduct
electricity.
In the molten state heat () breaks () the strong electrostatic forces allowing
the ions to move () and hence conduction of electricity.
Hard water contains calcium () and magnesium () ions. On addition of soap, scum
(b) (1) is formed which is an insoluble salt (1)of magnesium or calcium stearateas
follows:
Ca2+(aq) + 2St-(aq) Ca St2(s)(1) 5
It is only when all the calcium and magnesium ions have been removed() from
water that a lather will form() thus wastage of soap

(c) Sulphuric acid is an acid,(1) ammonia is alkaline,(1) and the two react () forming a
salt.(1)
H2SO4(aq) + 2NH3(g) (NH4)2SO4(aq)(1)
5
(d) Magnesium is higher in the reactivity series than copper() it reduces (1)
copper(II)oxide to copper(),; the brown solid and itself is oxidised(1) to
magnesium oxide(),; the white solid.
Equation: CuO(g) + Mg(s) Cu (s) + MgO(s) (1) 5

TOTAL 15
13(a (i) platinum-rhodium (1). 5
0
) (ii) temperature 900 C () , excess oxygen (), slight high pressure
(iii) 4NH3(g) + 5 O2(g) 4NO(g)+ 6H2O(l) (1)

2NO(g)+ O2(g) 2NO2(g) (1)


4NO2(g) +2H2O(l) + O2(g) 4HNO3(g) (1)

(i) Observations: a brown gas was given off (). The copper turnings dissolved
(b) forming a blue solution ()
4
(ii) Explanation: The acid is reduces () to nitrogen (IV) oxide,()
the brown gas and copper is oxidized() to copper (II) nitrate (), the blue
solution.

(ii) Cu (s) + 4HNO3 (aq)Cu(NO3)2 (aq + 2NO2(g) +2H2O(l) (1)


The original number of moles of nitric acid used = (50 x 1)/1000 = 5 x 10-2 ()
The number of moles of sodium hydroxide that reacted
= (30 x 1)/1000 = 3 x 10-2 () 5
The ratio NaOH:HNO3 is 1:1
The number of moles of nitric acid that reacted with sodium hydroxide
3 x 10-2 ()
The number of moles of nitric acid that reacted with the carbonate
( 5 x 10 -2-3 x 10-2) () = 2 x 10-2 ()
The ratio XCO3:HNO3 is 1:2 ()
-2 -2
The number of moles of XCO3 is (2 x 10 )/2 =1 x 10 ()
-2
1 x 10 moles of XCO3 are contained in 1.0g ()
-2
1mole of XCO3 is contained in 1.0/1 x 10 = 100 ()

X + 12+ (16 x3) = 100 ()


X = 100 (12+ (16 x3))
X =40 ()

TOTAL 15
14(a (i) hydrogen peroxide; room temperature ()and a catalyst() 4
)
(ii) (ii) potassium nitrate; heat()

(iii) Equation:
2H2O2(aq) 2H2O (l)) + O2(l) (1)
2KNO3(s) 2KNO2(s) + O2(l) (1)

(i) The rate of formation of oxygen is the increase in volume of oxygen per unit
time. (2)
(b)
(ii) adding a catalyst e.g. manganese (IV) oxide (1)
Increasing the concentration of hydrogen peroxide (1) 5
Exposing the mixture to sunlight. (1)

(iii) photosynthesis()

Sodium first melts and then burns with bright yellow flame ()forming a yellow
solid()
2Na(s) + O2(g) Na2O2(g) (1)
Phosphorous burns with yellow flame () and forms dense white fumes() 5
P4(s) + 5O2(g) 2P2O5(s) (1)

TOTAL 15
Name..Centre/Index No
Signature.

545/2
Chemistry
Paper 2
August 2015

Uganda Certificate of Education


UNNASE JOINT MOCK EXAMINATIONS
CHEMISTRY
PAPER 2
2 HOURS

Instructions:
This paper consists of two sections A and B
Attempt all questions in section A
Answers to section A must be written in the spaces provide only
Attempt two questions in section B

FOR EXAMINERS USE ONLY


Questio 01 02 03 04 05 06 07 08 09 10 11 12 13 14
n

Marks

SECTION A (50 marks)


1. Name one pair of substance in each case, which when mixed together
can be separated by
(a) Use of a separating funnel (1 marks)

(b) Fractional crystallization (1 mark)

(c) Filtration (1 mark)

(d) Chromatography (1 mark)

(e) Fractional distillation (1 mark)

2. 90 cm3 of 0.01M calcium hydroxide solution was added to a sample of


water containing 0.001 moles of calcium hydrogen carbonate.
(i) State what was observed (1 mark)


(ii) Write an equation for the reaction which took place. (1 marks)


(iii) Calculate the number of moles of calcium ions in 90cm3 of 0.01M
calcium hydroxide. (1mark)
...
...
...
..
..
...

..
(iv) State what would be observed if soap solution was added drop-wise to
a sample of the water after the addition of calcium hydroxide. Give a
reason. (1mark)


3. (a) When exposed to sunlight, hydrogen peroxide produces gas bubbles.


(i) Identify the gas that bubbles out from the hydrogen peroxide. (1mark)
...
(ii) Write an equation for the reaction leading to the formation of the gas you
have identified in (i) from hydrogen peroxide. (1mark)

(b) Manganese (IV) oxide was added to hydrogen peroxide and the mixture
was then exposed to sunlight
(i) State the rate of gas bubbling in this mixture would differ from that in (a)

(1mark)

..
(ii) Give reasons for your answer in (b) (i) (1 mark)




c) Other than the use of Manganese (IV) oxide, suggest one other thing
that could be done to enhance change in the rate of gas bubbling by the
hydrogen peroxide. (1 marks)



14. a) Write an equation for the ionization of sodium carbonate Na2CO3 in water


b) Water was added to 250cm3 of a 0.4M sodium carbonate solution to make
1 litre of a diluted solution. Calculate the concentration of sodium ions in
moles per litre of the diluted solution. (3 marks)










5. a) Excess carbon monoxide gas was passed over a heated sample of an
oxide of iron as shown in the diagram below. Study the diagram and the
data below to answer the questions that follow.

Mass of empty dish =10.98g


Mass of empty dish + oxide of iron =13.30g
Mass of empty dish + residue =12.66g
(iv) Determine the formula of the oxide of iron. (Fe=56, O=16) (3 marks)

(v) Write an equation for the reaction which took place in the dish (1 marks)


(b) Rusting is a destructive process in which iron is converted into hydrated
iron (iii) oxide.
(v) State two conditions necessary for rusting to occur
(1
mark)


One method to protect iron from rusting (1 mark)

6. Ethanol can be converted to Ethene according to the following equation


C2H6O(l) - H2 O C2H4(g)

(a) State the necessary conditions for conversion of ethanol to ethene.


(1
marks)


b) i) Name one reagent that can be used to distinguish between ethene and
sulphur dioxide gas. (1 marks)

(ii) State what would be observed if the reagent you have named in (b) (i)
was separately treated with each gas. (2marks)



iii) Write equations to illustrate your observation in (b) (ii) between Ethene
and the reagent you have named in (b) (i). (1 mark)

7. The elements: Aluminium, sulphur and calcium can each combine with
oxygen to form oxides.
a) State the class of the oxide of
(i) Aluminium ( mark)
.
(ii) Sulphur ( mark)
.
(iii) Calcium ( mark)

(b) (i) State what would be observed if a few drops of water were added to the
oxide of calcium. (1 marks)


(ii) Write the equation that takes place when water is added to the oxide of
calcium. (1 mark)


(c) Name one other oxide in the same class as the oxide of aluminium.
(1
marks)

8. The diagram below shows sketches of curves that were obtained in three
experiments to study the rates of reactions of dilute sulphuric acid and iron
metal.
mass of
flask and its
contents

Y Z

X
M

Time (minutes)

(a) In the first experiment, curve y was obtained.


State three factors that could have been changed to obtain curve x. (3
mks)



(b) State;
(i) The significance of point M (1 mark)


(ii) One factor that must have changed to obtain curve z from the conditions
that produced curve x. (1 mark)

9. Name one reagent which could be used to distinguish between members


of each of the following pairs of ions and in each case state what would be
observed if the reagent you have named was treated separately with each
member of the pair. (2 marks
each)

Pairs of ions Reagent Observation


(a) I- and Cl-

(b) CO32- and HCO3-

(c) Zn2+ and Pb2+

10. 1.9g of magnesium chloride were dissolved in distilled water. Silver nitrate
solution was added until in excess.
(a) Write equation for the reaction that took place. (1 marks)


(b) Calculate the mass of silver nitrate that was used for complete reaction.
(RFM of MgCl2 = 95, N = 14.0, O = 16.0, Ag = 108.0) (2 marks)

SECTION B
11. Explain the following observation. (Where necessary illustrate your answer
with equations)
(a) When heated, a mixture of ethanol and concentrated sulphuric acid
produces a gas that decolorizes bromine water. (4 marks)

(b) Sodium chloride in the solid state does not conduct electricity but it will
when molten and is decomposed by electric current. (6 marks)

(c) When aqueous ammonia is added to a solution containing zinc ions, a


white precipitate is formed, followed by a colourless solution. (5 marks)

12. Draw a labeled diagram for the set up of apparatus that can be used to
electrolyze lead (II) bromide. (3 marks)

(b) Describe the reactions that take place during the electrolysis of copper (ii)
sulphate solution using
(i) Graphite electrodes (6 marks)
(ii) Copper electrodes (4 marks)

(c) State one industrial application of electrolysis other than the purification of
copper. (1 mark)

13(a)(i) Define the term solubility of a solute. (2 marks)


(ii) State how temperature and quantity of a solvent c an affect the solubility
of a solute. (2 marks)

(b) The solubility of a salt X at various temperatures are shown in the table
below.

Temp (oC) 0 20 40 60 80 100


Solubility (g/100g of water) 4.0 5.0 12.0 21.5 39.5 60.0

Plot a graph of solubility of X (vertical axis) against temperature. (5 marks)

(c) Use the graph in (b) to determine


(i) The solubility of X at 15 oC and 75 oC (2 marks)
(ii) The mass of X which would crystallize if 39.5g of a saturated solution at
80oC was cooled to 15oC (1
mark)

(d) State
(i) How you would distinguish between soft water and hard water (1 marks)
(ii) One use of hard water (1 marks)

(e) Name one compound which when dissolved in water will cause;
(i) Permanent hardness of water
(iii) Temporary hardness of water

14( a)( i) Name the raw material from which sulphuric acid can be manufactured
by the contact process (1 mark)
(ii) With the help of equations, outline the reactions which take place during
the contact process. (5
marks)
(b) Explain why fuming sulphuric acid has no effect on litmus paper whereas
dilute sulphuric acid readily turns blue litmus paper red. (3 marks)

(c) State what would be observed and write equation for the reaction that
would take place when concentrated sulphuric acid was added to;
(i) Iron (ii) sulphate -7- water (3 marks)
(ii) Potassium chloride (3 marks)

END
Name..Centre/Index No

Signature.

545/2

Chemistry

Paper 2

August 2011

CHEMISTRY

PAPER 2

2 HOURS

Instructions:

Attempt All questions in section A and two questions in section B

Official use only


Qns. 1 2 3 4 5 6 7 8 9 10 11 12 13 14 Tota
l
Mark
s
1.(a) Give a reason in each case, why when heated strongly, the mass
of:

i) Zinc oxide does not change (


mark)

........................................................................................................
................................................... ............................................................
...............................................................................................

ii) Copper foil increases (


mark)

........................................................................................................
...................................................

........................................................................................................
...................................................

iii) Charcoal decreases (


mark)

........................................................................................................
...................................................

........................................................................................................
...................................................

b) A hydrate salt, XSO4.nH2O when heated decomposes according


to the following equation.

XSO4.H2O(s) XSO4(s) + nH2O(l)

When 9.28 of the salt was heated, 4.09g of the residue was
obtained. Calculate the number of moles of water of crystallization in
the hydrated salt

(H=1, O=16, S=32, X=46) (3


marks)
........................................................................................................
...................................................

........................................................................................................
...................................................

........................................................................................................
...................................................

........................................................................................................
...................................................

........................................................................................................
...................................................

........................................................................................................
...................................................

........................................................................................................
...................................................

........................................................................................................
...................................................

........................................................................................................
...................................................

........................................................................................................
...................................................

........................................................................................................
...................................................

........................................................................................................
...................................................

2. (a) Write equation for the reaction that would take place if burning
magnesium was lowered into a gas jar of;
i) Carbon dioxide (1
marks)

........................................................................................................
...................................................

........................................................................................................
...................................................

........................................................................................................
...................................................

ii) Nitrogen (1
marks)

........................................................................................................
................................................... ............................................................
............................................................................................... .................
.................................................................................................................
.........................

b) i)State what would be observed if the product(s) in (a) (i)


was/were shaken with water.
(1 mark)

........................................................................................................
.................................................. ............................................................
.............................................................................................. .................
.................................................................................................................
........................

ii) Write equation for the reaction that would take place if a few
drops of water were added to the product in (a) (ii) (1
marks)

........................................................................................................
.................................................. ............................................................
.............................................................................................. .................
.................................................................................................................
........................
3. Impure alcohol can be prepared from a solution of glucose,
C6H12O6 mixed with yeast.

a) (i)Name the process by which glucose in the presence of yeast


can be converted into alcohol.
(1 mark)

........................................................................................................
..................................................

........................................................................................................
..................................................

ii)State the role of yeast (1


mark)

........................................................................................................
..................................................

........................................................................................................
..................................................

iii)Write equation for the reaction leading to the formation of


alcohol

(1 mark)

........................................................................................................
..................................................

........................................................................................................
..................................................

b) State

i) How the alcohol can be purified (1


mark)
........................................................................................................
..................................................

........................................................................................................
..................................................

ii) One method that can be used to test for the purity of the
alcohol(1mark)
........................................................................................................
..................................................

........................................................................................................
..................................................

4. a) A hydrocarbon W, molecular mass = 58 contains 82.8% carbon


and 17.2 hydrogen

i) Calculate the empirical formula of W (3


marks)

........................................................................................................
.................................................. ............................................................
.............................................................................................. .................
.................................................................................................................
........................ .......................................................................................
................................................................... ...........................................
...............................................................................................................
........................................................................................................
..................................................

ii) Determine the molecular formula of W (1


mark)

........................................................................................................
..................................................
........................................................................................................
..................................................

........................................................................................................
..................................................

b) The empirical formula of another hydrocarbon Y, molecular mass


= 56 was found to be CH2

i) Deduce the molecular formula of Y (1


mark)

........................................................................................................
.................................................. ............................................................
.............................................................................................. .................
.................................................................................................................
........................

ii) State the structural difference between W and Y


(1 mark)

........................................................................................................
.................................................. ............................................................
.............................................................................................. .................
.................................................................................................................
........................

5. The atomic numbers of elements Q and R are 17 and 19


respectively.

a) Write the electronic configuration of

i) Q (1 mark)

........................................................................................................
..................................................

ii) R (1 mark)
........................................................................................................
..................................................

b) State

i) The group in the periodic table to which R belongs (1


marks)

........................................................................................................
...............................................

ii) The type of bond in the compound that can be formed when Q
reacts with R. (1
marks)

........................................................................................................
..................................................

c) Predict the effect of the heat on the carbonate of R.

........................................................................................................
..................................................

........................................................................................................
..................................................

d) If another element T was found to be below Q in the same group


in the periodic table, suggest whether T would be coloured and
monoatomic or colourless and diatomic or coloured and diatomic or
colourless and monoatomic.
(1 mark)

........................................................................................................
..................................................
6. a) State on use of sodium hydroxide in

i) A chemistry laboratory (1
marks)

........................................................................................................
..................................................

........................................................................................................
..................................................

ii) An industry (1
marks)

........................................................................................................
..................................................

........................................................................................................
..................................................

........................................................................................................
..................................................

b) Dilute sodium hydroxide solution was added to ammonium


carbonate and the mixture warmed

i) State what was observed (1 mark)

........................................................................................................
..................................................

........................................................................................................
..................................................

ii) Write an ionic equation for the reaction that took place.
(1 marks)

........................................................................................................
..................................................
........................................................................................................
..................................................

........................................................................................................
..................................................

c) Name one reagent that can be used for identifying a carbonate


ion

(
mark)

........................................................................................................
.................................................. ............................................................
..............................................................................................

ii) State what would be observed if the reagent you have named
in (c) (i) was treated with carbonate ion
( mark)

........................................................................................................
.................................................. ............................................................
..............................................................................................

7. Under suitable conditions, sulphuric acid can react with ethanol to


produce ethene.

a) State

i) The conditions under which the reaction takes place (1


marks)

........................................................................................................
.................................................. ............................................................
..............................................................................................

ii) The property of sulphuric acid that is shown by the reaction


(1mark)
........................................................................................................
.................................................. ............................................................
..............................................................................................

b) Write equation for the reaction that takes place when ethene is

i) Burnt in an unlimited supply of oxygen (1


marks)

........................................................................................................
.................................................. ............................................................
..............................................................................................

8. State what would be observed if a solution containing lead (II)


ions was added to dilute solution of

i) Hydrochloric acid (
mark)

........................................................................................................
.................................................. ............................................................
..............................................................................................

ii) Sulphuric acid (


mark)

........................................................................................................
.................................................. ............................................................
..............................................................................................

b) Describe briefly how the products in (a) (i) and (ii) can be
distinguished.

(1
marks)

........................................................................................................
.................................................. ............................................................
.............................................................................................. .................
.................................................................................................................
........................ .......................................................................................
................................................................... ...........................................
...............................................................................................................

c) Write an ionic equation to show how lead (II) ions can be


identified in solution.
(1 marks)

........................................................................................................
.................................................. ............................................................
.............................................................................................. .................
.................................................................................................................
........................

9. a) i) Name a substance which when reacted with sodium peroxide


can produce oxygen (
mark)

........................................................................................................
.................................................. ............................................................
..............................................................................................

ii) Write an equation for the reaction leading to the formation of


oxygen.

(1 marks)

........................................................................................................
.................................................. ............................................................
.............................................................................................. .................
.................................................................................................................
........................

b) Write an equation for the reaction that takes place when

i) Hydrogen is burnt in excess oxygen (1


marks)
........................................................................................................
.................................................. ............................................................
..............................................................................................

ii) Sodium is burnt in limited supply of oxygen (1


marks)

........................................................................................................
.................................................. ............................................................
..............................................................................................

........................................................................................................
..................................................

10. a) Copper (II) carbonate was heated strongly until there was
no further change.

i) State what was observed (1


mark)

.......................................................................................................
...................................................

........................................................................................................
..................................................

........................................................................................................
..................................................

ii) Write an equation for the reaction that took place (1


marks)

........................................................................................................
..................................................

........................................................................................................
..................................................

........................................................................................................
..................................................
b) When heated, zinc nitrate decomposes to give nitrogen dioxide
and oxygen according to the following equation

2Zn (NO3)2 (s) 2ZnO(s) + 4NO2 (g) + O2 (g)

Calculate the volume of nitrogen dioxide that would be evolved at


s.t.p when Zinc nitrate decomposed to give out 2.5g of oxygen.

(N=14, O=16, 1 mole of a gas occupies 22.4dm 3 at s.t.p)

........................................................................................................
..................................................

........................................................................................................
..................................................

........................................................................................................
..................................................

........................................................................................................
..................................................

........................................................................................................
..................................................

........................................................................................................
..................................................

........................................................................................................
..................................................

........................................................................................................
..................................................

........................................................................................................
..................................................

SECTION B: (30 MARKS)

Answer any Two questions from this section


11. a) i) Distinguish between the terms electrode and electrolyte
(2 marks)

ii) Name the particles by which electric current is conducted in an


electrode and in an electrolyte respectively. (1
mark)

b) Draw a labelled diagram to show how a solution of copper (II)


sulphate can be electrolyzed using graphite electrodes; and explain
how the products at the electrodes are formed.
(9 marks)

c) Electrolysis of copper (II) sulphate in (b) was repeated using


copper electrodes

i) Identify the substance that was formed at the anode and


explain briefly why the substance you have identified forms.
(1 marks)

ii) State two industrial applications of electrolysis of Copper (II)


sulphate solution using copper as the anode.
(1 mark)

d) When copper (II) chloride solution was electrolysed between


graphite electrodes, chlorine was evolved at the anode.

State the condition under which the electrolysis was carried out
( mark)

12. a) Define the term

i. Enthalpy of neutralization (1 mark)


ii. Enthalpy of combustion (1
mark)
b) When 70.0cm3 of a 1M nitric acid was mixed with 35.0cm3 of a 1M
calcium hydroxide solution, the temperature of the solution rose
from 27.0 C to 35.8oC. Calculate the enthalpy of neutralization of
o

calcium hydroxide by nitric acid and state the units for your answer.
(4 marks)
(Specify heat capacity of the solution and its density are 4.2Jg -1oC-
1
and 1.0gcm-3 respectively)

C) The enthalpy of combustion of sugar, C12H22O11 is -5865KJmol-1

i. Write the equation for the complete combustion of sugar


ii. Calculate the quantity of heat energy that would be produced if
6.84g of sugar was completely.
(2 marks)
(H=1, C=12, O=16)

d) The enthalpies of combustion of some carbon compounds and the


number of moles of carbon atoms per moles of each of the
compounds are shown in the table below

Enthalpy of 730 1400 2020 2700 3340


combustion/KJmol-1
Moles of carbon atoms 1 2 3 4 5

i. Plot a graph of enthalpy of combustion against number of moles


of carbon atoms
(4 marks)
ii. Comment on the shape of the graph (1
mark)
iii. Determine the enthalpy of combustion of the carbon compound
containing six moles of carbon atoms (1
mark)

13. a) i) Name the raw material for the manufacture of sulphuric acid
by the contact process. (
mark)

ii) Outline with aid of equations the reactions which take place
during the contact process. (6
marks)

iii) State one industrial use of sulphuric acid


b) Write an ionic equation for the reaction which can show that
sulphuric acid is an acid. (1
marks)

c) Warm concentrated sulphuric acid was added to sodium chloride.

i. State what was observed (1


mark)
ii. Write an equation for the reaction which took place. (
mark)

d) Identify which one of the products in (c) (ii) will

i. When dissolved in water, will give a solution with PH equal to 7.


( mark)
ii. React with iron to form a white solid (1
marks)

e) i) Write an equation for the reaction in (d) (ii) (1


marks)

ii) What would be observed if to an aqueous solution of the


product in (e) (i), was concentrated nitric acid.
(1 mark)

14 a) Describe how a sample of dry ammonia can be prepared in


the laboratory.

(Your answer should include equations for the reaction that takes
place; but No diagram is required)
(5 marks)

b) Give a reason why ammonia cannot be

i. collected over water (1


marks)
ii. dried using sulphuric acid or fused calcium chloride. (2
marks)

c) State the condition(s) under which ammonia can react with


oxygen and write equation(s) for the reaction(s) that took place.
(4 marks)

d) Dry ammonia was passed over hot lead (II) oxide. Write the
equation for the reaction which took place.
(1 marks)

e) State what would be observed; if aqueous ammonia was added


drop wise until in excess solution containing:

i. aluminium ion (1mark)


ii. zinc ion (1mark)

You might also like